Site Loader

Π‘ΠΎΠ΄Π΅Ρ€ΠΆΠ°Π½ΠΈΠ΅

Π’Π΅ΠΊΡ‚ΠΎΡ€Π½ΠΎΠ΅ ΠŸΡ€ΠΎΠΈΠ·Π²Π΅Π΄Π΅Π½ΠΈΠ΅ Π’Π΅ΠΊΡ‚ΠΎΡ€ΠΎΠ². Бвойства, ΠΎΠΏΡ€Π΅Π΄Π΅Π»Π΅Π½ΠΈΠ΅

ΠžΠΏΡ€Π΅Π΄Π΅Π»Π΅Π½ΠΈΠ΅ Π²Π΅ΠΊΡ‚ΠΎΡ€Π½ΠΎΠ³ΠΎ произвСдСния

БистСма ΠΊΠΎΠΎΡ€Π΄ΠΈΠ½Π°Ρ‚ β€” способ ΠΎΠΏΡ€Π΅Π΄Π΅Π»ΠΈΡ‚ΡŒ ΠΏΠΎΠ»ΠΎΠΆΠ΅Π½ΠΈΠ΅ ΠΈ ΠΏΠ΅Ρ€Π΅ΠΌΠ΅Ρ‰Π΅Π½ΠΈΠ΅ Ρ‚ΠΎΡ‡ΠΊΠΈ ΠΈΠ»ΠΈ Ρ‚Π΅Π»Π° с ΠΏΠΎΠΌΠΎΡ‰ΡŒΡŽ чисСл ΠΈΠ»ΠΈ Π΄Ρ€ΡƒΠ³ΠΈΡ… символов.

ΠšΠΎΠΎΡ€Π΄ΠΈΠ½Π°Ρ‚Ρ‹ β€” это ΡΠΎΠ²ΠΎΠΊΡƒΠΏΠ½ΠΎΡΡ‚ΡŒ чисСл, ΠΊΠΎΡ‚ΠΎΡ€Ρ‹Π΅ ΠΎΠΏΡ€Π΅Π΄Π΅Π»ΡΡŽΡ‚ ΠΏΠΎΠ»ΠΎΠΆΠ΅Π½ΠΈΠ΅ ΠΊΠ°ΠΊΠΎΠ³ΠΎ-Π»ΠΈΠ±ΠΎ ΠΎΠ±ΡŠΠ΅ΠΊΡ‚Π° Π½Π° прямой, плоскости, повСрхности ΠΈΠ»ΠΈ Π² пространствС. Как Π½Π°ΠΉΡ‚ΠΈ ΠΊΠΎΠΎΡ€Π΄ΠΈΠ½Π°Ρ‚Ρ‹ Ρ‚ΠΎΡ‡ΠΊΠΈ ΠΌΡ‹ рассказали Π² этой ΡΡ‚Π°Ρ‚ΡŒΠ΅.

Бкаляр β€” это Π²Π΅Π»ΠΈΡ‡ΠΈΠ½Π°, которая ΠΏΠΎΠ»Π½ΠΎΡΡ‚ΡŒΡŽ опрСдСляСтся Π² любой ΠΊΠΎΠΎΡ€Π΄ΠΈΠ½Π°Ρ‚Π½ΠΎΠΉ систСмС ΠΎΠ΄Π½ΠΈΠΌ числом ΠΈΠ»ΠΈ Ρ„ΡƒΠ½ΠΊΡ†ΠΈΠ΅ΠΉ.

Π’Π΅ΠΊΡ‚ΠΎΡ€ β€” Π½Π°ΠΏΡ€Π°Π²Π»Π΅Π½Π½Ρ‹ΠΉ ΠΎΡ‚Ρ€Π΅Π·ΠΎΠΊ прямой, для ΠΊΠΎΡ‚ΠΎΡ€ΠΎΠ³ΠΎ ΡƒΠΊΠ°Π·Π°Π½ΠΎ, какая Ρ‚ΠΎΡ‡ΠΊΠ° являСтся Π½Π°Ρ‡Π°Π»ΠΎΠΌ, Π° какая β€” ΠΊΠΎΠ½Ρ†ΠΎΠΌ.


Π’Π΅ΠΊΡ‚ΠΎΡ€ с Π½Π°Ρ‡Π°Π»ΠΎΠΌ Π² Ρ‚ΠΎΡ‡ΠΊΠ΅ A ΠΈ ΠΊΠΎΠ½Ρ†ΠΎΠΌ Π² Ρ‚ΠΎΡ‡ΠΊΠ΅ B принято ΠΎΠ±ΠΎΠ·Π½Π°Ρ‡Π°Ρ‚ΡŒ ΠΊΠ°ΠΊ β†’AB. Π’Π΅ΠΊΡ‚ΠΎΡ€Ρ‹ Ρ‚Π°ΠΊΠΆΠ΅ ΠΌΠΎΠΆΠ½ΠΎ ΠΎΠ±ΠΎΠ·Π½Π°Ρ‡Π°Ρ‚ΡŒ ΠΌΠ°Π»Ρ‹ΠΌΠΈ латинскими Π±ΡƒΠΊΠ²Π°ΠΌΠΈ со стрСлкой ΠΈΠ»ΠΈ Ρ‡Π΅Ρ€Ρ‚ΠΎΡ‡ΠΊΠΎΠΉ Π½Π°Π΄ Π½ΠΈΠΌΠΈ, Π²ΠΎΡ‚ Ρ‚Π°ΠΊ: β†’a.

ΠšΠΎΠ»Π»ΠΈΠ½Π΅Π°Ρ€Π½ΠΎΡΡ‚ΡŒ β€” ΠΎΡ‚Π½ΠΎΡˆΠ΅Π½ΠΈΠ΅ ΠΏΠ°Ρ€Π°Π»Π»Π΅Π»ΡŒΠ½ΠΎΡΡ‚ΠΈ Π²Π΅ΠΊΡ‚ΠΎΡ€ΠΎΠ². Π”Π²Π° Π½Π΅Π½ΡƒΠ»Π΅Π²Ρ‹Ρ… Π²Π΅ΠΊΡ‚ΠΎΡ€Π° Π½Π°Π·Ρ‹Π²Π°ΡŽΡ‚ΡΡ ΠΊΠΎΠ»Π»ΠΈΠ½Π΅Π°Ρ€Π½Ρ‹ΠΌΠΈ, Ссли ΠΎΠ½ΠΈ Π»Π΅ΠΆΠ°Ρ‚ Π½Π° ΠΏΠ°Ρ€Π°Π»Π»Π΅Π»ΡŒΠ½Ρ‹Ρ… прямых ΠΈΠ»ΠΈ Π½Π° ΠΎΠ΄Π½ΠΎΠΉ прямой.

ΠŸΡ€ΠΎΡ‰Π΅ говоря это Β«ΠΏΠ°Ρ€Π°Π»Π»Π΅Π»ΡŒΠ½Ρ‹Π΅Β» Π²Π΅ΠΊΡ‚ΠΎΡ€Ρ‹. ΠšΠΎΠ»Π»ΠΈΠ½Π΅Π°Ρ€Π½Ρ‹Π΅ Π²Π΅ΠΊΡ‚ΠΎΡ€Ρ‹ ΠΌΠΎΠ³ΡƒΡ‚ Π±Ρ‹Ρ‚ΡŒ ΠΎΠ΄ΠΈΠ½Π°ΠΊΠΎΠ²ΠΎ Π½Π°ΠΏΡ€Π°Π²Π»Π΅Π½Ρ‹ ΠΈΠ»ΠΈ ΠΏΡ€ΠΎΡ‚ΠΈΠ²ΠΎΠΏΠΎΠ»ΠΎΠΆΠ½ΠΎ Π½Π°ΠΏΡ€Π°Π²Π»Π΅Π½Ρ‹. ОсновноС ΠΎΠ±ΠΎΠ·Π½Π°Ρ‡Π΅Π½ΠΈΠ΅ β€” β†’a || β†’b. Π‘ΠΎΠ½Π°ΠΏΡ€Π°Π²Π»Π΅Π½Π½Ρ‹Π΅ ΠΊΠΎΠ»Π»ΠΈΠ½Π΅Π°Ρ€Π½Ρ‹Π΅ Π²Π΅ΠΊΡ‚ΠΎΡ€Ρ‹ ΠΎΠ±ΠΎΠ·Π½Π°Ρ‡Π°ΡŽΡ‚ΡΡ Ρ‚Π°ΠΊ β†’a ↑↑ β†’b, ΠΏΡ€ΠΎΡ‚ΠΈΠ²ΠΎΠΏΠΎΠ»ΠΎΠΆΠ½ΠΎ Π½Π°ΠΏΡ€Π°Π²Π»Π΅Π½Π½Ρ‹Π΅ β€” β†’a ↑↓ β†’b.

ΠŸΡ€Π΅ΠΆΠ΄Π΅ Ρ‡Π΅ΠΌ Π΄Π°Ρ‚ΡŒ ΠΎΠΏΡ€Π΅Π΄Π΅Π»Π΅Π½ΠΈΠ΅ Π²Π΅ΠΊΡ‚ΠΎΡ€Π½ΠΎΠ³ΠΎ произвСдСния, разбСрСмся с ΠΎΡ€ΠΈΠ΅Π½Ρ‚Π°Ρ†ΠΈΠ΅ΠΉ упорядочСнной Ρ‚Ρ€ΠΎΠΉΠΊΠΈ Π²Π΅ΠΊΡ‚ΠΎΡ€ΠΎΠ² β†’a, β†’b, β†’c Π² Ρ‚Ρ€Π΅Ρ…ΠΌΠ΅Ρ€Π½ΠΎΠΌ пространствС.

ΠžΡ‚Π»ΠΎΠΆΠΈΠΌ Π²Π΅ΠΊΡ‚ΠΎΡ€Ρ‹ β†’a, β†’b, β†’c ΠΎΡ‚ ΠΎΠ΄Π½ΠΎΠΉ Ρ‚ΠΎΡ‡ΠΊΠΈ. Π’ зависимости ΠΎΡ‚ направлСния Π²Π΅ΠΊΡ‚ΠΎΡ€Π° β†’c Ρ‚Ρ€ΠΎΠΉΠΊΠ° β†’a, β†’b, β†’c ΠΌΠΎΠΆΠ΅Ρ‚ Π±Ρ‹Ρ‚ΡŒ ΠΏΡ€Π°Π²ΠΎΠΉ ΠΈΠ»ΠΈ Π»Π΅Π²ΠΎΠΉ.

ΠŸΠΎΡΠΌΠΎΡ‚Ρ€ΠΈΠΌ с ΠΊΠΎΠ½Ρ†Π° Π²Π΅ΠΊΡ‚ΠΎΡ€Π° β†’c Π½Π° Ρ‚ΠΎ, ΠΊΠ°ΠΊ происходит ΠΊΡ€Π°Ρ‚Ρ‡Π°ΠΉΡˆΠΈΠΉ ΠΏΠΎΠ²ΠΎΡ€ΠΎΡ‚ ΠΎΡ‚ Π²Π΅ΠΊΡ‚ΠΎΡ€Π° β†’a ΠΊ β†’b. Если ΠΊΡ€Π°Ρ‚Ρ‡Π°ΠΉΡˆΠΈΠΉ ΠΏΠΎΠ²ΠΎΡ€ΠΎΡ‚ происходит ΠΏΡ€ΠΎΡ‚ΠΈΠ² часовой стрСлки, Ρ‚ΠΎ Ρ‚Ρ€ΠΎΠΉΠΊΠ° Π²Π΅ΠΊΡ‚ΠΎΡ€ΠΎΠ² β†’a, β†’b, β†’c называСтся ΠΏΡ€Π°Π²ΠΎΠΉ, ΠΏΠΎ часовой стрСлкС β€” Π»Π΅Π²ΠΎΠΉ.



Π’Π΅ΠΏΠ΅Ρ€ΡŒ возьмСм Π΄Π²Π° Π½Π΅ΠΊΠΎΠ»Π»ΠΈΠ½Π΅Π°Ρ€Π½Ρ‹Ρ… Π²Π΅ΠΊΡ‚ΠΎΡ€Π° β†’a ΠΈ β†’b. ΠžΡ‚Π»ΠΎΠΆΠΈΠΌ ΠΎΡ‚ Ρ‚ΠΎΡ‡ΠΊΠΈ А Π²Π΅ΠΊΡ‚ΠΎΡ€Ρ‹ β†’AB = β†’a ΠΈ β†’AC = β†’b. ΠŸΠΎΡΡ‚Ρ€ΠΎΠΈΠΌ Π½Π΅ΠΊΠΎΡ‚ΠΎΡ€Ρ‹ΠΉ Π²Π΅ΠΊΡ‚ΠΎΡ€ β†’AD = β†’c, пСрпСндикулярный ΠΎΠ΄Π½ΠΎΠ²Ρ€Π΅ΠΌΠ΅Π½Π½ΠΎ ΠΈ β†’AB ΠΈ β†’AC.

ΠžΡ‡Π΅Π²ΠΈΠ΄Π½ΠΎ, Ρ‡Ρ‚ΠΎ ΠΏΡ€ΠΈ построСнии Π²Π΅ΠΊΡ‚ΠΎΡ€Π° β†’AD = β†’c ΠΌΡ‹ ΠΌΠΎΠΆΠ΅ΠΌ ΠΏΠΎΡΡ‚ΡƒΠΏΠΈΡ‚ΡŒ ΠΏΠΎ-Ρ€Π°Π·Π½ΠΎΠΌΡƒ, Ссли Π·Π°Π΄Π°Π΄ΠΈΠΌ Π΅ΠΌΡƒ Π»ΠΈΠ±ΠΎ ΠΎΠ΄Π½ΠΎ Π½Π°ΠΏΡ€Π°Π²Π»Π΅Π½ΠΈΠ΅, Π»ΠΈΠ±ΠΎ ΠΏΡ€ΠΎΡ‚ΠΈΠ²ΠΎΠΏΠΎΠ»ΠΎΠΆΠ½ΠΎΠ΅.


Π’ зависимости ΠΎΡ‚ направлСния Π²Π΅ΠΊΡ‚ΠΎΡ€Π° β†’AD = β†’c упорядочСнная Ρ‚Ρ€ΠΎΠΉΠΊΠ° Π²Π΅ΠΊΡ‚ΠΎΡ€ΠΎΠ² β†’a, β†’b, β†’c ΠΌΠΎΠΆΠ΅Ρ‚ Π±Ρ‹Ρ‚ΡŒ ΠΏΡ€Π°Π²ΠΎΠΉ ΠΈΠ»ΠΈ Π»Π΅Π²ΠΎΠΉ.

И сСйчас ΠΌΡ‹ подошли ΠΊ ΠΎΠΏΡ€Π΅Π΄Π΅Π»Π΅Π½ΠΈΡŽ Π²Π΅ΠΊΡ‚ΠΎΡ€Π½ΠΎΠ³ΠΎ произвСдСния. Оно даСтся для Π΄Π²ΡƒΡ… Π²Π΅ΠΊΡ‚ΠΎΡ€ΠΎΠ², ΠΊΠΎΡ‚ΠΎΡ€Ρ‹Π΅ Π·Π°Π΄Π°Π½Ρ‹ Π² ΠΏΡ€ΡΠΌΠΎΡƒΠ³ΠΎΠ»ΡŒΠ½ΠΎΠΉ систСмС ΠΊΠΎΠΎΡ€Π΄ΠΈΠ½Π°Ρ‚ Ρ‚Ρ€Π΅Ρ…ΠΌΠ΅Ρ€Π½ΠΎΠ³ΠΎ пространства.

Π’Π΅ΠΊΡ‚ΠΎΡ€Π½Ρ‹ΠΌ ΠΏΡ€ΠΎΠΈΠ·Π²Π΅Π΄Π΅Π½ΠΈΠ΅ΠΌ Π΄Π²ΡƒΡ… Π²Π΅ΠΊΡ‚ΠΎΡ€ΠΎΠ² β†’a ΠΈ β†’b, ΠΊΠΎΡ‚ΠΎΡ€Ρ‹Π΅ Π·Π°Π΄Π°Π½Ρ‹ Π² ΠΏΡ€ΡΠΌΠΎΡƒΠ³ΠΎΠ»ΡŒΠ½ΠΎΠΉ систСмС ΠΊΠΎΠΎΡ€Π΄ΠΈΠ½Π°Ρ‚ Ρ‚Ρ€Π΅Ρ…ΠΌΠ΅Ρ€Π½ΠΎΠ³ΠΎ пространства, называСтся Ρ‚Π°ΠΊΠΎΠΉ Π²Π΅ΠΊΡ‚ΠΎΡ€ β†’c, Ρ‡Ρ‚ΠΎ:

  • ΠΎΠ½ являСтся Π½ΡƒΠ»Π΅Π²Ρ‹ΠΌ, Ссли Π²Π΅ΠΊΡ‚ΠΎΡ€Ρ‹ β†’a ΠΈ β†’b ΠΊΠΎΠ»Π»ΠΈΠ½Π΅Π°Ρ€Π½Ρ‹;
  • ΠΎΠ½ пСрпСндикулярСн ΠΈ Π²Π΅ΠΊΡ‚ΠΎΡ€Ρƒ β†’a ΠΈ Π²Π΅ΠΊΡ‚ΠΎΡ€Ρƒ β†’b;
  • Π΄Π»ΠΈΠ½Π° Π²Π΅ΠΊΡ‚ΠΎΡ€Π½ΠΎΠ³ΠΎ произвСдСния Ρ€Π°Π²Π½Π° ΠΏΡ€ΠΎΠΈΠ·Π²Π΅Π΄Π΅Π½ΠΈΡŽ Π΄Π»ΠΈΠ½ Π²Π΅ΠΊΡ‚ΠΎΡ€ΠΎΠ² β†’a ΠΈ β†’b Π½Π° синус ΡƒΠ³Π»Π° ΠΌΠ΅ΠΆΠ΄Ρƒ Π½ΠΈΠΌΠΈ
  • Ρ‚Ρ€ΠΎΠΉΠΊΠ° Π²Π΅ΠΊΡ‚ΠΎΡ€ΠΎΠ² β†’a, β†’b, β†’c ΠΎΡ€ΠΈΠ΅Π½Ρ‚ΠΈΡ€ΠΎΠ²Π°Π½Π° Ρ‚Π°ΠΊ ΠΆΠ΅, ΠΊΠ°ΠΊ ΠΈ заданная систСма ΠΊΠΎΠΎΡ€Π΄ΠΈΠ½Π°Ρ‚.

Π’Π΅ΠΊΡ‚ΠΎΡ€Π½Ρ‹ΠΌ ΠΏΡ€ΠΎΠΈΠ·Π²Π΅Π΄Π΅Π½ΠΈΠ΅ΠΌ Π²Π΅ΠΊΡ‚ΠΎΡ€Π° β†’a Π½Π° Π²Π΅ΠΊΡ‚ΠΎΡ€ β†’b называСтся Π²Π΅ΠΊΡ‚ΠΎΡ€ β†’c, Π΄Π»ΠΈΠ½Π° ΠΊΠΎΡ‚ΠΎΡ€ΠΎΠ³ΠΎ числСнно Ρ€Π°Π²Π½Π° ΠΏΠ»ΠΎΡ‰Π°Π΄ΠΈ ΠΏΠ°Ρ€Π°Π»Π»Π΅Π»ΠΎΠ³Ρ€Π°ΠΌΠΌΠ° построСнного Π½Π° Π²Π΅ΠΊΡ‚ΠΎΡ€Π°Ρ… β†’a ΠΈ β†’b, пСрпСндикулярный ΠΊ плоскости этих Π²Π΅ΠΊΡ‚ΠΎΡ€ΠΎΠ² ΠΈ Π½Π°ΠΏΡ€Π°Π²Π»Π΅Π½Π½Ρ‹ΠΉ Ρ‚Π°ΠΊ, Ρ‡Ρ‚ΠΎΠ±Ρ‹ наимСньшСС Π²Ρ€Π°Ρ‰Π΅Π½ΠΈΠ΅ ΠΎΡ‚ β†’a ΠΊ β†’b Π²ΠΎΠΊΡ€ΡƒΠ³ Π²Π΅ΠΊΡ‚ΠΎΡ€Π° c ΠΎΡΡƒΡ‰Π΅ΡΡ‚Π²Π»ΡΠ»ΠΎΡΡŒ ΠΏΡ€ΠΎΡ‚ΠΈΠ² часовой стрСлки, Ссли ΡΠΌΠΎΡ‚Ρ€Π΅Ρ‚ΡŒ с ΠΊΠΎΠ½Ρ†Π° Π²Π΅ΠΊΡ‚ΠΎΡ€Π° β†’c.


Π’Π΅ΠΊΡ‚ΠΎΡ€Π½ΠΎΠ΅ ΠΏΡ€ΠΎΠΈΠ·Π²Π΅Π΄Π΅Π½ΠΈΠ΅ Π΄Π²ΡƒΡ… Π²Π΅ΠΊΡ‚ΠΎΡ€ΠΎΠ² a = {ax; ay; az} ΠΈ b = {bx; by; bz} Π² Π΄Π΅ΠΊΠ°Ρ€Ρ‚ΠΎΠ²ΠΎΠΉ систСмС ΠΊΠΎΠΎΡ€Π΄ΠΈΠ½Π°Ρ‚ β€” это Π²Π΅ΠΊΡ‚ΠΎΡ€, Π·Π½Π°Ρ‡Π΅Π½ΠΈΠ΅ ΠΊΠΎΡ‚ΠΎΡ€ΠΎΠ³ΠΎ ΠΌΠΎΠΆΠ½ΠΎ Π²Ρ‹Ρ‡ΠΈΡΠ»ΠΈΡ‚ΡŒ, ΠΈΡΠΏΠΎΠ»ΡŒΠ·ΡƒΡ Ρ„ΠΎΡ€ΠΌΡƒΠ»Ρ‹ вычислСния Π²Π΅ΠΊΡ‚ΠΎΡ€Π½ΠΎΠ³ΠΎ произвСдСния Π²Π΅ΠΊΡ‚ΠΎΡ€ΠΎΠ²:



Π’Π΅ΠΊΡ‚ΠΎΡ€Π½ΠΎΠ΅ ΠΏΡ€ΠΎΠΈΠ·Π²Π΅Π΄Π΅Π½ΠΈΠ΅ Π²Π΅ΠΊΡ‚ΠΎΡ€ΠΎΠ² β†’a ΠΈ β†’b обозначаСтся ΠΊΠ°ΠΊ [β†’a β€’ β†’b].

Π”Ρ€ΡƒΠ³ΠΎΠ΅ ΠΎΠΏΡ€Π΅Π΄Π΅Π»Π΅Π½ΠΈΠ΅ связано с ΠΏΡ€Π°Π²ΠΎΠΉ Ρ€ΡƒΠΊΠΎΠΉ Ρ‡Π΅Π»ΠΎΠ²Π΅ΠΊΠ°, ΠΎΡ‚ΠΊΡƒΠ΄Π° ΠΈ Π΅ΡΡ‚ΡŒ Π½Π°Π·Π²Π°Π½ΠΈΠ΅. На рисункС Ρ‚Ρ€ΠΎΠΉΠΊΠ° Π²Π΅ΠΊΡ‚ΠΎΡ€ΠΎΠ² β†’a, β†’b, [β†’a β€’ β†’b] являСтся ΠΏΡ€Π°Π²ΠΎΠΉ

.


Π•Ρ‰Π΅ Π΅ΡΡ‚ΡŒ аналитичСский способ опрСдСлСния ΠΏΡ€Π°Π²ΠΎΠΉ ΠΈ Π»Π΅Π²ΠΎΠΉ Ρ‚Ρ€ΠΎΠΉΠΊΠΈ Π²Π΅ΠΊΡ‚ΠΎΡ€ΠΎΠ² β€” ΠΎΠ½ Ρ‚Ρ€Π΅Π±ΡƒΠ΅Ρ‚ задания Π² рассматриваСмом пространствС ΠΏΡ€Π°Π²ΠΎΠΉ ΠΈΠ»ΠΈ Π»Π΅Π²ΠΎΠΉ систСмы ΠΊΠΎΠΎΡ€Π΄ΠΈΠ½Π°Ρ‚, ΠΏΡ€ΠΈΡ‡Ρ‘ΠΌ Π½Π΅ ΠΎΠ±ΡΠ·Π°Ρ‚Π΅Π»ΡŒΠ½ΠΎ ΠΏΡ€ΡΠΌΠΎΡƒΠ³ΠΎΠ»ΡŒΠ½ΠΎΠΉ ΠΈ ΠΎΡ€Ρ‚ΠΎΠ½ΠΎΡ€ΠΌΠΈΡ€ΠΎΠ²Π°Π½Π½ΠΎΠΉ.

НуТно ΡΠΎΡΡ‚Π°Π²ΠΈΡ‚ΡŒ ΠΌΠ°Ρ‚Ρ€ΠΈΡ†Ρƒ, ΠΏΠ΅Ρ€Π²ΠΎΠΉ строкой ΠΊΠΎΡ‚ΠΎΡ€ΠΎΠΉ Π±ΡƒΠ΄ΡƒΡ‚ ΠΊΠΎΠΎΡ€Π΄ΠΈΠ½Π°Ρ‚Ρ‹ Π²Π΅ΠΊΡ‚ΠΎΡ€Π° β†’a, Π²Ρ‚ΠΎΡ€ΠΎΠΉ β€” Π²Π΅ΠΊΡ‚ΠΎΡ€Π° β†’b, Ρ‚Ρ€Π΅Ρ‚ΡŒΠ΅ΠΉ β€” Π²Π΅ΠΊΡ‚ΠΎΡ€Π° β†’c. Π—Π°Ρ‚Π΅ΠΌ, Π² зависимости ΠΎΡ‚ Π·Π½Π°ΠΊΠ° опрСдСлитСля этой ΠΌΠ°Ρ‚Ρ€ΠΈΡ†Ρ‹, ΠΌΠΎΠΆΠ½ΠΎ ΡΠ΄Π΅Π»Π°Ρ‚ΡŒ ΡΠ»Π΅Π΄ΡƒΡŽΡ‰ΠΈΠ΅ Π²Ρ‹Π²ΠΎΠ΄Ρ‹:

  • Если ΠΎΠΏΡ€Π΅Π΄Π΅Π»ΠΈΡ‚Π΅Π»ΡŒ ΠΏΠΎΠ»ΠΎΠΆΠΈΡ‚Π΅Π»Π΅Π½, Ρ‚ΠΎ Ρ‚Ρ€ΠΎΠΉΠΊΠ° Π²Π΅ΠΊΡ‚ΠΎΡ€ΠΎΠ² ΠΈΠΌΠ΅Π΅Ρ‚ Ρ‚Ρƒ ΠΆΠ΅ ΠΎΡ€ΠΈΠ΅Π½Ρ‚Π°Ρ†ΠΈΡŽ, Ρ‡Ρ‚ΠΎ ΠΈ систСма ΠΊΠΎΠΎΡ€Π΄ΠΈΠ½Π°Ρ‚.
  • Если ΠΎΠΏΡ€Π΅Π΄Π΅Π»ΠΈΡ‚Π΅Π»ΡŒ ΠΎΡ‚Ρ€ΠΈΡ†Π°Ρ‚Π΅Π»Π΅Π½, Ρ‚ΠΎ Ρ‚Ρ€ΠΎΠΉΠΊΠ° Π²Π΅ΠΊΡ‚ΠΎΡ€ΠΎΠ² ΠΈΠΌΠ΅Π΅Ρ‚ ΠΎΡ€ΠΈΠ΅Π½Ρ‚Π°Ρ†ΠΈΡŽ, ΠΏΡ€ΠΎΡ‚ΠΈΠ²ΠΎΠΏΠΎΠ»ΠΎΠΆΠ½ΡƒΡŽ ΠΎΡ€ΠΈΠ΅Π½Ρ‚Π°Ρ†ΠΈΠΈ систСмы ΠΊΠΎΠΎΡ€Π΄ΠΈΠ½Π°Ρ‚.
  • Если ΠΎΠΏΡ€Π΅Π΄Π΅Π»ΠΈΡ‚Π΅Π»ΡŒ Ρ€Π°Π²Π΅Π½ Π½ΡƒΠ»ΡŽ, Ρ‚ΠΎ Π²Π΅ΠΊΡ‚ΠΎΡ€Ρ‹ ΠΊΠΎΠΌΠΏΠ»Π°Π½Π°Ρ€Π½Ρ‹ (Π»ΠΈΠ½Π΅ΠΉΠ½ΠΎ зависимы).

ΠšΠΎΠΎΡ€Π΄ΠΈΠ½Π°Ρ‚Ρ‹ Π²Π΅ΠΊΡ‚ΠΎΡ€Π½ΠΎΠ³ΠΎ произвСдСния

Рассмотрим Π²Π΅ΠΊΡ‚ΠΎΡ€Π½ΠΎΠ΅ ΠΏΡ€ΠΎΠΈΠ·Π²Π΅Π΄Π΅Π½ΠΈΠ΅ Π²Π΅ΠΊΡ‚ΠΎΡ€ΠΎΠ² Π² ΠΊΠΎΠΎΡ€Π΄ΠΈΠ½Π°Ρ‚Π°Ρ….

Π‘Ρ„ΠΎΡ€ΠΌΡƒΠ»ΠΈΡ€ΡƒΠ΅ΠΌ Π²Ρ‚ΠΎΡ€ΠΎΠ΅ ΠΎΠΏΡ€Π΅Π΄Π΅Π»Π΅Π½ΠΈΠ΅ Π²Π΅ΠΊΡ‚ΠΎΡ€Π½ΠΎΠ³ΠΎ произвСдСния, ΠΊΠΎΡ‚ΠΎΡ€ΠΎΠ΅ позволяСт Π½Π°Ρ…ΠΎΠ΄ΠΈΡ‚ΡŒ Π΅Π³ΠΎ ΠΊΠΎΠΎΡ€Π΄ΠΈΠ½Π°Ρ‚Ρ‹ ΠΏΠΎ ΠΊΠΎΠΎΡ€Π΄ΠΈΠ½Π°Ρ‚Π°ΠΌ Π·Π°Π΄Π°Π½Π½Ρ‹Ρ… Π²Π΅ΠΊΡ‚ΠΎΡ€ΠΎΠ².

Π’ ΠΏΡ€ΡΠΌΠΎΡƒΠ³ΠΎΠ»ΡŒΠ½ΠΎΠΉ систСмС ΠΊΠΎΠΎΡ€Π΄ΠΈΠ½Π°Ρ‚ Ρ‚Ρ€Π΅Ρ…ΠΌΠ΅Ρ€Π½ΠΎΠ³ΠΎ пространства Π²Π΅ΠΊΡ‚ΠΎΡ€Π½ΠΎΠ΅ ΠΏΡ€ΠΎΠΈΠ·Π²Π΅Π΄Π΅Π½ΠΈΠ΅ Π΄Π²ΡƒΡ… Π²Π΅ΠΊΡ‚ΠΎΡ€ΠΎΠ² β†’a = (ax, ay, az) ΠΈ β†’b = (bx, by, bz) Π΅ΡΡ‚ΡŒ Π²Π΅ΠΊΡ‚ΠΎΡ€


, Π³Π΄Π΅

β†’i, β†’j, β†’k β€” ΠΊΠΎΠΎΡ€Π΄ΠΈΠ½Π°Ρ‚Π½Ρ‹Π΅ Π²Π΅ΠΊΡ‚ΠΎΡ€Ρ‹.

Π­Ρ‚ΠΎ ΠΎΠΏΡ€Π΅Π΄Π΅Π»Π΅Π½ΠΈΠ΅ ΠΏΠΎΠΊΠ°Π·Ρ‹Π²Π°Π΅Ρ‚ Π½Π°ΠΌ Π²Π΅ΠΊΡ‚ΠΎΡ€Π½ΠΎΠ΅ ΠΏΡ€ΠΎΠΈΠ·Π²Π΅Π΄Π΅Π½ΠΈΠ΅ Π² ΠΊΠΎΠΎΡ€Π΄ΠΈΠ½Π°Ρ‚Π½ΠΎΠΉ Ρ„ΠΎΡ€ΠΌΠ΅.

Π’Π΅ΠΊΡ‚ΠΎΡ€Π½ΠΎΠ΅ ΠΏΡ€ΠΎΠΈΠ·Π²Π΅Π΄Π΅Π½ΠΈΠ΅ ΡƒΠ΄ΠΎΠ±Π½ΠΎ ΠΏΡ€Π΅Π΄ΡΡ‚Π°Π²Π»ΡΡ‚ΡŒ Π² Π²ΠΈΠ΄Π΅ опрСдСлитСля ΠΊΠ²Π°Π΄Ρ€Π°Ρ‚Π½ΠΎΠΉ ΠΌΠ°Ρ‚Ρ€ΠΈΡ†Ρ‹ Ρ‚Ρ€Π΅Ρ‚ΡŒΠ΅Π³ΠΎ порядка, пСрвая строка ΠΊΠΎΡ‚ΠΎΡ€ΠΎΠΉ Π΅ΡΡ‚ΡŒ ΠΎΡ€Ρ‚Ρ‹ β†’i, β†’j, β†’k, Π²ΠΎ Π²Ρ‚ΠΎΡ€ΠΎΠΉ строкС находятся ΠΊΠΎΠΎΡ€Π΄ΠΈΠ½Π°Ρ‚Ρ‹ Π²Π΅ΠΊΡ‚ΠΎΡ€Π° β†’a, Π° Π² Ρ‚Ρ€Π΅Ρ‚ΡŒΠ΅ΠΉ β€” ΠΊΠΎΠΎΡ€Π΄ΠΈΠ½Π°Ρ‚Ρ‹ Π²Π΅ΠΊΡ‚ΠΎΡ€Π° β†’b Π² Π·Π°Π΄Π°Π½Π½ΠΎΠΉ ΠΏΡ€ΡΠΌΠΎΡƒΠ³ΠΎΠ»ΡŒΠ½ΠΎΠΉ систСмС ΠΊΠΎΠΎΡ€Π΄ΠΈΠ½Π°Ρ‚:


Если Ρ€Π°Π·Π»ΠΎΠΆΠΈΠΌ этот ΠΎΠΏΡ€Π΅Π΄Π΅Π»ΠΈΡ‚Π΅Π»ΡŒ ΠΏΠΎ элСмСнтам ΠΏΠ΅Ρ€Π²ΠΎΠΉ строки, Ρ‚ΠΎ ΠΏΠΎΠ»ΡƒΡ‡ΠΈΠΌ равСнство ΠΈΠ· опрСдСлСния Π²Π΅ΠΊΡ‚ΠΎΡ€Π½ΠΎΠ³ΠΎ произвСдСния Π² ΠΊΠΎΠΎΡ€Π΄ΠΈΠ½Π°Ρ‚Π°Ρ…:


Π’Π°ΠΆΠ½ΠΎ ΠΎΡ‚ΠΌΠ΅Ρ‚ΠΈΡ‚ΡŒ, Ρ‡Ρ‚ΠΎ координатная Ρ„ΠΎΡ€ΠΌΠ° Π²Π΅ΠΊΡ‚ΠΎΡ€Π½ΠΎΠ³ΠΎ произвСдСния согласуСтся с ΠΎΠΏΡ€Π΅Π΄Π΅Π»Π΅Π½ΠΈΠ΅ΠΌ,ΠΊΠΎΡ‚ΠΎΡ€ΠΎΠ΅ ΠΌΡ‹ Π΄Π°Π»ΠΈ Π² ΠΏΠ΅Ρ€Π²ΠΎΠΌ ΠΏΡƒΠ½ΠΊΡ‚Π΅ этой ΡΡ‚Π°Ρ‚ΡŒΠΈ. Π‘ΠΎΠ»Π΅Π΅ Ρ‚ΠΎΠ³ΠΎ, эти Π΄Π²Π° опрСдСлСния Π²Π΅ΠΊΡ‚ΠΎΡ€Π½ΠΎΠ³ΠΎ произвСдСния эквивалСнтны.

Бвойства Π²Π΅ΠΊΡ‚ΠΎΡ€Π½ΠΎΠ³ΠΎ произвСдСния

Π’Π΅ΠΊΡ‚ΠΎΡ€Π½ΠΎΠ΅ ΠΏΡ€ΠΎΠΈΠ·Π²Π΅Π΄Π΅Π½ΠΈΠ΅ Π² ΠΊΠΎΠΎΡ€Π΄ΠΈΠ½Π°Ρ‚Π°Ρ… прСдставляСтся Π² Π²ΠΈΠ΄Π΅ опрСдСлитСля ΠΌΠ°Ρ‚Ρ€ΠΈΡ†Ρ‹:


На основании свойств опрСдСлитСля ΠΌΠΎΠΆΠ½ΠΎ Π»Π΅Π³ΠΊΠΎ ΠΎΠ±ΠΎΡΠ½ΠΎΠ²Π°Ρ‚ΡŒ свойства Π²Π΅ΠΊΡ‚ΠΎΡ€Π½ΠΎΠ³ΠΎ произвСдСния Π²Π΅ΠΊΡ‚ΠΎΡ€ΠΎΠ²:


  1. ΠΠ½Ρ‚ΠΈΠΊΠΎΠΌΠΌΡƒΡ‚Π°Ρ‚ΠΈΠ²Π½ΠΎΡΡ‚ΡŒ
  2. Бвойство дистрибутивности

    ΠΈΠ»ΠΈ


  3. Π‘ΠΎΡ‡Π΅Ρ‚Π°Ρ‚Π΅Π»ΡŒΠ½ΠΎΠ΅ свойство

    ΠΈΠ»ΠΈ


    , Π³Π΄Π΅ Ξ» ΠΏΡ€ΠΎΠΈΠ·Π²ΠΎΠ»ΡŒΠ½ΠΎΠ΅ Π΄Π΅ΠΉΡΡ‚Π²ΠΈΡ‚Π΅Π»ΡŒΠ½ΠΎΠ΅ число.

Для большСй ясности Π΄ΠΎΠΊΠ°ΠΆΠ΅ΠΌ свойство антикоммутативности Π²Π΅ΠΊΡ‚ΠΎΡ€Π½ΠΎΠ³ΠΎ произвСдСния.

По ΠΎΠΏΡ€Π΅Π΄Π΅Π»Π΅Π½ΠΈΡŽ


ΠΈ


Нам извСстно, Ρ‡Ρ‚ΠΎ Π·Π½Π°Ρ‡Π΅Π½ΠΈΠ΅ опрСдСлитСля ΠΌΠ°Ρ‚Ρ€ΠΈΡ†Ρ‹ измСняСтся Π½Π° ΠΏΡ€ΠΎΡ‚ΠΈΠ²ΠΎΠΏΠΎΠ»ΠΎΠΆΠ½ΠΎΠ΅, Ссли ΠΏΠ΅Ρ€Π΅ΡΡ‚Π°Π²ΠΈΡ‚ΡŒ мСстами Π΄Π²Π΅ строки, поэтому


Ρ‡Ρ‚ΠΎ Π΄ΠΎΠΊΠ°Π·Ρ‹Π²Π°Π΅Ρ‚ свойство антикоммутативности Π²Π΅ΠΊΡ‚ΠΎΡ€Π½ΠΎΠ³ΠΎ произвСдСния.

Π§Ρ‚ΠΎΠ±Ρ‹ Π½Π°ΠΉΡ‚ΠΈ ΠΌΠΎΠ΄ΡƒΠ»ΡŒ Π²Π΅ΠΊΡ‚ΠΎΡ€Π½ΠΎΠ³ΠΎ произвСдСния Π²Π΅ΠΊΡ‚ΠΎΡ€ΠΎΠ² u ΠΈ v Π½ΡƒΠΆΠ½ΠΎ Π½Π°ΠΉΡ‚ΠΈ ΠΏΠ»ΠΎΡ‰Π°Π΄ΡŒ ΠΏΠ°Ρ€Π°Π»Π»Π΅Π»ΠΎΠ³Ρ€Π°ΠΌΠΌΠ°, ΠΊΠΎΡ‚ΠΎΡ€Ρ‹ΠΉ построСн Π½Π° Π΄Π°Π½Π½Ρ‹Ρ… Π²Π΅ΠΊΡ‚ΠΎΡ€Π°Ρ…: S = | u Γ— v | = | u | * | v | * sinΞΈ, Π³Π΄Π΅ ΞΈ β€” ΡƒΠ³ΠΎΠ» ΠΌΠ΅ΠΆΠ΄Ρƒ Π²Π΅ΠΊΡ‚ΠΎΡ€Π°ΠΌΠΈ.

Π’Π΅ΠΊΡ‚ΠΎΡ€Π½ΠΎΠ΅ ΠΏΡ€ΠΎΠΈΠ·Π²Π΅Π΄Π΅Π½ΠΈΠ΅ Π²Π΅ΠΊΡ‚ΠΎΡ€ΠΎΠ² u ΠΈ v Ρ€Π°Π²Π½ΠΎ Π½ΡƒΠ»Π΅Π²ΠΎΠΌΡƒ Π²Π΅ΠΊΡ‚ΠΎΡ€Ρƒ, Ссли u ΠΈ v ΠΏΠ°Ρ€Π°Π»Π»Π΅Π»ΡŒΠ½Ρ‹ (ΠΊΠΎΠ»Π»ΠΈΠ½Π΅Π°Ρ€Π½Ρ‹): u Γ— v = 0, Ссли u βˆ₯ v (ΞΈ = 0).

Β 

ΠŸΡ€ΠΈΠΌΠ΅Ρ€Ρ‹ Ρ€Π΅ΡˆΠ΅Π½ΠΈΡ Π·Π°Π΄Π°Ρ‡

ΠŸΡ€ΠΈΠΌΠ΅Ρ€ 1

Π°) Найти Π΄Π»ΠΈΠ½Ρƒ Π²Π΅ΠΊΡ‚ΠΎΡ€Π½ΠΎΠ³ΠΎ произвСдСния Π²Π΅ΠΊΡ‚ΠΎΡ€ΠΎΠ² β†’a ΠΈ β†’b, Ссли |β†’a| = 2, |β†’b| = 3, ∠(β†’a, β†’b) = Ο€/3.

Π±) Найти ΠΏΠ»ΠΎΡ‰Π°Π΄ΡŒ ΠΏΠ°Ρ€Π°Π»Π»Π΅Π»ΠΎΠ³Ρ€Π°ΠΌΠΌΠ°, построСнного Π½Π° Π²Π΅ΠΊΡ‚ΠΎΡ€Π°Ρ… β†’a ΠΈ β†’b, Ссли |β†’a| = 2, |β†’b| = 3, ∠(β†’a, β†’b) = Ο€/3.

Как Ρ€Π΅ΡˆΠ°Π΅ΠΌ:

Π°) По ΡƒΡΠ»ΠΎΠ²ΠΈΡŽ трСбуСтся Π½Π°ΠΉΡ‚ΠΈ Π΄Π»ΠΈΠ½Ρƒ Π²Π΅ΠΊΡ‚ΠΎΡ€Π½ΠΎΠ³ΠΎ произвСдСния. ΠŸΠΎΠ΄ΡΡ‚Π°Π²Π»ΡΠ΅ΠΌ Π΄Π°Π½Π½Ρ‹Π΅ Π² Ρ„ΠΎΡ€ΠΌΡƒΠ»Ρƒ:


ΠžΡ‚Π²Π΅Ρ‚:


Π’Π°ΠΊ ΠΊΠ°ΠΊ Π² Π·Π°Π΄Π°Ρ‡Π΅ Ρ€Π΅Ρ‡ΡŒ ΠΈΠ΄Π΅Ρ‚ ΠΎ Π΄Π»ΠΈΠ½Π΅, Ρ‚ΠΎ Π² ΠΎΡ‚Π²Π΅Ρ‚Π΅ ΡƒΠΊΠ°Π·Ρ‹Π²Π°Π΅ΠΌ Ρ€Π°Π·ΠΌΠ΅Ρ€Π½ΠΎΡΡ‚ΡŒ β€” Π΅Π΄ΠΈΠ½ΠΈΡ†Ρ‹.

Π±) По ΡƒΡΠ»ΠΎΠ²ΠΈΡŽ трСбуСтся Π½Π°ΠΉΡ‚ΠΈ ΠΏΠ»ΠΎΡ‰Π°Π΄ΡŒ ΠΏΠ°Ρ€Π°Π»Π»Π΅Π»ΠΎΠ³Ρ€Π°ΠΌΠΌΠ°, ΠΊΠΎΡ‚ΠΎΡ€Ρ‹ΠΉ построСн Π½Π° Π²Π΅ΠΊΡ‚ΠΎΡ€Π°Ρ… β†’a ΠΈ β†’b. ΠŸΠ»ΠΎΡ‰Π°Π΄ΡŒ Ρ‚Π°ΠΊΠΎΠ³ΠΎ ΠΏΠ°Ρ€Π°Π»Π»Π΅Π»ΠΎΠ³Ρ€Π°ΠΌΠΌΠ° числСнно Ρ€Π°Π²Π½Π° Π΄Π»ΠΈΠ½Π΅ Π²Π΅ΠΊΡ‚ΠΎΡ€Π½ΠΎΠ³ΠΎ произвСдСния:


ΠžΡ‚Π²Π΅Ρ‚:


ΠŸΡ€ΠΈΠΌΠ΅Ρ€ 2

Найти |[-3β†’a x 2β†’b]|, Ссли |β†’a| = 1/2, |β†’b| = 1/6, ∠(β†’a, β†’b) = Ο€/2.

Как Ρ€Π΅ΡˆΠ°Π΅ΠΌ:

По ΡƒΡΠ»ΠΎΠ²ΠΈΡŽ снова Π½ΡƒΠΆΠ½ΠΎ Π½Π°ΠΉΡ‚ΠΈ Π΄Π»ΠΈΠ½Ρƒ Π²Π΅ΠΊΡ‚ΠΎΡ€Π½ΠΎΠ³ΠΎ произвСдСния. Π˜ΡΠΏΠΎΠ»ΡŒΠ·ΡƒΠ΅ΠΌ Π½Π°ΡˆΡƒ Ρ„ΠΎΡ€ΠΌΡƒΠ»Ρƒ:


Богласно ассоциативным Π·Π°ΠΊΠΎΠ½Π°ΠΌ, выносим константы Π·Π° ΠΏΠ΅Ρ€Π΅Π΄Π΅Π»Ρ‹ Π²Π΅ΠΊΡ‚ΠΎΡ€Π½ΠΎΠ³ΠΎ произвСдСния.

Выносим константу Π·Π° ΠΏΡ€Π΅Π΄Π΅Π»Ρ‹ модуля, ΠΏΡ€ΠΈ этом ΠΌΠΎΠ΄ΡƒΠ»ΡŒ позволяСт ΡƒΠ±Ρ€Π°Ρ‚ΡŒ Π·Π½Π°ΠΊ минус. Π”Π»ΠΈΠ½Π° ΠΆΠ΅ Π½Π΅ ΠΌΠΎΠΆΠ΅Ρ‚ Π±Ρ‹Ρ‚ΡŒ ΠΎΡ‚Ρ€ΠΈΡ†Π°Ρ‚Π΅Π»ΡŒΠ½ΠΎΠΉ.

ΠžΡ‚Π²Π΅Ρ‚:


ΠŸΡ€ΠΈΠΌΠ΅Ρ€ 3

Π”Π°Π½Ρ‹ Π²Π΅Ρ€ΡˆΠΈΠ½Ρ‹ Ρ‚Ρ€Π΅ΡƒΠ³ΠΎΠ»ΡŒΠ½ΠΈΠΊΠ° A (0, 2, 0), B (-2, 5,0), C (-2, 2, 6). Найти Π΅Π³ΠΎ ΠΏΠ»ΠΎΡ‰Π°Π΄ΡŒ.

Как Ρ€Π΅ΡˆΠ°Π΅ΠΌ:

Π‘Π½Π°Ρ‡Π°Π»Π° Π½Π°ΠΉΠ΄Ρ‘ΠΌ Π²Π΅ΠΊΡ‚ΠΎΡ€Ρ‹:


Π—Π°Ρ‚Π΅ΠΌ Π²Π΅ΠΊΡ‚ΠΎΡ€Π½ΠΎΠ΅ ΠΏΡ€ΠΎΠΈΠ·Π²Π΅Π΄Π΅Π½ΠΈΠ΅:


Вычислим Π΅Π³ΠΎ Π΄Π»ΠΈΠ½Ρƒ:


ΠŸΠΎΠ΄ΡΡ‚Π°Π²ΠΈΠΌ Π΄Π°Π½Π½Ρ‹Π΅ Π² Ρ„ΠΎΡ€ΠΌΡƒΠ»Ρ‹ ΠΏΠ»ΠΎΡ‰Π°Π΄Π΅ΠΉ ΠΏΠ°Ρ€Π°Π»Π»Π΅Π»ΠΎΠ³Ρ€Π°ΠΌΠΌΠ° ΠΈ Ρ‚Ρ€Π΅ΡƒΠ³ΠΎΠ»ΡŒΠ½ΠΈΠΊΠ°:


ΠžΡ‚Π²Π΅Ρ‚:


ГСомСтричСский смысл Π²Π΅ΠΊΡ‚ΠΎΡ€Π½ΠΎΠ³ΠΎ произвСдСния

По ΠΎΠΏΡ€Π΅Π΄Π΅Π»Π΅Π½ΠΈΡŽ Π΄Π»ΠΈΠ½Π° Π²Π΅ΠΊΡ‚ΠΎΡ€Π½ΠΎΠ³ΠΎ произвСдСния Π²Π΅ΠΊΡ‚ΠΎΡ€ΠΎΠ² Ρ€Π°Π²Π½Π°


А ΠΈΠ· курса Π³Π΅ΠΎΠΌΠ΅Ρ‚Ρ€ΠΈΠΈ срСднСй ΡˆΠΊΠΎΠ»Ρ‹ ΠΌΡ‹ Π·Π½Π°Π΅ΠΌ, Ρ‡Ρ‚ΠΎ ΠΏΠ»ΠΎΡ‰Π°Π΄ΡŒ Ρ‚Ρ€Π΅ΡƒΠ³ΠΎΠ»ΡŒΠ½ΠΈΠΊΠ° Ρ€Π°Π²Π½Π° ΠΏΠΎΠ»ΠΎΠ²ΠΈΠ½Π΅ произвСдСния Π΄Π»ΠΈΠ½ Π΄Π²ΡƒΡ… сторон Ρ‚Ρ€Π΅ΡƒΠ³ΠΎΠ»ΡŒΠ½ΠΈΠΊΠ° Π½Π° синус ΡƒΠ³Π»Π° ΠΌΠ΅ΠΆΠ΄Ρƒ Π½ΠΈΠΌΠΈ.

ΠŸΠΎΡΡ‚ΠΎΠΌΡƒ Π΄Π»ΠΈΠ½Π° Π²Π΅ΠΊΡ‚ΠΎΡ€Π½ΠΎΠ³ΠΎ произвСдСния Ρ€Π°Π²Π½Π° ΡƒΠ΄Π²ΠΎΠ΅Π½Π½ΠΎΠΉ ΠΏΠ»ΠΎΡ‰Π°Π΄ΠΈ Ρ‚Ρ€Π΅ΡƒΠ³ΠΎΠ»ΡŒΠ½ΠΈΠΊΠ°, ΠΈΠΌΠ΅ΡŽΡ‰Π΅Π³ΠΎ сторонами Π²Π΅ΠΊΡ‚ΠΎΡ€Ρ‹ β†’a ΠΈ β†’b, Ссли ΠΈΡ… ΠΎΡ‚Π»ΠΎΠΆΠΈΡ‚ΡŒ ΠΎΡ‚ ΠΎΠ΄Π½ΠΎΠΉ Ρ‚ΠΎΡ‡ΠΊΠΈ. ΠŸΡ€ΠΎΡ‰Π΅ говоря, Π΄Π»ΠΈΠ½Π° Π²Π΅ΠΊΡ‚ΠΎΡ€Π½ΠΎΠ³ΠΎ произвСдСния Π²Π΅ΠΊΡ‚ΠΎΡ€ΠΎΠ² β†’a ΠΈ β†’b Ρ€Π°Π²Π½Π° ΠΏΠ»ΠΎΡ‰Π°Π΄ΠΈ ΠΏΠ°Ρ€Π°Π»Π»Π΅Π»ΠΎΠ³Ρ€Π°ΠΌΠΌΠ° со сторонами |β†’a| ΠΈ |β†’b| ΠΈ ΡƒΠ³Π»ΠΎΠΌ ΠΌΠ΅ΠΆΠ΄Ρƒ Π½ΠΈΠΌΠΈ, Ρ€Π°Π²Π½Ρ‹ΠΌ (β†’a, β†’b). Π’ этом состоит гСомСтричСский смысл Π²Π΅ΠΊΡ‚ΠΎΡ€Π½ΠΎΠ³ΠΎ произвСдСния.


ЀизичСский смысл Π²Π΅ΠΊΡ‚ΠΎΡ€Π½ΠΎΠ³ΠΎ произвСдСния

Π’ ΠΌΠ΅Ρ…Π°Π½ΠΈΠΊΠ΅ β€” ΠΎΠ΄Π½ΠΎΠΌ ΠΈΠ· Ρ€Π°Π·Π΄Π΅Π»ΠΎΠ² Ρ„ΠΈΠ·ΠΈΠΊΠΈ β€” благодаря Π²Π΅ΠΊΡ‚ΠΎΡ€Π½ΠΎΠΌΡƒ ΠΏΡ€ΠΎΠΈΠ·Π²Π΅Π΄Π΅Π½ΠΈΡŽ ΠΌΠΎΠΆΠ½ΠΎ ΠΎΠΏΡ€Π΅Π΄Π΅Π»ΠΈΡ‚ΡŒ ΠΌΠΎΠΌΠ΅Π½Ρ‚ силы ΠΎΡ‚Π½ΠΎΡΠΈΡ‚Π΅Π»ΡŒΠ½ΠΎ Ρ‚ΠΎΡ‡ΠΊΠΈ пространства. ΠŸΠΎΡΡ‚ΠΎΠΌΡƒ сформулируСм Π΅Ρ‰Π΅ ΠΎΠ΄Π½ΠΎ Π²Π°ΠΆΠ½ΠΎΠ΅ ΠΎΠΏΡ€Π΅Π΄Π΅Π»Π΅Π½ΠΈΠ΅.

Под ΠΌΠΎΠΌΠ΅Π½Ρ‚ΠΎΠΌ силы β†’F, ΠΏΡ€ΠΈΠ»ΠΎΠΆΠ΅Π½Π½ΠΎΠΉ ΠΊ Ρ‚ΠΎΡ‡ΠΊΠ΅ B, ΠΎΡ‚Π½ΠΎΡΠΈΡ‚Π΅Π»ΡŒΠ½ΠΎ Ρ‚ΠΎΡ‡ΠΊΠΈ A понимаСтся ΡΠ»Π΅Π΄ΡƒΡŽΡ‰Π΅Π΅ Π²Π΅ΠΊΡ‚ΠΎΡ€Π½ΠΎΠ΅ ΠΏΡ€ΠΎΠΈΠ·Π²Π΅Π΄Π΅Π½ΠΈΠ΅ [β†’A B Γ— β†’F].


Π’Π΅ΠΊΡ‚ΠΎΡ€ Π»ΠΈΠ½Π΅ΠΉΠ½ΠΎΠΉ скорости β†’V Ρ‚ΠΎΡ‡ΠΊΠΈ M колСса Ρ€Π°Π²Π΅Π½ Π²Π΅ΠΊΡ‚ΠΎΡ€Π½ΠΎΠΌΡƒ ΠΏΡ€ΠΎΠΈΠ·Π²Π΅Π΄Π΅Π½ΠΈΡŽ Π²Π΅ΠΊΡ‚ΠΎΡ€Π° ΡƒΠ³Π»ΠΎΠ²ΠΎΠΉ скорости β†’W ΠΈ радиус-Π²Π΅ΠΊΡ‚ΠΎΡ€Π° Ρ‚ΠΎΡ‡ΠΊΠΈ колСса, Ρ‚ΠΎ Π΅ΡΡ‚ΡŒ β†’V = β†’W`β†’rM.


опрСдСлСния, свойства, Ρ„ΠΎΡ€ΠΌΡƒΠ»Ρ‹, ΠΏΡ€ΠΈΠΌΠ΅Ρ€Ρ‹ ΠΈ Ρ€Π΅ΡˆΠ΅Π½ΠΈΡ

ΠžΠΏΡ€Π΅Π΄Π΅Π»Π΅Π½ΠΈΠ΅ Π²Π΅ΠΊΡ‚ΠΎΡ€Π½ΠΎΠ³ΠΎ произвСдСния

ΠŸΠ΅Ρ€Π΅Π΄ Ρ‚Π΅ΠΌ, ΠΊΠ°ΠΊ Π΄Π°Ρ‚ΡŒ понятиС Π²Π΅ΠΊΡ‚ΠΎΡ€Π½ΠΎΠ³ΠΎ произвСдСния, обратимся ΠΊ вопросу ΠΎ ΠΎΡ€ΠΈΠ΅Π½Ρ‚Π°Ρ†ΠΈΠΈ упорядочСнной Ρ‚Ρ€ΠΎΠΉΠΊΠΈ Π²Π΅ΠΊΡ‚ΠΎΡ€ΠΎΠ² aβ†’,Β bβ†’,Β cβ†’Β Π² Ρ‚Ρ€Π΅Ρ…ΠΌΠ΅Ρ€Π½ΠΎΠΌ пространствС.

ΠžΡ‚Π»ΠΎΠΆΠΈΠΌ для Π½Π°Ρ‡Π°Π»Π° Π²Π΅ΠΊΡ‚ΠΎΡ€Ρ‹ aβ†’,Β bβ†’,Β cβ†’ ΠΎΡ‚ ΠΎΠ΄Π½ΠΎΠΉ Ρ‚ΠΎΡ‡ΠΊΠΈ. ΠžΡ€ΠΈΠ΅Π½Ρ‚Π°Ρ†ΠΈΡ Ρ‚Ρ€ΠΎΠΉΠΊΠΈ aβ†’,Β bβ†’,Β cβ†’ Π±Ρ‹Π²Π°Π΅Ρ‚ ΠΏΡ€Π°Π²ΠΎΠΉ ΠΈΠ»ΠΈ Π»Π΅Π²ΠΎΠΉ, Π² зависимости ΠΎΡ‚ направлСния самого Π²Π΅ΠΊΡ‚ΠΎΡ€Π° cβ†’. ΠžΡ‚ Ρ‚ΠΎΠ³ΠΎ, Π² ΠΊΠ°ΠΊΡƒΡŽ сторону осущСствляСтся ΠΊΡ€Π°Ρ‚Ρ‡Π°ΠΉΡˆΠΈΠΉ ΠΏΠΎΠ²ΠΎΡ€ΠΎΡ‚ ΠΎΡ‚ Π²Π΅ΠΊΡ‚ΠΎΡ€Π° aβ†’ ΠΊ b→ с ΠΊΠΎΠ½Ρ†Π° Π²Π΅ΠΊΡ‚ΠΎΡ€Π° cβ†’, Π±ΡƒΠ΄Π΅Ρ‚ ΠΎΠΏΡ€Π΅Π΄Π΅Π»Π΅Π½ Π²ΠΈΠ΄ Ρ‚Ρ€ΠΎΠΉΠΊΠΈaβ†’,Β bβ†’,Β cβ†’.

Если ΠΊΡ€Π°Ρ‚Ρ‡Π°ΠΉΡˆΠΈΠΉ ΠΏΠΎΠ²ΠΎΡ€ΠΎΡ‚ осущСствляСтся ΠΏΡ€ΠΎΡ‚ΠΈΠ² часовой стрСлки, Ρ‚ΠΎ Ρ‚Ρ€ΠΎΠΉΠΊΠ° Π²Π΅ΠΊΡ‚ΠΎΡ€ΠΎΠ² aβ†’,Β bβ†’,Β cβ†’ называСтся ΠΏΡ€Π°Π²ΠΎΠΉ, Ссли ΠΏΠΎ часовой стрСлкС – Π»Π΅Π²ΠΎΠΉ.

Π”Π°Π»Π΅Π΅ возьмСм Π΄Π²Π° Π½Π΅ ΠΊΠΎΠ»Π»ΠΈΠ½Π΅Π°Ρ€Π½Ρ‹Ρ… Π²Π΅ΠΊΡ‚ΠΎΡ€Π° aβ†’ ΠΈ bβ†’. ΠžΡ‚Π»ΠΎΠΆΠΈΠΌ Π·Π°Ρ‚Π΅ΠΌ ΠΎΡ‚ Ρ‚ΠΎΡ‡ΠΊΠΈ A Π²Π΅ΠΊΡ‚ΠΎΡ€Ρ‹ ABβ†’=aβ†’ ΠΈ ACβ†’=bβ†’. ΠŸΠΎΡΡ‚Ρ€ΠΎΠΈΠΌ Π²Π΅ΠΊΡ‚ΠΎΡ€ ADβ†’=cβ†’, ΠΊΠΎΡ‚ΠΎΡ€Ρ‹ΠΉ ΠΎΠ΄Π½ΠΎΠ²Ρ€Π΅ΠΌΠ΅Π½Π½ΠΎ пСрпСндикулярный ΠΎΠ΄Π½ΠΎΠ²Ρ€Π΅ΠΌΠ΅Π½Π½ΠΎ ΠΈΒ ABβ†’ ΠΈ ACβ†’. Π’Π°ΠΊΠΈΠΌ ΠΎΠ±Ρ€Π°Π·ΠΎΠΌ, ΠΏΡ€ΠΈ построСнии самого Π²Π΅ΠΊΡ‚ΠΎΡ€Π° ADβ†’=cβ†’ ΠΌΡ‹ ΠΌΠΎΠΆΠ΅ΠΌ ΠΏΠΎΡΡ‚ΡƒΠΏΠΈΡ‚ΡŒ двояко, Π·Π°Π΄Π°Π² Π΅ΠΌΡƒ Π»ΠΈΠ±ΠΎ ΠΎΠ΄Π½ΠΎ Π½Π°ΠΏΡ€Π°Π²Π»Π΅Π½ΠΈΠ΅, Π»ΠΈΠ±ΠΎ ΠΏΡ€ΠΎΡ‚ΠΈΠ²ΠΎΠΏΠΎΠ»ΠΎΠΆΠ½ΠΎΠ΅ (смотритС ΠΈΠ»Π»ΡŽΡΡ‚Ρ€Π°Ρ†ΠΈΡŽ).

УпорядочСнная Ρ‚Ρ€ΠΎΠΉΠΊΠ° Π²Π΅ΠΊΡ‚ΠΎΡ€ΠΎΠ²Β aβ†’,Β bβ†’,Β cβ†’ ΠΌΠΎΠΆΠ΅Ρ‚ Π±Ρ‹Ρ‚ΡŒ, ΠΊΠ°ΠΊ ΠΌΡ‹ выяснили ΠΏΡ€Π°Π²ΠΎΠΉ ΠΈΠ»ΠΈ Π»Π΅Π²ΠΎΠΉ Π² зависимости ΠΎΡ‚ направлСния Π²Π΅ΠΊΡ‚ΠΎΡ€Π°.

Из Π²Ρ‹ΡˆΠ΅ΡΠΊΠ°Π·Π°Π½Π½ΠΎΠ³ΠΎ ΠΌΠΎΠΆΠ΅ΠΌ ввСсти ΠΎΠΏΡ€Π΅Π΄Π΅Π»Π΅Π½ΠΈΠ΅ Π²Π΅ΠΊΡ‚ΠΎΡ€Π½ΠΎΠ³ΠΎ произвСдСния. Π”Π°Π½Π½ΠΎΠ΅ ΠΎΠΏΡ€Π΅Π΄Π΅Π»Π΅Π½ΠΈΠ΅ даСтся для Π΄Π²ΡƒΡ… Π²Π΅ΠΊΡ‚ΠΎΡ€ΠΎΠ², ΠΎΠΏΡ€Π΅Π΄Π΅Π»Π΅Π½Π½Ρ‹Ρ… Π² ΠΏΡ€ΡΠΌΠΎΡƒΠ³ΠΎΠ»ΡŒΠ½ΠΎΠΉ систСмС ΠΊΠΎΠΎΡ€Π΄ΠΈΠ½Π°Ρ‚ Ρ‚Ρ€Π΅Ρ…ΠΌΠ΅Ρ€Π½ΠΎΠ³ΠΎ пространства.

ΠžΠΏΡ€Π΅Π΄Π΅Π»Π΅Π½ΠΈΠ΅ 1

Π’Π΅ΠΊΡ‚ΠΎΡ€Π½Ρ‹ΠΌ ΠΏΡ€ΠΎΠΈΠ·Π²Π΅Π΄Π΅Π½ΠΈΠ΅ΠΌ Π΄Π²ΡƒΡ… Π²Π΅ΠΊΡ‚ΠΎΡ€ΠΎΠ² aβ†’ ΠΈ bβ†’ Π±ΡƒΠ΄Π΅ΠΌ Π½Π°Π·Ρ‹Π²Π°Ρ‚ΡŒ Ρ‚Π°ΠΊΠΎΠΉ Π²Π΅ΠΊΡ‚ΠΎΡ€ Π·Π°Π΄Π°Π½Π½Ρ‹ΠΉ Π² ΠΏΡ€ΡΠΌΠΎΡƒΠ³ΠΎΠ»ΡŒΠ½ΠΎΠΉ систСмС ΠΊΠΎΠΎΡ€Π΄ΠΈΠ½Π°Ρ‚ Ρ‚Ρ€Π΅Ρ…ΠΌΠ΅Ρ€Π½ΠΎΠ³ΠΎ пространства Ρ‚Π°ΠΊΠΎΠΉ, Ρ‡Ρ‚ΠΎ:

  • Ссли Π²Π΅ΠΊΡ‚ΠΎΡ€Ρ‹ aβ†’ ΠΈ bβ†’ ΠΊΠΎΠ»Π»ΠΈΠ½Π΅Π°Ρ€Π½Ρ‹, ΠΎΠ½ Π±ΡƒΠ΄Π΅Ρ‚ Π½ΡƒΠ»Π΅Π²Ρ‹ΠΌ;
  • ΠΎΠ½ Π±ΡƒΠ΄Π΅Ρ‚ пСрпСндикулярСн ΠΈ Π²Π΅ΠΊΡ‚ΠΎΡ€Ρƒ a→​​​​ и Π²Π΅ΠΊΡ‚ΠΎΡ€Ρƒ bβ†’Β Ρ‚.Π΅. ∠aβ†’cβ†’=∠bβ†’cβ†’=Ο€2 ;
  • Π΅Π³ΠΎ Π΄Π»ΠΈΠ½Π° опрСдСляСтся ΠΏΠΎ Ρ„ΠΎΡ€ΠΌΡƒΠ»Π΅: cβ†’=aβ†’Β·bβ†’Β·sin∠aβ†’,bβ†’;
  • Ρ‚Ρ€ΠΎΠΉΠΊΠ° Π²Π΅ΠΊΡ‚ΠΎΡ€ΠΎΠ² aβ†’,Β bβ†’,Β cβ†’ ΠΈΠΌΠ΅Π΅Ρ‚ Ρ‚Π°ΠΊΡƒΡŽ ΠΆΠ΅ ΠΎΡ€ΠΈΠ΅Π½Ρ‚Π°Ρ†ΠΈΡŽ, Ρ‡Ρ‚ΠΎ ΠΈ заданная систСма ΠΊΠΎΠΎΡ€Π΄ΠΈΠ½Π°Ρ‚.

Π’Π΅ΠΊΡ‚ΠΎΡ€Π½ΠΎΠ΅ ΠΏΡ€ΠΎΠΈΠ·Π²Π΅Π΄Π΅Π½ΠΈΠ΅ Π²Π΅ΠΊΡ‚ΠΎΡ€ΠΎΠ² aβ†’ ΠΈ bβ†’ ΠΈΠΌΠ΅Π΅Ρ‚ слСдущСС ΠΎΠ±ΠΎΠ·Π½Π°Ρ‡Π΅Π½ΠΈΠ΅: aβ†’Γ—bβ†’.

ΠšΠΎΠΎΡ€Π΄ΠΈΠ½Π°Ρ‚Ρ‹ Π²Π΅ΠΊΡ‚ΠΎΡ€Π½ΠΎΠ³ΠΎ произвСдСния

Π’Π°ΠΊ ΠΊΠ°ΠΊ любой Π²Π΅ΠΊΡ‚ΠΎΡ€ ΠΈΠΌΠ΅Π΅Ρ‚ ΠΎΠΏΡ€Π΅Π΄Π΅Π»Π΅Π½Π½Ρ‹Π΅ ΠΊΠΎΠΎΡ€Π΄ΠΈΠ½Π°Ρ‚Ρ‹ Π² систСмС ΠΊΠΎΠΎΡ€Π΄ΠΈΠ½Π°Ρ‚, Ρ‚ΠΎ ΠΌΠΎΠΆΠ½ΠΎ ввСсти Π²Ρ‚ΠΎΡ€ΠΎΠ΅ ΠΎΠΏΡ€Π΅Π΄Π΅Π»Π΅Π½ΠΈΠ΅ Π²Π΅ΠΊΡ‚ΠΎΡ€Π½ΠΎΠ³ΠΎ произвСдСния, ΠΊΠΎΡ‚ΠΎΡ€ΠΎΠ΅ ΠΏΠΎΠ·Π²ΠΎΠ»ΠΈΡ‚ Π½Π°Ρ…ΠΎΠ΄ΠΈΡ‚ΡŒ Π΅Π³ΠΎ ΠΊΠΎΠΎΡ€Π΄ΠΈΠ½Π°Ρ‚Ρ‹ ΠΏΠΎ Π·Π°Π΄Π°Π½Π½Ρ‹ΠΌ ΠΊΠΎΠΎΡ€Π΄ΠΈΠ½Π°Ρ‚Π°ΠΌ Π²Π΅ΠΊΡ‚ΠΎΡ€ΠΎΠ².

ΠžΠΏΡ€Π΅Π΄Π΅Π»Π΅Π½ΠΈΠ΅ 2

Π’ ΠΏΡ€ΡΠΌΠΎΡƒΠ³ΠΎΠ»ΡŒΠ½ΠΎΠΉ систСмС ΠΊΠΎΠΎΡ€Π΄ΠΈΠ½Π°Ρ‚ Ρ‚Ρ€Π΅Ρ…ΠΌΠ΅Ρ€Π½ΠΎΠ³ΠΎ пространства Π²Π΅ΠΊΡ‚ΠΎΡ€Π½Ρ‹ΠΌ ΠΏΡ€ΠΎΠΈΠ·Π²Π΅Π΄Π΅Π½ΠΈΠ΅ΠΌ Π΄Π²ΡƒΡ… Π²Π΅ΠΊΡ‚ΠΎΡ€ΠΎΠ² aβ†’=(ax;Β ay;Β az) ΠΈ bβ†’=(bx;Β by;Β bz)Β Π½Π°Π·Ρ‹Π²Π°ΡŽΡ‚ Π²Π΅ΠΊΡ‚ΠΎΡ€ cβ†’=aβ†’Γ—bβ†’=(ayΒ·bz-azΒ·by)Β·iβ†’+(azΒ·bx-axΒ·bz)Β·jβ†’+(axΒ·by-ayΒ·bx)Β·kβ†’, Π³Π΄Π΅ iβ†’,Β jβ†’,Β kβ†’ ΡΠ²Π»ΡΡŽΡ‚ΡΡ ΠΊΠΎΠΎΡ€Π΄ΠΈΠ½Π°Ρ‚Π½Ρ‹ΠΌΠΈ Π²Π΅ΠΊΡ‚ΠΎΡ€Π°ΠΌΠΈ.

Π’Π΅ΠΊΡ‚ΠΎΡ€Π½ΠΎΠ΅ ΠΏΡ€ΠΎΠΈΠ·Π²Π΅Π΄Π΅Π½ΠΈΠ΅ ΠΌΠΎΠΆΠ½ΠΎ прСдставит ΠΊΠ°ΠΊ ΠΎΠΏΡ€Π΅Π΄Π΅Π»ΠΈΡ‚Π΅Π»ΡŒ ΠΊΠ²Π°Π΄Ρ€Π°Ρ‚Π½ΠΎΠΉ ΠΌΠ°Ρ‚Ρ€ΠΈΡ†Ρ‹ Ρ‚Ρ€Π΅Ρ‚ΡŒΠ΅Π³ΠΎ порядка, Π³Π΄Π΅ пСрвая строка Π΅ΡΡ‚ΡŒ Π²Π΅ΠΊΡ‚ΠΎΡ€Ρ‹ ΠΎΡ€Ρ‚Ρ‹ iβ†’,Β jβ†’,Β kβ†’, вторая строка содСрТит ΠΊΠΎΠΎΡ€Π΄ΠΈΠ½Π°Ρ‚Ρ‹ Π²Π΅ΠΊΡ‚ΠΎΡ€Π° aβ†’, Π° Ρ‚Ρ€Π΅Ρ‚ΡŒΡ – ΠΊΠΎΠΎΡ€Π΄ΠΈΠ½Π°Ρ‚Ρ‹ Π²Π΅ΠΊΡ‚ΠΎΡ€Π° bβ†’ Π² Π·Π°Π΄Π°Π½Π½ΠΎΠΉ ΠΏΡ€ΡΠΌΠΎΡƒΠ³ΠΎΠ»ΡŒΠ½ΠΎΠΉ систСмС ΠΊΠΎΠΎΡ€Π΄ΠΈΠ½Π°Ρ‚, Π΄Π°Π½Π½Ρ‹ΠΉ ΠΎΠΏΡ€Π΅Π΄Π΅Π»ΠΈΡ‚Π΅Π»ΡŒ ΠΌΠ°Ρ‚Ρ€ΠΈΡ†Ρ‹ выглядит Ρ‚Π°ΠΊ: cβ†’=aβ†’Γ—bβ†’=iβ†’jβ†’kβ†’axayazbxbybz

Π Π°Π·Π»ΠΎΠΆΠΈΠ² Π΄Π°Π½Π½Ρ‹ΠΉ ΠΎΠΏΡ€Π΅Π΄Π΅Π»ΠΈΡ‚Π΅Π»ΡŒ ΠΏΠΎ элСмСнтам ΠΏΠ΅Ρ€Π²ΠΎΠΉ строки, ΠΏΠΎΠ»ΡƒΡ‡ΠΈΠΌ равСнство:Β cβ†’=aβ†’Γ—bβ†’=iβ†’jβ†’kβ†’axayazbxbybz=ayazbybzΒ·iβ†’-axazbxbzΒ·jβ†’+axaybxbyΒ·kβ†’==aβ†’Γ—bβ†’=(ayΒ·bz-azΒ·by)Β·iβ†’+(azΒ·bx-axΒ·bz)Β·jβ†’+(axΒ·by-ayΒ·bx)Β·kβ†’

Бвойства Π²Π΅ΠΊΡ‚ΠΎΡ€Π½ΠΎΠ³ΠΎ произвСдСния

Π˜Π·Π²Π΅ΡΡ‚Π½ΠΎ, Ρ‡Ρ‚ΠΎ Π²Π΅ΠΊΡ‚ΠΎΡ€Π½ΠΎΠ΅ ΠΏΡ€ΠΎΠΈΠ·Π²Π΅Π΄Π΅Π½ΠΈΠ΅ Π² ΠΊΠΎΠΎΡ€Π΄ΠΈΠ½Π°Ρ‚Π°Ρ… прСдставляСтся ΠΊΠ°ΠΊ ΠΎΠΏΡ€Π΅Π΄Π΅Π»ΠΈΡ‚Π΅Π»ΡŒ ΠΌΠ°Ρ‚Ρ€ΠΈΡ†Ρ‹ cβ†’=aβ†’Γ—bβ†’=iβ†’jβ†’kβ†’axayazbxbybz, Ρ‚ΠΎ Π½Π° Π±Π°Π·Π΅ свойств опрСдСлитСля ΠΌΠ°Ρ‚Ρ€ΠΈΡ†Ρ‹ выводятся ΡΠ»Π΅Π΄ΡƒΡŽΡ‰ΠΈΠ΅ свойства Π²Π΅ΠΊΡ‚ΠΎΡ€Π½ΠΎΠ³ΠΎ произвСдСния:

  1. Π°Π½Ρ‚ΠΈΠΊΠΎΠΌΠΌΡƒΡ‚Π°Ρ‚ΠΈΠ²Π½ΠΎΡΡ‚ΡŒ aβ†’Γ—bβ†’=-bβ†’Γ—aβ†’;
  2. Π΄ΠΈΡΡ‚Ρ€ΠΈΠ±ΡƒΡ‚ΠΈΠ²Π½ΠΎΡΡ‚ΡŒ a(1)β†’+a(2)β†’Γ—b=a(1)β†’Γ—bβ†’+a(2)β†’Γ—bβ†’Β ΠΈΠ»ΠΈ aβ†’Γ—b(1)β†’+b(2)β†’=aβ†’Γ—b(1)β†’+aβ†’Γ—b(2)β†’;
  3. Π°ΡΡΠΎΡ†ΠΈΠ°Ρ‚ΠΈΠ²Π½ΠΎΡΡ‚ΡŒ λ·aβ†’Γ—bβ†’=λ·aβ†’Γ—bβ†’ ΠΈΠ»ΠΈΒ aβ†’Γ—(λ·bβ†’)=λ·aβ†’Γ—bβ†’, Π³Π΄Π΅ Ξ» — ΠΏΡ€ΠΎΠΈΠ·Π²ΠΎΠ»ΡŒΠ½ΠΎΠ΅ Π΄Π΅ΠΉΡΡ‚Π²ΠΈΡ‚Π΅Π»ΡŒΠ½ΠΎΠ΅ число.

Π”Π°Π½Π½Ρ‹Π΅ свойства ΠΈΠΌΠ΅ΡŽΡ‚ Π½Π΅ слоТныС Π΄ΠΎΠΊΠ°Π·Π°Ρ‚Π΅Π»ΡŒΡΡ‚Π²Π°.

Для ΠΏΡ€ΠΈΠΌΠ΅Ρ€Π° ΠΌΠΎΠΆΠ΅ΠΌ Π΄ΠΎΠΊΠ°Π·Π°Ρ‚ΡŒ свойство антикоммутативности Π²Π΅ΠΊΡ‚ΠΎΡ€Π½ΠΎΠ³ΠΎ произвСдСния.

Π”ΠΎΠΊΠ°Π·Π°Ρ‚Π΅Π»ΡŒΡΡ‚Π²ΠΎ антикоммутативности

По ΠΎΠΏΡ€Π΅Π΄Π΅Π»Π΅Π½ΠΈΡŽ aβ†’Γ—bβ†’=iβ†’jβ†’kβ†’axayazbxbybz ΠΈ bβ†’Γ—aβ†’=iβ†’jβ†’kβ†’bxbybzaxayaz. А Ссли Π΄Π²Π΅ строчки ΠΌΠ°Ρ‚Ρ€ΠΈΡ†Ρ‹ ΠΏΠ΅Ρ€Π΅ΡΡ‚Π°Π²ΠΈΡ‚ΡŒ мСстами, Ρ‚ΠΎ Π·Π½Π°Ρ‡Π΅Π½ΠΈΠ΅ опрСдСлитСля ΠΌΠ°Ρ‚Ρ€ΠΈΡ†Ρ‹ Π΄ΠΎΠ»ΠΆΠ½ΠΎ мСняСтся Π½Π° ΠΏΡ€ΠΎΡ‚ΠΈΠ²ΠΎΠΏΠΎΠ»ΠΎΠΆΠ½ΠΎΠ΅,ΡΠ»Π΅Π΄ΠΎΠ²Π°Ρ‚Π΅Π»ΡŒΠ½ΠΎ,aβ†’Γ—bβ†’=iβ†’jβ†’kβ†’axayazbxbybzΒ =-iβ†’jβ†’kβ†’bxbybzaxayaz=-bβ†’Γ—aβ†’, Ρ‡Ρ‚ΠΎ ΠΈ Π΄ΠΎΠΊΠ°Π·Ρ‹Π²Π°Π΅Ρ‚ Π°Π½Ρ‚ΠΈΠΊΠΎΠΌΠΌΡƒΡ‚Π°Ρ‚ΠΈΠ²Π½ΠΎΡΡ‚ΡŒ Π²Π΅ΠΊΡ‚ΠΎΡ€Π½ΠΎΠ³ΠΎ произвСдСния.

НуТна ΠΏΠΎΠΌΠΎΡ‰ΡŒ прСподаватСля?

Опиши Π·Π°Π΄Π°Π½ΠΈΠ΅Β β€” и наши экспСрты Ρ‚Π΅Π±Π΅ ΠΏΠΎΠΌΠΎΠ³ΡƒΡ‚!

ΠžΠΏΠΈΡΠ°Ρ‚ΡŒ Π·Π°Π΄Π°Π½ΠΈΠ΅

Π’Π΅ΠΊΡ‚ΠΎΡ€Π½ΠΎΠ΅ ΠΏΡ€ΠΎΠΈΠ·Π²Π΅Π΄Π΅Π½ΠΈΠ΅ – ΠΏΡ€ΠΈΠΌΠ΅Ρ€Ρ‹ ΠΈ Ρ€Π΅ΡˆΠ΅Π½ΠΈΡ

Π’ Π±ΠΎΠ»ΡŒΡˆΠΈΠ½ΡΡ‚Π²Π΅ случаСв Π²ΡΡ‚Ρ€Π΅Ρ‡Π°ΡŽΡ‚ΡΡ Ρ‚Ρ€ΠΈ Ρ‚ΠΈΠΏΠ° Π·Π°Π΄Π°Ρ‡.

Π’ Π·Π°Π΄Π°Ρ‡Π°Ρ… ΠΏΠ΅Ρ€Π²ΠΎΠ³ΠΎ Ρ‚ΠΈΠΏΠ° ΠΎΠ±Ρ‹Ρ‡Π½ΠΎ Π·Π°Π΄Π°Π½Ρ‹ Π΄Π»ΠΈΠ½Ρ‹ Π΄Π²ΡƒΡ… Π²Π΅ΠΊΡ‚ΠΎΡ€ΠΎΠ² ΠΈ ΡƒΠ³ΠΎΠ» ΠΌΠ΅ΠΆΠ΄Ρƒ Π½ΠΈΠΌΠΈ, Π° Π½ΡƒΠΆΠ½ΠΎ Π½Π°ΠΉΡ‚ΠΈ Π΄Π»ΠΈΠ½Ρƒ Π²Π΅ΠΊΡ‚ΠΎΡ€Π½ΠΎΠ³ΠΎ произвСдСния. Π’ этом случаС ΠΏΠΎΠ»ΡŒΠ·ΡƒΡŽΡ‚ΡΡ ΡΠ»Π΅Π΄ΡƒΡŽΡ‰Π΅ΠΉ Ρ„ΠΎΡ€ΠΌΡƒΠ»ΠΎΠΉcβ†’=aβ†’Β·bβ†’Β·sin∠aβ†’,bβ†’ .

ΠŸΡ€ΠΈΠΌΠ΅Ρ€ 1

НайдитС Π΄Π»ΠΈΠ½Ρƒ Π²Π΅ΠΊΡ‚ΠΎΡ€Π½ΠΎΠ³ΠΎ произвСдСния Π²Π΅ΠΊΡ‚ΠΎΡ€ΠΎΠ²Β aβ†’ ΠΈ bβ†’, Ссли извСстноaβ†’=3,Β bβ†’=5, ∠aβ†’,bβ†’=Ο€4.

РСшСниС

Π‘ ΠΏΠΎΠΌΠΎΡ‰ΡŒΡŽ опрСдСлСния Π΄Π»ΠΈΠ½Ρ‹ Π²Π΅ΠΊΡ‚ΠΎΡ€Π½ΠΎΠ³ΠΎ произвСдСния Π²Π΅ΠΊΡ‚ΠΎΡ€ΠΎΠ² aβ†’ ΠΈ bβ†’ Ρ€Π΅ΡˆΠΈΠΌ Π΄Π°Π½Π½ΡƒΡŽ Π·Π°Π΄Π°Ρ‡: aβ†’Γ—bβ†’=aβ†’Β·bβ†’Β·sin∠aβ†’,bβ†’=3Β·5Β·sinΟ€4=1522.

ΠžΡ‚Π²Π΅Ρ‚:Β 1522.

Π—Π°Π΄Π°Ρ‡ΠΈ Π²Ρ‚ΠΎΡ€ΠΎΠ³ΠΎ Ρ‚ΠΈΠΏΠ° ΠΈΠΌΠ΅ΡŽΡ‚ связь с ΠΊΠΎΠΎΡ€Π΄ΠΈΠ½Π°Ρ‚Π°ΠΌΠΈ Π²Π΅ΠΊΡ‚ΠΎΡ€ΠΎΠ², Π² Π½ΠΈΡ… Π²Π΅ΠΊΡ‚ΠΎΡ€Π½ΠΎΠ΅ ΠΏΡ€ΠΎΠΈΠ·Π²Π΅Π΄Π΅Π½ΠΈΠ΅, Π΅Π³ΠΎ Π΄Π»ΠΈΠ½Π° ΠΈ Ρ‚.Π΄. ищутся Ρ‡Π΅Ρ€Π΅Π· извСстныС ΠΊΠΎΠΎΡ€Π΄ΠΈΠ½Π°Ρ‚Ρ‹ Π·Π°Π΄Π°Π½Π½Ρ‹Ρ… Π²Π΅ΠΊΡ‚ΠΎΡ€ΠΎΠ² aβ†’=(ax;Β ay;Β az)Β ΠΈΒ bβ†’=(bx;Β by;Β bz).

Для Ρ‚Π°ΠΊΠΎΠ³ΠΎ Ρ‚ΠΈΠΏΠ° Π·Π°Π΄Π°Ρ‡, ΠΌΠΎΠΆΠ½ΠΎ Ρ€Π΅ΡˆΠΈΡ‚ΡŒ массу Π²Π°Ρ€ΠΈΠ°Π½Ρ‚ΠΎΠ² Π·Π°Π΄Π°Π½ΠΈΠΉ. НапримСр, ΠΌΠΎΠ³ΡƒΡ‚ Π±Ρ‹Ρ‚ΡŒ Π·Π°Π΄Π°Π½Ρ‹ Π½Π΅ ΠΊΠΎΠΎΡ€Π΄ΠΈΠ½Π°Ρ‚Ρ‹ Π²Π΅ΠΊΡ‚ΠΎΡ€ΠΎΠ² Β aβ†’ ΠΈ bβ†’, Π° ΠΈΡ… разлоТСния ΠΏΠΎ ΠΊΠΎΠΎΡ€Π΄ΠΈΠ½Π°Ρ‚Π½Ρ‹ΠΌ Π²Π΅ΠΊΡ‚ΠΎΡ€Π°ΠΌ Π²ΠΈΠ΄Π° bβ†’=bxΒ·iβ†’Β +byΒ·jβ†’+bzΒ·kβ†’Β ΠΈ cβ†’=aβ†’Γ—bβ†’=(ayΒ·bz-azΒ·by)Β·iβ†’+(azΒ·bx-axΒ·bz)Β·jβ†’+(axΒ·by-ayΒ·bx)Β·kβ†’, ΠΈΠ»ΠΈ Π²Π΅ΠΊΡ‚ΠΎΡ€Ρ‹Β aβ†’ ΠΈ bβ†’ ΠΌΠΎΠ³ΡƒΡ‚ Π±Ρ‹Ρ‚ΡŒ Π·Π°Π΄Π°Π½Ρ‹ ΠΊΠΎΠΎΡ€Π΄ΠΈΠ½Π°Ρ‚Π°ΠΌΠΈ Ρ‚ΠΎΡ‡Π΅ΠΊ ΠΈΡ… Π½Π°Ρ‡Π°Π»Π° ΠΈ ΠΊΠΎΠ½Ρ†Π°.

Рассмотрим ΡΠ»Π΅Π΄ΡƒΡŽΡ‰ΠΈΠ΅ ΠΏΡ€ΠΈΠΌΠ΅Ρ€Ρ‹.

ΠŸΡ€ΠΈΠΌΠ΅Ρ€ 2

Π’ ΠΏΡ€ΡΠΌΠΎΡƒΠ³ΠΎΠ»ΡŒΠ½ΠΎΠΉ систСмС ΠΊΠΎΠΎΡ€Π΄ΠΈΠ½Π°Ρ‚ Π·Π°Π΄Π°Π½Ρ‹ Π΄Π²Π° Π²Π΅ΠΊΡ‚ΠΎΡ€Π° aβ†’=(2;Β 1;Β -3),Β bβ†’=(0;Β -1;Β 1). НайдитС ΠΈΡ… Π²Π΅ΠΊΡ‚ΠΎΡ€Π½ΠΎΠ΅ ΠΏΡ€ΠΎΠΈΠ·Π²Π΅Π΄Π΅Π½ΠΈΠ΅.

РСшСниС

По Π²Ρ‚ΠΎΡ€ΠΎΠΌΡƒ ΠΎΠΏΡ€Π΅Π΄Π΅Π»Π΅Π½ΠΈΡŽ Π½Π°ΠΉΠ΄Π΅ΠΌ Π²Π΅ΠΊΡ‚ΠΎΡ€Π½ΠΎΠ΅ ΠΏΡ€ΠΎΠΈΠ·Π²Π΅Π΄Π΅Π½ΠΈΠ΅ Π΄Π²ΡƒΡ… Π²Π΅ΠΊΡ‚ΠΎΡ€ΠΎΠ² Π² Π·Π°Π΄Π°Π½Π½Ρ‹Ρ… ΠΊΠΎΠΎΡ€Π΄ΠΈΠ½Π°Ρ‚Π°Ρ…:aβ†’Γ—bβ†’=(ayΒ·bz-azΒ·by)Β·iβ†’+(azΒ·bx-axΒ·bz)Β·jβ†’+(axΒ·by-ayΒ·bx)Β·kβ†’==(1Β·1-(-3)Β·(-1))Β·iβ†’+((-3)Β·0-2Β·1)Β·jβ†’+(2Β·(-1)-1Β·0)Β·kβ†’==-2iβ†’-2jβ†’-2kβ†’.

Если Π·Π°ΠΏΠΈΡΠ°Ρ‚ΡŒ Π²Π΅ΠΊΡ‚ΠΎΡ€Π½ΠΎΠ΅ ΠΏΡ€ΠΎΠΈΠ·Π²Π΅Π΄Π΅Π½ΠΈΠ΅ Ρ‡Π΅Ρ€Π΅Π· ΠΎΠΏΡ€Π΅Π΄Π΅Π»ΠΈΡ‚Π΅Π»ΡŒ ΠΌΠ°Ρ‚Ρ€ΠΈΡ†Ρ‹, Ρ‚ΠΎ Ρ€Π΅ΡˆΠ΅Π½ΠΈΠ΅ Π΄Π°Π½Π½ΠΎΠ³ΠΎ ΠΏΡ€ΠΈΠΌΠ΅Ρ€Π° выглядит ΡΠ»Π΅Π΄ΡƒΡŽΡ‰ΠΈΠΌ ΠΎΠ±Ρ€Π°Π·ΠΎΠΌ:Β aβ†’Γ—bβ†’=iβ†’jβ†’kβ†’axayazbxbybz=iβ†’jβ†’kβ†’21-30-11=-2iβ†’-2jβ†’-2kβ†’.

ΠžΡ‚Π²Π΅Ρ‚:Β aβ†’Γ—bβ†’=-2iβ†’-2jβ†’-2kβ†’.

ΠŸΡ€ΠΈΠΌΠ΅Ρ€ 3

НайдитС Π΄Π»ΠΈΠ½Ρƒ Π²Π΅ΠΊΡ‚ΠΎΡ€Π½ΠΎΠ³ΠΎ произвСдСния Π²Π΅ΠΊΡ‚ΠΎΡ€ΠΎΠ² iβ†’-jβ†’ ΠΈ iβ†’+jβ†’+kβ†’, Π³Π΄Π΅ iβ†’,Β jβ†’,Β kβ†’ — ΠΎΡ€Ρ‚Ρ‹ ΠΏΡ€ΡΠΌΠΎΡƒΠ³ΠΎΠ»ΡŒΠ½ΠΎΠΉ Π΄Π΅ΠΊΠ°Ρ€Ρ‚ΠΎΠ²ΠΎΠΉ систСмы ΠΊΠΎΠΎΡ€Π΄ΠΈΠ½Π°Ρ‚.

РСшСниС

Для Π½Π°Ρ‡Π°Π»Π° Π½Π°ΠΉΠ΄Π΅ΠΌ ΠΊΠΎΠΎΡ€Π΄ΠΈΠ½Π°Ρ‚Ρ‹ Π·Π°Π΄Π°Π½Π½ΠΎΠ³ΠΎ Π²Π΅ΠΊΡ‚ΠΎΡ€Π½ΠΎΠ³ΠΎ произвСдСния iβ†’-jβ†’Γ—iβ†’+jβ†’+kβ†’Β Π² Π΄Π°Π½Π½ΠΎΠΉ ΠΏΡ€ΡΠΌΠΎΡƒΠ³ΠΎΠ»ΡŒΠ½ΠΎΠΉ систСмС ΠΊΠΎΠΎΡ€Π΄ΠΈΠ½Π°Ρ‚.

Π˜Π·Π²Π΅ΡΡ‚Π½ΠΎ, Ρ‡Ρ‚ΠΎ Π²Π΅ΠΊΡ‚ΠΎΡ€Ρ‹ iβ†’-jβ†’Β ΠΈ iβ†’+jβ†’+kβ†’Β ΠΈΠΌΠ΅ΡŽΡ‚ ΠΊΠΎΠΎΡ€Π΄ΠΈΠ½Π°Ρ‚Ρ‹ (1;Β -1;Β 0)Β  ΠΈ (1;Β 1;Β 1) соотвСтствСнно. НайдСм Π΄Π»ΠΈΠ½Ρƒ Π²Π΅ΠΊΡ‚ΠΎΡ€Π½ΠΎΠ³ΠΎ произвСдСния ΠΏΡ€ΠΈ ΠΏΠΎΠΌΠΎΡ‰ΠΈ опрСдСлитСля ΠΌΠ°Ρ‚Ρ€ΠΈΡ†Ρ‹, Ρ‚ΠΎΠ³Π΄Π° ΠΈΠΌΠ΅Π΅ΠΌ iβ†’-jβ†’Γ—iβ†’+jβ†’+kβ†’=iβ†’jβ†’kβ†’1-10111=-iβ†’-jβ†’+2kβ†’.

Π‘Π»Π΅Π΄ΠΎΠ²Π°Ρ‚Π΅Π»ΡŒΠ½ΠΎ, Π²Π΅ΠΊΡ‚ΠΎΡ€Π½ΠΎΠ΅ ΠΏΡ€ΠΎΠΈΠ·Π²Π΅Π΄Π΅Π½ΠΈΠ΅ iβ†’-jβ†’Γ—iβ†’+jβ†’+kβ†’ ΠΈΠΌΠ΅Π΅Ρ‚ ΠΊΠΎΠΎΡ€Π΄ΠΈΠ½Π°Ρ‚Ρ‹ (-1;Β -1;Β 2) Π² Π·Π°Π΄Π°Π½Π½ΠΎΠΉ систСмС ΠΊΠΎΠΎΡ€Π΄ΠΈΠ½Π°Ρ‚.

Π”Π»ΠΈΠ½Ρƒ Π²Π΅ΠΊΡ‚ΠΎΡ€Π½ΠΎΠ³ΠΎ произвСдСния Π½Π°ΠΉΠ΄Π΅ΠΌ ΠΏΠΎ Ρ„ΠΎΡ€ΠΌΡƒΠ»Π΅ (см. Π² Ρ€Π°Π·Π΄Π΅Π»Π΅ Π½Π°Ρ…ΠΎΠΆΠ΄Π΅Π½ΠΈΠ΅ Π΄Π»ΠΈΠ½Ρ‹ Π²Π΅ΠΊΡ‚ΠΎΡ€Π°):Β iβ†’-jβ†’Γ—iβ†’+jβ†’+kβ†’=-12+-12+22=6.

ΠžΡ‚Π²Π΅Ρ‚: iβ†’-jβ†’Γ—iβ†’+jβ†’+kβ†’=6..

ΠŸΡ€ΠΈΠΌΠ΅Ρ€ 4

Π’ ΠΏΡ€ΡΠΌΠΎΡƒΠ³ΠΎΠ»ΡŒΠ½ΠΎΠΉ Π΄Π΅ΠΊΠ°Ρ€Ρ‚ΠΎΠ²ΠΎΠΉ систСмС ΠΊΠΎΠΎΡ€Π΄ΠΈΠ½Π°Ρ‚ Π·Π°Π΄Π°Π½Ρ‹ ΠΊΠΎΠΎΡ€Π΄ΠΈΠ½Π°Ρ‚Ρ‹ Ρ‚Ρ€Π΅Ρ… Ρ‚ΠΎΡ‡Π΅ΠΊΒ A(1,0,1),Β B(0,2,3),Β C(1,4,2)Β . НайдитС ΠΊΠ°ΠΊΠΎΠΉ-Π½ΠΈΠ±ΡƒΠ΄ΡŒ Π²Π΅ΠΊΡ‚ΠΎΡ€, пСрпСндикулярный ABβ†’ ΠΈ ACβ†’ ΠΎΠ΄Π½ΠΎΠ²Ρ€Π΅ΠΌΠ΅Π½Π½ΠΎ.

РСшСниС

Π’Π΅ΠΊΡ‚ΠΎΡ€Ρ‹ Β ABβ†’ ΠΈ ACβ†’ ΠΈΠΌΠ΅ΡŽΡ‚ ΡΠ»Π΅Π΄ΡƒΡŽΡ‰ΠΈΠ΅ ΠΊΠΎΠΎΡ€Π΄ΠΈΠ½Π°Ρ‚Ρ‹ (-1;Β 2;Β 2) ΠΈ (0;Β 4;Β 1) соотвСтствСнно. Найдя Π²Π΅ΠΊΡ‚ΠΎΡ€Π½ΠΎΠ΅ ΠΏΡ€ΠΎΠΈΠ·Π²Π΅Π΄Π΅Π½ΠΈΠ΅ Π²Π΅ΠΊΡ‚ΠΎΡ€ΠΎΠ²Β ABβ†’ ΠΈ ACβ†’, ΠΎΡ‡Π΅Π²ΠΈΠ΄Π½ΠΎ, Ρ‡Ρ‚ΠΎ ΠΎΠ½ΠΎ являСтся пСрпСндикулярным Π²Π΅ΠΊΡ‚ΠΎΡ€ΠΎΠΌ ΠΏΠΎ ΠΎΠΏΡ€Π΅Π΄Π΅Π»Π΅Π½ΠΈΡŽ ΠΈ ΠΊ Β AB→​​​​​ и ΠΊΒ ACβ†’, Ρ‚ΠΎ Π΅ΡΡ‚ΡŒ, являСтся Ρ€Π΅ΡˆΠ΅Π½ΠΈΠ΅ΠΌ нашСй Π·Π°Π΄Π°Ρ‡ΠΈ. НайдСм Π΅Π³ΠΎΒ ABβ†’Γ—ACβ†’=iβ†’jβ†’kβ†’-122041=-6iβ†’+jβ†’-4kβ†’.

ΠžΡ‚Π²Π΅Ρ‚:Β -6iβ†’+jβ†’-4kβ†’. — ΠΎΠ΄ΠΈΠ½ ΠΈΠ· пСрпСндикулярных Π²Π΅ΠΊΡ‚ΠΎΡ€ΠΎΠ².

Π—Π°Π΄Π°Ρ‡ΠΈ Ρ‚Ρ€Π΅Ρ‚ΡŒΠ΅Π³ΠΎ Ρ‚ΠΈΠΏΠ° ΠΎΡ€ΠΈΠ΅Π½Ρ‚ΠΈΡ€ΠΎΠ²Π°Π½Ρ‹ Π½Π° использованиС свойств Π²Π΅ΠΊΡ‚ΠΎΡ€Π½ΠΎΠ³ΠΎ произвСдСния Π²Π΅ΠΊΡ‚ΠΎΡ€ΠΎΠ². ПослС примСнСния ΠΊΠΎΡ‚ΠΎΡ€Ρ‹Ρ…, Π±ΡƒΠ΄Π΅ΠΌ ΠΏΠΎΠ»ΡƒΡ‡Π°Ρ‚ΡŒ Ρ€Π΅ΡˆΠ΅Π½ΠΈΠ΅ Π·Π°Π΄Π°Π½Π½ΠΎΠΉ Π·Π°Π΄Π°Ρ‡ΠΈ.

ΠŸΡ€ΠΈΠΌΠ΅Ρ€ 5

Π’Π΅ΠΊΡ‚ΠΎΡ€Ρ‹ Β aβ†’ ΠΈ bβ†’ пСрпСндикулярны ΠΈ ΠΈΡ… Π΄Π»ΠΈΠ½Ρ‹ Ρ€Π°Π²Π½Ρ‹ соотвСтствСнно 3Β ΠΈ 4. НайдитС Π΄Π»ΠΈΠ½Ρƒ Π²Π΅ΠΊΡ‚ΠΎΡ€Π½ΠΎΠ³ΠΎ произвСдСния 3Β·aβ†’-bβ†’Γ—aβ†’-2Β·bβ†’=3Β·aβ†’Γ—aβ†’-2Β·bβ†’+-bβ†’Γ—aβ†’-2Β·bβ†’==3Β·aβ†’Γ—aβ†’+3Β·aβ†’Γ—-2Β·bβ†’+-bβ†’Γ—aβ†’+-bβ†’Γ—-2Β·bβ†’.

РСшСниС

По свойству дистрибутивности Π²Π΅ΠΊΡ‚ΠΎΡ€Π½ΠΎΠ³ΠΎ произвСдСния ΠΌΡ‹ ΠΌΠΎΠΆΠ΅ΠΌ Π·Π°ΠΏΠΈΡΠ°Ρ‚ΡŒΒ 3Β·aβ†’-bβ†’Γ—aβ†’-2Β·bβ†’=3Β·aβ†’Γ—aβ†’-2Β·bβ†’+-bβ†’Γ—aβ†’-2Β·bβ†’==3Β·aβ†’Γ—aβ†’+3Β·aβ†’Γ—-2Β·bβ†’+-bβ†’Γ—aβ†’+-bβ†’Γ—-2Β·bβ†’

По свойству ассоциативности вынСсСм числовыС коэффициСнты Π·Π° Π·Π½Π°ΠΊ Π²Π΅ΠΊΡ‚ΠΎΡ€Π½Ρ‹Ρ… ΠΏΡ€ΠΎΠΈΠ·Π²Π΅Π΄Π΅Π½ΠΈΠΉ Π² послСднСм Π²Ρ‹Ρ€Π°ΠΆΠ΅Π½ΠΈΠΈ: 3Β·aβ†’Γ—aβ†’+3Β·aβ†’Γ—-2Β·bβ†’+-bβ†’Γ—aβ†’+-bβ†’Γ—-2Β·bβ†’==3Β·aβ†’Γ—aβ†’+3Β·(-2)Β·aβ†’Γ—bβ†’+(-1)Β·bβ†’Γ—aβ†’+(-1)Β·(-2)Β·bβ†’Γ—bβ†’==3Β·aβ†’Γ—aβ†’-6Β·aβ†’Γ—bβ†’-bβ†’Γ—aβ†’+2Β·bβ†’Γ—bβ†’

Π’Π΅ΠΊΡ‚ΠΎΡ€Π½Ρ‹Π΅ произвСдСния aβ†’Γ—aβ†’ ΠΈ bβ†’Γ—bβ†’ Ρ€Π°Π²Π½Ρ‹ 0, Ρ‚Π°ΠΊ ΠΊΠ°ΠΊ aβ†’Γ—aβ†’=aβ†’Β·aβ†’Β·sin0=0 ΠΈ bβ†’Γ—bβ†’=bβ†’Β·bβ†’Β·sin0=0, Ρ‚ΠΎΠ³Π΄Π° 3Β·aβ†’Γ—aβ†’-6Β·aβ†’Γ—bβ†’-bβ†’Γ—aβ†’+2Β·bβ†’Γ—bβ†’=-6Β·aβ†’Γ—bβ†’-bβ†’Γ—aβ†’..

Из антикоммутативности Π²Π΅ΠΊΡ‚ΠΎΡ€Π½ΠΎΠ³ΠΎ произвСдСния слСдуСт -6Β·aβ†’Γ—bβ†’-bβ†’Γ—aβ†’=-6Β·aβ†’Γ—bβ†’-(-1)Β·aβ†’Γ—bβ†’=-5Β·aβ†’Γ—bβ†’..

Π’ΠΎΡΠΏΠΎΠ»ΡŒΠ·ΠΎΠ²Π°Π²ΡˆΠΈΡΡŒ свойствами Π²Π΅ΠΊΡ‚ΠΎΡ€Π½ΠΎΠ³ΠΎ произвСдСния, ΠΏΠΎΠ»ΡƒΡ‡Π°Π΅ΠΌ равСнство 3Β·aβ†’-bβ†’Γ—aβ†’-2Β·bβ†’==-5Β·aβ†’Γ—bβ†’.

По ΡƒΡΠ»ΠΎΠ²ΠΈΡŽ Π²Π΅ΠΊΡ‚ΠΎΡ€Ρ‹ Β aβ†’ ΠΈ bβ†’ пСрпСндикулярны, Ρ‚ΠΎ Π΅ΡΡ‚ΡŒ ΡƒΠ³ΠΎΠ» ΠΌΠ΅ΠΆΠ΄Ρƒ Π½ΠΈΠΌΠΈ Ρ€Π°Π²Π΅Π½ Ο€2. Π’Π΅ΠΏΠ΅Ρ€ΡŒ остаСтся лишь ΠΏΠΎΠ΄ΡΡ‚Π°Π²ΠΈΡ‚ΡŒ Π½Π°ΠΉΠ΄Π΅Π½Π½Ρ‹Π΅ значСния Π² ΡΠΎΠΎΡ‚Π²Π΅Ρ‚ΡΡ‚Π²ΡƒΡŽΡ‰ΠΈΠ΅ Ρ„ΠΎΡ€ΠΌΡƒΠ»Ρ‹: 3Β·aβ†’-bβ†’Γ—aβ†’-2Β·bβ†’=-5Β·aβ†’Γ—bβ†’==5Β·aβ†’Γ—bβ†’=5Β·aβ†’Β·bβ†’Β·sin(aβ†’,bβ†’)=5Β·3Β·4Β·sinΟ€2=60.

ΠžΡ‚Π²Π΅Ρ‚: 3Β·aβ†’-bβ†’Γ—aβ†’-2Β·bβ†’=60.

ГСомСтричСский смысл Π²Π΅ΠΊΡ‚ΠΎΡ€Π½ΠΎΠ³ΠΎ произвСдСния

Π”Π»ΠΈΠ½Π° Π²Π΅ΠΊΡ‚ΠΎΡ€Π½ΠΎΠ³ΠΎ произвСдСния Π²Π΅ΠΊΡ‚ΠΎΡ€ΠΎΠ² ΠΏΠΎ ΠΎΡ€ΠΏΠ΅Π΄Π΅Π»Π΅Π½ΠΈΡŽ Ρ€Π°Π²Π½Π°Β aβ†’Γ—bβ†’=aβ†’Β·bβ†’Β·sin∠aβ†’,bβ†’. Π’Π°ΠΊ ΠΊΠ°ΠΊ ΡƒΠΆΠ΅ извСстно (ΠΈΠ· школьного курса), Ρ‡Ρ‚ΠΎ ΠΏΠ»ΠΎΡ‰Π°Π΄ΡŒ Ρ‚Ρ€Π΅ΡƒΠ³ΠΎΠ»ΡŒΠ½ΠΈΠΊΠ° Ρ€Π°Π²Π½Π° ΠΏΠΎΠ»ΠΎΠ²ΠΈΠ½Π΅ произвСдСния Π΄Π»ΠΈΠ½ Π΄Π²ΡƒΡ… Π΅Π³ΠΎ сторон ΡƒΠΌΠ½ΠΎΠΆΠ΅Π½Π½ΠΎΠ΅ Π½Π° синус ΡƒΠ³Π»Π° ΠΌΠ΅ΠΆΠ΄Ρƒ Π΄Π°Π½Π½Ρ‹ΠΌΠΈ сторонами. Π‘Π»Π΅Π΄ΠΎΠ²Π°Ρ‚Π΅Π»ΡŒΠ½ΠΎ, Π΄Π»ΠΈΠ½Π° Π²Π΅ΠΊΡ‚ΠΎΡ€Π½ΠΎΠ³ΠΎ произвСдСния Ρ€Π°Π²Π½Π° ΠΏΠ»ΠΎΡ‰Π°Π΄ΠΈ ΠΏΠ°Ρ€Π°Π»Π»Π΅Π»ΠΎΠ³Ρ€Π°ΠΌΠΌΠ° — ΡƒΠ΄Π²ΠΎΠ΅Π½Π½ΠΎΠ³ΠΎ Ρ‚Ρ€Π΅ΡƒΠ³ΠΎΠ»ΡŒΠ½ΠΈΠΊΠ°, Π° ΠΈΠΌΠ΅Π½Π½ΠΎ ΠΏΡ€ΠΎΠΈΠ·Π²Π΅Π΄Π΅Π½ΠΈΡŽ сторон Π² Π²ΠΈΠ΄Π΅ Π²Π΅ΠΊΡ‚ΠΎΡ€ΠΎΠ² Β aβ†’ ΠΈ bβ†’, ΠΎΡ‚Π»ΠΎΠΆΠ΅Π½Π½Ρ‹Π΅ ΠΎΡ‚ ΠΎΠ΄Π½ΠΎΠΉ Ρ‚ΠΎΡ‡ΠΊΠΈ, Π½Π° синус ΡƒΠ³Π»Π° ΠΌΠ΅ΠΆΠ΄Ρƒ Π½ΠΈΠΌΠΈ sin∠aβ†’,bβ†’.

Π­Ρ‚ΠΎ ΠΈ Π΅ΡΡ‚ΡŒ гСомСтричСский смысл Π²Π΅ΠΊΡ‚ΠΎΡ€Π½ΠΎΠ³ΠΎ произвСдСния.

ЀизичСский смысл Π²Π΅ΠΊΡ‚ΠΎΡ€Π½ΠΎΠ³ΠΎ произвСдСния

Π’ ΠΌΠ΅Ρ…Π°Π½ΠΈΠΊΠ΅, ΠΎΠ΄Π½ΠΎΠΌ ΠΈΠ· Ρ€Π°Π·Π΄Π΅Π»ΠΎΠ² Ρ„ΠΈΠ·ΠΈΠΊΠΈ, благодаря Π²Π΅ΠΊΡ‚ΠΎΡ€Π½ΠΎΠΌΡƒ ΠΏΡ€ΠΎΠΈΠ·Π²Π΅Π΄Π΅Π½ΠΈΡŽ ΠΌΠΎΠΆΠ½ΠΎ ΠΎΠΏΡ€Π΅Π΄Π΅Π»ΠΈΡ‚ΡŒ ΠΌΠΎΠΌΠ΅Π½Ρ‚ силы ΠΎΡ‚Π½ΠΎΡΠΈΡ‚Π΅Π»ΡŒΠ½ΠΎ Ρ‚ΠΎΡ‡ΠΊΠΈ пространства.

ΠžΠΏΡ€Π΅Π΄Π΅Π»Π΅Π½ΠΈΠ΅ 3

Под ΠΌΠΎΠΌΠ΅Π½Ρ‚ΠΎΠΌ силы Fβ†’, ΠΏΡ€ΠΈΠ»ΠΎΠΆΠ΅Π½Π½ΠΎΠΉ ΠΊ Ρ‚ΠΎΡ‡ΠΊΠ΅ B, ΠΎΡ‚Π½ΠΎΡΠΈΡ‚Π΅Π»ΡŒΠ½ΠΎ Ρ‚ΠΎΡ‡ΠΊΠΈ A Π±ΡƒΠ΄Π΅ΠΌ ΠΏΠΎΠ½ΠΈΠΌΠ°Ρ‚ΡŒ ΡΠ»Π΅Π΄ΡƒΡŽΡ‰Π΅Π΅ Π²Π΅ΠΊΡ‚ΠΎΡ€Π½ΠΎΠ΅ ΠΏΡ€ΠΎΠΈΠ·Π²Π΅Π΄Π΅Π½ΠΈΠ΅ ABβ†’Γ—Fβ†’.

Π’Π΅ΠΊΡ‚ΠΎΡ€Π½ΠΎΠ΅ ΠΏΡ€ΠΎΠΈΠ·Π²Π΅Π΄Π΅Π½ΠΈΠ΅ Π΄Π²ΡƒΡ… Π²Π΅ΠΊΡ‚ΠΎΡ€ΠΎΠ². Он-Π»Π°ΠΉΠ½ ΠΊΠ°Π»ΡŒΠΊΡƒΠ»ΡΡ‚ΠΎΡ€.

Π’Π΅ΠΊΡ‚ΠΎΡ€Π½ΠΎΠ΅ ΠΏΡ€ΠΎΠΈΠ·Π²Π΅Π΄Π΅Π½ΠΈΠ΅ Π΄Π²ΡƒΡ… Π²Π΅ΠΊΡ‚ΠΎΡ€ΠΎΠ². Он-Π»Π°ΠΉΠ½ ΠΊΠ°Π»ΡŒΠΊΡƒΠ»ΡΡ‚ΠΎΡ€.

Π’Π΅ΠΊΡ‚ΠΎΡ€Π½ΠΎΠ΅ ΠΏΡ€ΠΎΠΈΠ·Π²Π΅Π΄Π΅Π½ΠΈΠ΅ Π΄Π²ΡƒΡ… Π²Π΅ΠΊΡ‚ΠΎΡ€ΠΎΠ² Π° ΠΈ b — это опСрация Π½Π°Π΄ Π½ΠΈΠΌΠΈ, опрСдСлСнная лишь Π² Ρ‚Ρ€Π΅Ρ…ΠΌΠ΅Ρ€Π½ΠΎΠΌ пространствС, Ρ€Π΅Π·ΡƒΠ»ΡŒΡ‚Π°Ρ‚ΠΎΠΌ ΠΊΠΎΡ‚ΠΎΡ€ΠΎΠΉ являСтся Π²Π΅ΠΊΡ‚ΠΎΡ€ со ΡΠ»Π΅Π΄ΡƒΡŽΡ‰ΠΈΠΌΠΈ свойствами:

Для большСй ясности ΠΏΡ€ΠΈΠ²Π΅Π΄Π΅ΠΌ ΠΏΡ€ΠΈΠΌΠ΅Ρ€ — Π½Π° рисункС справа Π²Π΅ΠΊΡ‚ΠΎΡ€ [a,b] — Π²Π΅ΠΊΡ‚ΠΎΡ€Π½ΠΎΠ΅ ΠΏΡ€ΠΎΠΈΠ·Π²Π΅Π΄Π΅Π½ΠΈΠ΅ Π²Π΅ΠΊΡ‚ΠΎΡ€ΠΎΠ² Π° ΠΈ b. Как сказано Π² ΠΎΠΏΡ€Π΅Π΄Π΅Π»Π΅Π½ΠΈΠΈ, ΠΌΡ‹ ΠΏΡ€ΠΈΠ²Π΅Π»ΠΈ всС Ρ‚Ρ€ΠΈ Π²Π΅ΠΊΡ‚ΠΎΡ€Π° ΠΊ ΠΎΠ±Ρ‰Π΅ΠΌΡƒ Π½Π°Ρ‡Π°Π»Ρƒ, ΠΈ Ρ‚ΠΎΠ³Π΄Π°, Ссли ΡΠΌΠΎΡ‚Ρ€Π΅Ρ‚ΡŒ Π½Π° Π²Π΅ΠΊΡ‚ΠΎΡ€Π° a ΠΈ b с ΠΊΠΎΠ½Ρ†Π° Π²Π΅ΠΊΡ‚ΠΎΡ€Π° [a,b], ΠΊΡ€Π°Ρ‚Ρ‡Π°ΠΉΡˆΠΈΠΉ ΠΏΠΎΠ²ΠΎΡ€ΠΎΡ‚ ΠΎΡ‚ Π²Π΅ΠΊΡ‚ΠΎΡ€Π° Π° Π΄ΠΎ Π²Π΅ΠΊΡ‚ΠΎΡ€Π° b Π±ΡƒΠ΄Π΅Ρ‚ ΠΏΡ€ΠΎΡ‚ΠΈΠ² часовой стрСлки .

  • ΠžΡ‡Π΅Π²ΠΈΠ΄Π½ΠΎ, Ρ‡Ρ‚ΠΎ Π² случаС Π²Π΅ΠΊΡ‚ΠΎΡ€Π½ΠΎΠ³ΠΎ произвСдСния, ΠΈΠΌΠ΅Π΅Ρ‚ Π·Π½Π°Ρ‡Π΅Π½ΠΈΠ΅ порядок, Π² ΠΊΠΎΡ‚ΠΎΡ€ΠΎΠΌ бСрутся Π²Π΅ΠΊΡ‚ΠΎΡ€Π°, Π±ΠΎΠ»Π΅Π΅ Ρ‚ΠΎΠ³ΠΎ,
  • Π’Π°ΠΊ ΠΆΠ΅, нСпосрСдствСнно ΠΈΠ· опрСдСлСния слСдуСт, Ρ‡Ρ‚ΠΎ для любого скалярного мноТитСля k (числа) Π²Π΅Ρ€Π½ΠΎ ΡΠ»Π΅Π΄ΡƒΡŽΡ‰Π΅Π΅:
  • Π’Π΅ΠΊΡ‚ΠΎΡ€Π½ΠΎΠ΅ ΠΏΡ€ΠΎΠΈΠ·Π²Π΅Π΄Π΅Π½ΠΈΠ΅ ΠΊΠΎΠ»Π»ΠΈΠ½Π΅Π°Ρ€Π½Ρ‹Ρ… Π²Π΅ΠΊΡ‚ΠΎΡ€ΠΎΠ² Ρ€Π°Π²Π½ΠΎ Π½ΡƒΠ»Π΅Π²ΠΎΠΌΡƒ Π²Π΅ΠΊΡ‚ΠΎΡ€Ρƒ. Π‘ΠΎΠ»Π΅Π΅ Ρ‚ΠΎΠ³ΠΎ, Π²Π΅ΠΊΡ‚ΠΎΡ€Π½ΠΎΠ΅ ΠΏΡ€ΠΎΠΈΠ·Π²Π΅Π΄Π΅Π½ΠΈΠ΅ Π΄Π²ΡƒΡ… Π²Π΅ΠΊΡ‚ΠΎΡ€ΠΎΠ² Ρ€Π°Π²Π½ΠΎ Π½ΡƒΠ»ΡŽ Ρ‚ΠΎΠ³Π΄Π° ΠΈ Ρ‚ΠΎΠ»ΡŒΠΊΠΎ Ρ‚ΠΎΠ³Π΄Π°, ΠΊΠΎΠ³Π΄Π° ΠΎΠ½ΠΈ ΠΊΠΎΠ»Π»ΠΈΠ½Π΅Π°Ρ€Π½Ρ‹. (Π’ случаС, Ссли ΠΎΠ΄ΠΈΠ½ ΠΈΠ· Π½ΠΈΡ… Π½ΡƒΠ»Π΅Π²ΠΎΠΉ Π²Π΅ΠΊΡ‚ΠΎΡ€ Π½Π΅ΠΎΠ±Ρ…ΠΎΠ΄ΠΈΠΌΠΎ Π²ΡΠΏΠΎΠΌΠ½ΠΈΡ‚ΡŒ, Ρ‡Ρ‚ΠΎ Π½ΡƒΠ»Π΅Π²ΠΎΠΉ Π²Π΅ΠΊΡ‚ΠΎΡ€ ΠΊΠΎΠ»Π»ΠΈΠ½Π΅Π°Ρ€Π΅Π½ Π»ΡŽΠ±ΠΎΠΌΡƒ Π²Π΅ΠΊΡ‚ΠΎΡ€Ρƒ ΠΏΠΎ ΠΎΠΏΡ€Π΅Π΄Π΅Π»Π΅Π½ΠΈΡŽ).
  • Π’Π΅ΠΊΡ‚ΠΎΡ€Π½ΠΎΠ΅ ΠΏΡ€ΠΎΠΈΠ·Π²Π΅Π΄Π΅Π½ΠΈΠ΅ ΠΎΠ±Π»Π°Π΄Π°Π΅Ρ‚ Ρ€Π°ΡΠΏΡ€Π΅Π΄Π΅Π»ΠΈΡ‚Π΅Π»ΡŒΠ½Ρ‹ΠΌ свойством, Ρ‚ΠΎ Π΅ΡΡ‚ΡŒ

Π’Ρ‹Ρ€Π°ΠΆΠ΅Π½ΠΈΠ΅ Π²Π΅ΠΊΡ‚ΠΎΡ€Π½ΠΎΠ³ΠΎ произвСдСния Ρ‡Π΅Ρ€Π΅Π· ΠΊΠΎΠΎΡ€Π΄ΠΈΠ½Π°Ρ‚Ρ‹ Π²Π΅ΠΊΡ‚ΠΎΡ€ΠΎΠ².

  • ΠŸΡƒΡΡ‚ΡŒ Π΄Π°Π½Ρ‹ Π΄Π²Π° Π²Π΅ΠΊΡ‚ΠΎΡ€Π°
  • (ΠΊΠ°ΠΊ Π½Π°ΠΉΡ‚ΠΈ ΠΊΠΎΠΎΡ€Π΄ΠΈΠ½Π°Ρ‚Ρ‹ Π²Π΅ΠΊΡ‚ΠΎΡ€Π° ΠΏΠΎ ΠΊΠΎΠΎΡ€Π΄ΠΈΠ½Π°Ρ‚Π°ΠΌ Π΅Π³ΠΎ Π½Π°Ρ‡Π°Π»Π° ΠΈ ΠΊΠΎΠ½Ρ†Π° — см. ΡΡ‚Π°Ρ‚ΡŒΡŽ БкалярноС ΠΏΡ€ΠΎΠΈΠ·Π²Π΅Π΄Π΅Π½ΠΈΠ΅ Π²Π΅ΠΊΡ‚ΠΎΡ€ΠΎΠ², ΠΏΡƒΠ½ΠΊΡ‚ ΠΠ»ΡŒΡ‚Π΅Ρ€Π½Π°Ρ‚ΠΈΠ²Π½ΠΎΠ΅ ΠΎΠΏΡ€Π΅Π΄Π΅Π»Π΅Π½ΠΈΠ΅ скалярного произвСдСния, ΠΈΠ»ΠΈ вычислСниС скалярного произвСдСния Π΄Π²ΡƒΡ… Π²Π΅ΠΊΡ‚ΠΎΡ€ΠΎΠ², Π·Π°Π΄Π°Π½Π½Ρ‹Ρ… своими ΠΊΠΎΠΎΡ€Π΄ΠΈΠ½Π°Ρ‚Π°ΠΌΠΈ.)
    • Π’ΠΎΠ³Π΄Π°

Π—Π°Ρ‡Π΅ΠΌ Π½ΡƒΠΆΠ½ΠΎ Π²Π΅ΠΊΡ‚ΠΎΡ€Π½ΠΎΠ΅ ΠΏΡ€ΠΎΠΈΠ·Π²Π΅Π΄Π΅Π½ΠΈΠ΅?

БущСствуСт мноТСство способов примСнСния Π²Π΅ΠΊΡ‚ΠΎΡ€Π½ΠΎΠ³ΠΎ произвСдСния, Π½Π°ΠΏΡ€ΠΈΠΌΠ΅Ρ€, ΠΊΠ°ΠΊ ΡƒΠΆΠ΅ написано Π²Ρ‹ΡˆΠ΅, вычислив Π²Π΅ΠΊΡ‚ΠΎΡ€Π½ΠΎΠ΅ ΠΏΡ€ΠΎΠΈΠ·Π²Π΅Π΄Π΅Π½ΠΈΠ΅ Π΄Π²ΡƒΡ… Π²Π΅ΠΊΡ‚ΠΎΡ€ΠΎΠ² ΠΌΠΎΠΆΠ½ΠΎ Π²Ρ‹ΡΡΠ½ΠΈΡ‚ΡŒ, ΠΊΠΎΠ»Π»ΠΈΠ½Π΅Π°Ρ€Π½Ρ‹ Π»ΠΈ ΠΎΠ½ΠΈ.

Или ΠΆΠ΅ Π΅Π³ΠΎ ΠΌΠΎΠΆΠ½ΠΎ ΠΈΡΠΏΠΎΠ»ΡŒΠ·ΠΎΠ²Π°Ρ‚ΡŒ ΠΊΠ°ΠΊ способ вычислСния ΠΏΠ»ΠΎΡ‰Π°Π΄ΠΈ ΠΏΠ°Ρ€Π°Π»Π»Π΅Π»ΠΎΠ³Ρ€Π°ΠΌΠΌΠ°, построСнного Π½Π° этих Π²Π΅ΠΊΡ‚ΠΎΡ€Π°Ρ…. Π˜ΡΡ…ΠΎΠ΄Ρ ΠΈΠ· опрСдСлСния, Π΄Π»ΠΈΠ½Π° Ρ€Π΅Π·ΡƒΠ»ΡŒΡ‚ΠΈΡ€ΡƒΡŽΡ‰Π΅Π³ΠΎ Π²Π΅ΠΊΡ‚ΠΎΡ€Π° ΠΈ Π΅ΡΡ‚ΡŒ ΠΏΠ»ΠΎΡ‰Π°Π΄ΡŒ Π΄Π°Π½Π½ΠΎΠ³ΠΎ ΠΏΠ°Ρ€Π°Π»Π»Π΅Π»ΠΎΠ³Ρ€Π°ΠΌΠΌΠ°.

Β Β Β Β Β Β 

Π’Π°ΠΊΠΆΠ΅ ΠΎΠ³Ρ€ΠΎΠΌΠ½ΠΎΠ΅ количСство ΠΏΡ€ΠΈΠΌΠ΅Π½Π΅Π½ΠΈΠΉ сущСствуСт Π² элСктричСствС ΠΈ ΠΌΠ°Π³Π½Π΅Ρ‚ΠΈΠ·ΠΌΠ΅.

Он-Π»Π°ΠΉΠ½ ΠΊΠ°Π»ΡŒΠΊΡƒΠ»ΡΡ‚ΠΎΡ€ Π²Π΅ΠΊΡ‚ΠΎΡ€Π½ΠΎΠ³ΠΎ произвСдСния.

Π§Ρ‚ΠΎΠ±Ρ‹ Π½Π°ΠΉΡ‚ΠΈ скалярноС ΠΏΡ€ΠΎΠΈΠ·Π²Π΅Π΄Π΅Π½ΠΈΠ΅ Π΄Π²ΡƒΡ… Π²Π΅ΠΊΡ‚ΠΎΡ€ΠΎΠ² с ΠΏΠΎΠΌΠΎΡ‰ΡŒΡŽ Π΄Π°Π½Π½ΠΎΠ³ΠΎ ΠΊΠ°Π»ΡŒΠΊΡƒΠ»ΡΡ‚ΠΎΡ€Π°, Π½ΡƒΠΆΠ½ΠΎ ввСсти Π² ΠΏΠ΅Ρ€Π²ΡƒΡŽ строку ΠΏΠΎ порядку ΠΊΠΎΠΎΡ€Π΄ΠΈΠ½Π°Ρ‚Ρ‹ ΠΏΠ΅Ρ€Π²ΠΎΠ³ΠΎ Π²Π΅ΠΊΡ‚ΠΎΡ€Π°, Π²ΠΎ Π²Ρ‚ΠΎΡ€ΡƒΡŽ- Π²Ρ‚ΠΎΡ€ΠΎΠ³ΠΎ. ΠšΠΎΠΎΡ€Π΄ΠΈΠ½Π°Ρ‚Ρ‹ Π²Π΅ΠΊΡ‚ΠΎΡ€ΠΎΠ² ΠΌΠΎΠ³ΡƒΡ‚ Π±Ρ‹Ρ‚ΡŒ вычислСны ΠΏΠΎ ΠΊΠΎΠΎΡ€Π΄ΠΈΠ½Π°Ρ‚Π°ΠΌ ΠΈΡ… Π½Π°Ρ‡Π°Π»Π° ΠΈ ΠΊΠΎΠ½Ρ†Π° (см. ΡΡ‚Π°Ρ‚ΡŒΡŽ БкалярноС ΠΏΡ€ΠΎΠΈΠ·Π²Π΅Π΄Π΅Π½ΠΈΠ΅ Π²Π΅ΠΊΡ‚ΠΎΡ€ΠΎΠ², ΠΏΡƒΠ½ΠΊΡ‚ ΠΠ»ΡŒΡ‚Π΅Ρ€Π½Π°Ρ‚ΠΈΠ²Π½ΠΎΠ΅ ΠΎΠΏΡ€Π΅Π΄Π΅Π»Π΅Π½ΠΈΠ΅ скалярного произвСдСния, ΠΈΠ»ΠΈ вычислСниС скалярного произвСдСния Π΄Π²ΡƒΡ… Π²Π΅ΠΊΡ‚ΠΎΡ€ΠΎΠ², Π·Π°Π΄Π°Π½Π½Ρ‹Ρ… своими ΠΊΠΎΠΎΡ€Π΄ΠΈΠ½Π°Ρ‚Π°ΠΌΠΈ. )

Π§Ρ‚ΠΎΠ±Ρ‹ Π½Π°ΠΉΡ‚ΠΈ скалярноС ΠΏΡ€ΠΎΠΈΠ·Π²Π΅Π΄Π΅Π½ΠΈΠ΅ Π΄Π²ΡƒΡ… Π²Π΅ΠΊΡ‚ΠΎΡ€ΠΎΠ² с ΠΏΠΎΠΌΠΎΡ‰ΡŒΡŽ Π΄Π°Π½Π½ΠΎΠ³ΠΎ ΠΊΠ°Π»ΡŒΠΊΡƒΠ»ΡΡ‚ΠΎΡ€Π°, Π½ΡƒΠΆΠ½ΠΎ ввСсти Π² ΠΏΠ΅Ρ€Π²ΡƒΡŽ строку ΠΏΠΎ порядку ΠΊΠΎΠΎΡ€Π΄ΠΈΠ½Π°Ρ‚Ρ‹ ΠΏΠ΅Ρ€Π²ΠΎΠ³ΠΎ Π²Π΅ΠΊΡ‚ΠΎΡ€Π°, Π²ΠΎ Π²Ρ‚ΠΎΡ€ΡƒΡŽ- Π²Ρ‚ΠΎΡ€ΠΎΠ³ΠΎ. ΠšΠΎΠΎΡ€Π΄ΠΈΠ½Π°Ρ‚Ρ‹ Π²Π΅ΠΊΡ‚ΠΎΡ€ΠΎΠ² ΠΌΠΎΠ³ΡƒΡ‚ Π±Ρ‹Ρ‚ΡŒ вычислСны ΠΏΠΎ ΠΊΠΎΠΎΡ€Π΄ΠΈΠ½Π°Ρ‚Π°ΠΌ ΠΈΡ… Π½Π°Ρ‡Π°Π»Π° ΠΈ ΠΊΠΎΠ½Ρ†Π° (см. ΡΡ‚Π°Ρ‚ΡŒΡŽ БкалярноС ΠΏΡ€ΠΎΠΈΠ·Π²Π΅Π΄Π΅Π½ΠΈΠ΅ Π²Π΅ΠΊΡ‚ΠΎΡ€ΠΎΠ², ΠΏΡƒΠ½ΠΊΡ‚ ΠΠ»ΡŒΡ‚Π΅Ρ€Π½Π°Ρ‚ΠΈΠ²Π½ΠΎΠ΅ ΠΎΠΏΡ€Π΅Π΄Π΅Π»Π΅Π½ΠΈΠ΅ скалярного произвСдСния, ΠΈΠ»ΠΈ вычислСниС скалярного произвСдСния Π΄Π²ΡƒΡ… Π²Π΅ΠΊΡ‚ΠΎΡ€ΠΎΠ², Π·Π°Π΄Π°Π½Π½Ρ‹Ρ… своими ΠΊΠΎΠΎΡ€Π΄ΠΈΠ½Π°Ρ‚Π°ΠΌΠΈ. )

Β 

Π’Π΅ΠΊΡ‚ΠΎΡ€Π½ΠΎΠ΅ ΠΏΡ€ΠΎΠΈΠ·Π²Π΅Π΄Π΅Π½ΠΈΠ΅ Π²Π΅ΠΊΡ‚ΠΎΡ€ΠΎΠ² ΠΈ Π΅Π³ΠΎ свойства с ΠΏΡ€ΠΈΠΌΠ΅Ρ€Π°ΠΌΠΈ Ρ€Π΅ΡˆΠ΅Π½ΠΈΡ ΠΈ ΠΎΠ±Ρ€Π°Π·Ρ†Π°ΠΌΠΈ выполнСния

ΠžΠΏΡ€Π΅Π΄Π΅Π»Π΅Π½ΠΈΠ΅ Π²Π΅ΠΊΡ‚ΠΎΡ€Π½ΠΎΠ³ΠΎ произвСдСния:

Π’Ρ€ΠΈ Π½Π΅ΠΊΠΎΠΌΠΏΠ»Π°Π½Π°Ρ€Π½Ρ‹Ρ… Π²Π΅ΠΊΡ‚ΠΎΡ€Π°

, взятыС Π² ΡƒΠΊΠ°Π·Π°Π½Π½ΠΎΠΌ порядкС, ΠΎΠ±Ρ€Π°Π·ΡƒΡŽΡ‚ ΠΏΡ€Π°Π²ΡƒΡŽ Ρ‚Ρ€ΠΎΠΉΠΊΡƒ, Ссли с ΠΊΠΎΠ½Ρ†Π° Ρ‚Ρ€Π΅Ρ‚ΡŒΠ΅Π³ΠΎ Π²Π΅ΠΊΡ‚ΠΎΡ€Π° с ΠΊΡ€Π°Ρ‚Ρ‡Π°ΠΉΡˆΠΈΠΉ ΠΏΠΎΠ²ΠΎΡ€ΠΎΡ‚ ΠΎΡ‚ ΠΏΠ΅Ρ€Π²ΠΎΠ³ΠΎ Π²Π΅ΠΊΡ‚ΠΎΡ€Π° ΠΊΠΎ Π²Ρ‚ΠΎΡ€ΠΎΠΌΡƒ Π²Π΅ΠΊΡ‚ΠΎΡ€Ρƒ Π²ΠΈΠ΄Π΅Π½ ΡΠΎΠ²Π΅Ρ€ΡˆΠ°ΡŽΡ‰ΠΈΠΌΡΡ ΠΏΡ€ΠΎΡ‚ΠΈΠ² часовой стрСлки, ΠΈ Π»Π΅Π²ΡƒΡŽ, Ссли ΠΏΠΎ часовой

Π’Π΅ΠΊΡ‚ΠΎΡ€Π½Ρ‹ΠΌ ΠΏΡ€ΠΎΠΈΠ·Π²Π΅Π΄Π΅Π½ΠΈΠ΅ΠΌ Π²Π΅ΠΊΡ‚ΠΎΡ€Π°

Π½Π° Π²Π΅ΠΊΡ‚ΠΎΡ€ называСтся Π²Π΅ΠΊΡ‚ΠΎΡ€ , ΠΊΠΎΡ‚ΠΎΡ€Ρ‹ΠΉ:
  1. пСрпСндикулярСн Π²Π΅ΠΊΡ‚ΠΎΡ€Π°ΠΌ
  2. ΠΈΠΌΠ΅Π΅Ρ‚ Π΄Π»ΠΈΠ½Ρƒ, числСнно Ρ€Π°Π²Π½ΡƒΡŽ ΠΏΠ»ΠΎΡ‰Π°Π΄ΠΈ ΠΏΠ°Ρ€Π°Π»Π»Π΅Π»ΠΎΠ³Ρ€Π°ΠΌΠΌΠ°, построСнного Π½Π° Π²Π΅ΠΊΡ‚ΠΎΡ€Π°Ρ… ΠΊΠ°ΠΊ Π½Π° сторонах (см. рис. 17), Ρ‚. Π΅.
  3. Π²Π΅ΠΊΡ‚ΠΎΡ€Ρ‹ ΠΎΠ±Ρ€Π°Π·ΡƒΡŽΡ‚ ΠΏΡ€Π°Π²ΡƒΡŽ Ρ‚Ρ€ΠΎΠΉΠΊΡƒ.

Π’Π΅ΠΊΡ‚ΠΎΡ€Π½ΠΎΠ΅ ΠΏΡ€ΠΎΠΈΠ·Π²Π΅Π΄Π΅Π½ΠΈΠ΅ обозначаСтся

ΠΈΠ»ΠΈ . Из опрСдСлСния Π²Π΅ΠΊΡ‚ΠΎΡ€Π½ΠΎΠ³ΠΎ произвСдСния нСпосрСдствСнно Π²Ρ‹Ρ‚Π΅ΠΊΠ°ΡŽΡ‚ ΡΠ»Π΅Π΄ΡƒΡŽΡ‰ΠΈΠ΅ ΡΠΎΠΎΡ‚Π½ΠΎΡˆΠ΅Π½ΠΈΡ ΠΌΠ΅ΠΆΠ΄Ρƒ ΠΎΡ€Ρ‚Π°ΠΌΠΈ i, j ΠΈ k (см. рис. 18):

Π”ΠΎΠΊΠ°ΠΆΠ΅ΠΌ, Π½Π°ΠΏΡ€ΠΈΠΌΠ΅Ρ€, Ρ‡Ρ‚ΠΎ

.
  1. Π²Π΅ΠΊΡ‚ΠΎΡ€Ρ‹ i, j ΠΈ k ΠΎΠ±Ρ€Π°Π·ΡƒΡŽΡ‚ ΠΏΡ€Π°Π²ΡƒΡŽ Ρ‚Ρ€ΠΎΠΉΠΊΡƒ (см. рис. 16).

Бвойства Π²Π΅ΠΊΡ‚ΠΎΡ€Π½ΠΎΠ³ΠΎ произвСдСния

1.ΠŸΡ€ΠΈ пСрСстановкС сомноТитСлСй

Π²Π΅ΠΊΡ‚ΠΎΡ€Π½ΠΎΠ΅ ΠΏΡ€ΠΎΠΈΠ·Π²Π΅Π΄Π΅Π½ΠΈΠ΅ мСняСт Π·Π½Π°ΠΊ, Ρ‚. Π΅. (см. рис. 19).

Π’Π΅ΠΊΡ‚ΠΎΡ€Ρ‹

ΠΊΠΎΠ»Π»ΠΈΠ½Π΅Π°Ρ€Π½Ρ‹, ΠΈΠΌΠ΅ΡŽΡ‚ ΠΎΠ΄ΠΈΠ½Π°ΠΊΠΎΠ²Ρ‹Π΅ ΠΌΠΎΠ΄ΡƒΠ»ΠΈ (ΠΏΠ»ΠΎΡ‰Π°Π΄ΡŒ ΠΏΠ°Ρ€Π°Π»Π»Π΅Π»ΠΎΠ³Ρ€Π°ΠΌΠΌΠ° остаСтся Π½Π΅ΠΈΠ·ΠΌΠ΅Π½Π½ΠΎΠΉ), Π½ΠΎ ΠΏΡ€ΠΎΡ‚ΠΈΠ²ΠΎΠΏΠΎΠ»ΠΎΠΆΠ½ΠΎ Π½Π°ΠΏΡ€Π°Π²Π»Π΅Π½Ρ‹ (Ρ‚Ρ€ΠΎΠΉΠΊΠΈ ΠΏΡ€ΠΎΡ‚ΠΈΠ²ΠΎΠΏΠΎΠ»ΠΎΠΆΠ½ΠΎΠΉ ΠΎΡ€ΠΈΠ΅Π½Ρ‚Π°Ρ†ΠΈΠΈ). Π‘Ρ‚Π°Π»ΠΎ Π±Ρ‹Ρ‚ΡŒ, .

2.Π’Π΅ΠΊΡ‚ΠΎΡ€Π½ΠΎΠ΅ ΠΏΡ€ΠΎΠΈΠ·Π²Π΅Π΄Π΅Π½ΠΈΠ΅ ΠΎΠ±Π»Π°Π΄Π°Π΅Ρ‚ ΡΠΎΡ‡Π΅Ρ‚Π°Ρ‚Π΅Π»ΡŒΠ½Ρ‹ΠΌ свойством ΠΎΡ‚Π½ΠΎΡΠΈΡ‚Π΅Π»ΡŒΠ½ΠΎ скалярного мноТитСля, Ρ‚. Π΅.

ΠŸΡƒΡΡ‚ΡŒ

. Π’Π΅ΠΊΡ‚ΠΎΡ€ пСрпСндикулярСн Π²Π΅ΠΊΡ‚ΠΎΡ€Π°ΠΌ . Π’Π΅ΠΊΡ‚ΠΎΡ€ Ρ‚Π°ΠΊΠΆΠ΅ пСрпСндикулярСн Π²Π΅ΠΊΡ‚ΠΎΡ€Π°ΠΌ (Π²Π΅ΠΊΡ‚ΠΎΡ€Ρ‹ Π»Π΅ΠΆΠ°Ρ‚ Π² ΠΎΠ΄Π½ΠΎΠΉ плоскости). Π—Π½Π°Ρ‡ΠΈΡ‚, Π²Π΅ΠΊΡ‚ΠΎΡ€Ρ‹ ΠΈ ΠΊΠΎΠ»Π»ΠΈΠ½Π΅Π°Ρ€Π½Ρ‹. ΠžΡ‡Π΅Π²ΠΈΠ΄Π½ΠΎ, Ρ‡Ρ‚ΠΎ ΠΈ направлСния ΠΈΡ… ΡΠΎΠ²ΠΏΠ°Π΄Π°ΡŽΡ‚. Π˜ΠΌΠ΅ΡŽΡ‚ ΠΎΠ΄ΠΈΠ½Π°ΠΊΠΎΠ²ΡƒΡŽ Π΄Π»ΠΈΠ½Ρƒ:

ΠΈ

ΠŸΠΎΡΡ‚ΠΎΠΌΡƒ

Аналогично доказываСтся ΠΏΡ€ΠΈ

3. Π”Π²Π° Π½Π΅Π½ΡƒΠ»Π΅Π²Ρ‹Ρ… Π²Π΅ΠΊΡ‚ΠΎΡ€Π°

ΠΊΠΎΠ»Π»ΠΈΠ½Π΅Π°Ρ€Π½Ρ‹ Ρ‚ΠΎΠ³Π΄Π° ΠΈ Ρ‚ΠΎΠ»ΡŒΠΊΠΎ Ρ‚ΠΎΠ³Π΄Π°, ΠΊΠΎΠ³Π΄Π° ΠΈΡ… Π²Π΅ΠΊΡ‚ΠΎΡ€Π½ΠΎΠ΅ ΠΏΡ€ΠΎΠΈΠ·Π²Π΅Π΄Π΅Π½ΠΈΠ΅ Ρ€Π°Π²Π½ΠΎ Π½ΡƒΠ»Π΅Π²ΠΎΠΌΡƒ Π²Π΅ΠΊΡ‚ΠΎΡ€Ρƒ, Ρ‚. Π΅.

Если

, Ρ‚ΠΎ ΡƒΠ³ΠΎΠ» ΠΌΠ΅ΠΆΠ΄Ρƒ Π½ΠΈΠΌΠΈ Ρ€Π°Π²Π΅Π½ 0Β° ΠΈΠ»ΠΈ 180Β°. Но Ρ‚ΠΎΠ³Π΄Π° Π—Π½Π°Ρ‡ΠΈΡ‚,

Если ΠΆΠ΅

, Ρ‚ΠΎ . Но Ρ‚ΠΎΠ³Π΄Π° , Ρ‚. Π΅. .

Π’ частности,

4. Π’Π΅ΠΊΡ‚ΠΎΡ€Π½ΠΎΠ΅ ΠΏΡ€ΠΎΠΈΠ·Π²Π΅Π΄Π΅Π½ΠΈΠ΅ ΠΎΠ±Π»Π°Π΄Π°Π΅Ρ‚ Ρ€Π°ΡΠΏΡ€Π΅Π΄Π΅Π»ΠΈΡ‚Π΅Π»ΡŒΠ½Ρ‹ΠΌ свойством:

ΠŸΡ€ΠΈΠΌΠ΅ΠΌ Π±Π΅Π· Π΄ΠΎΠΊΠ°Π·Π°Ρ‚Π΅Π»ΡŒΡΡ‚Π²Π°.

Π’Ρ‹Ρ€Π°ΠΆΠ΅Π½ΠΈΠ΅ Π²Π΅ΠΊΡ‚ΠΎΡ€Π½ΠΎΠ³ΠΎ произвСдСния Ρ‡Π΅Ρ€Π΅Π· ΠΊΠΎΠΎΡ€Π΄ΠΈΠ½Π°Ρ‚Ρ‹

ΠœΡ‹ Π±ΡƒΠ΄Π΅ΠΌ ΠΈΡΠΏΠΎΠ»ΡŒΠ·ΠΎΠ²Π°Ρ‚ΡŒ Ρ‚Π°Π±Π»ΠΈΡ†Ρƒ Π²Π΅ΠΊΡ‚ΠΎΡ€Π½ΠΎΠ³ΠΎ произвСдСния Π²Π΅ΠΊΡ‚ΠΎΡ€ΠΎΠ² i, j ΠΈ k:

Ссли Π½Π°ΠΏΡ€Π°Π²Π»Π΅Π½ΠΈΠ΅ ΠΊΡ€Π°Ρ‚Ρ‡Π°ΠΉΡˆΠ΅Π³ΠΎ ΠΏΡƒΡ‚ΠΈ ΠΎΡ‚ ΠΏΠ΅Ρ€Π²ΠΎΠ³ΠΎ Π²Π΅ΠΊΡ‚ΠΎΡ€Π° ΠΊ Π²Ρ‚ΠΎΡ€ΠΎΠΌΡƒ совпадаСт с Π½Π°ΠΏΡ€Π°Π²Π»Π΅Π½ΠΈΠ΅ΠΌ стрСлки, Ρ‚ΠΎ ΠΏΡ€ΠΎΠΈΠ·Π²Π΅Π΄Π΅Π½ΠΈΠ΅ Ρ€Π°Π²Π½ΠΎ Ρ‚Ρ€Π΅Ρ‚ΡŒΠ΅ΠΌΡƒ Π²Π΅ΠΊΡ‚ΠΎΡ€Ρƒ, Ссли Π½Π΅ совпадаСт β€” Ρ‚Ρ€Π΅Ρ‚ΠΈΠΉ Π²Π΅ΠΊΡ‚ΠΎΡ€ бСрСтся со Π·Π½Π°ΠΊΠΎΠΌ «минус».

ΠŸΡƒΡΡ‚ΡŒ Π·Π°Π΄Π°Π½Ρ‹ Π΄Π²Π° Π²Π΅ΠΊΡ‚ΠΎΡ€Π°

НайдСм Π²Π΅ΠΊΡ‚ΠΎΡ€Π½ΠΎΠ΅ ΠΏΡ€ΠΎΠΈΠ·Π²Π΅Π΄Π΅Π½ΠΈΠ΅ этих Π²Π΅ΠΊΡ‚ΠΎΡ€ΠΎΠ², пСрСмноТая ΠΈΡ… ΠΊΠ°ΠΊ ΠΌΠ½ΠΎΠ³ΠΎΡ‡Π»Π΅Π½Ρ‹ (согласно свойств Π²Π΅ΠΊΡ‚ΠΎΡ€Π½ΠΎΠ³ΠΎ произвСдСния):

Ρ‚.Π΅.

ΠŸΠΎΠ»ΡƒΡ‡Π΅Π½Π½ΡƒΡŽ Ρ„ΠΎΡ€ΠΌΡƒΠ»Ρƒ ΠΌΠΎΠΆΠ½ΠΎ Π·Π°ΠΏΠΈΡΠ°Ρ‚ΡŒ Π΅Ρ‰Π΅ ΠΊΠΎΡ€ΠΎΡ‡Π΅:

Ρ‚Π°ΠΊ ΠΊΠ°ΠΊ правая Ρ‡Π°ΡΡ‚ΡŒ равСнства (7.1) соотвСтствуСт Ρ€Π°Π·Π»ΠΎΠΆΠ΅Π½ΠΈΡŽ опрСдСлитСля Ρ‚Ρ€Π΅Ρ‚ΡŒΠ΅Π³ΠΎ порядка ΠΏΠΎ элСмСнтам ΠΏΠ΅Ρ€Π²ΠΎΠΉ строки. РавСнство (7.2) Π»Π΅Π³ΠΊΠΎ запоминаСтся.

НСкоторыС прилоТСния Π²Π΅ΠΊΡ‚ΠΎΡ€Π½ΠΎΠ³ΠΎ произвСдСния
УстановлСниС коллинСарности Π²Π΅ΠΊΡ‚ΠΎΡ€ΠΎΠ²

Если

, Ρ‚ΠΎ (ΠΈ Π½Π°ΠΎΠ±ΠΎΡ€ΠΎΡ‚), Ρ‚. Π΅.

НахоТдСниС ΠΏΠ»ΠΎΡ‰Π°Π΄ΠΈ ΠΏΠ°Ρ€Π°Π»Π»Π΅Π»ΠΎΠ³Ρ€Π°ΠΌΠΌΠ° ΠΈ Ρ‚Ρ€Π΅ΡƒΠ³ΠΎΠ»ΡŒΠ½ΠΈΠΊΠ°

Богласно ΠΎΠΏΡ€Π΅Π΄Π΅Π»Π΅Π½ΠΈΡŽ Π²Π΅ΠΊΡ‚ΠΎΡ€Π½ΠΎΠ³ΠΎ произвСдСния Π²Π΅ΠΊΡ‚ΠΎΡ€ΠΎΠ²

И, Π·Π½Π°Ρ‡ΠΈΡ‚,

ΠžΠΏΡ€Π΅Π΄Π΅Π»Π΅Π½ΠΈΠ΅ ΠΌΠΎΠΌΠ΅Π½Ρ‚Π° силы ΠΎΡ‚Π½ΠΎΡΠΈΡ‚Π΅Π»ΡŒΠ½ΠΎ Ρ‚ΠΎΡ‡ΠΊΠΈ

ΠŸΡƒΡΡ‚ΡŒ Π² Ρ‚ΠΎΡ‡ΠΊΠ΅ А ΠΏΡ€ΠΈΠ»ΠΎΠΆΠ΅Π½Π° сила

ΠΈ ΠΏΡƒΡΡ‚ΡŒ О β€” нСкоторая Ρ‚ΠΎΡ‡ΠΊΠ° пространства (см. рис. 20).

Из Ρ„ΠΈΠ·ΠΈΠΊΠΈ извСстно, Ρ‡Ρ‚ΠΎ ΠΌΠΎΠΌΠ΅Π½Ρ‚ΠΎΠΌ силы

ΠΎΡ‚Π½ΠΎΡΠΈΡ‚Π΅Π»ΡŒΠ½ΠΎ Ρ‚ΠΎΡ‡ΠΊΠΈ О называСтся Π²Π΅ΠΊΡ‚ΠΎΡ€ , ΠΊΠΎΡ‚ΠΎΡ€Ρ‹ΠΉ ΠΏΡ€ΠΎΡ…ΠΎΠ΄ΠΈΡ‚ Ρ‡Π΅Ρ€Π΅Π· Ρ‚ΠΎΡ‡ΠΊΡƒ О ΠΈ:

1) пСрпСндикулярСн плоскости, проходящСй Ρ‡Π΅Ρ€Π΅Π· Ρ‚ΠΎΡ‡ΠΊΠΈ О, А, Π’;

2) числСнно Ρ€Π°Π²Π΅Π½ ΠΏΡ€ΠΎΠΈΠ·Π²Π΅Π΄Π΅Π½ΠΈΡŽ силы Π½Π° ΠΏΠ»Π΅Ρ‡ΠΎ

3) ΠΎΠ±Ρ€Π°Π·ΡƒΠ΅Ρ‚ ΠΏΡ€Π°Π²ΡƒΡŽ Ρ‚Ρ€ΠΎΠΉΠΊΡƒ с Π²Π΅ΠΊΡ‚ΠΎΡ€Π°ΠΌΠΈ

. Π‘Ρ‚Π°Π»ΠΎ Π±Ρ‹Ρ‚ΡŒ,

НахоТдСниС Π»ΠΈΠ½Π΅ΠΉΠ½ΠΎΠΉ скорости вращСния

Π‘ΠΊΠΎΡ€ΠΎΡΡ‚ΡŒ

Ρ‚ΠΎΡ‡ΠΊΠΈ М Ρ‚Π²Π΅Ρ€Π΄ΠΎΠ³ΠΎ Ρ‚Π΅Π»Π°, Π²Ρ€Π°Ρ‰Π°ΡŽΡ‰Π΅Π³ΠΎΡΡ с ΡƒΠ³Π»ΠΎΠ²ΠΎΠΉ ΡΠΊΠΎΡ€ΠΎΡΡ‚ΡŒΡŽ Π²ΠΎΠΊΡ€ΡƒΠ³ Π½Π΅ΠΏΠΎΠ΄Π²ΠΈΠΆΠ½ΠΎΠΉ оси, опрСдСляСтся Ρ„ΠΎΡ€ΠΌΡƒΠ»ΠΎΠΉ Π­ΠΉΠ»Π΅Ρ€Π° , Π³Π΄Π΅ , Π³Π΄Π΅ О β€” нСкоторая нСподвиТная Ρ‚ΠΎΡ‡ΠΊΠ° оси (см. рис. 21).

Π’Π΅ΠΊΡ‚ΠΎΡ€Π½ΠΎΠ΅ ΠΏΡ€ΠΎΠΈΠ·Π²Π΅Π΄Π΅Π½ΠΈΠ΅

Π‘ΠΌΠΎΡ‚Ρ€ΠΈΡ‚Π΅ Ρ‚Π°ΠΊΠΆΠ΅:

ΠŸΡ€Π΅Π΄ΠΌΠ΅Ρ‚ Π²Ρ‹ΡΡˆΠ°Ρ ΠΌΠ°Ρ‚Π΅ΠΌΠ°Ρ‚ΠΈΠΊΠ°

РСшСниС Π·Π°Π΄Π°Π½ΠΈΠΉ ΠΈ Π·Π°Π΄Π°Ρ‡ ΠΏΠΎ ΠΏΡ€Π΅Π΄ΠΌΠ΅Ρ‚Π°ΠΌ:

Π”ΠΎΠΏΠΎΠ»Π½ΠΈΡ‚Π΅Π»ΡŒΠ½Ρ‹Π΅ Π»Π΅ΠΊΡ†ΠΈΠΈ ΠΏΠΎ Π²Ρ‹ΡΡˆΠ΅ΠΉ ΠΌΠ°Ρ‚Π΅ΠΌΠ°Ρ‚ΠΈΠΊΠ΅:

  1. ВоТдСствСнныС прСобразования алгСбраичСских Π²Ρ‹Ρ€Π°ΠΆΠ΅Π½ΠΈΠΉ
  2. Π€ΡƒΠ½ΠΊΡ†ΠΈΠΈ ΠΈ Π³Ρ€Π°Ρ„ΠΈΠΊΠΈ
  3. ΠŸΡ€Π΅ΠΎΠ±Ρ€Π°Π·ΠΎΠ²Π°Π½ΠΈΡ Π³Ρ€Π°Ρ„ΠΈΠΊΠΎΠ² Ρ„ΡƒΠ½ΠΊΡ†ΠΈΠΉ
  4. ΠšΠ²Π°Π΄Ρ€Π°Ρ‚Π½Π°Ρ функция ΠΈ Π΅Ρ‘ Π³Ρ€Π°Ρ„ΠΈΠΊΠΈ
  5. АлгСбраичСскиС нСравСнства
  6. НСравСнства
  7. НСравСнства с ΠΏΠ΅Ρ€Π΅ΠΌΠ΅Π½Π½Ρ‹ΠΌΠΈ
  8. ΠŸΡ€ΠΎΠ³Ρ€Π΅ΡΡΠΈΠΈ Π² ΠΌΠ°Ρ‚Π΅ΠΌΠ°Ρ‚ΠΈΠΊΠ΅
  9. АрифмСтичСская прогрСссия
  10. ГСомСтричСская прогрСссия
  11. ΠŸΠΎΠΊΠ°Π·Π°Ρ‚Π΅Π»ΠΈ Π² ΠΌΠ°Ρ‚Π΅ΠΌΠ°Ρ‚ΠΈΠΊΠ΅
  12. Π›ΠΎΠ³Π°Ρ€ΠΈΡ„ΠΌΡ‹ Π² ΠΌΠ°Ρ‚Π΅ΠΌΠ°Ρ‚ΠΈΠΊΠ΅
  13. ИсслСдованиС ΡƒΡ€Π°Π²Π½Π΅Π½ΠΈΠΉ
  14. УравнСния Π²Ρ‹ΡΡˆΠΈΡ… стСпСнСй
  15. УравнСния Π²Ρ‹ΡΡˆΠΈΡ… стСпСнСй с ΠΎΠ΄Π½ΠΈΠΌ нСизвСстным
  16. ΠšΠΎΠΌΠΏΠ»Π΅ΠΊΡΠ½Ρ‹Π΅ числа
  17. НСпрСрывная Π΄Ρ€ΠΎΠ±ΡŒ (цСпная Π΄Ρ€ΠΎΠ±ΡŒ)
  18. АлгСбраичСскиС уравнСния
  19. НСопрСдСлСнныС уравнСния
  20. БоСдинСния
  21. Π‘ΠΈΠ½ΠΎΠΌ ΠΡŒΡŽΡ‚ΠΎΠ½Π°
  22. Число С
  23. НСпрСрывныС Π΄Ρ€ΠΎΠ±ΠΈ
  24. Ѐункция
  25. ИсслСдованиС Ρ„ΡƒΠ½ΠΊΡ†ΠΈΠΉ
  26. ΠŸΡ€Π΅Π΄Π΅Π»
  27. Π˜Π½Ρ‚Π΅Π³Ρ€Π°Π»
  28. Π”Π²ΠΎΠΉΠ½ΠΎΠΉ ΠΈΠ½Ρ‚Π΅Π³Ρ€Π°Π»
  29. Π’Ρ€ΠΎΠΉΠ½ΠΎΠΉ ΠΈΠ½Ρ‚Π΅Π³Ρ€Π°Π»
  30. Π˜Π½Ρ‚Π΅Π³Ρ€ΠΈΡ€ΠΎΠ²Π°Π½ΠΈΠ΅
  31. НСопрСдСлённый ΠΈΠ½Ρ‚Π΅Π³Ρ€Π°Π»
  32. ΠžΠΏΡ€Π΅Π΄Π΅Π»Π΅Π½Π½Ρ‹ΠΉ ΠΈΠ½Ρ‚Π΅Π³Ρ€Π°Π»
  33. ΠšΡ€ΠΈΠ²ΠΎΠ»ΠΈΠ½Π΅ΠΉΠ½Ρ‹Π΅ ΠΈΠ½Ρ‚Π΅Π³Ρ€Π°Π»Ρ‹
  34. ΠŸΠΎΠ²Π΅Ρ€Ρ…Π½ΠΎΡΡ‚Π½Ρ‹Π΅ ΠΈΠ½Ρ‚Π΅Π³Ρ€Π°Π»Ρ‹
  35. НСсобствСнныС ΠΈΠ½Ρ‚Π΅Π³Ρ€Π°Π»Ρ‹
  36. ΠšΡ€Π°Ρ‚Π½Ρ‹Π΅ ΠΈΠ½Ρ‚Π΅Π³Ρ€Π°Π»Ρ‹
  37. Π˜Π½Ρ‚Π΅Π³Ρ€Π°Π»Ρ‹, зависящиС ΠΎΡ‚ ΠΏΠ°Ρ€Π°ΠΌΠ΅Ρ‚Ρ€Π°
  38. ΠšΠ²Π°Π΄Ρ€Π°Ρ‚Π½Ρ‹ΠΉ Ρ‚Ρ€Π΅Ρ…Ρ‡Π»Π΅Π½
  39. ΠŸΡ€ΠΎΠΈΠ·Π²ΠΎΠ΄Π½Π°Ρ
  40. ΠŸΡ€ΠΈΠΌΠ΅Π½Π΅Π½ΠΈΠ΅ ΠΏΡ€ΠΎΠΈΠ·Π²ΠΎΠ΄Π½ΠΎΠΉ ΠΊ исслСдованию Ρ„ΡƒΠ½ΠΊΡ†ΠΈΠΉ
  41. ΠŸΡ€ΠΈΠ»ΠΎΠΆΠ΅Π½ΠΈΡ ΠΏΡ€ΠΎΠΈΠ·Π²ΠΎΠ΄Π½ΠΎΠΉ
  42. Π”ΠΈΡ„Ρ„Π΅Ρ€Π΅Π½Ρ†ΠΈΠ°Π» Ρ„ΡƒΠ½ΠΊΡ†ΠΈΠΈ
  43. Π”ΠΈΡ„Ρ„Π΅Ρ€Π΅Π½Ρ†ΠΈΡ€ΠΎΠ²Π°Π½ΠΈΠ΅ Π² ΠΌΠ°Ρ‚Π΅ΠΌΠ°Ρ‚ΠΈΠΊΠ΅
  44. Π€ΠΎΡ€ΠΌΡƒΠ»Ρ‹ ΠΈ ΠΏΡ€Π°Π²ΠΈΠ»Π° диффСрСнцирования
  45. Π”ΠΈΡ„Ρ„Π΅Ρ€Π΅Π½Ρ†ΠΈΠ°Π»ΡŒΠ½ΠΎΠ΅ исчислСниС
  46. Π”ΠΈΡ„Ρ„Π΅Ρ€Π΅Π½Ρ†ΠΈΠ°Π»ΡŒΠ½Ρ‹Π΅ уравнСния
  47. Π”ΠΈΡ„Ρ„Π΅Ρ€Π΅Π½Ρ†ΠΈΠ°Π»ΡŒΠ½Ρ‹Π΅ уравнСния ΠΏΠ΅Ρ€Π²ΠΎΠ³ΠΎ порядка
  48. Π”ΠΈΡ„Ρ„Π΅Ρ€Π΅Π½Ρ†ΠΈΠ°Π»ΡŒΠ½Ρ‹Π΅ уравнСния Π²Ρ‹ΡΡˆΠΈΡ… порядков
  49. Π”ΠΈΡ„Ρ„Π΅Ρ€Π΅Π½Ρ†ΠΈΠ°Π»ΡŒΠ½Ρ‹Π΅ уравнСния Π² частных ΠΏΡ€ΠΎΠΈΠ·Π²ΠΎΠ΄Π½Ρ‹Ρ…
  50. ВригономСтричСскиС Ρ„ΡƒΠ½ΠΊΡ†ΠΈΠΈ
  51. ВригономСтричСскиС уравнСния ΠΈ нСравСнства
  52. ΠŸΠΎΠΊΠ°Π·Π°Ρ‚Π΅Π»ΡŒΠ½Π°Ρ функция
  53. ΠŸΠΎΠΊΠ°Π·Π°Ρ‚Π΅Π»ΡŒΠ½Ρ‹Π΅ уравнСния
  54. ΠžΠ±ΠΎΠ±Ρ‰Π΅Π½Π½Π°Ρ ΡΡ‚Π΅ΠΏΠ΅Π½ΡŒ
  55. Π’Π·Π°ΠΈΠΌΠ½ΠΎ ΠΎΠ±Ρ€Π°Ρ‚Π½Ρ‹Π΅ Ρ„ΡƒΠ½ΠΊΡ†ΠΈΠΈ
  56. ЛогарифмичСская функция
  57. УравнСния ΠΈ нСравСнства
  58. ΠŸΠΎΠ»ΠΎΠΆΠΈΡ‚Π΅Π»ΡŒΠ½Ρ‹Π΅ ΠΈ ΠΎΡ‚Ρ€ΠΈΡ†Π°Ρ‚Π΅Π»ΡŒΠ½Ρ‹Π΅ числа
  59. АлгСбраичСскиС выраТСния
  60. Π˜Ρ€Ρ€Π°Ρ†ΠΈΠΎΠ½Π°Π»ΡŒΠ½Ρ‹Π΅ алгСбраичСскиС выраТСния
  61. ΠŸΡ€Π΅ΠΎΠ±Ρ€Π°Π·ΠΎΠ²Π°Π½ΠΈΠ΅ алгСбраичСских Π²Ρ‹Ρ€Π°ΠΆΠ΅Π½ΠΈΠΉ
  62. ΠŸΡ€Π΅ΠΎΠ±Ρ€Π°Π·ΠΎΠ²Π°Π½ΠΈΠ΅ Π΄Ρ€ΠΎΠ±Π½Ρ‹Ρ… алгСбраичСских Π²Ρ‹Ρ€Π°ΠΆΠ΅Π½ΠΈΠΉ
  63. Π Π°Π·Π»ΠΎΠΆΠ΅Π½ΠΈΠ΅ ΠΌΠ½ΠΎΠ³ΠΎΡ‡Π»Π΅Π½ΠΎΠ² Π½Π° ΠΌΠ½ΠΎΠΆΠΈΡ‚Π΅Π»ΠΈ
  64. ΠœΠ½ΠΎΠ³ΠΎΡ‡Π»Π΅Π½Ρ‹ ΠΎΡ‚ ΠΎΠ΄Π½ΠΎΠ³ΠΎ ΠΏΠ΅Ρ€Π΅ΠΌΠ΅Π½Π½ΠΎΠ³ΠΎ
  65. АлгСбраичСскиС Π΄Ρ€ΠΎΠ±ΠΈ
  66. ΠŸΡ€ΠΎΠΏΠΎΡ€Ρ†ΠΈΠΈ
  67. УравнСния
  68. БистСмы ΡƒΡ€Π°Π²Π½Π΅Π½ΠΈΠΉ
  69. БистСмы ΡƒΡ€Π°Π²Π½Π΅Π½ΠΈΠΉ Π²Ρ‹ΡΡˆΠΈΡ… стСпСнСй
  70. БистСмы алгСбраичСских ΡƒΡ€Π°Π²Π½Π΅Π½ΠΈΠΉ
  71. БистСмы Π»ΠΈΠ½Π΅ΠΉΠ½Ρ‹Ρ… ΡƒΡ€Π°Π²Π½Π΅Π½ΠΈΠΉ
  72. БистСмы Π΄ΠΈΡ„Ρ„Π΅Ρ€Π΅Π½Ρ†ΠΈΠ°Π»ΡŒΠ½Ρ‹Ρ… ΡƒΡ€Π°Π²Π½Π΅Π½ΠΈΠΉ
  73. АрифмСтичСский ΠΊΠ²Π°Π΄Ρ€Π°Ρ‚Π½Ρ‹ΠΉ ΠΊΠΎΡ€Π΅Π½ΡŒ
  74. ΠšΠ²Π°Π΄Ρ€Π°Ρ‚Π½Ρ‹Π΅ ΠΈ кубичСскиС ΠΊΠΎΡ€Π½ΠΈ
  75. Π˜Π·Π²Π»Π΅Ρ‡Π΅Π½ΠΈΠ΅ ΠΊΠ²Π°Π΄Ρ€Π°Ρ‚Π½ΠΎΠ³ΠΎ корня
  76. Π Π°Ρ†ΠΈΠΎΠ½Π°Π»ΡŒΠ½Ρ‹Π΅ числа
  77. Π˜Ρ€Ρ€Π°Ρ†ΠΈΠΎΠ½Π°Π»ΡŒΠ½Ρ‹Π΅ числа
  78. АрифмСтичСский ΠΊΠΎΡ€Π΅Π½ΡŒ
  79. ΠšΠ²Π°Π΄Ρ€Π°Ρ‚Π½Ρ‹Π΅ уравнСния
  80. Π˜Ρ€Ρ€Π°Ρ†ΠΈΠΎΠ½Π°Π»ΡŒΠ½Ρ‹Π΅ уравнСния
  81. ΠŸΠΎΡΠ»Π΅Π΄ΠΎΠ²Π°Ρ‚Π΅Π»ΡŒΠ½ΠΎΡΡ‚ΡŒ
  82. Ряды сходящиСся ΠΈ расходящиСся
  83. ВригономСтричСскиС Ρ„ΡƒΠ½ΠΊΡ†ΠΈΠΈ ΠΏΡ€ΠΎΠΈΠ·Π²ΠΎΠ»ΡŒΠ½ΠΎΠ³ΠΎ ΡƒΠ³Π»Π°
  84. ВригономСтричСскиС Ρ„ΠΎΡ€ΠΌΡƒΠ»Ρ‹
  85. ΠžΠ±Ρ€Π°Ρ‚Π½Ρ‹Π΅ тригономСтричСскиС Ρ„ΡƒΠ½ΠΊΡ†ΠΈΠΈ
  86. Π’Π΅ΠΎΡ€Π΅ΠΌΠ° Π‘Π΅Π·Ρƒ
  87. ΠœΠ°Ρ‚Π΅ΠΌΠ°Ρ‚ΠΈΡ‡Π΅ΡΠΊΠ°Ρ индукция
  88. ΠŸΠΎΠΊΠ°Π·Π°Ρ‚Π΅Π»ΡŒ стСпСни
  89. ΠŸΠΎΠΊΠ°Π·Π°Ρ‚Π΅Π»ΡŒΠ½Ρ‹Π΅ Ρ„ΡƒΠ½ΠΊΡ†ΠΈΠΈ ΠΈ Π»ΠΎΠ³Π°Ρ€ΠΈΡ„ΠΌΡ‹
  90. ΠœΠ½ΠΎΠΆΠ΅ΡΡ‚Π²ΠΎ
  91. ΠœΠ½ΠΎΠΆΠ΅ΡΡ‚Π²ΠΎ Π΄Π΅ΠΉΡΡ‚Π²ΠΈΡ‚Π΅Π»ΡŒΠ½Ρ‹Ρ… чисСл
  92. ЧисловыС мноТСства
  93. ΠŸΡ€Π΅ΠΎΠ±Ρ€Π°Π·ΠΎΠ²Π°Π½ΠΈΠ΅ Ρ€Π°Ρ†ΠΈΠΎΠ½Π°Π»ΡŒΠ½Ρ‹Ρ… Π²Ρ‹Ρ€Π°ΠΆΠ΅Π½ΠΈΠΉ
  94. ΠŸΡ€Π΅ΠΎΠ±Ρ€Π°Π·ΠΎΠ²Π°Π½ΠΈΠ΅ ΠΈΡ€Ρ€Π°Ρ†ΠΈΠΎΠ½Π°Π»ΡŒΠ½Ρ‹Ρ… Π²Ρ‹Ρ€Π°ΠΆΠ΅Π½ΠΈΠΉ
  95. ГСомСтрия
  96. Π”Π΅ΠΉΡΡ‚Π²ΠΈΡ‚Π΅Π»ΡŒΠ½Ρ‹Π΅ числа
  97. Π‘Ρ‚Π΅ΠΏΠ΅Π½ΠΈ ΠΈ ΠΊΠΎΡ€Π½ΠΈ
  98. Π‘Ρ‚Π΅ΠΏΠ΅Π½ΡŒ с Ρ€Π°Ρ†ΠΈΠΎΠ½Π°Π»ΡŒΠ½Ρ‹ΠΌ ΠΏΠΎΠΊΠ°Π·Π°Ρ‚Π΅Π»Π΅ΠΌ
  99. ВригономСтричСскиС Ρ„ΡƒΠ½ΠΊΡ†ΠΈΠΈ ΡƒΠ³Π»Π°
  100. ВригономСтричСскиС Ρ„ΡƒΠ½ΠΊΡ†ΠΈΠΈ числового Π°Ρ€Π³ΡƒΠΌΠ΅Π½Ρ‚Π°
  101. ВригономСтричСскиС выраТСния ΠΈ ΠΈΡ… прСобразования
  102. ΠŸΡ€Π΅ΠΎΠ±Ρ€Π°Π·ΠΎΠ²Π°Π½ΠΈΠ΅ тригономСтричСских Π²Ρ‹Ρ€Π°ΠΆΠ΅Π½ΠΈΠΉ
  103. ΠšΠΎΠΌΠ±ΠΈΠ½Π°Ρ‚ΠΎΡ€ΠΈΠΊΠ°
  104. Π’Ρ‹Ρ‡ΠΈΡΠ»ΠΈΡ‚Π΅Π»ΡŒΠ½Π°Ρ ΠΌΠ°Ρ‚Π΅ΠΌΠ°Ρ‚ΠΈΠΊΠ°
  105. ΠŸΡ€ΡΠΌΠ°Ρ линия Π½Π° плоскости ΠΈ Π΅Π΅ уравнСния
  106. ΠŸΡ€ΡΠΌΠ°Ρ ΠΈ ΠΏΠ»ΠΎΡΠΊΠΎΡΡ‚ΡŒ
  107. Π›ΠΈΠ½ΠΈΠΈ ΠΈ уравнСния
  108. ΠŸΡ€ΡΠΌΠ°Ρ линия
  109. УравнСния прямой ΠΈ плоскости Π² пространствС
  110. ΠšΡ€ΠΈΠ²Ρ‹Π΅ Π²Ρ‚ΠΎΡ€ΠΎΠ³ΠΎ порядка
  111. ΠšΡ€ΠΈΠ²Ρ‹Π΅ ΠΈ повСрхности Π²Ρ‚ΠΎΡ€ΠΎΠ³ΠΎ порядка
  112. ЧисловыС ряды
  113. Π‘Ρ‚Π΅ΠΏΠ΅Π½Π½Ρ‹Π΅ ряды
  114. Ряды Π€ΡƒΡ€ΡŒΠ΅
  115. ΠŸΡ€Π΅ΠΎΠ±Ρ€Π°Π·ΠΎΠ²Π°Π½ΠΈΠ΅ Π€ΡƒΡ€ΡŒΠ΅
  116. Π€ΡƒΠ½ΠΊΡ†ΠΈΠΎΠ½Π°Π»ΡŒΠ½Ρ‹Π΅ ряды
  117. Π€ΡƒΠ½ΠΊΡ†ΠΈΠΈ ΠΌΠ½ΠΎΠ³ΠΈΡ… ΠΏΠ΅Ρ€Π΅ΠΌΠ΅Π½Π½Ρ‹Ρ…
  118. ΠœΠ΅Ρ‚ΠΎΠ΄ ΠΊΠΎΠΎΡ€Π΄ΠΈΠ½Π°Ρ‚
  119. ГармоничСский Π°Π½Π°Π»ΠΈΠ·
  120. ВСщСствСнныС числа
  121. ΠŸΡ€Π΅Π΄Π΅Π» ΠΏΠΎΡΠ»Π΅Π΄ΠΎΠ²Π°Ρ‚Π΅Π»ΡŒΠ½ΠΎΡΡ‚ΠΈ
  122. АналитичСская гСомСтрия
  123. АналитичСская гСомСтрия Π½Π° плоскости
  124. АналитичСская гСомСтрия Π² пространствС
  125. Π€ΡƒΠ½ΠΊΡ†ΠΈΠΈ ΠΎΠ΄Π½ΠΎΠΉ ΠΏΠ΅Ρ€Π΅ΠΌΠ΅Π½Π½ΠΎΠΉ
  126. Π’Ρ‹ΡΡˆΠ°Ρ Π°Π»Π³Π΅Π±Ρ€Π°
  127. ВСкторная Π°Π»Π³Π΅Π±Ρ€Π°
  128. Π’Π΅ΠΊΡ‚ΠΎΡ€Π½Ρ‹ΠΉ Π°Π½Π°Π»ΠΈΠ·
  129. Π’Π΅ΠΊΡ‚ΠΎΡ€Ρ‹
  130. БкалярноС ΠΏΡ€ΠΎΠΈΠ·Π²Π΅Π΄Π΅Π½ΠΈΠ΅ Π²Π΅ΠΊΡ‚ΠΎΡ€ΠΎΠ²
  131. БмСшанноС ΠΏΡ€ΠΎΠΈΠ·Π²Π΅Π΄Π΅Π½ΠΈΠ΅ Π²Π΅ΠΊΡ‚ΠΎΡ€ΠΎΠ²
  132. ΠžΠΏΠ΅Ρ€Π°Ρ†ΠΈΠΈ Π½Π°Π΄ Π²Π΅ΠΊΡ‚ΠΎΡ€Π°ΠΌΠΈ
  133. ΠΠ΅ΠΏΡ€Π΅Ρ€Ρ‹Π²Π½ΠΎΡΡ‚ΡŒ Ρ„ΡƒΠ½ΠΊΡ†ΠΈΠΉ
  134. ΠŸΡ€Π΅Π΄Π΅Π» ΠΈ Π½Π΅ΠΏΡ€Π΅Ρ€Ρ‹Π²Π½ΠΎΡΡ‚ΡŒ Ρ„ΡƒΠ½ΠΊΡ†ΠΈΠΉ Π½Π΅ΡΠΊΠΎΠ»ΡŒΠΊΠΈΡ… ΠΏΠ΅Ρ€Π΅ΠΌΠ΅Π½Π½Ρ‹Ρ…
  135. ΠŸΡ€Π΅Π΄Π΅Π» ΠΈ Π½Π΅ΠΏΡ€Π΅Ρ€Ρ‹Π²Π½ΠΎΡΡ‚ΡŒ Ρ„ΡƒΠ½ΠΊΡ†ΠΈΠΈ ΠΎΠ΄Π½ΠΎΠΉ ΠΏΠ΅Ρ€Π΅ΠΌΠ΅Π½Π½ΠΎΠΉ
  136. ΠŸΡ€ΠΎΠΈΠ·Π²ΠΎΠ΄Π½Ρ‹Π΅ ΠΈ Π΄ΠΈΡ„Ρ„Π΅Ρ€Π΅Π½Ρ†ΠΈΠ°Π»Ρ‹ Ρ„ΡƒΠ½ΠΊΡ†ΠΈΠΈ ΠΎΠ΄Π½ΠΎΠΉ ΠΏΠ΅Ρ€Π΅ΠΌΠ΅Π½Π½ΠΎΠΉ
  137. ЧастныС ΠΏΡ€ΠΎΠΈΠ·Π²ΠΎΠ΄Π½Ρ‹Π΅ ΠΈ Π΄ΠΈΡ„Ρ„Π΅Ρ€Π΅Π½Ρ†ΠΈΡ€ΡƒΠ΅ΠΌΠΎΡΡ‚ΡŒ Ρ„ΡƒΠ½ΠΊΡ†ΠΈΠΉ Π½Π΅ΡΠΊΠΎΠ»ΡŒΠΊΠΈΡ… ΠΏΠ΅Ρ€Π΅ΠΌΠ΅Π½Π½Ρ‹Ρ…
  138. Π”ΠΈΡ„Ρ„Π΅Ρ€Π΅Π½Ρ†ΠΈΠ°Π»ΡŒΠ½ΠΎΠ΅ исчислСниС Ρ„ΡƒΠ½ΠΊΡ†ΠΈΠΈ ΠΎΠ΄Π½ΠΎΠΉ ΠΏΠ΅Ρ€Π΅ΠΌΠ΅Π½Π½ΠΎΠΉ
  139. ΠœΠ°Ρ‚Ρ€ΠΈΡ†Ρ‹
  140. Π›ΠΈΠ½Π΅ΠΉΠ½Ρ‹Π΅ ΠΈ Π΅Π²ΠΊΠ»ΠΈΠ΄ΠΎΠ²Ρ‹ пространства
  141. Π›ΠΈΠ½Π΅ΠΉΠ½Ρ‹Π΅ отобраТСния
  142. Π”ΠΈΡ„Ρ„Π΅Ρ€Π΅Π½Ρ†ΠΈΠ°Π»ΡŒΠ½Ρ‹Π΅ Ρ‚Π΅ΠΎΡ€Π΅ΠΌΡ‹ ΠΎ срСднСм
  143. ВСория устойчивости Π΄ΠΈΡ„Ρ„Π΅Ρ€Π΅Π½Ρ†ΠΈΠ°Π»ΡŒΠ½Ρ‹Ρ… ΡƒΡ€Π°Π²Π½Π΅Π½ΠΈΠΉ
  144. Π€ΡƒΠ½ΠΊΡ†ΠΈΠΈ комплСксного ΠΏΠ΅Ρ€Π΅ΠΌΠ΅Π½Π½ΠΎΠ³ΠΎ
  145. ΠŸΡ€Π΅ΠΎΠ±Ρ€Π°Π·ΠΎΠ²Π°Π½ΠΈΠ΅ Лапласа
  146. Π’Π΅ΠΎΡ€ΠΈΠΈ поля
  147. ΠžΠΏΠ΅Ρ€Π°Ρ†ΠΈΠΎΠ½Π½ΠΎΠ΅ исчислСниС
  148. БистСмы ΠΊΠΎΠΎΡ€Π΄ΠΈΠ½Π°Ρ‚
  149. Π Π°Ρ†ΠΈΠΎΠ½Π°Π»ΡŒΠ½Π°Ρ функция
  150. Π˜Π½Ρ‚Π΅Π³Ρ€Π°Π»ΡŒΠ½ΠΎΠ΅ исчислСниС
  151. Π˜Π½Ρ‚Π΅Π³Ρ€Π°Π»ΡŒΠ½ΠΎΠ΅ исчислСниС Ρ„ΡƒΠ½ΠΊΡ†ΠΈΠΉ ΠΎΠ΄Π½ΠΎΠΉ ΠΏΠ΅Ρ€Π΅ΠΌΠ΅Π½Π½ΠΎΠΉ
  152. Π”ΠΈΡ„Ρ„Π΅Ρ€Π΅Π½Ρ†ΠΈΠ°Π»ΡŒΠ½ΠΎΠ΅ исчислСниС Ρ„ΡƒΠ½ΠΊΡ†ΠΈΠΉ Π½Π΅ΡΠΊΠΎΠ»ΡŒΠΊΠΈΡ… ΠΏΠ΅Ρ€Π΅ΠΌΠ΅Π½Π½Ρ‹Ρ…
  153. ΠžΡ‚Π½ΠΎΡˆΠ΅Π½ΠΈΠ΅ Π² ΠΌΠ°Ρ‚Π΅ΠΌΠ°Ρ‚ΠΈΠΊΠ΅
  154. ΠœΠ°Ρ‚Π΅ΠΌΠ°Ρ‚ΠΈΡ‡Π΅ΡΠΊΠ°Ρ Π»ΠΎΠ³ΠΈΠΊΠ°
  155. Π“Ρ€Π°Ρ„Ρ‹ Π² ΠΌΠ°Ρ‚Π΅ΠΌΠ°Ρ‚ΠΈΠΊΠ΅
  156. Π›ΠΈΠ½Π΅ΠΉΠ½Ρ‹Π΅ пространства
  157. ΠŸΠ΅Ρ€Π²ΠΎΠΎΠ±Ρ€Π°Π·Π½Π°Ρ ΠΈ Π½Π΅ΠΎΠΏΡ€Π΅Π΄Π΅Π»Π΅Π½Π½Ρ‹ΠΉ ΠΈΠ½Ρ‚Π΅Π³Ρ€Π°Π»
  158. ЛинСйная функция
  159. Π’Ρ‹ΠΏΡƒΠΊΠ»Ρ‹Π΅ мноТСства Ρ‚ΠΎΡ‡Π΅ΠΊ
  160. БистСма ΠΊΠΎΠΎΡ€Π΄ΠΈΠ½Π°Ρ‚

Как Π½Π°ΠΉΡ‚ΠΈ Π²Π΅ΠΊΡ‚ΠΎΡ€Π½ΠΎΠ΅ ΠΏΡ€ΠΎΠΈΠ·Π²Π΅Π΄Π΅Π½ΠΈΠ΅ Π²Π΅ΠΊΡ‚ΠΎΡ€ΠΎΠ²

Π£Π³ΠΎΠ» ΠΌΠ΅ΠΆΠ΄Ρƒ Π²Π΅ΠΊΡ‚ΠΎΡ€Π°ΠΌΠΈ

Для Ρ‚ΠΎΠ³ΠΎ Ρ‡Ρ‚ΠΎΠ±Ρ‹ ΠΌΡ‹ ΠΌΠΎΠ³Π»ΠΈ ввСсти понятиС Π²Π΅ΠΊΡ‚ΠΎΡ€Π½ΠΎΠ³ΠΎ произвСдСния Π΄Π²ΡƒΡ… Π²Π΅ΠΊΡ‚ΠΎΡ€ΠΎΠ², Π½ΡƒΠΆΠ½ΠΎ сначала Ρ€Π°Π·ΠΎΠ±Ρ€Π°Ρ‚ΡŒΡΡ с Ρ‚Π°ΠΊΠΈΠΌ понятиС, ΠΊΠ°ΠΊ ΡƒΠ³ΠΎΠ» ΠΌΠ΅ΠΆΠ΄Ρƒ этими Π²Π΅ΠΊΡ‚ΠΎΡ€Π°ΠΌΠΈ.\circ$.

ΠžΠ±ΠΎΠ·Π½Π°Ρ‡Π΅Π½ΠΈΠ΅: $∠(\overline{Ξ±},\overline{Ξ²})$

ΠŸΠΎΠ½ΡΡ‚ΠΈΠ΅ Π²Π΅ΠΊΡ‚ΠΎΡ€Π½ΠΎΠ³ΠΎ произвСдСния Π²Π΅ΠΊΡ‚ΠΎΡ€ΠΎΠ² ΠΈ Ρ„ΠΎΡ€ΠΌΡƒΠ»Π° нахоТдСния

ΠžΠΏΡ€Π΅Π΄Π΅Π»Π΅Π½ΠΈΠ΅ 1

Π’Π΅ΠΊΡ‚ΠΎΡ€Π½Ρ‹ΠΌ ΠΏΡ€ΠΎΠΈΠ·Π²Π΅Π΄Π΅Π½ΠΈΠ΅ΠΌ Π΄Π²ΡƒΡ… Π²Π΅ΠΊΡ‚ΠΎΡ€ΠΎΠ² называСтся Π²Π΅ΠΊΡ‚ΠΎΡ€, пСрпСндикулярный ΠΎΠ±ΠΎΠΈΠΌ Π΄Π°Π½Π½Ρ‹ΠΌ Π²Π΅ΠΊΡ‚ΠΎΡ€Π°ΠΌ, ΠΈ Π΅Π³ΠΎ Π΄Π»ΠΈΠ½Π° Π±ΡƒΠ΄Π΅Ρ‚ Ρ€Π°Π²Π½ΡΡ‚ΡŒΡΡ ΠΏΡ€ΠΎΠΈΠ·Π²Π΅Π΄Π΅Π½ΠΈΡŽ Π΄Π»ΠΈΠ½ этих Π²Π΅ΠΊΡ‚ΠΎΡ€ΠΎΠ² с синусом ΡƒΠ³Π»Π° ΠΌΠ΅ΠΆΠ΄Ρƒ Π΄Π°Π½Π½Ρ‹ΠΌΠΈ Π²Π΅ΠΊΡ‚ΠΎΡ€Π°ΠΌΠΈ, Π° Ρ‚Π°ΠΊΠΆΠ΅ этот Π²Π΅ΠΊΡ‚ΠΎΡ€ с двумя Π½Π°Ρ‡Π°Π»ΡŒΠ½Ρ‹ΠΌΠΈ ΠΈΠΌΠ΅ΡŽΡ‚ Ρ‚ΡƒΠΆΠ΅ ΠΎΡ€ΠΈΠ΅Π½Ρ‚Π°Ρ†ΠΈΡŽ, ΠΊΠ°ΠΊ ΠΈ Π΄Π΅ΠΊΠ°Ρ€Ρ‚ΠΎΠ²Π° систСма ΠΊΠΎΠΎΡ€Π΄ΠΈΠ½Π°Ρ‚.

ΠžΠ±ΠΎΠ·Π½Π°Ρ‡Π΅Π½ΠΈΠ΅: $\overline{Ξ±}Ρ…\overline{Ξ²}$.

ΠœΠ°Ρ‚Π΅ΠΌΠ°Ρ‚ΠΈΡ‡Π΅ΡΠΊΠΈ это выглядит ΡΠ»Π΅Π΄ΡƒΡŽΡ‰ΠΈΠΌ ΠΎΠ±Ρ€Π°Π·ΠΎΠΌ:

  1. $|\overline{Ξ±}Ρ…\overline{Ξ²}|=|\overline{Ξ±}||\overline{Ξ²}|sin⁑∠(\overline{Ξ±},\overline{Ξ²})$
  2. $\overline{Ξ±}Ρ…\overline{Ξ²}βŠ₯\overline{Ξ±}$, $\overline{Ξ±}Ρ…\overline{Ξ²}βŠ₯\overline{Ξ²}$
  3. $(\overline{Ξ±}Ρ…\overline{Ξ²},\overline{Ξ±},\overline{Ξ²})$ ΠΈ $(\overline{i},\overline{j},\overline{k})$ ΠΎΠ΄ΠΈΠ½Π°ΠΊΠΎΠ²ΠΎ ΠΎΡ€ΠΈΠ΅Π½Ρ‚ΠΈΡ€ΠΎΠ²Π°Π½Ρ‹ (рис. 2)

Рисунок 2. ΠŸΡ€ΠΎΠΈΠ·Π²Π΅Π΄Π΅Π½ΠΈΠ΅ Π²Π΅ΠΊΡ‚ΠΎΡ€ΠΎΠ². Автор24 β€” ΠΈΠ½Ρ‚Π΅Ρ€Π½Π΅Ρ‚-Π±ΠΈΡ€ΠΆΠ° студСнчСских Ρ€Π°Π±ΠΎΡ‚

ΠžΡ‡Π΅Π²ΠΈΠ΄Π½ΠΎ, Ρ‡Ρ‚ΠΎ внСшнСС ΠΏΡ€ΠΎΠΈΠ·Π²Π΅Π΄Π΅Π½ΠΈΠ΅ Π²Π΅ΠΊΡ‚ΠΎΡ€ΠΎΠ² Π±ΡƒΠ΄Π΅Ρ‚ Ρ€Π°Π²Π½ΡΡ‚ΡŒΡΡ Π½ΡƒΠ»Π΅Π²ΠΎΠΌΡƒ Π²Π΅ΠΊΡ‚ΠΎΡ€Ρƒ Π² Π΄Π²ΡƒΡ… случаях:

  1. Если Π΄Π»ΠΈΠ½Π° ΠΎΠ΄Π½ΠΎΠ³ΠΎ ΠΈΠ»ΠΈ ΠΎΠ±ΠΎΠΈΡ… Π²Π΅ΠΊΡ‚ΠΎΡ€ΠΎΠ² равняСтся Π½ΡƒΠ»ΡŽ.\circ=4\cdot 3\cdot 1=12$

    ΠžΡ‚Π²Π΅Ρ‚: $12$.

    ВычислСниС Π²Π΅ΠΊΡ‚ΠΎΡ€Π½ΠΎΠ³ΠΎ произвСдСния ΠΏΠΎ ΠΊΠΎΠΎΡ€Π΄ΠΈΠ½Π°Ρ‚Π°ΠΌ Π²Π΅ΠΊΡ‚ΠΎΡ€ΠΎΠ²

    Из опрСдСлСния 1 сразу ΠΆΠ΅ Π²Ρ‹Ρ‚Π΅ΠΊΠ°Π΅Ρ‚ ΠΈ способ нахоТдСния Π²Π΅ΠΊΡ‚ΠΎΡ€Π½ΠΎΠ³ΠΎ произвСдСния для Π΄Π²ΡƒΡ… Π²Π΅ΠΊΡ‚ΠΎΡ€ΠΎΠ². ΠŸΠΎΡΠΊΠΎΠ»ΡŒΠΊΡƒ Π²Π΅ΠΊΡ‚ΠΎΡ€ ΠΊΡ€ΠΎΠΌΠ΅ значСния ΠΈΠΌΠ΅Π΅Ρ‚ Π΅Ρ‰Π΅ ΠΈ Π½Π°ΠΏΡ€Π°Π²Π»Π΅Π½ΠΈΠ΅, Π½Π°Ρ…ΠΎΠ΄ΠΈΡ‚ΡŒ Π΅Π³ΠΎ Ρ‚ΠΎΠ»ΡŒΠΊΠΎ ΠΏΡ€ΠΈ ΠΏΠΎΠΌΠΎΡ‰ΠΈ скалярной Π²Π΅Π»ΠΈΡ‡ΠΈΠ½Ρ‹ Π½Π΅Π²ΠΎΠ·ΠΌΠΎΠΆΠ½ΠΎ. Но ΠΏΠΎΠΌΠΈΠΌΠΎ Π½Π΅Π³ΠΎ сущСствуСт Π΅Ρ‰Π΅ способ нахоТдСния с ΠΏΠΎΠΌΠΎΡ‰ΡŒΡŽ ΠΊΠΎΠΎΡ€Π΄ΠΈΠ½Π°Ρ‚ Π΄Π°Π½Π½Ρ‹Ρ… Π½Π°ΠΌ Π²Π΅ΠΊΡ‚ΠΎΡ€ΠΎΠ².

    ΠŸΡƒΡΡ‚ΡŒ Π½Π°ΠΌ Π΄Π°Π½Ρ‹ Π²Π΅ΠΊΡ‚ΠΎΡ€Ρ‹ $\overline{Ξ±}$ ΠΈ $\overline{Ξ²}$, ΠΊΠΎΡ‚ΠΎΡ€Ρ‹Π΅ Π±ΡƒΠ΄ΡƒΡ‚ ΠΈΠΌΠ΅Ρ‚ΡŒ ΠΊΠΎΠΎΡ€Π΄ΠΈΠ½Π°Ρ‚Ρ‹ $(Ξ±_1,Ξ±_2,Ξ±_3)$ ΠΈ $(Ξ²_1,Ξ²_2,Ξ²_3)$, соотвСтствСнно. Π’ΠΎΠ³Π΄Π° Π²Π΅ΠΊΡ‚ΠΎΡ€ Π²Π΅ΠΊΡ‚ΠΎΡ€Π½ΠΎΠ³ΠΎ произвСдСния (Π° ΠΈΠΌΠ΅Π½Π½ΠΎ Π΅Π³ΠΎ ΠΊΠΎΠΎΡ€Π΄ΠΈΠ½Π°Ρ‚Ρ‹) ΠΌΠΎΠΆΠ½ΠΎ Π½Π°ΠΉΡ‚ΠΈ ΠΏΠΎ ΡΠ»Π΅Π΄ΡƒΡŽΡ‰Π΅ΠΉ Ρ„ΠΎΡ€ΠΌΡƒΠ»Π΅:

    $\overline{Ξ±}Ρ…\overline{Ξ²}=\begin{vmatrix}\overline{i}&\overline{j}&\overline{k}\\Ξ±_1&Ξ±_2&Ξ±_3\\Ξ²_1&Ξ²_2&Ξ²_3\end{vmatrix}$

    Π˜Π½Π°Ρ‡Π΅, раскрывая ΠΎΠΏΡ€Π΅Π΄Π΅Π»ΠΈΡ‚Π΅Π»ΡŒ, ΠΏΠΎΠ»ΡƒΡ‡ΠΈΠΌ ΡΠ»Π΅Π΄ΡƒΡŽΡ‰ΠΈΠ΅ ΠΊΠΎΠΎΡ€Π΄ΠΈΠ½Π°Ρ‚Ρ‹

    $\overline{Ξ±}Ρ…\overline{Ξ²}=(Ξ±_2 Ξ²_3-Ξ±_3 Ξ²_2,Ξ±_3 Ξ²_1-Ξ±_1 Ξ²_3,Ξ±_1 Ξ²_2-Ξ±_2 Ξ²_1)$

    ΠŸΡ€ΠΈΠΌΠ΅Ρ€ 2

    Найти Π²Π΅ΠΊΡ‚ΠΎΡ€ Π²Π΅ΠΊΡ‚ΠΎΡ€Π½ΠΎΠ³ΠΎ произвСдСния ΠΊΠΎΠ»Π»ΠΈΠ½Π΅Π°Ρ€Π½Ρ‹Ρ… Π²Π΅ΠΊΡ‚ΠΎΡ€ΠΎΠ² $\overline{Ξ±}$ ΠΈ $\overline{Ξ²}$ с ΠΊΠΎΠΎΡ€Π΄ΠΈΠ½Π°Ρ‚Π°ΠΌΠΈ $(0,3,3)$ ΠΈ $(-1,2,6)$.

    РСшСниС.

    Π’ΠΎΡΠΏΠΎΠ»ΡŒΠ·ΡƒΠ΅ΠΌΡΡ Ρ„ΠΎΡ€ΠΌΡƒΠ»ΠΎΠΉ, ΠΏΡ€ΠΈΠ²Π΅Π΄Π΅Π½Π½ΠΎΠΉ Π²Ρ‹ΡˆΠ΅. ΠŸΠΎΠ»ΡƒΡ‡ΠΈΠΌ

    $\overline{Ξ±}Ρ…\overline{Ξ²}=\begin{vmatrix}\overline{i}&\overline{j}&\overline{k}\\0&3&3\\-1&2&6\end{vmatrix}=(18-6)\overline{i}-(0+3)\overline{j}+(0+3)\overline{k}=12\overline{i}-3\overline{j}+3\overline{k}=(12,-3,3)$

    ΠžΡ‚Π²Π΅Ρ‚: $(12,-3,3)$.

    Бвойства Π²Π΅ΠΊΡ‚ΠΎΡ€Π½ΠΎΠ³ΠΎ произвСдСния Π²Π΅ΠΊΡ‚ΠΎΡ€ΠΎΠ²

    Для ΠΏΡ€ΠΎΠΈΠ·Π²ΠΎΠ»ΡŒΠ½Ρ‹Ρ… ΡΠΌΠ΅ΡˆΠ°Π½Π½Ρ‹Ρ… Ρ‚Ρ€Π΅Ρ… Π²Π΅ΠΊΡ‚ΠΎΡ€ΠΎΠ² $\overline{Ξ±}$, $\overline{Ξ²}$ ΠΈ $\overline{Ξ³}$, Π° Ρ‚Π°ΠΊΠΆΠ΅ $r∈R$ справСдливы ΡΠ»Π΅Π΄ΡƒΡŽΡ‰ΠΈΠ΅ свойства:

    1. $\overline{Ξ±}Ρ…\overline{Ξ²}=-(\overline{Ξ²}Ρ…\overline{Ξ±})$

      Π’Π΅Ρ€Π½ΠΎΡΡ‚ΡŒ этого свойства Π±ΡƒΠ΄Π΅Ρ‚ ΡΠ»Π΅Π΄ΠΎΠ²Π°Ρ‚ΡŒ ΠΈΠ· Ρ‚Ρ€Π΅Ρ‚ΡŒΠ΅Π³ΠΎ ΠΏΡƒΠ½ΠΊΡ‚Π° опрСдСлСния 1.

    2. $(r\overline{Ξ±})Ρ…\overline{Ξ²}=r(\overline{Ξ±}Ρ…\overline{Ξ²})$ ΠΈ $\overline{Ξ±}Ρ…(r\overline{Ξ²})=r(\overline{Ξ±}Ρ…\overline{Ξ²})$

      Из Ρ„ΠΎΡ€ΠΌΡƒΠ»Ρ‹ для нахоТдСния Π²Π΅ΠΊΡ‚ΠΎΡ€Π½ΠΎΠ³ΠΎ произвСдСния Π±ΡƒΠ΄Π΅ΠΌ ΠΏΠΎΠ»ΡƒΡ‡Π°Ρ‚ΡŒ:

      $(r\overline{Ξ±})\overline{Ξ²}=\begin{vmatrix}\overline{i}&\overline{j}&\overline{k}\\rΞ±_1&rΞ±_2&rΞ±_3\\Ξ²_1&Ξ²_2&Ξ²_3\end{vmatrix}=r\begin{vmatrix}\overline{i}&\overline{j}&\overline{k}\\Ξ±_1&Ξ±_2&Ξ±_3\\Ξ²_1&Ξ²_2&Ξ²_3\end{vmatrix}=r(\overline{Ξ±}Ρ…\overline{Ξ²})$

      $\overline{Ξ±}Ρ…(r\overline{Ξ²})=\begin{vmatrix}\overline{i}&\overline{j}&\overline{k}\\Ξ±_1&Ξ±_2&Ξ±_3\\rΞ²_1&rΞ²_2&rΞ²_3\end{vmatrix}=r\begin{vmatrix}\overline{i}&\overline{j}&\overline{k}\\Ξ±_1&Ξ±_2&Ξ±_3\\Ξ²_1&Ξ²_2&Ξ²_3\end{vmatrix}=r(\overline{Ξ±}Ρ…\overline{Ξ²})$

    3. $\overline{Ξ±}Ρ…(\overline{Ξ²}+\overline{Ξ³})=\overline{Ξ±}\overline{Ξ²}+\overline{Ξ±}\overline{Ξ³}$ ΠΈ $(\overline{Ξ±}+\overline{Ξ²})\overline{Ξ³}=\overline{Ξ±}\overline{Ξ³}+\overline{Ξ²}\overline{Ξ³}$.

      Π”Π°Π½Π½ΠΎΠ΅ свойство Π²Π΅ΠΊΡ‚ΠΎΡ€Π½ΠΎΠ³ΠΎ произвСдСния Π²Π΅ΠΊΡ‚ΠΎΡ€ΠΎΠ² Ρ‚Π°ΠΊΠΆΠ΅ ΠΌΠΎΠΆΠ½ΠΎ ΠΏΡ€ΠΎΠ²Π΅Ρ€ΠΈΡ‚ΡŒ с ΠΏΠΎΠΌΠΎΡ‰ΡŒΡŽ Ρ„ΠΎΡ€ΠΌΡƒΠ»Ρ‹.

      Π‘Π»Π΅Π΄ΡƒΡŽΡ‰Π΅Π΅ свойство Π½Π°Π·Ρ‹Π²Π°ΡŽΡ‚ гСомСтричСским смыслом Π²Π΅ΠΊΡ‚ΠΎΡ€Π½ΠΎΠ³ΠΎ произвСдСния:

    4. Π”Π»ΠΈΠ½Π° Π²Π΅ΠΊΡ‚ΠΎΡ€Π° Π²Π΅ΠΊΡ‚ΠΎΡ€Π½ΠΎΠ³ΠΎ произвСдСния равняСтся ΠΏΠ»ΠΎΡ‰Π°Π΄ΠΈ ΠΏΠ°Ρ€Π°Π»Π»Π΅Π»ΠΎΠ³Ρ€Π°ΠΌΠΌΠ°, ΠΊΠΎΡ‚ΠΎΡ€Ρ‹ΠΉ Π½ΡƒΠΆΠ½ΠΎ Π±Ρ‹Π»ΠΎ ΠΏΠΎΡΡ‚Ρ€ΠΎΠΈΡ‚ΡŒ ΠΌΠ΅ΠΆΠ΄Ρƒ Π½ΠΈΠΌΠΈ (рис. 4)

      Рисунок 4. Π”Π»ΠΈΠ½Π° Π²Π΅ΠΊΡ‚ΠΎΡ€Π° Π²Π΅ΠΊΡ‚ΠΎΡ€Π½ΠΎΠ³ΠΎ произвСдСния. Автор24 β€” ΠΈΠ½Ρ‚Π΅Ρ€Π½Π΅Ρ‚-Π±ΠΈΡ€ΠΆΠ° студСнчСских Ρ€Π°Π±ΠΎΡ‚

    ΠŸΡ€ΠΈΠΌΠ΅Ρ€ 3

    НайдитС ΠΏΠ»ΠΎΡ‰Π°Π΄ΡŒ ΠΏΠ°Ρ€Π°Π»Π»Π΅Π»ΠΎΠ³Ρ€Π°ΠΌΠΌΠ°, Π²Π΅Ρ€ΡˆΠΈΠ½Ρ‹ ΠΊΠΎΡ‚ΠΎΡ€ΠΎΠ³ΠΎ ΠΈΠΌΠ΅ΡŽΡ‚ ΠΊΠΎΠΎΡ€Π΄ΠΈΠ½Π°Ρ‚Ρ‹ $(3,0,0)$, $(0,0,0)$, $(0,8,0)$ ΠΈ $(3,8,0)$.

    РСшСниС.

    Π’Π½Π°Ρ‡Π°Π»Π΅ ΠΈΠ·ΠΎΠ±Ρ€Π°Π·ΠΈΠΌ Π΄Π°Π½Π½Ρ‹ΠΉ ΠΏΠ°Ρ€Π°Π»Π»Π΅Π»ΠΎΠ³Ρ€Π°ΠΌΠΌ Π² ΠΊΠΎΠΎΡ€Π΄ΠΈΠ½Π°Ρ‚Π½ΠΎΠΌ пространствС (рис.5):

    Рисунок 5. ΠŸΠ°Ρ€Π°Π»Π»Π΅Π»ΠΎΠ³Ρ€Π°ΠΌΠΌ Π² ΠΊΠΎΠΎΡ€Π΄ΠΈΠ½Π°Ρ‚Π½ΠΎΠΌ пространствС. Автор24 β€” ΠΈΠ½Ρ‚Π΅Ρ€Π½Π΅Ρ‚-Π±ΠΈΡ€ΠΆΠ° студСнчСских Ρ€Π°Π±ΠΎΡ‚

    Π’ΠΈΠ΄ΠΈΠΌ, Ρ‡Ρ‚ΠΎ Π΄Π²Π΅ стороны этого ΠΏΠ°Ρ€Π°Π»Π»Π΅Π»ΠΎΠ³Ρ€Π°ΠΌΠΌΠ° построСны с ΠΏΠΎΠΌΠΎΡ‰ΡŒΡŽ ΠΊΠΎΠ»Π»ΠΈΠ½Π΅Π°Ρ€Π½Ρ‹Ρ… Π²Π΅ΠΊΡ‚ΠΎΡ€ΠΎΠ² с ΠΊΠΎΠΎΡ€Π΄ΠΈΠ½Π°Ρ‚Π°ΠΌΠΈ $\overline{Ξ±}=(3,0,0)$ ΠΈ $\overline{Ξ²}=(0,8,0)$. Π˜ΡΠΏΠΎΠ»ΡŒΠ·ΡƒΡ Ρ‡Π΅Ρ‚Π²Π΅Ρ€Ρ‚ΠΎΠ΅ свойство, ΠΏΠΎΠ»ΡƒΡ‡ΠΈΠΌ:

    $S=|\overline{Ξ±}Ρ…\overline{Ξ²}|$

    НайдСм Π²Π΅ΠΊΡ‚ΠΎΡ€ $\overline{Ξ±}Ρ…\overline{Ξ²}$:

    $\overline{Ξ±}Ρ…\overline{Ξ²}=\begin{vmatrix}\overline{i}&\overline{j}&\overline{k}\\3&0&0\\0&8&0\end{vmatrix}=0\overline{i}-0\overline{j}+24\overline{k}=(0,0,24)$

    Π‘Π»Π΅Π΄ΠΎΠ²Π°Ρ‚Π΅Π»ΡŒΠ½ΠΎ

    $S=|\overline{Ξ±}Ρ…\overline{Ξ²}|=\sqrt{0+0+24^2}=24$

    ΠžΡ‚Π²Π΅Ρ‚: $24$.

    Π’Π΅ΠΊΡ‚ΠΎΡ€Π½ΠΎΠ΅ ΠΏΡ€ΠΎΠΈΠ·Π²Π΅Π΄Π΅Π½ΠΈΠ΅ Π²Π΅ΠΊΡ‚ΠΎΡ€ΠΎΠ² — это… Π§Ρ‚ΠΎ Ρ‚Π°ΠΊΠΎΠ΅ Π’Π΅ΠΊΡ‚ΠΎΡ€Π½ΠΎΠ΅ ΠΏΡ€ΠΎΠΈΠ·Π²Π΅Π΄Π΅Π½ΠΈΠ΅ Π²Π΅ΠΊΡ‚ΠΎΡ€ΠΎΠ²?

    ΠŸΡ€Π°Π²Ρ‹Π΅ ΠΈ Π»Π΅Π²Ρ‹Π΅ Ρ‚Ρ€ΠΎΠΉΠΊΠΈ Π²Π΅ΠΊΡ‚ΠΎΡ€ΠΎΠ²

    Π’Ρ€ΠΈ Π²Π΅ΠΊΡ‚ΠΎΡ€Π° Π½Π°Π·Ρ‹Π²Π°ΡŽΡ‚ΡΡ упорядочСнной Ρ‚Ρ€ΠΎΠΉΠΊΠΎΠΉ, Ссли ΡƒΠΊΠ°Π·Π°Π½ΠΎ, ΠΊΠ°ΠΊΠΎΠΉ ΠΈΠ· этих Π²Π΅ΠΊΡ‚ΠΎΡ€ΠΎΠ² являСтся ΠΏΠ΅Ρ€Π²Ρ‹ΠΌ, ΠΊΠ°ΠΊΠΎΠΉΒ β€” Π²Ρ‚ΠΎΡ€Ρ‹ΠΌ, Π° ΠΊΠ°ΠΊΠΎΠΉΒ β€” Ρ‚Ρ€Π΅Ρ‚ΡŒΠΈΠΌ.

    Π’Ρ€ΠΎΠΉΠΊΠ° Π½Π΅ΠΊΠΎΠΌΠΏΠ»Π°Π½Π°Ρ€Π½Ρ‹Ρ… Π²Π΅ΠΊΡ‚ΠΎΡ€ΠΎΠ² называСтся ΠΏΡ€Π°Π²ΠΎΠΉ (Π»Π΅Π²ΠΎΠΉ), Ссли, Π±ΡƒΠ΄ΡƒΡ‡ΠΈ ΠΏΡ€ΠΈΠ²Π΅Π΄Ρ‘Π½Π½Ρ‹ΠΌΠΈ ΠΊ ΠΎΠ±Ρ‰Π΅ΠΌΡƒ Π½Π°Ρ‡Π°Π»Ρƒ, эти Π²Π΅ΠΊΡ‚ΠΎΡ€Ρ‹ Ρ€Π°ΡΠΏΠΎΠ»Π°Π³Π°ΡŽΡ‚ΡΡ Ρ‚Π°ΠΊ, ΠΊΠ°ΠΊ ΠΌΠΎΠ³ΡƒΡ‚ Π±Ρ‹Ρ‚ΡŒ располоТСны соотвСтствСнно большой, нСсогнутый ΡƒΠΊΠ°Π·Π°Ρ‚Π΅Π»ΡŒΠ½Ρ‹ΠΉ ΠΈ срСдний ΠΏΠ°Π»ΡŒΡ†Ρ‹ ΠΏΡ€Π°Π²ΠΎΠΉ (Π»Π΅Π²ΠΎΠΉ) Ρ€ΡƒΠΊΠΈ.

    ΠžΠΏΡ€Π΅Π΄Π΅Π»Π΅Π½ΠΈΠ΅

    Π’Π΅ΠΊΡ‚ΠΎΡ€Π½Ρ‹ΠΌ ΠΏΡ€ΠΎΠΈΠ·Π²Π΅Π΄Π΅Π½ΠΈΠ΅ΠΌ Π²Π΅ΠΊΡ‚ΠΎΡ€Π° Π½Π° Π²Π΅ΠΊΡ‚ΠΎΡ€ называСтся Π²Π΅ΠΊΡ‚ΠΎΡ€ , ΡƒΠ΄ΠΎΠ²Π»Π΅Ρ‚Π²ΠΎΡ€ΡΡŽΡ‰ΠΈΠΉ ΡΠ»Π΅Π΄ΡƒΡŽΡ‰ΠΈΠΌ трСбованиям:

    • Π΄Π»ΠΈΠ½Π° Π²Π΅ΠΊΡ‚ΠΎΡ€Π° Ρ€Π°Π²Π½Π° ΠΏΡ€ΠΎΠΈΠ·Π²Π΅Π΄Π΅Π½ΠΈΡŽ Π΄Π»ΠΈΠ½ Π²Π΅ΠΊΡ‚ΠΎΡ€ΠΎΠ² ΠΈ Π½Π° синус ΡƒΠ³Π»Π° ; ΠΌΠ΅ΠΆΠ΄Ρƒ Π½ΠΈΠΌΠΈ

    ΠžΠ±ΠΎΠ·Π½Π°Ρ‡Π΅Π½ΠΈΠ΅:

    Π’ Ρ€Π°Π·Π»ΠΈΡ‡Π½Ρ‹Ρ… ΡƒΡ‡Π΅Π±Π½Ρ‹Ρ… завСдСниях ΠΎΠΏΡ€Π΅Π΄Π΅Π»Π΅Π½ΠΈΠ΅ Π²Π΅ΠΊΡ‚ΠΎΡ€Π½ΠΎΠ³ΠΎ произвСдСния даётся ΠΏΠΎ-Ρ€Π°Π·Π½ΠΎΠΌΡƒ. НапримСр, Π² качСствС опрСдСлСния даётся описанноС Π΄Π°Π»Π΅Π΅ Π²Ρ‹Ρ€Π°ΠΆΠ΅Π½ΠΈΠ΅ Π²Π΅ΠΊΡ‚ΠΎΡ€Π½ΠΎΠ³ΠΎ произвСдСния Π² ΠΊΠΎΠΎΡ€Π΄ΠΈΠ½Π°Ρ‚Π°Ρ…. А Π΄Π°Π»Π΅Π΅ выводится Π΄Π°Π½Π½ΠΎΠ΅ Π²Ρ‹ΡˆΠ΅ ΠΎΠΏΡ€Π΅Π΄Π΅Π»Π΅Π½ΠΈΠ΅.

    Бвойства

    ГСомСтричСскиС свойства Π²Π΅ΠΊΡ‚ΠΎΡ€Π½ΠΎΠ³ΠΎ произвСдСния

    АлгСбраичСскиС свойства Π²Π΅ΠΊΡ‚ΠΎΡ€Π½ΠΎΠ³ΠΎ произвСдСния

    Π’Ρ‹Ρ€Π°ΠΆΠ΅Π½ΠΈΠ΅ для Π²Π΅ΠΊΡ‚ΠΎΡ€Π½ΠΎΠ³ΠΎ произвСдСния Π² Π΄Π΅ΠΊΠ°Ρ€Ρ‚ΠΎΠ²Ρ‹Ρ… ΠΊΠΎΠΎΡ€Π΄ΠΈΠ½Π°Ρ‚Π°Ρ…

    Если Π΄Π²Π° Π²Π΅ΠΊΡ‚ΠΎΡ€Π° ΠΈ ΠΎΠΏΡ€Π΅Π΄Π΅Π»Π΅Π½Ρ‹ своими ΠΏΡ€ΡΠΌΠΎΡƒΠ³ΠΎΠ»ΡŒΠ½Ρ‹ΠΌΠΈ Π΄Π΅ΠΊΠ°Ρ€Ρ‚ΠΎΠ²Ρ‹ΠΌΠΈ ΠΊΠΎΠΎΡ€Π΄ΠΈΠ½Π°Ρ‚Π°ΠΌΠΈ, Π° говоря Ρ‚ΠΎΡ‡Π½Π΅Π΅Β β€” прСдставлСны Π² ΠΎΡ€Ρ‚ΠΎΠ½ΠΎΡ€ΠΌΠΈΡ€ΠΎΠ²Π°Π½Π½ΠΎΠΌ базисС

    Ρ‚ΠΎ ΠΈΡ… Π²Π΅ΠΊΡ‚ΠΎΡ€Π½ΠΎΠ΅ ΠΏΡ€ΠΎΠΈΠ·Π²Π΅Π΄Π΅Π½ΠΈΠ΅ ΠΈΠΌΠ΅Π΅Ρ‚ Π²ΠΈΠ΄

    Для запоминания этой Ρ„ΠΎΡ€ΠΌΡƒΠ»Ρ‹ ΡƒΠ΄ΠΎΠ±Π½ΠΎ ΠΈΡΠΏΠΎΠ»ΡŒΠ·ΠΎΠ²Π°Ρ‚ΡŒ ΠΎΠΏΡ€Π΅Π΄Π΅Π»ΠΈΡ‚Π΅Π»ΡŒ:

    ΠΈΠ»ΠΈ

    Π³Π΄Π΅ Β β€” символ Π›Π΅Π²ΠΈ-Π§ΠΈΠ²ΠΈΡ‚Ρ‹.

    ΠžΠ±ΠΎΠ±Ρ‰Π΅Π½ΠΈΡ

    ΠšΠ²Π°Ρ‚Π΅Ρ€Π½ΠΈΠΎΠ½Ρ‹

    Π’Π΅ΠΊΡ‚ΠΎΡ€Π½ΠΎΠ΅ ΠΏΡ€ΠΎΠΈΠ·Π²Π΅Π΄Π΅Π½ΠΈΠ΅ ΠΌΠΎΠΆΠ½ΠΎ Ρ‚Π°ΠΊΠΆΠ΅ Π·Π°ΠΏΠΈΡΠ°Ρ‚ΡŒ Π² ΠΊΠ²Π°Ρ‚Π΅Ρ€Π½ΠΈΠΎΠ½Π½ΠΎΠΉ Ρ„ΠΎΡ€ΠΌΠ΅, поэтому Π±ΡƒΠΊΠ²Ρ‹ , , Β β€” стандартныС обозначСния для ΠΎΡ€Ρ‚ΠΎΠ² Π² : ΠΎΠ½ΠΈ Ρ€Π°ΡΡΠΌΠ°Ρ‚Ρ€ΠΈΠ²Π°ΡŽΡ‚ΡΡ ΠΊΠ°ΠΊ Π²ΠΎΠΎΠ±Ρ€Π°ΠΆΠ°Π΅ΠΌΡ‹Π΅ ΠΊΠ²Π°Ρ‚Π΅Ρ€Π½ΠΈΠΎΠ½Ρ‹.

    Π—Π°ΠΌΠ΅Ρ‚ΠΈΠΌ, Ρ‡Ρ‚ΠΎ ΡΠΎΠΎΡ‚Π½ΠΎΡˆΠ΅Π½ΠΈΡ Ρ‡Π΅Ρ€Π΅Π· Π²Π΅ΠΊΡ‚ΠΎΡ€Π½ΠΎΠ΅ ΠΏΡ€ΠΎΠΈΠ·Π²Π΅Π΄Π΅Π½ΠΈΠ΅ ΠΌΠ΅ΠΆΠ΄Ρƒ , ΠΈ ΡΠΎΠΎΡ‚Π²Π΅Ρ‚ΡΡ‚Π²ΡƒΡŽΡ‚ ΠΏΡ€Π°Π²ΠΈΠ»Π°ΠΌ умноТСния для ΠΊΠ²Π°Ρ‚Π΅Ρ€Π½ΠΈΠΎΠ½ΠΎΠ² i, j ΠΈ k. Если ΠΏΡ€Π΅Π΄ΡΡ‚Π°Π²ΠΈΡ‚ΡŒ Π²Π΅ΠΊΡ‚ΠΎΡ€ ΠΊΠ°ΠΊ ΠΊΠ²Π°Ρ‚Π΅Ρ€Π½ΠΈΠΎΠ½ a1i + a2j + a3k, Ρ‚ΠΎ Π²Π΅ΠΊΡ‚ΠΎΡ€Π½ΠΎΠ΅ ΠΏΡ€ΠΎΠΈΠ·Π²Π΅Π΄Π΅Π½ΠΈΠ΅ Π΄Π²ΡƒΡ… Π²Π΅ΠΊΡ‚ΠΎΡ€ΠΎΠ² получаСтся взятиСм Π²Π΅ΠΊΡ‚ΠΎΡ€Π½ΠΎΠΉ части ΠΎΡ‚ произвСдСния ΡΠΎΠΎΡ‚Π²Π΅Ρ‚ΡΡ‚Π²ΡƒΡŽΡ‰ΠΈΡ… ΠΈΠΌ ΠΊΠ²Π°Ρ‚Π΅Ρ€Π½ΠΈΠΎΠ½ΠΎΠ². БкалярноС ΠΏΡ€ΠΎΠΈΠ·Π²Π΅Π΄Π΅Π½ΠΈΠ΅ этих Π²Π΅ΠΊΡ‚ΠΎΡ€ΠΎΠ² ΠΏΡ€ΠΎΡ‚ΠΈΠ²ΠΎΠΏΠΎΠ»ΠΎΠΆΠ½ΠΎ скалярной части произвСдСния этих ΠΊΠ²Π°Ρ‚Π΅Ρ€Π½ΠΈΠΎΠ½ΠΎΠ².

    ΠŸΡ€Π΅ΠΎΠ±Ρ€Π°Π·ΠΎΠ²Π°Π½ΠΈΠ΅ ΠΊ ΠΌΠ°Ρ‚Ρ€ΠΈΡ‡Π½ΠΎΠΉ Ρ„ΠΎΡ€ΠΌΠ΅

    Π’Π΅ΠΊΡ‚ΠΎΡ€Π½ΠΎΠ΅ ΠΏΡ€ΠΎΠΈΠ·Π²Π΅Π΄Π΅Π½ΠΈΠ΅ Π΄Π²ΡƒΡ… Π²Π΅ΠΊΡ‚ΠΎΡ€ΠΎΠ² ΠΌΠΎΠΆΠ½ΠΎ Π·Π°ΠΏΠΈΡΠ°Ρ‚ΡŒ ΠΊΠ°ΠΊ ΠΏΡ€ΠΎΠΈΠ·Π²Π΅Π΄Π΅Π½ΠΈΠ΅ кососиммСтричСской ΠΌΠ°Ρ‚Ρ€ΠΈΡ†Ρ‹ ΠΈ Π²Π΅ΠΊΡ‚ΠΎΡ€Π°:

    Π³Π΄Π΅

    ΠŸΡƒΡΡ‚ΡŒ Ρ€Π°Π²Π΅Π½ Π²Π΅ΠΊΡ‚ΠΎΡ€Π½ΠΎΠΌΡƒ ΠΏΡ€ΠΎΠΈΠ·Π²Π΅Π΄Π΅Π½ΠΈΡŽ:

    Ρ‚ΠΎΠ³Π΄Π°

    Вакая Ρ„ΠΎΡ€ΠΌΠ° записи позволяСт ΠΎΠ±ΠΎΠ±Ρ‰ΠΈΡ‚ΡŒ Π²Π΅ΠΊΡ‚ΠΎΡ€Π½ΠΎΠ΅ ΠΏΡ€ΠΎΠΈΠ·Π²Π΅Π΄Π΅Π½ΠΈΠ΅ Π½Π° Π²Ρ‹ΡΡˆΠΈΠ΅ размСрности, прСдставляя псСвдовСкторы (угловая ΡΠΊΠΎΡ€ΠΎΡΡ‚ΡŒ, индукция ΠΈΒ Ρ‚.Β ΠΏ.) ΠΊΠ°ΠΊ Ρ‚Π°ΠΊΠΈΠ΅ кососиммСтричныС ΠΌΠ°Ρ‚Ρ€ΠΈΡ†Ρ‹. Ясно, Ρ‡Ρ‚ΠΎ Ρ‚Π°ΠΊΠΈΠ΅ физичСскиС Π²Π΅Π»ΠΈΡ‡ΠΈΠ½Ρ‹ Π±ΡƒΠ΄ΡƒΡ‚ ΠΈΠΌΠ΅Ρ‚ΡŒ n(n βˆ’ 1) / 2 нСзависимых ΠΊΠΎΠΌΠΏΠΎΠ½Π΅Π½Ρ‚ Π² n-ΠΌΠ΅Ρ€Π½ΠΎΠΌ пространствС. Π’ Ρ‚Ρ€Ρ‘Ρ…ΠΌΠ΅Ρ€Π½ΠΎΠΌ пространствС ΠΏΠΎΠ»ΡƒΡ‡Π°ΡŽΡ‚ΡΡ Ρ‚Ρ€ΠΈ нСзависимыС ΠΊΠΎΠΌΠΏΠΎΠ½Π΅Π½Ρ‚Ρ‹, поэтому Ρ‚Π°ΠΊΠΈΠ΅ Π²Π΅Π»ΠΈΡ‡ΠΈΠ½Ρ‹ ΠΌΠΎΠΆΠ½ΠΎ ΠΏΡ€Π΅Π΄ΡΡ‚Π°Π²Π»ΡΡ‚ΡŒ ΠΊΠ°ΠΊ Π²Π΅ΠΊΡ‚ΠΎΡ€Ρ‹ этого пространства.

    Π‘ Ρ‚Π°ΠΊΠΎΠΉ Ρ„ΠΎΡ€ΠΌΠΎΠΉ записи Ρ‚Π°ΠΊΠΆΠ΅ Π·Π°Ρ‡Π°ΡΡ‚ΡƒΡŽ ΠΏΡ€ΠΎΡ‰Π΅ Ρ€Π°Π±ΠΎΡ‚Π°Ρ‚ΡŒ (Π½Π°ΠΏΡ€ΠΈΠΌΠ΅Ρ€, Π² en:epipolar geometry).

    Из ΠΎΠ±Ρ‰ΠΈΡ… свойств Π²Π΅ΠΊΡ‚ΠΎΡ€Π½ΠΎΠ³ΠΎ произвСдСния слСдуСт, Ρ‡Ρ‚ΠΎ

    Β  ΠΈ Β 

    Π° Ρ‚Π°ΠΊ ΠΊΠ°ΠΊ кососиммСтрична, Ρ‚ΠΎ

    Π’ Ρ‚Π°ΠΊΠΎΠΉ Ρ„ΠΎΡ€ΠΌΠ΅ записи Π»Π΅Π³ΠΊΠΎ доказываСтся тоТдСство Π›Π°Π³Ρ€Π°Π½ΠΆΠ° (ΠΏΡ€Π°Π²ΠΈΠ»ΠΎ Β«Π±Π°Ρ† минус Ρ†Π°Π±Β»).

    РаспространСниС Π½Π° ΠΌΠ°Ρ‚Ρ€ΠΈΡ†Ρ‹

    Π’ 3-Ρ…ΠΌΠ΅Ρ€Π½ΠΎΠΌ случаС ΠΌΠΎΠΆΠ½ΠΎ ΠΎΠΏΡ€Π΅Π΄Π΅Π»ΠΈΡ‚ΡŒ Π²Π΅ΠΊΡ‚ΠΎΡ€Π½ΠΎΠ΅ ΠΏΡ€ΠΎΠΈΠ·Π²Π΅Π΄Π΅Π½ΠΈΠ΅ ΠΌΠ°Ρ‚Ρ€ΠΈΡ† ΠΈ ΠΏΡ€ΠΎΠΈΠ·Π²Π΅Π΄Π΅Π½ΠΈΠ΅ ΠΌΠ°Ρ‚Ρ€ΠΈΡ†Ρ‹ Π½Π° Π²Π΅ΠΊΡ‚ΠΎΡ€. Π­Ρ‚ΠΎ Π΄Π΅Π»Π°Π΅Ρ‚ ΠΎΡ‡Π΅Π²ΠΈΠ΄Π½Ρ‹ΠΌ ΡƒΠΊΠ°Π·Π°Π½Π½Ρ‹ΠΉ Π²Ρ‹ΡˆΠ΅ ΠΈΠ·ΠΎΠΌΠΎΡ€Ρ„ΠΈΠ·ΠΌ ΠΈ позволяСт ΡƒΠΏΡ€ΠΎΡΡ‚ΠΈΡ‚ΡŒ ΠΌΠ½ΠΎΠ³ΠΈΠ΅ Π²Ρ‹ΠΊΠ»Π°Π΄ΠΊΠΈ. ΠŸΡ€Π΅Π΄ΡΡ‚Π°Π²ΠΈΠΌ ΠΌΠ°Ρ‚Ρ€ΠΈΡ†Ρƒ A ΠΊΠ°ΠΊ столбСц Π²Π΅ΠΊΡ‚ΠΎΡ€ΠΎΠ², Ρ‚ΠΎΠ³Π΄Π°

    Π£ΠΌΠ½ΠΎΠΆΠ΅Π½ΠΈΠ΅ ΠΌΠ°Ρ‚Ρ€ΠΈΡ†Ρ‹ Π½Π° Π²Π΅ΠΊΡ‚ΠΎΡ€ слСва опрСдСляСтся Π°Π½Π°Π»ΠΎΠ³ΠΈΡ‡Π½ΠΎ, Ссли ΠΏΡ€Π΅Π΄ΡΡ‚Π°Π²ΠΈΡ‚ΡŒ A ΠΊΠ°ΠΊ строку Π²Π΅ΠΊΡ‚ΠΎΡ€ΠΎΠ². ВранспонированиС ΠΌΠ°Ρ‚Ρ€ΠΈΡ†Ρ‹, соотвСтствСнно, ΠΏΠ΅Ρ€Π΅Π²ΠΎΠ΄ΠΈΡ‚ строку Π²Π΅ΠΊΡ‚ΠΎΡ€ΠΎΠ² Π² столбСц Π²Π΅ΠΊΡ‚ΠΎΡ€ΠΎΠ², ΠΈ Π½Π°ΠΎΠ±ΠΎΡ€ΠΎΡ‚. Π›Π΅Π³ΠΊΠΎ ΠΎΠ±ΠΎΠ±Ρ‰ΠΈΡ‚ΡŒ ΠΌΠ½ΠΎΠ³ΠΈΠ΅ ΡΠΎΠΎΡ‚Π½ΠΎΡˆΠ΅Π½ΠΈΡ для Π²Π΅ΠΊΡ‚ΠΎΡ€ΠΎΠ² Π½Π° ΡΠΎΠΎΡ‚Π½ΠΎΡˆΠ΅Π½ΠΈΡ для Π²Π΅ΠΊΡ‚ΠΎΡ€ΠΎΠ² ΠΈ ΠΌΠ°Ρ‚Ρ€ΠΈΡ†, Π½Π°ΠΏΡ€ΠΈΠΌΠ΅Ρ€ (AΒ β€” ΠΌΠ°Ρ‚Ρ€ΠΈΡ†Π°, , Β β€” Π²Π΅ΠΊΡ‚ΠΎΡ€Ρ‹):

    ПослС этого ΠΌΠΎΠΆΠ½ΠΎ ΠΈΠ·ΠΌΠ΅Π½ΠΈΡ‚ΡŒ Ρ„ΠΎΡ€ΠΌΡƒ записи для Π²Π΅ΠΊΡ‚ΠΎΡ€Π½ΠΎΠ³ΠΎ произвСдСния:

    EΒ β€” Сдиничная ΠΌΠ°Ρ‚Ρ€ΠΈΡ†Π°. ΠžΡ‚ΡΡŽΠ΄Π° ΠΎΡ‡Π΅Π²ΠΈΠ΄Π½Ρ‹ сущСствованиС ΠΈ Π²ΠΈΠ΄ ΠΌΠ°Ρ‚Ρ€ΠΈΡ†Ρ‹, ΡΠΎΠΎΡ‚Π²Π΅Ρ‚ΡΡ‚Π²ΡƒΡŽΡ‰Π΅ΠΉ Π²Π΅ΠΊΡ‚ΠΎΡ€Π½ΠΎΠΌΡƒ ΡƒΠΌΠ½ΠΎΠΆΠ΅Π½ΠΈΡŽ Π½Π° Π²Π΅ΠΊΡ‚ΠΎΡ€ слСва. Аналогично ΠΌΠΎΠΆΠ½ΠΎ ΠΏΠΎΠ»ΡƒΡ‡ΠΈΡ‚ΡŒ Π²Ρ‹Ρ€Π°ΠΆΠ΅Π½ΠΈΠ΅ для ΠΌΠ°Ρ‚Ρ€ΠΈΡ†Ρ‹ умноТСния Π½Π° Π²Π΅ΠΊΡ‚ΠΎΡ€ справа. Распространяя ΠΎΠΏΠ΅Ρ€Π°Ρ†ΠΈΠΈ Π½Π°Π΄ Π²Π΅ΠΊΡ‚ΠΎΡ€Π°ΠΌΠΈ Π½Π° ΠΌΠ°Ρ‚Ρ€ΠΈΡ†Ρ‹ ΠΏΠΎΠΊΠΎΠΌΠΏΠΎΠ½Π΅Π½Ρ‚Π½ΠΎ, прСдставляя ΠΈΡ… ΠΊΠ°ΠΊ Β«Π²Π΅ΠΊΡ‚ΠΎΡ€Ρ‹ ΠΈΠ· Π²Π΅ΠΊΡ‚ΠΎΡ€ΠΎΠ²Β», стандартныС ΡΠΎΠΎΡ‚Π½ΠΎΡˆΠ΅Π½ΠΈΡ для Π²Π΅ΠΊΡ‚ΠΎΡ€ΠΎΠ² Π»Π΅Π³ΠΊΠΎ ΠΎΠ±ΠΎΠ±Ρ‰Π°ΡŽΡ‚ΡΡ Π½Π° ΠΌΠ°Ρ‚Ρ€ΠΈΡ†Ρ‹. НапримСр, Ρ‚Π΅ΠΎΡ€Π΅ΠΌΠ° Бтокса Π² ΠΏΡ€ΠΈΠΌΠ΅Ρ‚ Π²ΠΈΠ΄:

    Π³Π΄Π΅ Ρ€ΠΎΡ‚ΠΎΡ€ ΠΌΠ°Ρ‚Ρ€ΠΈΡ†Ρ‹ A вычисляСтся ΠΊΠ°ΠΊ Π²Π΅ΠΊΡ‚ΠΎΡ€Π½ΠΎΠ΅ ΠΏΡ€ΠΎΠΈΠ·Π²Π΅Π΄Π΅Π½ΠΈΠ΅ ΠΌΠ°Ρ‚Ρ€ΠΈΡ†Ρ‹ A Π½Π° ΠΎΠΏΠ΅Ρ€Π°Ρ‚ΠΎΡ€ Π“Π°ΠΌΠΈΠ»ΡŒΡ‚ΠΎΠ½Π° слСва. Π’ этих обозначСниях ΠΎΡ‡Π΅Π½ΡŒ Π»Π΅Π³ΠΊΠΎ Π΄ΠΎΠΊΠ°Π·Π°Ρ‚ΡŒ, Π½Π°ΠΏΡ€ΠΈΠΌΠ΅Ρ€, ΡΠ»Π΅Π΄ΡƒΡŽΡ‰ΠΈΠ΅ Ρ„ΠΎΡ€ΠΌΡ‹ Ρ‚Π΅ΠΎΡ€Π΅ΠΌΡ‹ Бтокса:

    РазмСрности, Π½Π΅ Ρ€Π°Π²Π½Ρ‹Π΅ Ρ‚Ρ€Ρ‘ΠΌ

    ΠŸΡƒΡΡ‚ΡŒ DΒ β€” Ρ€Π°Π·ΠΌΠ΅Ρ€Π½ΠΎΡΡ‚ΡŒ пространства.

    Π’Π΅ΠΊΡ‚ΠΎΡ€Π½ΠΎΠ΅ ΠΏΡ€ΠΎΠΈΠ·Π²Π΅Π΄Π΅Π½ΠΈΠ΅, ΠΎΠ±Π»Π°Π΄Π°ΡŽΡ‰Π΅Π΅ всСми свойствами ΠΎΠ±Ρ‹Ρ‡Π½ΠΎΠ³ΠΎ Ρ‚Ρ€Ρ‘Ρ…ΠΌΠ΅Ρ€Π½ΠΎΠ³ΠΎ Π²Π΅ΠΊΡ‚ΠΎΡ€Π½ΠΎΠ³ΠΎ произвСдСния, Ρ‚ΠΎ Π΅ΡΡ‚ΡŒ Π±ΠΈΠ½Π°Ρ€Π½ΠΎΠ΅ Π±ΠΈΠ»ΠΈΠ½Π΅ΠΉΠ½ΠΎΠ΅ антисиммСтричноС Π½Π΅Π²Ρ‹Ρ€ΠΎΠΆΠ΄Π΅Π½Π½ΠΎΠ΅ ΠΎΡ‚ΠΎΠ±Ρ€Π°ΠΆΠ΅Π½ΠΈΠ΅ , ΠΌΠΎΠΆΠ½ΠΎ ввСсти Ρ‚ΠΎΠ»ΡŒΠΊΠΎ для размСрности 3.

    Однако Π΅ΡΡ‚ΡŒ простоС ΠΎΠ±ΠΎΠ±Ρ‰Π΅Π½ΠΈΠ΅ Π½Π° ΠΎΡΡ‚Π°Π»ΡŒΠ½Ρ‹Π΅ Π½Π°Ρ‚ΡƒΡ€Π°Π»ΡŒΠ½Ρ‹Π΅ размСрности, начиная с 3, Π° Ссли Π½ΡƒΠΆΠ½ΠΎΒ β€” ΠΈ Π½Π° Ρ€Π°Π·ΠΌΠ΅Ρ€Π½ΠΎΡΡ‚ΡŒ 2 (послСднСС, ΠΏΡ€Π°Π²Π΄Π°, ΡΡ€Π°Π²Π½ΠΈΡ‚Π΅Π»ΡŒΠ½ΠΎ спСцифичСским ΠΎΠ±Ρ€Π°Π·ΠΎΠΌ). Π’ΠΎΠ³Π΄Π° это ΠΎΠ±ΠΎΠ±Ρ‰Π΅Π½ΠΈΠ΅, Π² ΠΎΡ‚Π»ΠΈΡ‡ΠΈΠ΅ ΠΎΡ‚ Π½Π΅Π²ΠΎΠ·ΠΌΠΎΠΆΠ½ΠΎΠ³ΠΎ, описанного Ρ‡ΡƒΡ‚ΡŒ Π²Ρ‹ΡˆΠ΅, вводится Π½Π΅ для ΠΏΠ°Ρ€Ρ‹ Π²Π΅ΠΊΡ‚ΠΎΡ€ΠΎΠ², Π° лишь для Π½Π°Π±ΠΎΡ€Π° (D βˆ’ 1) Π²Π΅ΠΊΡ‚ΠΎΡ€ΠΎΠ²-сомноТитСлСй. Π’ΠΏΠΎΠ»Π½Π΅ Π°Π½Π°Π»ΠΎΠ³ΠΈΡ‡Π½ΠΎ ΡΠΌΠ΅ΡˆΠ°Π½Π½ΠΎΠΌΡƒ ΠΏΡ€ΠΎΠΈΠ·Π²Π΅Π΄Π΅Π½ΠΈΡŽ, СстСствСнно ΠΎΠ±ΠΎΠ±Ρ‰Π°Π΅ΠΌΠΎΠΌΡƒ Π² D-ΠΌΠ΅Ρ€Π½ΠΎΠΌ пространствС Π½Π° ΠΎΠΏΠ΅Ρ€Π°Ρ†ΠΈΡŽ с D сомноТитСлями. Π˜ΡΠΏΠΎΠ»ΡŒΠ·ΡƒΡ символ Π›Π΅Π²ΠΈ-Π§ΠΈΠ²ΠΈΡ‚Ρ‹ с D индСксами, ΠΌΠΎΠΆΠ½ΠΎ явно Π·Π°ΠΏΠΈΡΠ°Ρ‚ΡŒ Ρ‚Π°ΠΊΠΎΠ΅ (D βˆ’ 1)-Π²Π°Π»Π΅Π½Ρ‚Π½ΠΎΠ΅ Π²Π΅ΠΊΡ‚ΠΎΡ€Π½ΠΎΠ΅ ΠΏΡ€ΠΎΠΈΠ·Π²Π΅Π΄Π΅Π½ΠΈΠ΅ ΠΊΠ°ΠΊ

    Π’Π°ΠΊΠΎΠ΅ ΠΎΠ±ΠΎΠ±Ρ‰Π΅Π½ΠΈΠ΅ Π΄Π°Π΅Ρ‚ Π³ΠΈΠΏΠ΅Ρ€ΠΏΠ»ΠΎΡ‰Π°Π΄ΡŒ размСрности (D βˆ’ 1).

    Если Π½ΡƒΠΆΠ½ΠΎ ввСсти ΠΎΠΏΠ΅Ρ€Π°Ρ†ΠΈΡŽ ΠΈΠΌΠ΅Π½Π½ΠΎ для Π΄Π²ΡƒΡ… сомноТитСлСй, ΠΈΠΌΠ΅ΡŽΡ‰ΡƒΡŽ гСомСтричСский смысл, ΠΏΡ€Π΅Π΄Π΅Π»ΡŒΠ½ΠΎ Π±Π»ΠΈΠ·ΠΊΠΈΠΉ ΠΊ смыслу Π²Π΅ΠΊΡ‚ΠΎΡ€Π½ΠΎΠ³ΠΎ произвСдСния (Ρ‚ΠΎ Π΅ΡΡ‚ΡŒ ΠΏΡ€Π΅Π΄ΡΡ‚Π°Π²Π»ΡΡŽΡ‰ΡƒΡŽ ΠΎΡ€ΠΈΠ΅Π½Ρ‚ΠΈΡ€ΠΎΠ²Π°Π½Π½ΡƒΡŽ ΠΏΠ»ΠΎΡ‰Π°Π΄ΡŒ), Ρ‚ΠΎ Ρ€Π΅Π·ΡƒΠ»ΡŒΡ‚Π°Ρ‚ ΡƒΠΆΠ΅ Π½Π΅ Π±ΡƒΠ΄Π΅Ρ‚ Π²Π΅ΠΊΡ‚ΠΎΡ€ΠΎΠΌ, Ρ‚Π°ΠΊ ΠΊΠ°ΠΊ ΠΏΡ€ΠΈ D < > 3 Π½Π΅ найдСтся СдинствСнной, ΠΎΠ΄Π½ΠΎΠ·Π½Π°Ρ‡Π½ΠΎ ΠΎΠΏΡ€Π΅Π΄Π΅Π»Ρ‘Π½Π½ΠΎΠΉ Π½ΠΎΡ€ΠΌΠ°Π»ΠΈ ΠΊ Π΄Π²ΡƒΠΌΠ΅Ρ€Π½ΠΎΠΉ плоскости, натянутой Π½Π° ΠΌΠ½ΠΎΠΆΠΈΡ‚Π΅Π»ΠΈ. МоТно ввСсти Π±ΠΈΠ²Π΅ΠΊΡ‚ΠΎΡ€, ΠΊΠΎΠΌΠΏΠΎΠ½Π΅Π½Ρ‚Ρ‹ ΠΊΠΎΡ‚ΠΎΡ€ΠΎΠ³ΠΎ Ρ€Π°Π²Π½Ρ‹ проСкциям ΠΎΡ€ΠΈΠ΅Π½Ρ‚ΠΈΡ€ΠΎΠ²Π°Π½Π½ΠΎΠΉ ΠΏΠ»ΠΎΡ‰Π°Π΄ΠΈ ΠΏΠ°Ρ€Π°Π»Π»Π΅Π»ΠΎΠ³Ρ€Π°ΠΌΠΌΠ°, натянутого Π½Π° ΠΏΠ°Ρ€Ρƒ Π²Π΅ΠΊΡ‚ΠΎΡ€ΠΎΠ², Π½Π° ΠΊΠΎΠΎΡ€Π΄ΠΈΠ½Π°Ρ‚Π½Ρ‹Π΅ плоскости:

    .

    Π­Ρ‚Π° конструкция называСтся внСшним ΠΏΡ€ΠΎΠΈΠ·Π²Π΅Π΄Π΅Π½ΠΈΠ΅ΠΌ.

    Для Π΄Π²ΡƒΠΌΠ΅Ρ€Π½ΠΎΠ³ΠΎ случая эта опСрация называСтся псСвдоскалярным ΠΏΡ€ΠΎΠΈΠ·Π²Π΅Π΄Π΅Π½ΠΈΠ΅ΠΌ, Ρ‚Π°ΠΊ ΠΊΠ°ΠΊ ΠΏΠΎΠ»ΡƒΡ‡Π°ΡŽΡ‰Π΅Π΅ΡΡ пространство ΠΎΠ΄Π½ΠΎΠΌΠ΅Ρ€Π½ΠΎ ΠΈ Ρ€Π΅Π·ΡƒΠ»ΡŒΡ‚Π°Ρ‚ ΠΌΠΎΠΆΠ½ΠΎ ΠΎΡ‚ΠΎΠΆΠ΄Π΅ΡΡ‚Π²ΠΈΡ‚ΡŒ с псСвдоскаляром.

    АлгСбра Π›ΠΈ Π²Π΅ΠΊΡ‚ΠΎΡ€ΠΎΠ²

    Π’Π΅ΠΊΡ‚ΠΎΡ€Π½ΠΎΠ΅ ΠΏΡ€ΠΎΠΈΠ·Π²Π΅Π΄Π΅Π½ΠΈΠ΅ Π²Π²ΠΎΠ΄ΠΈΡ‚ Π½Π° структуру Π°Π»Π³Π΅Π±Ρ€Ρ‹ Π›ΠΈ (ΠΏΠΎΡΠΊΠΎΠ»ΡŒΠΊΡƒ ΠΎΠ½ΠΎ удовлСтворяСт ΠΎΠ±Π΅ΠΈΠΌ аксиомам — антисиммСтричности ΠΈ тоТдСству Π―ΠΊΠΎΠ±ΠΈ). Π­Ρ‚Π° структура соотвСтствуСт ΠΎΡ‚ΠΎΠΆΠ΄Π΅ΡΡ‚Π²Π»Π΅Π½ΠΈΡŽ с ΠΊΠ°ΡΠ°Ρ‚Π΅Π»ΡŒΠ½ΠΎΠΉ Π°Π»Π³Π΅Π±Ρ€ΠΎΠΉ Π›ΠΈ so(3) ΠΊ Π³Ρ€ΡƒΠΏΠΏΠ΅ Π›ΠΈ SO(3) ΠΎΡ€Ρ‚ΠΎΠ³ΠΎΠ½Π°Π»ΡŒΠ½Ρ‹Ρ… Π»ΠΈΠ½Π΅ΠΉΠ½Ρ‹Ρ… ΠΏΡ€Π΅ΠΎΠ±Ρ€Π°Π·ΠΎΠ²Π°Π½ΠΈΠΉ Ρ‚Ρ€Ρ‘Ρ…ΠΌΠ΅Ρ€Π½ΠΎΠ³ΠΎ пространства.

    Π‘ΠΌ. Ρ‚Π°ΠΊΠΆΠ΅

    ΠŸΡ€ΠΎΠΈΠ·Π²Π΅Π΄Π΅Π½ΠΈΡ Π²Π΅ΠΊΡ‚ΠΎΡ€ΠΎΠ²

    Π”Ρ€ΡƒΠ³ΠΎΠ΅

    Бсылки

    Π›ΠΈΡ‚Π΅Ρ€Π°Ρ‚ΡƒΡ€Π°

    • ΠšΠΎΡ‡ΠΈΠ½ Н. Π•. Π’Π²Π΅Π΄Π΅Π½ΠΈΠ΅ Π² Π²Π΅ΠΊΡ‚ΠΎΡ€Π½Ρ‹ΠΉ ΠΈ Ρ‚Π΅Π½Π·ΠΎΡ€Π½Ρ‹ΠΉ Π°Π½Π°Π»ΠΈΠ·.

    Wikimedia Foundation. 2010.

    Π˜Π‘ΠŸΠžΠ›Π¬Π—ΠžΠ’ΠΠΠ˜Π• Π’Π•ΠšΠ’ΠžΠ ΠΠžΠ“Πž ΠŸΠ ΠžΠ˜Π—Π’Π•Π”Π•ΠΠ˜Π― Π’Π•ΠšΠ’ΠžΠ ΠžΠ’ Π”Π›Π― Π’Π«Π§Π˜Π‘Π›Π•ΠΠ˜Π― ΠŸΠ›ΠžΠ©ΠΠ”Π˜ ΠΠ•ΠšΠžΠ’ΠžΠ Π«Π₯ Π“Π•ΠžΠœΠ•Π’Π Π˜Π§Π•Π‘ΠšΠ˜Π₯ Π€Π˜Π“Π£Π  Π˜ΡΡΠ»Π΅Π΄ΠΎΠ²Π°Ρ‚Π΅Π»ΡŒΡΠΊΠ°Ρ Ρ€Π°Π±ΠΎΡ‚Π° ΠΏΠΎ ΠΌΠ°Ρ‚Π΅ΠΌΠ°Ρ‚ΠΈΠΊΠ΅ ΡƒΡ‡Π΅Π½ΠΈΠΊΠ° 10 Π‘ класса МОУ БОШ β„–73 ΠŸΠ΅Ρ€Π΅Π²ΠΎΠ·Π½ΠΈΠΊΠΎΠ²Π° ΠœΠΈΡ…Π°ΠΈΠ»Π°

    ИспользованиС Π²Π΅ΠΊΡ‚ΠΎΡ€Π½ΠΎΠ³ΠΎ произвСдСния Π’Π•ΠšΠ’ΠžΠ ΠžΠ’

    для вычислСния ΠΏΠ»ΠΎΡ‰Π°Π΄ΠΈ

    Π½Π΅ΠΊΠΎΡ‚ΠΎΡ€Ρ‹Ρ… гСомСтричСских Ρ„ΠΈΠ³ΡƒΡ€

    Π˜ΡΡΠ»Π΅Π΄ΠΎΠ²Π°Ρ‚Π΅Π»ΡŒΡΠΊΠ°Ρ Ρ€Π°Π±ΠΎΡ‚Π° ΠΏΠΎ ΠΌΠ°Ρ‚Π΅ΠΌΠ°Ρ‚ΠΈΠΊΠ΅

    Π£Ρ‡Π΅Π½ΠΈΠΊΠ° 10 Π‘ класса

    МОУ БОШ β„–73

    ΠŸΠ΅Ρ€Π΅Π²ΠΎΠ·Π½ΠΈΠΊΠΎΠ²Π° ΠœΠΈΡ…Π°ΠΈΠ»Π°

    Π ΡƒΠΊΠΎΠ²ΠΎΠ΄ΠΈΡ‚Π΅Π»ΠΈ:

    АссистСнт ΠΊΠ°Ρ„. матСматичСского Π°Π½Π°Π»ΠΈΠ·Π° ΠΌΠ΅Ρ…Π°Π½ΠΈΠΊΠΎ-матСматичСского Ρ„Π°ΠΊΡƒΠ»ΡŒΡ‚Π΅Ρ‚Π° Π‘Π“Π£ ΠΈΠΌ. Н.Π“. Π§Π΅Ρ€Π½Ρ‹ΡˆΠ΅Π²ΡΠΊΠΎΠ³ΠΎ Π‘Π΅Ρ€Π΄Π½ΠΈΠΊΠΎΠ² Π“Π»Π΅Π± Π‘Π΅Ρ€Π³Π΅Π΅Π²ΠΈΡ‡

    Π‘Π°Ρ€Π°Ρ‚ΠΎΠ², 2015

    Π‘ΠΎΠ΄Π΅Ρ€ΠΆΠ°Π½ΠΈΠ΅

    Π’Π²Π΅Π΄Π΅Π½ΠΈΠ΅.

    1. ВСорСтичСский ΠΎΠ±Π·ΠΎΡ€.

    1.1. Π’Π΅ΠΊΡ‚ΠΎΡ€Ρ‹ ΠΈ вычислСния с Π²Π΅ΠΊΡ‚ΠΎΡ€Π°ΠΌΠΈ.

    1.2. ИспользованиС скалярного произвСдСния Π²Π΅ΠΊΡ‚ΠΎΡ€ΠΎΠ² Π² Ρ€Π΅ΡˆΠ΅Π½ΠΈΠΈ Π·Π°Π΄Π°Ρ‡

    1.3 БкалярноС ΠΏΡ€ΠΎΠΈΠ·Π²Π΅Π΄Π΅Π½ΠΈΠ΅ Π²Π΅ΠΊΡ‚ΠΎΡ€ΠΎΠ² Π² ΠΊΠΎΠΎΡ€Π΄ΠΈΠ½Π°Ρ‚Π°Ρ…

    1.4. Π’Π΅ΠΊΡ‚ΠΎΡ€Π½ΠΎΠ΅ ΠΏΡ€ΠΎΠΈΠ·Π²Π΅Π΄Π΅Π½ΠΈΠ΅ Π²Π΅ΠΊΡ‚ΠΎΡ€ΠΎΠ² Π² Ρ‚Ρ€Ρ‘Ρ…ΠΌΠ΅Ρ€Π½ΠΎΠΌ Π•Π²ΠΊΠ»ΠΈΠ΄ΠΎΠ²ΠΎΠΌ пространствС: ΠΎΠΏΡ€Π΅Π΄Π΅Π»Π΅Π½ΠΈΠ΅ понятия.

    1.5. ΠšΠΎΠΎΡ€Π΄ΠΈΠ½Π°Ρ‚Ρ‹ Π²Π΅ΠΊΡ‚ΠΎΡ€Π½ΠΎΠ³ΠΎ произвСдСния Π²Π΅ΠΊΡ‚ΠΎΡ€ΠΎΠ².

    2. ΠŸΡ€Π°ΠΊΡ‚ΠΈΡ‡Π΅ΡΠΊΠ°Ρ Ρ‡Π°ΡΡ‚ΡŒ.

    2.1. Бвязь Π²Π΅ΠΊΡ‚ΠΎΡ€Π½ΠΎΠ³ΠΎ произвСдСния с ΠΏΠ»ΠΎΡ‰Π°Π΄ΡŒΡŽ Ρ‚Ρ€Π΅ΡƒΠ³ΠΎΠ»ΡŒΠ½ΠΈΠΊΠ° ΠΈ ΠΏΠ°Ρ€Π°Π»Π»Π΅Π»ΠΎΠ³Ρ€Π°ΠΌΠΌΠ°. Π’Ρ‹Π²Π΅Π΄Π΅Π½ΠΈΠ΅ Ρ„ΠΎΡ€ΠΌΡƒΠ»Ρ‹ ΠΈ гСомСтричСский смысл Π²Π΅ΠΊΡ‚ΠΎΡ€Π½ΠΎΠ³ΠΎ произвСдСния Π²Π΅ΠΊΡ‚ΠΎΡ€ΠΎΠ².

    2.2. Зная Ρ‚ΠΎΠ»ΡŒΠΊΠΎ ΠΊΠΎΠΎΡ€Π΄ΠΈΠ½Π°Ρ‚Ρ‹ Ρ‚ΠΎΡ‡Π΅ΠΊ, Π½Π°ΠΉΡ‚ΠΈ ΠΏΠ»ΠΎΡ‰Π°Π΄ΡŒ Ρ‚Ρ€Π΅ΡƒΠ³ΠΎΠ»ΡŒΠ½ΠΈΠΊΠ°. Π”ΠΎΠΊΠ°Π·Π°Ρ‚Π΅Π»ΡŒΡΡ‚Π²ΠΎ Ρ‚Π΅ΠΎΡ€Π΅ΠΌΡ‹

    2.3. ΠŸΡ€ΠΎΠ²Π΅Ρ€ΠΊΠ° Π½Π° ΠΏΡ€ΠΈΠΌΠ΅Ρ€Π°Ρ… ΠΏΡ€Π°Π²ΠΈΠ»ΡŒΠ½ΠΎΡΡ‚ΠΈ Ρ„ΠΎΡ€ΠΌΡƒΠ»Ρ‹.

    2.4. ΠŸΡ€Π°ΠΊΡ‚ΠΈΡ‡Π΅ΡΠΊΠΎΠ΅ использованиС Π²Π΅ΠΊΡ‚ΠΎΡ€Π½ΠΎΠΉ Π°Π»Π³Π΅Π±Ρ€Ρ‹ ΠΈ произвСдСния Π²Π΅ΠΊΡ‚ΠΎΡ€ΠΎΠ².

    Π—Π°ΠΊΠ»ΡŽΡ‡Π΅Π½ΠΈΠ΅

    Π’Π²Π΅Π΄Π΅Π½ΠΈΠ΅

    Как извСстно, ΠΌΠ½ΠΎΠ³ΠΈΠ΅ гСомСтричСскиС Π·Π°Π΄Π°Ρ‡ΠΈ ΠΈΠΌΠ΅ΡŽΡ‚ Π΄Π²Π° ΠΊΠ»ΡŽΡ‡Π΅Π²Ρ‹Ρ… способа Ρ€Π΅ΡˆΠ΅Π½ΠΈΡ – графичСский ΠΈ аналитичСский. ГрафичСский ΠΌΠ΅Ρ‚ΠΎΠ΄ связан с построСниСм Π³Ρ€Π°Ρ„ΠΈΠΊΠΎΠ² ΠΈ Ρ‡Π΅Ρ€Ρ‚Π΅ΠΆΠ΅ΠΉ, Π° аналитичСский   Β ΠΏΡ€Π΅Π΄ΠΏΠΎΠ»Π°Π³Π°Π΅Ρ‚ Ρ€Π΅ΡˆΠ΅Π½ΠΈΠ΅ Π·Π°Π΄Π°Ρ‡ прСимущСствСнно с ΠΏΠΎΠΌΠΎΡ‰ΡŒΡŽ алгСбраичСских дСйствий. Π’ послСднСм случаС Π°Π»Π³ΠΎΡ€ΠΈΡ‚ΠΌ Ρ€Π΅ΡˆΠ΅Π½ΠΈΠΉ Π·Π°Π΄Π°Ρ‡ связан с аналитичСской Π³Π΅ΠΎΠΌΠ΅Ρ‚Ρ€ΠΈΠ΅ΠΉ. АналитичСская гСомСтрия – это ΠΎΠ±Π»Π°ΡΡ‚ΡŒ ΠΌΠ°Ρ‚Π΅ΠΌΠ°Ρ‚ΠΈΠΊΠΈ, Π° Ρ‚ΠΎΡ‡Π½Π΅Π΅ Π»ΠΈΠ½Π΅ΠΉΠ½ΠΎΠΉ Π°Π»Π³Π΅Π±Ρ€Ρ‹, которая рассматриваСт Ρ€Π΅ΡˆΠ΅Π½ΠΈΠ΅ гСомСтричСских Π·Π°Π΄Π°Ρ‡ срСдствами Π°Π»Π³Π΅Π±Ρ€Ρ‹ Π½Π° основС ΠΌΠ΅Ρ‚ΠΎΠ΄Π° ΠΊΠΎΠΎΡ€Π΄ΠΈΠ½Π°Ρ‚ Π½Π° плоскости ΠΈ Π² пространствС. АналитичСская гСомСтрия позволяСт Π°Π½Π°Π»ΠΈΠ·ΠΈΡ€ΠΎΠ²Π°Ρ‚ΡŒ гСомСтричСскиС ΠΎΠ±Ρ€Π°Π·Ρ‹, ΠΈΡΡΠ»Π΅Π΄ΠΎΠ²Π°Ρ‚ΡŒ Π»ΠΈΠ½ΠΈΠΈ ΠΈ повСрхности, Π²Π°ΠΆΠ½Ρ‹Π΅ для практичСских ΠΏΡ€ΠΈΠ»ΠΎΠΆΠ΅Π½ΠΈΠΉ. ΠŸΡ€ΠΈ этом Π² этой Π½Π°ΡƒΠΊΠ΅ для Ρ€Π°ΡΡˆΠΈΡ€Π΅Π½ΠΈΡ пространствСнного понимания Ρ„ΠΈΠ³ΡƒΡ€ ΠΏΠΎΠΌΠΈΠΌΠΎ скалярного произвСдСния Π²Π΅ΠΊΡ‚ΠΎΡ€ΠΎΠ² ΠΈΠ½ΠΎΠ³Π΄Π° примСняСтся Π²Π΅ΠΊΡ‚ΠΎΡ€Π½ΠΎΠ΅ ΠΏΡ€ΠΎΠΈΠ·Π²Π΅Π΄Π΅Π½ΠΈΠ΅ Π²Π΅ΠΊΡ‚ΠΎΡ€ΠΎΠ².

    Π’ связи с ΡˆΠΈΡ€ΠΎΠΊΠΈΠΌ распространСниСм Ρ‚Ρ€Π΅Ρ…ΠΌΠ΅Ρ€Π½Ρ‹Ρ… пространствСнных Ρ‚Π΅Ρ…Π½ΠΎΠ»ΠΎΠ³ΠΈΠΉ, ΠΈΠ·ΡƒΡ‡Π΅Π½ΠΈΠ΅ свойств Π½Π΅ΠΊΠΎΡ‚ΠΎΡ€Ρ‹Ρ… гСомСтричСских Ρ„ΠΈΠ³ΡƒΡ€ с использованиСм Π²Π΅ΠΊΡ‚ΠΎΡ€Π½ΠΎΠ³ΠΎ произвСдСния прСдставляСтся Π°ΠΊΡ‚ΡƒΠ°Π»ΡŒΠ½Ρ‹ΠΌ.

    Π’ связи с этим Π±Ρ‹Π»Π° ΠΎΠ±ΠΎΠ·Π½Π°Ρ‡Π΅Π½Π° Ρ†Π΅Π»ΡŒ Π΄Π°Π½Π½ΠΎΠ³ΠΎ проСкта – использованиС Π²Π΅ΠΊΡ‚ΠΎΡ€Π½ΠΎΠ³ΠΎ произвСдСния Π²Π΅ΠΊΡ‚ΠΎΡ€ΠΎΠ² для вычислСния ΠΏΠ»ΠΎΡ‰Π°Π΄ΠΈ Π½Π΅ΠΊΠΎΡ‚ΠΎΡ€Ρ‹Ρ… гСомСтричСских Ρ„ΠΈΠ³ΡƒΡ€.

    Π’ связи с поставлСнной Ρ†Π΅Π»ΡŒΡŽ Ρ€Π΅ΡˆΠ°Π»ΠΈΡΡŒ ΡΠ»Π΅Π΄ΡƒΡŽΡ‰ΠΈΠ΅ Π·Π°Π΄Π°Ρ‡ΠΈ:

    1. ВСорСтичСски ΠΈΠ·ΡƒΡ‡ΠΈΡ‚ΡŒ Π½Π΅ΠΎΠ±Ρ…ΠΎΠ΄ΠΈΠΌΡ‹Π΅ основы Π²Π΅ΠΊΡ‚ΠΎΡ€Π½ΠΎΠΉ Π°Π»Π³Π΅Π±Ρ€Ρ‹ ΠΈ Π΄Π°Ρ‚ΡŒ ΠΎΠΏΡ€Π΅Π΄Π΅Π»Π΅Π½ΠΈΠ΅ Π²Π΅ΠΊΡ‚ΠΎΡ€Π½ΠΎΠΌΡƒ ΠΏΡ€ΠΎΠΈΠ·Π²Π΅Π΄Π΅Π½ΠΈΡŽ Π²Π΅ΠΊΡ‚ΠΎΡ€ΠΎΠ² Π² систСмС ΠΊΠΎΠΎΡ€Π΄ΠΈΠ½Π°Ρ‚;

    2. ΠŸΡ€ΠΎΠ°Π½Π°Π»ΠΈΠ·ΠΈΡ€ΠΎΠ²Π°Ρ‚ΡŒ Π½Π°Π»ΠΈΡ‡ΠΈΠ΅ связи Π²Π΅ΠΊΡ‚ΠΎΡ€Π½ΠΎΠ³ΠΎ произвСдСния с ΠΏΠ»ΠΎΡ‰Π°Π΄ΡŒΡŽ Ρ‚Ρ€Π΅ΡƒΠ³ΠΎΠ»ΡŒΠ½ΠΈΠΊΠ° ΠΈ ΠΏΠ°Ρ€Π°Π»Π»Π΅Π»ΠΎΠ³Ρ€Π°ΠΌΠΌΠ°;

    3. ВывСсти Ρ„ΠΎΡ€ΠΌΡƒΠ»Ρƒ ΠΏΠ»ΠΎΡ‰Π°Π΄ΠΈ Ρ‚Ρ€Π΅ΡƒΠ³ΠΎΠ»ΡŒΠ½ΠΈΠΊΠ° ΠΈ ΠΏΠ°Ρ€Π°Π»Π»Π΅Π»ΠΎΠ³Ρ€Π°ΠΌΠΌΠ° Π² ΠΊΠΎΠΎΡ€Π΄ΠΈΠ½Π°Ρ‚Π°Ρ…;

    4. ΠŸΡ€ΠΎΠ²Π΅Ρ€ΠΈΡ‚ΡŒ Π½Π° ΠΊΠΎΠ½ΠΊΡ€Π΅Ρ‚Π½Ρ‹Ρ… ΠΏΡ€ΠΈΠΌΠ΅Ρ€Π°Ρ… Π²Π΅Ρ€Π½ΠΎΡΡ‚ΡŒ Π²Ρ‹Π²Π΅Π΄Π΅Π½Π½ΠΎΠΉ Ρ„ΠΎΡ€ΠΌΡƒΠ»Ρ‹.

    1. ВСорСтичСский ΠΎΠ±Π·ΠΎΡ€.

      1. Π’Π΅ΠΊΡ‚ΠΎΡ€Ρ‹ ΠΈ вычислСния с Π²Π΅ΠΊΡ‚ΠΎΡ€Π°ΠΌΠΈ

    Π’Π΅ΠΊΡ‚ΠΎΡ€ΠΎΠΌ называСтся направлСнный ΠΎΡ‚Ρ€Π΅Π·ΠΎΠΊ, для ΠΊΠΎΡ‚ΠΎΡ€ΠΎΠ³ΠΎ ΡƒΠΊΠ°Π·Π°Π½ΠΎ Π΅Π³ΠΎ Π½Π°Ρ‡Π°Π»ΠΎ ΠΈ ΠΊΠΎΠ½Π΅Ρ†:

    Π’ Π΄Π°Π½Π½ΠΎΠΌ случаС Π½Π°Ρ‡Π°Π»ΠΎΠΌ ΠΎΡ‚Ρ€Π΅Π·ΠΊΠ° являСтся Ρ‚ΠΎΡ‡ΠΊΠ° А, ΠΊΠΎΠ½Ρ†ΠΎΠΌ ΠΎΡ‚Ρ€Π΅Π·ΠΊΠ° – Ρ‚ΠΎΡ‡ΠΊΠ° Π’. Π‘Π°ΠΌ Π²Π΅ΠΊΡ‚ΠΎΡ€ ΠΎΠ±ΠΎΠ·Π½Π°Ρ‡Π΅Π½ Ρ‡Π΅Ρ€Π΅Π· ΠΈΠ»ΠΈ Β . Π§Ρ‚ΠΎΠ±Ρ‹ Π½Π°ΠΉΡ‚ΠΈ ΠΊΠΎΠΎΡ€Π΄ΠΈΠ½Π°Ρ‚Ρ‹ Π²Π΅ΠΊΡ‚ΠΎΡ€Π° , зная ΠΊΠΎΠΎΡ€Π΄ΠΈΠ½Π°Ρ‚Ρ‹ Π΅Π³ΠΎ Π½Π°Ρ‡Π°Π»ΡŒΠ½ΠΎΠΉ Ρ‚ΠΎΡ‡Π΅ΠΊ А ΠΈ ΠΊΠΎΠ½Π΅Ρ‡Π½ΠΎΠΉ Ρ‚ΠΎΡ‡ΠΊΠΈ Π’, Π½Π΅ΠΎΠ±Ρ…ΠΎΠ΄ΠΈΠΌΠΎ ΠΈΠ· ΠΊΠΎΠΎΡ€Π΄ΠΈΠ½Π°Ρ‚ ΠΊΠΎΠ½Π΅Ρ‡Π½ΠΎΠΉ Ρ‚ΠΎΡ‡ΠΊΠΈ Π²Ρ‹Ρ‡Π΅ΡΡ‚ΡŒ ΡΠΎΠΎΡ‚Π²Π΅Ρ‚ΡΡ‚Π²ΡƒΡŽΡ‰ΠΈΠ΅ ΠΊΠΎΠΎΡ€Π΄ΠΈΠ½Π°Ρ‚Ρ‹ Π½Π°Ρ‡Π°Π»ΡŒΠ½ΠΎΠΉ Ρ‚ΠΎΡ‡ΠΊΠΈ:

    Β = {BxΒ — AxΒ ;Β ByΒ — Ay}

    ΠšΠΎΠ»Π»ΠΈΠ½Π΅Π°Ρ€Π½Ρ‹ΠΌΠΈ Π½Π°Π·Ρ‹Π²Π°ΡŽΡ‚ΡΡ Π²Π΅ΠΊΡ‚ΠΎΡ€Ρ‹, Π»Π΅ΠΆΠ°Ρ‰ΠΈΠ΅ Π½Π° ΠΏΠ°Ρ€Π°Π»Π»Π΅Π»ΡŒΠ½Ρ‹Ρ… прямых ΠΈΠ»ΠΈ Π½Π° ΠΎΠ΄Π½ΠΎΠΉ прямой. ΠŸΡ€ΠΈ этом Π²Π΅ΠΊΡ‚ΠΎΡ€ Β  ΠΎΡ‚Ρ€Π΅Π·ΠΎΠΊ, Ρ…Π°Ρ€Π°ΠΊΡ‚Π΅Ρ€ΠΈΠ·ΡƒΡŽΡ‰ΠΈΠΉΡΡ Π΄Π»ΠΈΠ½ΠΎΠΉ ΠΈ Π½Π°ΠΏΡ€Π°Π²Π»Π΅Π½ΠΈΠ΅ΠΌ.

    Π”Π»ΠΈΠ½Π° Π½Π°ΠΏΡ€Π°Π²Π»Π΅Π½Π½ΠΎΠ³ΠΎ ΠΎΡ‚Ρ€Π΅Π·ΠΊΠ° опрСдСляСт числовоС Π·Π½Π°Ρ‡Π΅Π½ΠΈΠ΅ Π²Π΅ΠΊΡ‚ΠΎΡ€Π° ΠΈ называСтся длиной Π²Π΅ΠΊΡ‚ΠΎΡ€Π° ΠΈΠ»ΠΈ ΠΌΠΎΠ΄ΡƒΠ»Π΅ΠΌ Π²Π΅ΠΊΡ‚ΠΎΡ€Π°.

    Π”Π»ΠΈΠ½Π° Π²Π΅ΠΊΡ‚ΠΎΡ€Π°Β || Π² ΠΏΡ€ΡΠΌΠΎΡƒΠ³ΠΎΠ»ΡŒΠ½Ρ‹Ρ… Π΄Π΅ΠΊΠ°Ρ€Ρ‚ΠΎΠ²Ρ‹Ρ… ΠΊΠΎΠΎΡ€Π΄ΠΈΠ½Π°Ρ‚Π°Ρ… Ρ€Π°Π²Π½Π° ΠΊΠ²Π°Π΄Ρ€Π°Ρ‚Π½ΠΎΠΌΡƒ ΠΊΠΎΡ€Π½ΡŽ ΠΈΠ· суммы ΠΊΠ²Π°Π΄Ρ€Π°Ρ‚ΠΎΠ² Π΅Π³ΠΎ ΠΊΠΎΠΎΡ€Π΄ΠΈΠ½Π°Ρ‚.

    Π‘ Π²Π΅ΠΊΡ‚ΠΎΡ€Π°ΠΌΠΈ ΠΌΠΎΠΆΠ½ΠΎ ΡΠΎΠ²Π΅Ρ€ΡˆΠ°Ρ‚ΡŒ Ρ€Π°Π·Π»ΠΈΡ‡Π½Ρ‹Π΅ дСйствия.

    НапримСр, слоТСниС. Π§Ρ‚ΠΎΠ±Ρ‹ ΠΈΡ… ΡΠ»ΠΎΠΆΠΈΡ‚ΡŒ, Π½ΡƒΠΆΠ½ΠΎ провСсти сначала Π²Ρ‚ΠΎΡ€ΠΎΠΉ Π²Π΅ΠΊΡ‚ΠΎΡ€ ΠΈΠ· ΠΊΠΎΠ½Ρ†Π° ΠΏΠ΅Ρ€Π²ΠΎΠ³ΠΎ, Π° ΠΏΠΎΡ‚ΠΎΠΌ ΡΠΎΠ΅Π΄ΠΈΠ½ΠΈΡ‚ΡŒ Π½Π°Ρ‡Π°Π»ΠΎ ΠΏΠ΅Ρ€Π²ΠΎΠ³ΠΎ с ΠΊΠΎΠ½Ρ†ΠΎΠΌ Π²Ρ‚ΠΎΡ€ΠΎΠ³ΠΎ (рис. 1). Π‘ΡƒΠΌΠΌΠΎΠΉ Π²Π΅ΠΊΡ‚ΠΎΡ€ΠΎΠ² являСтся Π΄Ρ€ΡƒΠ³ΠΎΠΉ Π²Π΅ΠΊΡ‚ΠΎΡ€ с Π½ΠΎΠ²Ρ‹ΠΌΠΈ ΠΊΠΎΠΎΡ€Π΄ΠΈΠ½Π°Ρ‚Π°ΠΌΠΈ.

    Π‘ΡƒΠΌΠΌΡƒ Π²Π΅ΠΊΡ‚ΠΎΡ€ΠΎΠ² Β Β = {axΒ ;Β ay} ΠΈ = {bxΒ ;Β by} ΠΌΠΎΠΆΠ½ΠΎ Π½Π°ΠΉΡ‚ΠΈ воспользовавшись ΡΠ»Π΅Π΄ΡƒΡŽΡ‰Π΅ΠΉ Ρ„ΠΎΡ€ΠΌΡƒΠ»ΠΎΠΉ:

    +Β = {axΒ +Β bx;Β ayΒ +Β by}

    Рис. 1. ДСйствия с Π²Π΅ΠΊΡ‚ΠΎΡ€Π°ΠΌΠΈ

    Вычитая Π²Π΅ΠΊΡ‚ΠΎΡ€Ρ‹, Π½ΡƒΠΆΠ½ΠΎ сначала провСсти ΠΈΡ… ΠΈΠ· ΠΎΠ΄Π½ΠΎΠΉ Ρ‚ΠΎΡ‡ΠΊΠΈ, Π° ΠΏΠΎΡ‚ΠΎΠΌ ΡΠΎΠ΅Π΄ΠΈΠ½ΠΈΡ‚ΡŒ ΠΊΠΎΠ½Π΅Ρ† Π²Ρ‚ΠΎΡ€ΠΎΠ³ΠΎ с ΠΊΠΎΠ½Ρ†ΠΎΠΌ ΠΏΠ΅Ρ€Π²ΠΎΠ³ΠΎ.

    Π Π°Π·Π½ΠΎΡΡ‚ΡŒ Π²Π΅ΠΊΡ‚ΠΎΡ€ΠΎΠ²Β Β = {axΒ ;Β ay} ΠΈΒ Β = {bxΒ ;Β by} ΠΌΠΎΠΆΠ½ΠΎ Π½Π°ΠΉΡ‚ΠΈ ΠΏΠΎ Ρ„ΠΎΡ€ΠΌΡƒΠ»Π΅:

    —Β Β = {axΒ —Β bx;Β ayΒ —Β by}

    Π’Π°ΠΊΠΆΠ΅, Π²Π΅ΠΊΡ‚ΠΎΡ€Ρ‹ ΠΌΠΎΠΆΠ½ΠΎ ΡƒΠΌΠ½ΠΎΠΆΠ°Ρ‚ΡŒ Π½Π° число. Π Π΅Π·ΡƒΠ»ΡŒΡ‚Π°Ρ‚ΠΎΠΌ Ρ‚Π°ΠΊΠΆΠ΅ Π±ΡƒΠ΄Π΅Ρ‚ Π²Π΅ΠΊΡ‚ΠΎΡ€, ΠΊΠΎΡ‚ΠΎΡ€Ρ‹ΠΉ Π² k Ρ€Π°Π· большС (ΠΈΠ»ΠΈ мСньшС) Π΄Π°Π½Π½ΠΎΠ³ΠΎ. Π•Π³ΠΎ Π½Π°ΠΏΡ€Π°Π²Π»Π΅Π½ΠΈΠ΅ Π±ΡƒΠ΄Π΅Ρ‚ Π·Π°Π²ΠΈΡΠ΅Ρ‚ΡŒ ΠΎΡ‚ Π·Π½Π°ΠΊΠ° k: ΠΏΡ€ΠΈ ΠΏΠΎΠ»ΠΎΠΆΠΈΡ‚Π΅Π»ΡŒΠ½ΠΎΠΌ k Π²Π΅ΠΊΡ‚ΠΎΡ€Ρ‹ сонаправлСны, Π° ΠΏΡ€ΠΈ ΠΎΡ‚Ρ€ΠΈΡ†Π°Ρ‚Π΅Π»ΡŒΠ½ΠΎΠΌ – ΠΏΡ€ΠΎΡ‚ΠΈΠ²ΠΎΠΏΠΎΠ»ΠΎΠΆΠ½ΠΎ Π½Π°ΠΏΡ€Π°Π²Π»Π΅Π½Ρ‹.

    ΠŸΡ€ΠΎΠΈΠ·Π²Π΅Π΄Π΅Π½ΠΈΠ΅ Π²Π΅ΠΊΡ‚ΠΎΡ€Π°Β Β = {axΒ ;Β ay} ΠΈ числа k ΠΌΠΎΠΆΠ½ΠΎ Π½Π°ΠΉΡ‚ΠΈ воспользовавшись ΡΠ»Π΅Π΄ΡƒΡŽΡ‰Π΅ΠΉ Ρ„ΠΎΡ€ΠΌΡƒΠ»ΠΎΠΉ:

    kΒ Β·Β Β = {kΒ Β·Β ax;Β kΒ Β·Β ay}

    А ΠΌΠΎΠΆΠ½ΠΎ Π»ΠΈ ΡƒΠΌΠ½ΠΎΠΆΠ°Ρ‚ΡŒ Π²Π΅ΠΊΡ‚ΠΎΡ€ Π½Π° Π²Π΅ΠΊΡ‚ΠΎΡ€? ΠšΠΎΠ½Π΅Ρ‡Π½ΠΎ, ΠΈ Π΄Π°ΠΆΠ΅ двумя Π²Π°Ρ€ΠΈΠ°Π½Ρ‚Π°ΠΌΠΈ!

    ΠŸΠ΅Ρ€Π²Ρ‹ΠΉ Π²Π°Ρ€ΠΈΠ°Π½Ρ‚ – скалярноС ΠΏΡ€ΠΎΠΈΠ·Π²Π΅Π΄Π΅Π½ΠΈΠ΅.

    Рис. 2. БкалярноС ΠΏΡ€ΠΎΠΈΠ·Π²Π΅Π΄Π΅Π½ΠΈΠ΅ Π² ΠΊΠΎΠΎΡ€Π΄ΠΈΠ½Π°Ρ‚Π°Ρ…

    Для нахоТдСния произвСдСния Π²Π΅ΠΊΡ‚ΠΎΡ€ΠΎΠ² ΠΌΠΎΠΆΠ½ΠΎ ΠΈΡΠΏΠΎΠ»ΡŒΠ·ΠΎΠ²Π°Ρ‚ΡŒ ΡƒΠ³ΠΎΠ»  ΠΌΠ΅ΠΆΠ΄Ρƒ Π΄Π°Π½Π½Ρ‹ΠΌΠΈ Π²Π΅ΠΊΡ‚ΠΎΡ€Π°ΠΌΠΈ, ΠΏΠΎΠΊΠ°Π·Π°Π½Π½Ρ‹ΠΉ Π½Π° рисункС 3.

    Из Ρ„ΠΎΡ€ΠΌΡƒΠ»Ρ‹ слСдуСт, Ρ‡Ρ‚ΠΎ скалярноС ΠΏΡ€ΠΎΠΈΠ·Π²Π΅Π΄Π΅Π½ΠΈΠ΅ Ρ€Π°Π²Π½ΠΎ ΠΏΡ€ΠΎΠΈΠ·Π²Π΅Π΄Π΅Π½ΠΈΡŽ Π΄Π»ΠΈΠ½ Π΄Π°Π½Π½Ρ‹Ρ… Π²Π΅ΠΊΡ‚ΠΎΡ€ΠΎΠ² Π½Π° косинус ΡƒΠ³Π»Π° ΠΌΠ΅ΠΆΠ΄Ρƒ Π½ΠΈΠΌΠΈ, Π΅Π³ΠΎ Ρ€Π΅Π·ΡƒΠ»ΡŒΡ‚Π°Ρ‚ΠΎΠΌ являСтся число. Π’Π°ΠΆΠ½ΠΎ, Ρ‡Ρ‚ΠΎ Ссли Π²Π΅ΠΊΡ‚ΠΎΡ€Ρ‹ пСрпСндикулярны, Ρ‚ΠΎ ΠΈΡ… скалярноС ΠΏΡ€ΠΎΠΈΠ·Π²Π΅Π΄Π΅Π½ΠΈΠ΅ Ρ€Π°Π²Π½ΠΎ Π½ΡƒΠ»ΡŽ, Ρ‚.ΠΊ. косинус прямого ΡƒΠ³Π»Π° ΠΌΠ΅ΠΆΠ΄Ρƒ Π½ΠΈΠΌΠΈ Ρ€Π°Π²Π΅Π½ Π½ΡƒΠ»ΡŽ.

    Π’ ΠΊΠΎΠΎΡ€Π΄ΠΈΠ½Π°Ρ‚Π½ΠΎΠΉ плоскости Π²Π΅ΠΊΡ‚ΠΎΡ€ Ρ‚Π°ΠΊΠΆΠ΅ ΠΈΠΌΠ΅Π΅Ρ‚ ΠΊΠΎΠΎΡ€Π΄ΠΈΠ½Π°Ρ‚Ρ‹. Π’Π΅ΠΊΡ‚ΠΎΡ€Π°, ΠΈΡ… ΠΊΠΎΠΎΡ€Π΄ΠΈΠ½Π°Ρ‚Ρ‹ ΠΈ скалярноС ΠΏΡ€ΠΎΠΈΠ·Π²Π΅Π΄Π΅Π½ΠΈΠ΅ ΡΠ²Π»ΡΡŽΡ‚ΡΡ ΠΎΠ΄Π½ΠΈΠΌΠΈ ΠΈΠ· самых ΡƒΠ΄ΠΎΠ±Π½Ρ‹Ρ… ΠΌΠ΅Ρ‚ΠΎΠ΄ΠΎΠ² вычислСния ΡƒΠ³Π»Π° ΠΌΠ΅ΠΆΠ΄Ρƒ прямыми (ΠΈΠ»ΠΈ ΠΈΡ… ΠΎΡ‚Ρ€Π΅Π·ΠΊΠ°ΠΌΠΈ), Ссли Π²Π²Π΅Π΄Π΅Π½Π° систСма ΠΊΠΎΠΎΡ€Π΄ΠΈΠ½Π°Ρ‚. И Ссли ΠΊΠΎΠΎΡ€Π΄ΠΈΠ½Π°Ρ‚Ρ‹ , Ρ‚ΠΎ ΠΈΡ… скалярноС ΠΏΡ€ΠΎΠΈΠ·Π²Π΅Π΄Π΅Π½ΠΈΠ΅ Ρ€Π°Π²Π½ΠΎ:

    Π’ Ρ‚Ρ€Π΅Ρ…ΠΌΠ΅Ρ€Π½ΠΎΠΌ пространствС сущСствуСт 3 оси ΠΈ, соотвСтствСнно, Ρƒ Ρ‚ΠΎΡ‡Π΅ΠΊ ΠΈ Π²Π΅ΠΊΡ‚ΠΎΡ€ΠΎΠ² Π² Ρ‚Π°ΠΊΠΎΠΉ систСмС Π±ΡƒΠ΄Π΅Ρ‚ ΠΏΠΎ 3 ΠΊΠΎΠΎΡ€Π΄ΠΈΠ½Π°Ρ‚Ρ‹, Π° скалярноС ΠΏΡ€ΠΎΠΈΠ·Π²Π΅Π΄Π΅Π½ΠΈΠ΅ Π²Π΅ΠΊΡ‚ΠΎΡ€ΠΎΠ² вычисляСтся ΠΏΠΎ Ρ„ΠΎΡ€ΠΌΡƒΠ»Π΅:

    .

    1.2. Π’Π΅ΠΊΡ‚ΠΎΡ€Π½ΠΎΠ΅ ΠΏΡ€ΠΎΠΈΠ·Π²Π΅Π΄Π΅Π½ΠΈΠ΅ Π²Π΅ΠΊΡ‚ΠΎΡ€ΠΎΠ² Π² Ρ‚Ρ€Π΅Ρ…ΠΌΠ΅Ρ€Π½ΠΎΠΌ пространствС.

    Π’Ρ‚ΠΎΡ€Ρ‹ΠΌ Π²Π°Ρ€ΠΈΠ°Π½Ρ‚ΠΎΠΌ вычислСния произвСдСния Π²Π΅ΠΊΡ‚ΠΎΡ€ΠΎΠ² являСтся Π²Π΅ΠΊΡ‚ΠΎΡ€Π½ΠΎΠ΅ ΠΏΡ€ΠΎΠΈΠ·Π²Π΅Π΄Π΅Π½ΠΈΠ΅. Но, Ρ‡Ρ‚ΠΎΠ±Ρ‹ Π΅Π³ΠΎ ΠΎΠΏΡ€Π΅Π΄Π΅Π»ΠΈΡ‚ΡŒ трСбуСтся ΡƒΠΆΠ΅ Π½Π΅ ΠΏΠ»ΠΎΡΠΊΠΎΡΡ‚ΡŒ, Π° Ρ‚Ρ€Π΅Ρ…ΠΌΠ΅Ρ€Π½ΠΎΠ΅ пространство, Π² ΠΊΠΎΡ‚ΠΎΡ€ΠΎΠΌ Π½Π°Ρ‡Π°Π»ΠΎ ΠΈ ΠΊΠΎΠ½Π΅Ρ† Π²Π΅ΠΊΡ‚ΠΎΡ€Π° ΠΈΠΌΠ΅ΡŽΡ‚ ΠΏΠΎ 3 ΠΊΠΎΠΎΡ€Π΄ΠΈΠ½Π°Ρ‚Ρ‹.

    Π’ ΠΎΡ‚Π»ΠΈΡ‡ΠΈΠ΅ ΠΎΡ‚ скалярного произвСдСния Π²Π΅ΠΊΡ‚ΠΎΡ€ΠΎΠ² Π² Ρ‚Ρ€Ρ‘Ρ…ΠΌΠ΅Ρ€Π½ΠΎΠΌ пространствС опСрация Β«Π²Π΅ΠΊΡ‚ΠΎΡ€Π½ΠΎΠ΅ ΡƒΠΌΠ½ΠΎΠΆΠ΅Π½ΠΈΠ΅Β» Π½Π°Π΄ Π²Π΅ΠΊΡ‚ΠΎΡ€Π°ΠΌΠΈ ΠΏΡ€ΠΈΠ²ΠΎΠ΄ΠΈΡ‚ ΠΊ ΠΈΠ½ΠΎΠΌΡƒ Ρ€Π΅Π·ΡƒΠ»ΡŒΡ‚Π°Ρ‚Ρƒ. Если Π² ΠΏΡ€Π΅Π΄Ρ‹Π΄ΡƒΡ‰Π΅ΠΌ случаС скалярного умноТСния Π΄Π²ΡƒΡ… Π²Π΅ΠΊΡ‚ΠΎΡ€ΠΎΠ² Ρ€Π΅Π·ΡƒΠ»ΡŒΡ‚Π°Ρ‚ΠΎΠΌ Π±Ρ‹Π»ΠΎ число, Ρ‚ΠΎ Π² случаС Π²Π΅ΠΊΡ‚ΠΎΡ€Π½ΠΎΠ³ΠΎ умноТСния Π²Π΅ΠΊΡ‚ΠΎΡ€ΠΎΠ² Ρ€Π΅Π·ΡƒΠ»ΡŒΡ‚Π°Ρ‚ΠΎΠΌ Π±ΡƒΠ΄Π΅Ρ‚ Π΄Ρ€ΡƒΠ³ΠΎΠΉ Π²Π΅ΠΊΡ‚ΠΎΡ€, пСрпСндикулярный ΠΎΠ±ΠΎΠΈΠΌ Π²ΡΡ‚ΡƒΠΏΠΈΠ²ΡˆΠΈΠΌ Π² ΠΏΡ€ΠΎΠΈΠ·Π²Π΅Π΄Π΅Π½ΠΈΠ΅ Π²Π΅ΠΊΡ‚ΠΎΡ€Π°ΠΌ. ΠŸΠΎΡΡ‚ΠΎΠΌΡƒ это ΠΏΡ€ΠΎΠΈΠ·Π²Π΅Π΄Π΅Π½ΠΈΠ΅ Π²Π΅ΠΊΡ‚ΠΎΡ€ΠΎΠ² называСтся Π²Π΅ΠΊΡ‚ΠΎΡ€Π½Ρ‹ΠΌ.

    ΠžΡ‡Π΅Π²ΠΈΠ΄Π½ΠΎ, Ρ‡Ρ‚ΠΎ ΠΏΡ€ΠΈ построСнии Ρ€Π΅Π·ΡƒΠ»ΡŒΡ‚ΠΈΡ€ΡƒΡŽΡ‰Π΅Π³ΠΎ Π²Π΅ΠΊΡ‚ΠΎΡ€Π° , пСрпСндикулярного Π΄Π²ΡƒΠΌ, Π²ΡΡ‚ΡƒΠΏΠΈΠ²ΡˆΠΈΠΌ Π² ΠΏΡ€ΠΎΠΈΠ·Π²Π΅Π΄Π΅Π½ΠΈΠ΅ — ΠΈ , ΠΌΠΎΠΆΠ΅Ρ‚ Π±Ρ‹Ρ‚ΡŒ Π²Ρ‹Π±Ρ€Π°Π½ΠΎ Π΄Π²Π° ΠΏΡ€ΠΎΡ‚ΠΈΠ²ΠΎΠΏΠΎΠ»ΠΎΠΆΠ½Ρ‹Ρ… направлСния. ΠŸΡ€ΠΈ этом Π½Π°ΠΏΡ€Π°Π²Π»Π΅Π½ΠΈΠ΅ Ρ€Π΅Π·ΡƒΠ»ΡŒΡ‚ΠΈΡ€ΡƒΡŽΡ‰Π΅Π³ΠΎ Π²Π΅ΠΊΡ‚ΠΎΡ€Π° опрСдСляСтся ΠΏΠΎ ΠΏΡ€Π°Π²ΠΈΠ»Ρƒ ΠΏΡ€Π°Π²ΠΎΠΉ Ρ€ΡƒΠΊΠΈ, ΠΈΠ»ΠΈ ΠΏΡ€Π°Π²ΠΈΠ»Ρƒ Π±ΡƒΡ€Π°Π²Ρ‡ΠΈΠΊΠ°. Если Π½Π°Ρ€ΠΈΡΠΎΠ²Π°Ρ‚ΡŒ Π²Π΅ΠΊΡ‚ΠΎΡ€Ρ‹ Ρ‚Π°ΠΊ, Ρ‡Ρ‚ΠΎΠ±Ρ‹ ΠΈΡ… Π½Π°Ρ‡Π°Π»Π° совпадали ΠΈ Π²Ρ€Π°Ρ‰Π°Ρ‚ΡŒ ΠΏΠ΅Ρ€Π²Ρ‹ΠΉ Π²Π΅ΠΊΡ‚ΠΎΡ€-ΡΠΎΠΌΠ½ΠΎΠΆΠΈΡ‚Π΅Π»ΡŒ ΠΊΡ€Π°Ρ‚Ρ‡Π°ΠΉΡˆΠΈΠΌ ΠΎΠ±Ρ€Π°Π·ΠΎΠΌ ΠΊΠΎ Π²Ρ‚ΠΎΡ€ΠΎΠΌΡƒ Π²Π΅ΠΊΡ‚ΠΎΡ€Ρƒ-ΡΠΎΠΌΠ½ΠΎΠΆΠΈΡ‚Π΅Π»ΡŽ, Π° Ρ‡Π΅Ρ‚Ρ‹Ρ€Π΅ ΠΏΠ°Π»ΡŒΡ†Π° ΠΏΡ€Π°Π²ΠΎΠΉ Ρ€ΡƒΠΊΠΈ ΠΏΠΎΠΊΠ°Π·Ρ‹Π²Π°Π»ΠΈ Π½Π°ΠΏΡ€Π°Π²Π»Π΅Π½ΠΈΠ΅ вращСния (ΠΊΠ°ΠΊ Π±Ρ‹ охватывая Π²Ρ€Π°Ρ‰Π°ΡŽΡ‰ΠΈΠΉΡΡ Ρ†ΠΈΠ»ΠΈΠ½Π΄Ρ€), Ρ‚ΠΎ ΠΎΡ‚Ρ‚ΠΎΠΏΡ‹Ρ€Π΅Π½Π½Ρ‹ΠΉ большой ΠΏΠ°Π»Π΅Ρ† ΠΏΠΎΠΊΠ°ΠΆΠ΅Ρ‚ Π½Π°ΠΏΡ€Π°Π²Π»Π΅Π½ΠΈΠ΅ Π²Π΅ΠΊΡ‚ΠΎΡ€Π°-произвСдСния (рис. 7).

    Рис. 7. ΠŸΡ€Π°Π²ΠΈΠ»ΠΎ ΠΏΡ€Π°Π²ΠΎΠΉ Ρ€ΡƒΠΊΠΈ

    1.3. Бвойства Π²Π΅ΠΊΡ‚ΠΎΡ€Π½ΠΎΠ³ΠΎ произвСдСния Π²Π΅ΠΊΡ‚ΠΎΡ€ΠΎΠ².

    Π”Π»ΠΈΠ½Π° Ρ€Π΅Π·ΡƒΠ»ΡŒΡ‚ΠΈΡ€ΡƒΡŽΡ‰Π΅Π³ΠΎ Π²Π΅ΠΊΡ‚ΠΎΡ€Π° опрСдСляСтся ΠΏΠΎ Ρ„ΠΎΡ€ΠΌΡƒΠ»Π΅

    .

    ΠŸΡ€ΠΈ этом Β  Π²Π΅ΠΊΡ‚ΠΎΡ€Π½ΠΎΠ΅ ΠΏΡ€ΠΎΠΈΠ·Π²Π΅Π΄Π΅Π½ΠΈΠ΅. Как Π±Ρ‹Π»ΠΎ сказано Π²Ρ‹ΡˆΠ΅, Ρ€Π΅Π·ΡƒΠ»ΡŒΡ‚ΠΈΡ€ΡƒΡŽΡ‰ΠΈΠΉ Π²Π΅ΠΊΡ‚ΠΎΡ€ Π±ΡƒΠ΄Π΅Ρ‚ пСрпСндикулярСн , Π° Π΅Π³ΠΎ Π½Π°ΠΏΡ€Π°Π²Π»Π΅Π½ΠΈΠ΅ опрСдСляСтся ΠΏΠΎ ΠΏΡ€Π°Π²ΠΈΠ»Ρƒ ΠΏΡ€Π°Π²ΠΎΠΉ Ρ€ΡƒΠΊΠΈ.

    Π’Π΅ΠΊΡ‚ΠΎΡ€Π½ΠΎΠ΅ ΠΏΡ€ΠΎΠΈΠ·Π²Π΅Π΄Π΅Π½ΠΈΠ΅ зависит ΠΎΡ‚ порядка сомноТитСлСй, ΠΈΠΌΠ΅Π½Π½ΠΎ:

    .

    Π’Π΅ΠΊΡ‚ΠΎΡ€Π½ΠΎΠ΅ ΠΏΡ€ΠΎΠΈΠ·Π²Π΅Π΄Π΅Π½ΠΈΠ΅ Π½Π΅Π½ΡƒΠ»Π΅Π²Ρ‹Ρ… Π²Π΅ΠΊΡ‚ΠΎΡ€ΠΎΠ² Ρ€Π°Π²Π½ΠΎ 0, Ссли ΠΎΠ½ΠΈ ΠΊΠΎΠ»Π»ΠΈΠ½Π΅Π°Ρ€Π½Ρ‹, Ρ‚ΠΎΠ³Π΄Π° синус ΡƒΠ³Π»Π° ΠΌΠ΅ΠΆΠ΄Ρƒ Π½ΠΈΠΌΠΈ Π±ΡƒΠ΄Π΅Ρ‚ Ρ€Π°Π²Π΅Π½ 0.

    ΠšΠΎΠΎΡ€Π΄ΠΈΠ½Π°Ρ‚Ρ‹ Π²Π΅ΠΊΡ‚ΠΎΡ€ΠΎΠ² Π² Ρ‚Ρ€Π΅Ρ…ΠΌΠ΅Ρ€Π½ΠΎΠΌ пространствС Π²Ρ‹Ρ€Π°ΠΆΠ°ΡŽΡ‚ΡΡ ΡΠ»Π΅Π΄ΡƒΡŽΡ‰ΠΈΠΌ ΠΎΠ±Ρ€Π°Π·ΠΎΠΌ: . Π’ΠΎΠ³Π΄Π° ΠΊΠΎΠΎΡ€Π΄ΠΈΠ½Π°Ρ‚Ρ‹ Ρ€Π΅Π·ΡƒΠ»ΡŒΡ‚ΠΈΡ€ΡƒΡŽΡ‰Π΅Π³ΠΎ Π²Π΅ΠΊΡ‚ΠΎΡ€Π° Π½Π°Ρ…ΠΎΠ΄ΠΈΠΌ ΠΏΠΎ Ρ„ΠΎΡ€ΠΌΡƒΠ»Π΅

    .

    Π”Π»ΠΈΠ½Π° Ρ€Π΅Π·ΡƒΠ»ΡŒΡ‚ΠΈΡ€ΡƒΡŽΡ‰Π΅Π³ΠΎ Π²Π΅ΠΊΡ‚ΠΎΡ€Π° находится ΠΏΠΎ Ρ„ΠΎΡ€ΠΌΡƒΠ»Π΅:

    .



    2. ΠŸΡ€Π°ΠΊΡ‚ΠΈΡ‡Π΅ΡΠΊΠ°Ρ Ρ‡Π°ΡΡ‚ΡŒ.

    2.1. Бвязь Π²Π΅ΠΊΡ‚ΠΎΡ€Π½ΠΎΠ³ΠΎ произвСдСния с ΠΏΠ»ΠΎΡ‰Π°Π΄ΡŒΡŽ Ρ‚Ρ€Π΅ΡƒΠ³ΠΎΠ»ΡŒΠ½ΠΈΠΊΠ° ΠΈ ΠΏΠ°Ρ€Π°Π»Π»Π΅Π»ΠΎΠ³Ρ€Π°ΠΌΠΌΠ° Π² плоскости. ГСомСтричСский смысл Π²Π΅ΠΊΡ‚ΠΎΡ€Π½ΠΎΠ³ΠΎ произвСдСния Π²Π΅ΠΊΡ‚ΠΎΡ€ΠΎΠ².

    ΠŸΡƒΡΡ‚ΡŒ Π½Π°ΠΌ Π΄Π°Π½ Ρ‚Ρ€Π΅ΡƒΠ³ΠΎΠ»ΡŒΠ½ΠΈΠΊ ABC (рис. 8). Π˜Π·Π²Π΅ΡΡ‚Π½ΠΎ, Ρ‡Ρ‚ΠΎ .

    Если ΠΏΡ€Π΅Π΄ΡΡ‚Π°Π²ΠΈΡ‚ΡŒ стороны Ρ‚Ρ€Π΅ΡƒΠ³ΠΎΠ»ΡŒΠ½ΠΈΠΊΠ° АВ ΠΈ АБ Π² Π²ΠΈΠ΄Π΅ Π΄Π²ΡƒΡ… Π²Π΅ΠΊΡ‚ΠΎΡ€ΠΎΠ², Ρ‚ΠΎ Π² Ρ„ΠΎΡ€ΠΌΡƒΠ»Π΅ ΠΏΠ»ΠΎΡ‰Π°Π΄ΠΈ Ρ‚Ρ€Π΅ΡƒΠ³ΠΎΠ»ΡŒΠ½ΠΈΠΊΠ° ΠΌΡ‹ Π½Π°Ρ…ΠΎΠ΄ΠΈΠΌ Π²Ρ‹Ρ€Π°ΠΆΠ΅Π½ΠΈΠ΅ Π²Π΅ΠΊΡ‚ΠΎΡ€Π½ΠΎΠ³ΠΎ произвСдСния Π²Π΅ΠΊΡ‚ΠΎΡ€ΠΎΠ²:

    Из Π²Ρ‹ΡˆΠ΅ сказанного ΠΌΠΎΠΆΠ½ΠΎ ΠΎΠΏΡ€Π΅Π΄Π΅Π»ΠΈΡ‚ΡŒ гСомСтричСский смысл Π²Π΅ΠΊΡ‚ΠΎΡ€Π½ΠΎΠ³ΠΎ произвСдСния (рис. 9):

    Π΄Π»ΠΈΠ½Π° Π²Π΅ΠΊΡ‚ΠΎΡ€Π½ΠΎΠ³ΠΎ произвСдСния Π²Π΅ΠΊΡ‚ΠΎΡ€ΠΎΠ² Ρ€Π°Π²Π½Π° ΡƒΠ΄Π²ΠΎΠ΅Π½Π½ΠΎΠΉ ΠΏΠ»ΠΎΡ‰Π°Π΄ΠΈ Ρ‚Ρ€Π΅ΡƒΠ³ΠΎΠ»ΡŒΠ½ΠΈΠΊΠ°, ΠΈΠΌΠ΅ΡŽΡ‰Π΅Π³ΠΎ сторонами Π²Π΅ΠΊΡ‚ΠΎΡ€Ρ‹Β Β ΠΈΒ , Ссли ΠΈΡ… ΠΎΡ‚Π»ΠΎΠΆΠΈΡ‚ΡŒ ΠΎΡ‚ ΠΎΠ΄Π½ΠΎΠΉ Ρ‚ΠΎΡ‡ΠΊΠΈ.

    Π”Ρ€ΡƒΠ³ΠΈΠΌΠΈ словами, Π΄Π»ΠΈΠ½Π° Π²Π΅ΠΊΡ‚ΠΎΡ€Π½ΠΎΠ³ΠΎ произвСдСния Π²Π΅ΠΊΡ‚ΠΎΡ€ΠΎΠ²Β Β ΠΈΒ Β Ρ€Π°Π²Π½Π° ΠΏΠ»ΠΎΡ‰Π°Π΄ΠΈ ΠΏΠ°Ρ€Π°Π»Π»Π΅Π»ΠΎΠ³Ρ€Π°ΠΌΠΌΠ°, построСнного Π½Π° Π²Π΅ΠΊΡ‚ΠΎΡ€Π°Ρ…Β Β ΠΈΒ , со сторонами  и  и ΡƒΠ³Π»ΠΎΠΌ ΠΌΠ΅ΠΆΠ΄Ρƒ Π½ΠΈΠΌΠΈ, Ρ€Π°Π²Π½Ρ‹ΠΌΒ .

    .

    Рис. 9. ГСомСтричСский смысл Π²Π΅ΠΊΡ‚ΠΎΡ€Π½ΠΎΠ³ΠΎ произвСдСния Π²Π΅ΠΊΡ‚ΠΎΡ€ΠΎΠ²

    Π’ связи с этим, ΠΌΠΎΠΆΠ½ΠΎ привСсти Π΅Ρ‰Π΅ ΠΎΠ΄Π½ΠΎ ΠΎΠΏΡ€Π΅Π΄Π΅Π»Π΅Π½ΠΈΠ΅ Π²Π΅ΠΊΡ‚ΠΎΡ€Π½ΠΎΠ³ΠΎ произвСдСния Π²Π΅ΠΊΡ‚ΠΎΡ€ΠΎΠ²:

    Π’Π΅ΠΊΡ‚ΠΎΡ€Π½Ρ‹ΠΌ ΠΏΡ€ΠΎΠΈΠ·Π²Π΅Π΄Π΅Π½ΠΈΠ΅ΠΌ Π²Π΅ΠΊΡ‚ΠΎΡ€Π° Β Π½Π° Π²Π΅ΠΊΡ‚ΠΎΡ€ Β  называСтся Π²Π΅ΠΊΡ‚ΠΎΡ€ , Π΄Π»ΠΈΠ½Π° ΠΊΠΎΡ‚ΠΎΡ€ΠΎΠ³ΠΎ числСнно Ρ€Π°Π²Π½Π° ΠΏΠ»ΠΎΡ‰Π°Π΄ΠΈ ΠΏΠ°Ρ€Π°Π»Π»Π΅Π»ΠΎΠ³Ρ€Π°ΠΌΠΌΠ° построСнного Π½Π° Π²Π΅ΠΊΡ‚ΠΎΡ€Π°Ρ…Β  ΠΈΒ , пСрпСндикулярный ΠΊ плоскости этих Π²Π΅ΠΊΡ‚ΠΎΡ€ΠΎΠ² ΠΈ Π½Π°ΠΏΡ€Π°Π²Π»Π΅Π½Π½Ρ‹ΠΉ Ρ‚Π°ΠΊ, Ρ‡Ρ‚ΠΎΠ± наимСньшСС Π²Ρ€Π°Ρ‰Π΅Π½ΠΈΠ΅ ΠΎΡ‚Β Β ΠΊΒ  Π²ΠΎΠΊΡ€ΡƒΠ³ Π²Π΅ΠΊΡ‚ΠΎΡ€Π°Β  ΠΎΡΡƒΡ‰Π΅ΡΡ‚Π²Π»ΡΠ»ΠΎΡΡŒ ΠΏΡ€ΠΎΡ‚ΠΈΠ² часовой стрСлки, Ссли ΡΠΌΠΎΡ‚Ρ€Π΅Ρ‚ΡŒ с ΠΊΠΎΠ½Ρ†Π° Π²Π΅ΠΊΡ‚ΠΎΡ€Π°Β Β (рис. 10).


    Рис. 10. ΠžΠΏΡ€Π΅Π΄Π΅Π»Π΅Π½ΠΈΠ΅ Π²Π΅ΠΊΡ‚ΠΎΡ€Π½ΠΎΠ³ΠΎ произвСдСния Π²Π΅ΠΊΡ‚ΠΎΡ€ΠΎΠ²

    с использованиСм ΠΏΠ°Ρ€Π°Π»Π»Π΅Π»ΠΎΠ³Ρ€Π°ΠΌΠΌΠ°

    2.2. Π’Ρ‹Π²ΠΎΠ΄ Ρ„ΠΎΡ€ΠΌΡƒΠ»Ρ‹ для нахоТдСния ΠΏΠ»ΠΎΡ‰Π°Π΄ΠΈ Ρ‚Ρ€Π΅ΡƒΠ³ΠΎΠ»ΡŒΠ½ΠΈΠΊΠ° Π² ΠΊΠΎΠΎΡ€Π΄ΠΈΠ½Π°Ρ‚Π°Ρ….

    Π˜Ρ‚Π°ΠΊ, Π½Π°ΠΌ Π΄Π°Π½ Ρ‚Ρ€Π΅ΡƒΠ³ΠΎΠ»ΡŒΠ½ΠΈΠΊ АВБ Π² плоскости ΠΈ ΠΊΠΎΠΎΡ€Π΄ΠΈΠ½Π°Ρ‚Ρ‹ Π΅Π³ΠΎ Π²Π΅Ρ€ΡˆΠΈΠ½. НайдСм ΠΏΠ»ΠΎΡ‰Π°Π΄ΡŒ этого Ρ‚Ρ€Π΅ΡƒΠ³ΠΎΠ»ΡŒΠ½ΠΈΠΊΠ° (рис. 11).

    Рис. 11. ΠŸΡ€ΠΈΠΌΠ΅Ρ€ Ρ€Π΅ΡˆΠ΅Π½ΠΈΡ Π·Π°Π΄Π°Ρ‡ΠΈ Π½Π° Π½Π°Ρ…ΠΎΠΆΠ΄Π΅Π½ΠΈΠ΅ ΠΏΠ»ΠΎΡ‰Π°Π΄ΠΈ Ρ‚Ρ€Π΅ΡƒΠ³ΠΎΠ»ΡŒΠ½ΠΈΠΊΠ° ΠΏΠΎ ΠΊΠΎΠΎΡ€Π΄ΠΈΠ½Π°Ρ‚Π°ΠΌ Π΅Π³ΠΎ Π²Π΅Ρ€ΡˆΠΈΠ½

    РСшСниС.

    Для Π½Π°Ρ‡Π°Π»Π°, рассмотрим ΠΊΠΎΠΎΡ€Π΄ΠΈΠ½Π°Ρ‚Ρ‹ Π²Π΅Ρ€ΡˆΠΈΠ½ Π² пространствС ΠΈ вычислим ΠΊΠΎΠΎΡ€Π΄ΠΈΠ½Π°Ρ‚Ρ‹ Π²Π΅ΠΊΡ‚ΠΎΡ€ΠΎΠ² АВ ΠΈ АБ.

    По Π΄Π°Π½Π½ΠΎΠΉ ΠΏΡ€Π΅ΠΆΠ΄Π΅ Ρ„ΠΎΡ€ΠΌΡƒΠ»Π΅ подсчитаСм ΠΊΠΎΠΎΡ€Π΄ΠΈΠ½Π°Ρ‚Ρ‹ ΠΈΡ… Π²Π΅ΠΊΡ‚ΠΎΡ€Π½ΠΎΠ³ΠΎ произвСдСния. Π”Π»ΠΈΠ½Π° этого Π²Π΅ΠΊΡ‚ΠΎΡ€Π° Ρ€Π°Π²Π½Π° 2 площадям Ρ‚Ρ€Π΅ΡƒΠ³ΠΎΠ»ΡŒΠ½ΠΈΠΊΠ° АВБ. ΠŸΠ»ΠΎΡ‰Π°Π΄ΡŒ Ρ‚Ρ€Π΅ΡƒΠ³ΠΎΠ»ΡŒΠ½ΠΈΠΊΠ° Ρ€Π°Π²Π½Π° 10.

    Π‘ΠΎΠ»Π΅Π΅ Ρ‚ΠΎΠ³ΠΎ, Ссли ΠΌΡ‹ рассмотрим Ρ‚Ρ€Π΅ΡƒΠ³ΠΎΠ»ΡŒΠ½ΠΈΠΊ Π½Π° плоскости, Ρ‚ΠΎ ΠΏΠ΅Ρ€Π²Ρ‹Π΅ 2 ΠΊΠΎΠΎΡ€Π΄ΠΈΠ½Π°Ρ‚Ρ‹ Π²Π΅ΠΊΡ‚ΠΎΡ€Π½ΠΎΠ³ΠΎ произвСдСния всСгда Π±ΡƒΠ΄ΡƒΡ‚ Ρ€Π°Π²Π½Ρ‹ Π½ΡƒΠ»ΡŽ, поэтому ΠΌΡ‹ ΠΌΠΎΠΆΠ΅ΠΌ ΡΡ„ΠΎΡ€ΠΌΡƒΠ»ΠΈΡ€ΠΎΠ²Π°Ρ‚ΡŒ ΡΠ»Π΅Π΄ΡƒΡŽΡ‰ΡƒΡŽ Ρ‚Π΅ΠΎΡ€Π΅ΠΌΡƒ.

    Π’Π΅ΠΎΡ€Π΅ΠΌΠ°: ΠŸΡƒΡΡ‚ΡŒ Π΄Π°Π½ Ρ‚Ρ€Π΅ΡƒΠ³ΠΎΠ»ΡŒΠ½ΠΈΠΊ АВБ ΠΈ ΠΊΠΎΠΎΡ€Π΄ΠΈΠ½Π°Ρ‚Ρ‹ Π΅Π³ΠΎ Π²Π΅Ρ€ΡˆΠΈΠ½ (рис. 12).

    Π’ΠΎΠ³Π΄Π° .

    Рис. 12. Π”ΠΎΠΊΠ°Π·Π°Ρ‚Π΅Π»ΡŒΡΡ‚Π²ΠΎ Ρ‚Π΅ΠΎΡ€Π΅ΠΌΡ‹

    Π”ΠΎΠΊΠ°Π·Π°Ρ‚Π΅Π»ΡŒΡΡ‚Π²ΠΎ.

    Рассмотрим Ρ‚ΠΎΡ‡ΠΊΠΈ Π² пространствС ΠΈ вычислим ΠΊΠΎΠΎΡ€Π΄ΠΈΠ½Π°Ρ‚Ρ‹ Π²Π΅ΠΊΡ‚ΠΎΡ€ΠΎΠ² Π’Π‘ ΠΈ ВА. . По ΠΏΡ€ΠΈΠ²Π΅Π΄Π΅Π½Π½ΠΎΠΉ Ρ€Π°Π½ΡŒΡˆΠ΅ Ρ„ΠΎΡ€ΠΌΡƒΠ»Π΅ вычислим ΠΊΠΎΠΎΡ€Π΄ΠΈΠ½Π°Ρ‚Ρ‹ Π²Π΅ΠΊΡ‚ΠΎΡ€Π½ΠΎΠ³ΠΎ произвСдСния этих Π²Π΅ΠΊΡ‚ΠΎΡ€ΠΎΠ². ΠžΠ±Ρ€Π°Ρ‚ΠΈΠΌ Π²Π½ΠΈΠΌΠ°Π½ΠΈΠ΅, Ρ‡Ρ‚ΠΎ всС Ρ‡Π»Π΅Π½Ρ‹, содСрТащиС z1 ΠΈΠ»ΠΈ z2, Ρ€Π°Π²Π½Ρ‹ 0, Ρ‚.ΠΊ. z1ΠΈ z2 = 0. УБРАВЬ!!!

    Π˜Ρ‚Π°ΠΊ, ΡΠ»Π΅Π΄ΠΎΠ²Π°Ρ‚Π΅Π»ΡŒΠ½ΠΎ,

    2.3. ΠŸΡ€ΠΎΠ²Π΅Ρ€ΠΊΠ° ΠΏΡ€Π°Π²ΠΈΠ»ΡŒΠ½ΠΎΡΡ‚ΠΈ Ρ„ΠΎΡ€ΠΌΡƒΠ»Ρ‹ Π½Π° ΠΏΡ€ΠΈΠΌΠ΅Ρ€Π°Ρ…

    Найти ΠΏΠ»ΠΎΡ‰Π°Π΄ΡŒ Ρ‚Ρ€Π΅ΡƒΠ³ΠΎΠ»ΡŒΠ½ΠΈΠΊΠ° ΠΎΠ±Ρ€Π°Π·ΠΎΠ²Π°Π½Π½ΠΎΠ³ΠΎ Π²Π΅ΠΊΡ‚ΠΎΡ€Π°ΠΌΠΈΒ aΒ =Β {-1;Β 2;Β -2} ΠΈΒ bΒ =Β {2;Β 1;Β -1}.

    РСшСниС: НайдСм Π²Π΅ΠΊΡ‚ΠΎΡ€Π½ΠΎΠ΅ ΠΏΡ€ΠΎΠΈΠ·Π²Π΅Π΄Π΅Π½ΠΈΠ΅ этих Π²Π΅ΠΊΡ‚ΠΎΡ€ΠΎΠ²:

    aΒ Γ—Β b=Β 

    Β iΒ 

    Β jΒ 

    Β kΒ 

    Β =

    Β -1Β 

    Β 2Β 

    Β -2Β 

    Β 2Β 

    Β 1Β 

    Β -1Β 

    =Β i(2 Β· (-1) — (-2) Β· 1) —Β j((-1) Β· (-1) — (-2) Β· 2) +Β k((-1) Β· 1 — 2 Β· 2) =

    =Β i(-2 + 2) —Β j(1 + 4) +Β k(-1 — 4) = -5jΒ — 5kΒ = {0; -5; -5}

    Из свойств Π²Π΅ΠΊΡ‚ΠΎΡ€Π½ΠΎΠ³ΠΎ произвСдСния:

    SΞ” =Β 

    1

    |aΒ Γ—Β b| =Β 

    1

    √02 + 52 + 52 = 

    1

    √25 + 25 = 

    1

    √50 = 

    5√2

    2

    2

    2

    2

    2

    ΠžΡ‚Π²Π΅Ρ‚:Β SΞ” = 2.5√2.

    Π—Π°ΠΊΠ»ΡŽΡ‡Π΅Π½ΠΈΠ΅

    2.4. ΠŸΡ€ΠΈΠ»ΠΎΠΆΠ΅Π½ΠΈΡ Π²Π΅ΠΊΡ‚ΠΎΡ€Π½ΠΎΠΉ Π°Π»Π³Π΅Π±Ρ€Ρ‹

    ΠΈ скалярного ΠΈ Π²Π΅ΠΊΡ‚ΠΎΡ€Π½ΠΎΠ³ΠΎ произвСдСния Π²Π΅ΠΊΡ‚ΠΎΡ€ΠΎΠ².

    Π“Π΄Π΅ ΠΆΠ΅ Π½ΡƒΠΆΠ½Ρ‹ Π²Π΅ΠΊΡ‚ΠΎΡ€Ρ‹? Π’Π΅ΠΊΡ‚ΠΎΡ€Π½ΠΎΠ΅ пространство ΠΈ Π²Π΅ΠΊΡ‚ΠΎΡ€Ρ‹ носят Π½Π΅ Ρ‚ΠΎΠ»ΡŒΠΊΠΎ тСорСтичСский Ρ…Π°Ρ€Π°ΠΊΡ‚Π΅Ρ€, Π½ΠΎ ΠΈ ΠΈΠΌΠ΅ΡŽΡ‚ Π²ΠΏΠΎΠ»Π½Π΅ Ρ€Π΅Π°Π»ΡŒΠ½ΠΎΠ΅ практичСскоС ΠΏΡ€ΠΈΠΌΠ΅Π½Π΅Π½ΠΈΠ΅ Π² соврСмСнном ΠΌΠΈΡ€Π΅.

    Π’ ΠΌΠ΅Ρ…Π°Π½ΠΈΠΊΠ΅ ΠΈ Ρ„ΠΈΠ·ΠΈΠΊΠ΅ ΠΌΠ½ΠΎΠ³ΠΈΠ΅ Π²Π΅Π»ΠΈΡ‡ΠΈΠ½Ρ‹ ΠΈΠΌΠ΅ΡŽΡ‚ Π½Π΅ Ρ‚ΠΎΠ»ΡŒΠΊΠΎ числСнноС Π·Π½Π°Ρ‡Π΅Π½ΠΈΠ΅, Π½ΠΎ ΠΈ Π½Π°ΠΏΡ€Π°Π²Π»Π΅Π½ΠΈΠ΅. Π’Π°ΠΊΠΈΠ΅ Π²Π΅Π»ΠΈΡ‡ΠΈΠ½Ρ‹ Π½Π°Π·Ρ‹Π²Π°ΡŽΡ‚ΡΡ Π²Π΅ΠΊΡ‚ΠΎΡ€Π½Ρ‹ΠΌΠΈ. ВмСстС с использованиСм элСмСнтарных мСханичСских понятий, ΠΎΠΏΠΈΡ€Π°ΡΡΡŒ Π½Π° ΠΈΡ… физичСский смысл, ΠΌΠ½ΠΎΠ³ΠΈΠ΅ Π²Π΅Π»ΠΈΡ‡ΠΈΠ½Ρ‹ Ρ€Π°ΡΡΠΌΠ°Ρ‚Ρ€ΠΈΠ²Π°ΡŽΡ‚ΡΡ ΠΊΠ°ΠΊ ΡΠΊΠΎΠ»ΡŒΠ·ΡΡ‰ΠΈΠ΅ Π²Π΅ΠΊΡ‚ΠΎΡ€Ρ‹, Π° ΠΈΡ… свойства ΠΎΠΏΠΈΡΡ‹Π²Π°ΡŽΡ‚ΡΡ ΠΊΠ°ΠΊ аксиомами, ΠΊΠ°ΠΊ это принято Π² тСорСтичСской ΠΌΠ΅Ρ…Π°Π½ΠΈΠΊΠ΅, Ρ‚Π°ΠΊ ΠΈ ΠΏΡ€ΠΈ ΠΏΠΎΠΌΠΎΡ‰ΠΈ матСматичСских свойств Π²Π΅ΠΊΡ‚ΠΎΡ€ΠΎΠ². НаиболСС яркими ΠΏΡ€ΠΈΠΌΠ΅Ρ€Π°ΠΌΠΈ Π²Π΅ΠΊΡ‚ΠΎΡ€Π½Ρ‹Ρ… Π²Π΅Π»ΠΈΡ‡ΠΈΠ½ ΡΠ²Π»ΡΡŽΡ‚ΡΡ ΡΠΊΠΎΡ€ΠΎΡΡ‚ΡŒ, ΠΈΠΌΠΏΡƒΠ»ΡŒΡ ΠΈ сила (рис. 12). НапримСр, ΠΌΠΎΠΌΠ΅Π½Ρ‚ ΠΈΠΌΠΏΡƒΠ»ΡŒΡΠ° ΠΈ сила Π›ΠΎΡ€Π΅Π½Ρ†Π° матСматичСски Π·Π°ΠΏΠΈΡΡ‹Π²Π°ΡŽΡ‚ΡΡ с ΠΏΠΎΠΌΠΎΡ‰ΡŒΡŽ Π²Π΅ΠΊΡ‚ΠΎΡ€ΠΎΠ².

    Π’ Ρ„ΠΈΠ·ΠΈΠΊΠ΅ Π²Π°ΠΆΠ½Ρ‹ Π½Π΅ Ρ‚ΠΎΠ»ΡŒΠΊΠΎ сами Π²Π΅ΠΊΡ‚ΠΎΡ€Π°, Π½ΠΎ Π² большой стСпСни Π²Π°ΠΆΠ½Ρ‹ ΠΈ ΠΈΡ… произвСдСния, ΠΊΠΎΡ‚ΠΎΡ€Ρ‹Π΅ ΠΏΠΎΠΌΠΎΠ³Π°ΡŽΡ‚ Π²Ρ‹Ρ‡ΠΈΡΠ»ΡΡ‚ΡŒ Π½Π΅ΠΊΠΎΡ‚ΠΎΡ€Ρ‹Π΅ Π²Π΅Π»ΠΈΡ‡ΠΈΠ½Ρ‹. Π’Π΅ΠΊΡ‚ΠΎΡ€Π½ΠΎΠ΅ ΠΏΡ€ΠΎΠΈΠ·Π²Π΅Π΄Π΅Π½ΠΈΠ΅ ΠΏΠΎΠ»Π΅Π·Π½ΠΎ для опрСдСлСния коллинСарности Π²Π΅ΠΊΡ‚ΠΎΡ€ΠΎΠ² Β  ΠΌΠΎΠ΄ΡƒΠ»ΡŒ Π²Π΅ΠΊΡ‚ΠΎΡ€Π½ΠΎΠ³ΠΎ произвСдСния Π΄Π²ΡƒΡ… Π²Π΅ΠΊΡ‚ΠΎΡ€ΠΎΠ² Ρ€Π°Π²Π΅Π½ ΠΏΡ€ΠΎΠΈΠ·Π²Π΅Π΄Π΅Π½ΠΈΡŽ ΠΈΡ… ΠΌΠΎΠ΄ΡƒΠ»Π΅ΠΉ, Ссли ΠΎΠ½ΠΈ пСрпСндикулярны, ΠΈ ΡƒΠΌΠ΅Π½ΡŒΡˆΠ°Π΅Ρ‚ΡΡ Π΄ΠΎ нуля, Ссли Π²Π΅ΠΊΡ‚ΠΎΡ€Ρ‹ сонаправлСнны ΠΈΠ»ΠΈ ΠΏΡ€ΠΎΡ‚ΠΈΠ²ΠΎΠΏΠΎΠ»ΠΎΠΆΠ½ΠΎ Π½Π°ΠΏΡ€Π°Π²Π»Π΅Π½Π½Ρ‹.

    Π•Ρ‰Π΅ ΠΎΠ΄ΠΈΠ½ ΠΏΡ€ΠΈΠΌΠ΅Ρ€: скалярноС ΠΏΡ€ΠΎΠΈΠ·Π²Π΅Π΄Π΅Π½ΠΈΠ΅ ΠΈΡΠΏΠΎΠ»ΡŒΠ·ΡƒΠ΅Ρ‚ΡΡ для вычислСния Ρ€Π°Π±ΠΎΡ‚Ρ‹ ΠΏΠΎ ΠΏΡ€ΠΈΠ²Π΅Π΄Π΅Π½Π½ΠΎΠΉ Π½ΠΈΠΆΠ΅ Ρ„ΠΎΡ€ΠΌΡƒΠ»Π΅, Π³Π΄Π΅ F – Π²Π΅ΠΊΡ‚ΠΎΡ€ силы, Π° s – Π²Π΅ΠΊΡ‚ΠΎΡ€ пСрСмСщСния.

    Одним ΠΈΠ· ΠΏΡ€ΠΈΠΌΠ΅Ρ€ΠΎΠ² использования произвСдСния Π²Π΅ΠΊΡ‚ΠΎΡ€ΠΎΠ² являСтся ΠΌΠΎΠΌΠ΅Π½Ρ‚ силы, Ρ€Π°Π²Π½Ρ‹ΠΉ ΠΏΡ€ΠΎΠΈΠ·Π²Π΅Π΄Π΅Π½ΠΈΡŽ радиус-Π²Π΅ΠΊΡ‚ΠΎΡ€Π°, ΠΏΡ€ΠΎΠ²Π΅Π΄Π΅Π½Π½ΠΎΠ³ΠΎ ΠΎΡ‚ оси вращСния ΠΊ Ρ‚ΠΎΡ‡ΠΊΠ΅ прилоТСния силы, Π½Π° Π²Π΅ΠΊΡ‚ΠΎΡ€ этой силы.

    МногоС ΠΈΠ· Ρ‚ΠΎΠ³ΠΎ, Ρ‡Ρ‚ΠΎ вычисляСтся Π² Ρ„ΠΈΠ·ΠΈΠΊΠ΅ ΠΏΠΎ ΠΏΡ€Π°Π²ΠΈΠ»Ρƒ ΠΏΡ€Π°Π²ΠΎΠΉ Ρ€ΡƒΠΊΠΈ являСтся Π²Π΅ΠΊΡ‚ΠΎΡ€Π½Ρ‹ΠΌ ΠΏΡ€ΠΎΠΈΠ·Π²Π΅Π΄Π΅Π½ΠΈΠ΅ΠΌ. Найти подтвСрТдСния, привСсти ΠΏΡ€ΠΈΠΌΠ΅Ρ€Ρ‹.

    Π‘Ρ‚ΠΎΠΈΡ‚ Π΅Ρ‰Π΅ Π·Π°ΠΌΠ΅Ρ‚ΠΈΡ‚ΡŒ, Ρ‡Ρ‚ΠΎ Π΄Π²ΡƒΡ…ΠΌΠ΅Ρ€Π½Ρ‹ΠΌ ΠΈ Ρ‚Ρ€Π΅Ρ…ΠΌΠ΅Ρ€Π½Ρ‹ΠΌ пространством Π½Π΅ ΠΈΡΡ‡Π΅Ρ€ΠΏΡ‹Π²Π°ΡŽΡ‚ΡΡ Π²ΠΎΠ·ΠΌΠΎΠΆΠ½Ρ‹Π΅ Π²Π°Ρ€ΠΈΠ°Π½Ρ‚Ρ‹ Π²Π΅ΠΊΡ‚ΠΎΡ€Π½Ρ‹Ρ… пространств. Π’Ρ‹ΡΡˆΠ°Ρ ΠΌΠ°Ρ‚Π΅ΠΌΠ°Ρ‚ΠΈΠΊΠ° рассматриваСт пространства большСй размСрности, Π² ΠΊΠΎΡ‚ΠΎΡ€Ρ‹Ρ… Ρ‚Π°ΠΊΠΆΠ΅ ΠΎΠΏΡ€Π΅Π΄Π΅Π»ΡΡŽΡ‚ΡΡ Π°Π½Π°Π»ΠΎΠ³ΠΈ Ρ„ΠΎΡ€ΠΌΡƒΠ» для скалярного ΠΈ Π²Π΅ΠΊΡ‚ΠΎΡ€Π½ΠΎΠ³ΠΎ произвСдСния. НСсмотря Π½Π° Ρ‚ΠΎ, Ρ‡Ρ‚ΠΎ пространства большСй размСрности, Ρ‡Π΅ΠΌ 3, чСловСчСскоС сознаниС нСспособно ΠΏΡ€Π΅Π΄ΡΡ‚Π°Π²ΠΈΡ‚ΡŒ Π²ΠΈΠ·ΡƒΠ°Π»ΡŒΠ½ΠΎ, ΠΎΠ½ΠΈ ΡƒΠ΄ΠΈΠ²ΠΈΡ‚Π΅Π»ΡŒΠ½Ρ‹ΠΌ ΠΎΠ±Ρ€Π°Π·ΠΎΠΌ находят сСбС прилоТСния Π²ΠΎ ΠΌΠ½ΠΎΠ³ΠΈΡ… областях Π½Π°ΡƒΠΊΠΈ ΠΈ ΠΏΡ€ΠΎΠΌΡ‹ΡˆΠ»Π΅Π½Π½ΠΎΡΡ‚ΠΈ.

    Π’ Ρ‚ΠΎ ΠΆΠ΅ врСмя Ρ€Π΅Π·ΡƒΠ»ΡŒΡ‚Π°Ρ‚ΠΎΠΌ Π²Π΅ΠΊΡ‚ΠΎΡ€Π½ΠΎΠ³ΠΎ произвСдСния Π²Π΅ΠΊΡ‚ΠΎΡ€ΠΎΠ² Π² Ρ‚Ρ€Ρ‘Ρ…ΠΌΠ΅Ρ€Π½ΠΎΠΌ Π•Π²ΠΊΠ»ΠΈΠ΄ΠΎΠ²ΠΎΠΌ пространствС являСтся Π½Π΅ число, Π° Ρ€Π΅Π·ΡƒΠ»ΡŒΡ‚ΠΈΡ€ΡƒΡŽΡ‰ΠΈΠΉ Π²Π΅ΠΊΡ‚ΠΎΡ€ со своими ΠΊΠΎΠΎΡ€Π΄ΠΈΠ½Π°Ρ‚Π°ΠΌΠΈ, Π½Π°ΠΏΡ€Π°Π²Π»Π΅Π½ΠΈΠ΅ΠΌ ΠΈ Π΄Π»ΠΈΠ½ΠΎΠΉ.

    НаправлСниС Ρ€Π΅Π·ΡƒΠ»ΡŒΡ‚ΠΈΡ€ΡƒΡŽΡ‰Π΅Π³ΠΎ Π²Π΅ΠΊΡ‚ΠΎΡ€Π° опрСдСляСтся ΠΏΠΎ ΠΏΡ€Π°Π²ΠΈΠ»Ρƒ ΠΏΡ€Π°Π²ΠΎΠΉ Ρ€ΡƒΠΊΠΈ, Ρ‡Ρ‚ΠΎ являСтся ΠΎΠ΄Π½ΠΈΠΌ ΠΈΠ· самых ΡƒΠ΄ΠΈΠ²ΠΈΡ‚Π΅Π»ΡŒΠ½Ρ‹Ρ… ΠΏΠΎΠ»ΠΎΠΆΠ΅Π½ΠΈΠΉ аналитичСской Π³Π΅ΠΎΠΌΠ΅Ρ‚Ρ€ΠΈΠΈ.

    Π’Π΅ΠΊΡ‚ΠΎΡ€Π½ΠΎΠ΅ ΠΏΡ€ΠΎΠΈΠ·Π²Π΅Π΄Π΅Π½ΠΈΠ΅ Π²Π΅ΠΊΡ‚ΠΎΡ€ΠΎΠ² ΠΌΠΎΠΆΠ΅Ρ‚ Π±Ρ‹Ρ‚ΡŒ использовано Π² Π½Π°Ρ…ΠΎΠΆΠ΄Π΅Π½ΠΈΠΈ ΠΏΠ»ΠΎΡ‰Π°Π΄ΠΈ Ρ‚Ρ€Π΅ΡƒΠ³ΠΎΠ»ΡŒΠ½ΠΈΠΊΠ° ΠΈΠ»ΠΈ ΠΏΠ°Ρ€Π°Π»Π»Π΅Π»ΠΎΠ³Ρ€Π°ΠΌΠΌΠ° ΠΏΠΎ Π·Π°Π΄Π°Π½Π½Ρ‹ΠΌ ΠΊΠΎΠΎΡ€Π΄ΠΈΠ½Π°Ρ‚Π°ΠΌ Π²Π΅Ρ€ΡˆΠΈΠ½, Ρ‡Ρ‚ΠΎ Π±Ρ‹Π»ΠΎ ΠΏΠΎΠ΄Ρ‚Π²Π΅Ρ€ΠΆΠ΄Π΅Π½ΠΎ Π²Ρ‹Π²Π΅Π΄Π΅Π½ΠΈΠ΅ΠΌ Ρ„ΠΎΡ€ΠΌΡƒΠ»Ρ‹, Π΄ΠΎΠΊΠ°Π·Π°Ρ‚Π΅Π»ΡŒΡΡ‚Π²ΠΎΠΌ Ρ‚Π΅ΠΎΡ€Π΅ΠΌΡ‹ ΠΈ Ρ€Π΅ΡˆΠ΅Π½ΠΈΠ΅ΠΌ практичСских Π·Π°Π΄Π°Ρ‡.

    Π’Π΅ΠΊΡ‚ΠΎΡ€Ρ‹ ΡˆΠΈΡ€ΠΎΠΊΠΎ ΠΈΡΠΏΠΎΠ»ΡŒΠ·ΡƒΡŽΡ‚ΡΡ Π² Ρ„ΠΈΠ·ΠΈΠΊΠ΅, Π³Π΄Π΅ Ρ‚Π°ΠΊΠΈΠ΅ ΠΏΠΎΠΊΠ°Π·Π°Ρ‚Π΅Π»ΠΈ ΠΊΠ°ΠΊ ΡΠΊΠΎΡ€ΠΎΡΡ‚ΡŒ, ΠΈΠΌΠΏΡƒΠ»ΡŒΡ ΠΈ сила ΠΌΠΎΠ³ΡƒΡ‚ Π±Ρ‹Ρ‚ΡŒ прСдставлСны Π² Π²ΠΈΠ΄Π΅ Π²Π΅ΠΊΡ‚ΠΎΡ€Π½Ρ‹Ρ… Π²Π΅Π»ΠΈΡ‡ΠΈΠ½ ΠΈ Π²Ρ‹Ρ‡ΠΈΡΠ»ΡΡŽΡ‚ΡΡ гСомСтричСски.

    Бписок ΠΈΡΠΏΠΎΠ»ΡŒΠ·ΠΎΠ²Π°Π½Π½Ρ‹Ρ… источников

    Атанасян Π›. Π‘., Π‘ΡƒΡ‚ΡƒΠ·ΠΎΠ² Π’. Π€., ΠšΠ°Π΄ΠΎΠΌΡ†Π΅Π² Π‘. Π‘. ΠΈ Π΄Ρ€. ГСомСтрия. 7-9 классы: ΡƒΡ‡Π΅Π±Π½ΠΈΠΊ для ΠΎΠ±Ρ‰Π΅ΠΎΠ±Ρ€Π°Π·ΠΎΠ²Π°Ρ‚Π΅Π»ΡŒΠ½Ρ‹Ρ… ΠΎΡ€Π³Π°Π½ΠΈΠ·Π°Ρ†ΠΈΠΉ. М.: ΠŸΡ€ΠΎΡΠ²Π΅Ρ‰Π΅Π½ΠΈΠ΅, 2013. 383 с.

    Атанасян Π›.Π‘., Π‘ΡƒΡ‚ΡƒΠ·ΠΎΠ² Π’. Π€., ΠšΠ°Π΄ΠΎΠΌΡ†Π΅Π² Π‘. Π‘. ΠΈ Π΄Ρ€. ГСомСтрия. 10-11 классы: ΡƒΡ‡Π΅Π±Π½ΠΈΠΊ для ΠΎΠ±Ρ‰Π΅ΠΎΠ±Ρ€Π°Π·ΠΎΠ²Π°Ρ‚Π΅Π»ΡŒΠ½Ρ‹Ρ… ΠΎΡ€Π³Π°Π½ΠΈΠ·Π°Ρ†ΠΈΠΉ: Π±Π°Π·ΠΎΠ²Ρ‹ΠΉ ΠΈ ΠΏΡ€ΠΎΡ„ΠΈΠ»ΡŒΠ½Ρ‹ΠΉ ΡƒΡ€ΠΎΠ²Π½ΠΈ. М.: ΠŸΡ€ΠΎΡΠ²Π΅Ρ‰Π΅Π½ΠΈΠ΅, 2013. 255 с.

    Π‘ΡƒΠ³Ρ€ΠΎΠ² Π―.Π‘., Никольский Π‘.М. Π’Ρ‹ΡΡˆΠ°Ρ ΠΌΠ°Ρ‚Π΅ΠΌΠ°Ρ‚ΠΈΠΊΠ°. Π’ΠΎΠΌ ΠΏΠ΅Ρ€Π²Ρ‹ΠΉ: элСмСнты Π»ΠΈΠ½Π΅ΠΉΠ½ΠΎΠΉ Π°Π»Π³Π΅Π±Ρ€Ρ‹ ΠΈ аналитичСской Π³Π΅ΠΎΠΌΠ΅Ρ‚Ρ€ΠΈΠΈ.

    ????ΠšΠ»Π΅Ρ‚Π΅Π½ΠΈΠΊ Π”.Π’. Π‘Π±ΠΎΡ€Π½ΠΈΠΊ Π·Π°Π΄Π°Ρ‡ ΠΏΠΎ аналитичСской Π³Π΅ΠΎΠΌΠ΅Ρ‚Ρ€ΠΈΠΈ. М.: Наука, Π€ΠΈΠ·ΠΌΠ°Ρ‚Π»ΠΈΡ‚, 1998.

    АналитичСская гСомСтрия.

    http://a-geometry.narod.ru/problems/problems_32.htm

    ΠœΠ°Ρ‚Π΅ΠΌΠ°Ρ‚ΠΈΠΊΠ°. ΠšΠ»Π΅Π²Π΅Ρ€.

    http://www.cleverstudents.ru/vectors/vector_product_of_vectors.html

    ——Π˜Π·ΡƒΡ‡Π΅Π½ΠΈΠ΅ ΠΌΠ°Ρ‚Π΅ΠΌΠ°Ρ‚ΠΈΠΊΠΈ ΠΎΠ½Π»Π°ΠΉΠ½.

    http://ru.onlinemschool.com/math/library/vector/multiply1/

    Π‘Π°ΠΉΡ‚ Π’. Π“Π»Π°Π·Π½Π΅Π²Π°.

    http://glaznev.sibcity.ru/1kurs/analit/common/html/anlek7.htm

    ——ВикипСдия.

    https://ru.wikipedia.org/wiki/%C2%E5%EA%F2%EE%F0%ED%EE%E5_%EF%F0%EE%E8%E7%E2%E5%E4%E5%ED%E8%E5

    ΠŸΠ΅Ρ€Π΅ΠΊΡ€Π΅ΡΡ‚Π½ΠΎΠ΅ ΠΏΡ€ΠΎΠΈΠ·Π²Π΅Π΄Π΅Π½ΠΈΠ΅

    Π’Π΅ΠΊΡ‚ΠΎΡ€ ΠΈΠΌΠ΅Π΅Ρ‚ Π²Π΅Π»ΠΈΡ‡ΠΈΠ½Ρƒ , (Π΄Π»ΠΈΠ½Π°) ΠΈ Π½Π°ΠΏΡ€Π°Π²Π»Π΅Π½ΠΈΠ΅ :

    Π”Π²Π° Π²Π΅ΠΊΡ‚ΠΎΡ€Π° ΠΌΠΎΠΆΠ½ΠΎ ΡƒΠΌΠ½ΠΎΠΆΠΈΡ‚ΡŒ Π½Π° с ΠΏΠΎΠΌΠΎΡ‰ΡŒΡŽ Β« Cross Product Β» (Ρ‚Π°ΠΊΠΆΠ΅ см. Π’ΠΎΡ‡Π΅Ρ‡Π½ΠΎΠ΅ ΠΏΡ€ΠΎΠΈΠ·Π²Π΅Π΄Π΅Π½ΠΈΠ΅)

    ΠŸΠ΅Ρ€Π΅ΠΊΡ€Π΅ΡΡ‚Π½ΠΎΠ΅ ΠΏΡ€ΠΎΠΈΠ·Π²Π΅Π΄Π΅Π½ΠΈΠ΅ a Γ— b Π΄Π²ΡƒΡ… Π²Π΅ΠΊΡ‚ΠΎΡ€ΠΎΠ² — это Π΄Ρ€ΡƒΠ³ΠΎΠΉ Π²Π΅ΠΊΡ‚ΠΎΡ€ , располоТСнный ΠΏΠΎΠ΄ прямым ΡƒΠ³Π»ΠΎΠΌ ΠΊ ​​обоим:

    И всС это происходит Π² Ρ‚Ρ€Π΅Ρ… измСрСниях!

    Π’Π΅Π»ΠΈΡ‡ΠΈΠ½Π° (Π΄Π»ΠΈΠ½Π°) Π²Π΅ΠΊΡ‚ΠΎΡ€Π½ΠΎΠ³ΠΎ произвСдСния Ρ€Π°Π²Π½Π° ΠΏΠ»ΠΎΡ‰Π°Π΄ΠΈ ΠΏΠ°Ρ€Π°Π»Π»Π΅Π»ΠΎΠ³Ρ€Π°ΠΌΠΌΠ° с Π²Π΅ΠΊΡ‚ΠΎΡ€Π°ΠΌΠΈ a ΠΈ b для сторон:

    ΠŸΠΎΡΠΌΠΎΡ‚Ρ€ΠΈΡ‚Π΅, ΠΊΠ°ΠΊ ΠΎΠ½ мСняСтся ΠΏΠΎΠ΄ Ρ€Π°Π·Π½Ρ‹ΠΌΠΈ ΡƒΠ³Π»Π°ΠΌΠΈ:

    ΠŸΠ΅Ρ€Π΅ΠΊΡ€Π΅ΡΡ‚Π½ΠΎΠ΅ ΠΏΡ€ΠΎΠΈΠ·Π²Π΅Π΄Π΅Π½ΠΈΠ΅ ( синий ):

    • Π½ΡƒΠ»Π΅Π²ΠΎΠΉ Π΄Π»ΠΈΠ½Ρ‹, ΠΊΠΎΠ³Π΄Π° Π²Π΅ΠΊΡ‚ΠΎΡ€Ρ‹ a ΠΈ b ΡƒΠΊΠ°Π·Ρ‹Π²Π°ΡŽΡ‚ Π² Ρ‚ΠΎΠΌ ΠΆΠ΅ ΠΈΠ»ΠΈ ΠΏΡ€ΠΎΡ‚ΠΈΠ²ΠΎΠΏΠΎΠ»ΠΎΠΆΠ½ΠΎΠΌ Π½Π°ΠΏΡ€Π°Π²Π»Π΅Π½ΠΈΠΈ
    • достигаСт максимальной Π΄Π»ΠΈΠ½Ρ‹, ΠΊΠΎΠ³Π΄Π° Π²Π΅ΠΊΡ‚ΠΎΡ€Ρ‹ a ΠΈ b располоТСны ΠΏΠΎΠ΄ прямым ΡƒΠ³Π»ΠΎΠΌ

    И это ΠΌΠΎΠΆΠ΅Ρ‚ ΡƒΠΊΠ°Π·Ρ‹Π²Π°Ρ‚ΡŒ Π² Ρ‚Ρƒ ΠΈΠ»ΠΈ ΠΈΠ½ΡƒΡŽ сторону!

    Π’Π°ΠΊ ΠΊΠ°ΠΊ ΠΆΠ΅ это Π²Ρ‹Ρ‡ΠΈΡΠ»ΠΈΡ‚ΡŒ?

    РасчСт

    ΠœΡ‹ ΠΌΠΎΠΆΠ΅ΠΌ Π²Ρ‹Ρ‡ΠΈΡΠ»ΠΈΡ‚ΡŒ пСрСкрСстноС ΠΏΡ€ΠΎΠΈΠ·Π²Π΅Π΄Π΅Π½ΠΈΠ΅ ΡΠ»Π΅Π΄ΡƒΡŽΡ‰ΠΈΠΌ ΠΎΠ±Ρ€Π°Π·ΠΎΠΌ:

    a Γ— b = | a | | b | sin (ΞΈ) n

    • | a | Π²Π΅Π»ΠΈΡ‡ΠΈΠ½Π° (Π΄Π»ΠΈΠ½Π°) Π²Π΅ΠΊΡ‚ΠΎΡ€Π° a
    • | b | Π²Π΅Π»ΠΈΡ‡ΠΈΠ½Π° (Π΄Π»ΠΈΠ½Π°) Π²Π΅ΠΊΡ‚ΠΎΡ€Π° b
    • ΞΈ — ΡƒΠ³ΠΎΠ» ΠΌΠ΅ΠΆΠ΄Ρƒ a ΠΈ b
    • n — Π΅Π΄ΠΈΠ½ΠΈΡ‡Π½Ρ‹ΠΉ Π²Π΅ΠΊΡ‚ΠΎΡ€ ΠΏΠΎΠ΄ прямым ΡƒΠ³Π»ΠΎΠΌ ΠΊ ​​ a ΠΈ b

    Π’Π°ΠΊΠΈΠΌ ΠΎΠ±Ρ€Π°Π·ΠΎΠΌ, Π΄Π»ΠΈΠ½Π° Ρ€Π°Π²Π½Π°: Π΄Π»ΠΈΠ½Π° a умноТСнная Π½Π° Π΄Π»ΠΈΠ½Ρƒ b умноТСнная Π½Π° синус ΡƒΠ³Π»Π° ΠΌΠ΅ΠΆΠ΄Ρƒ a ΠΈ b ,

    Π—Π°Ρ‚Π΅ΠΌ ΠΌΡ‹ ΡƒΠΌΠ½ΠΎΠΆΠ°Π΅ΠΌ Π½Π° Π²Π΅ΠΊΡ‚ΠΎΡ€ n , Ρ‡Ρ‚ΠΎΠ±Ρ‹ ΠΎΠ½ направлялся Π² ΠΏΡ€Π°Π²ΠΈΠ»ΡŒΠ½ΠΎΠΌ Π½Π°ΠΏΡ€Π°Π²Π»Π΅Π½ΠΈΠΈ (ΠΏΠΎΠ΄ прямым ΡƒΠ³Π»ΠΎΠΌ ΠΊ ​​ a ΠΈ b ).

    Π˜Π›Π˜ ΠΌΡ‹ ΠΌΠΎΠΆΠ΅ΠΌ Π²Ρ‹Ρ‡ΠΈΡΠ»ΠΈΡ‚ΡŒ это Ρ‚Π°ΠΊ:

    Когда a ΠΈ b Π½Π°Ρ‡ΠΈΠ½Π°ΡŽΡ‚ΡΡ Π² исходной Ρ‚ΠΎΡ‡ΠΊΠ΅ (0,0,0), пСрСкрСстноС ΠΏΡ€ΠΎΠΈΠ·Π²Π΅Π΄Π΅Π½ΠΈΠ΅ заканчиваСтся Π½Π°:

    • c x = a y b z — a z b y
    • c y = a z b x — a x b z
    • c z = a x b y — a y b x

    ΠŸΡ€ΠΈΠΌΠ΅Ρ€: пСрСкрСстноС ΠΏΡ€ΠΎΠΈΠ·Π²Π΅Π΄Π΅Π½ΠΈΠ΅

    a = (2,3,4) ΠΈ b = (5,6,7)
    • c x = a y b z — a z b y = 3 Γ— 7 — 4 Γ— 6 = βˆ’3
    • c y = a z b x — a x b z = 4 Γ— 5 — 2 Γ— 7 = 6
    • c z = a x b y — a y b x = 2 Γ— 6 — 3 Γ— 5 = βˆ’3

    ΠžΡ‚Π²Π΅Ρ‚: a Γ— b = (βˆ’3,6, βˆ’3)

    Π’ ΠΊΠ°ΠΊΠΎΠΌ Π½Π°ΠΏΡ€Π°Π²Π»Π΅Π½ΠΈΠΈ?

    ΠŸΠ΅Ρ€Π΅ΠΊΡ€Π΅ΡΡ‚Π½ΠΎΠ΅ ΠΏΡ€ΠΎΠΈΠ·Π²Π΅Π΄Π΅Π½ΠΈΠ΅ ΠΌΠΎΠΆΠ΅Ρ‚ ΡƒΠΊΠ°Π·Ρ‹Π²Π°Ρ‚ΡŒ Π² ΡΠΎΠ²Π΅Ρ€ΡˆΠ΅Π½Π½ΠΎ ΠΏΡ€ΠΎΡ‚ΠΈΠ²ΠΎΠΏΠΎΠ»ΠΎΠΆΠ½ΠΎΠΌ Π½Π°ΠΏΡ€Π°Π²Π»Π΅Π½ΠΈΠΈ ΠΈ ΠΏΠΎ-ΠΏΡ€Π΅ΠΆΠ½Π΅ΠΌΡƒ Π½Π°Ρ…ΠΎΠ΄ΠΈΡ‚ΡŒΡΡ ΠΏΠΎΠ΄ прямым ΡƒΠ³Π»ΠΎΠΌ ΠΊ ​​двум Π΄Ρ€ΡƒΠ³ΠΈΠΌ Π²Π΅ΠΊΡ‚ΠΎΡ€Π°ΠΌ, поэтому ΠΌΡ‹ ΠΈΠΌΠ΅Π΅ΠΌ:

    «ΠŸΡ€Π°Π²ΠΈΠ»ΠΎ ΠΏΡ€Π°Π²ΠΎΠΉ Ρ€ΡƒΠΊΠΈ»

    ΠŸΡ€Π°Π²ΠΎΠΉ Ρ€ΡƒΠΊΠΎΠΉ ΡƒΠΊΠ°ΠΆΠΈΡ‚Π΅ ΡƒΠΊΠ°Π·Π°Ρ‚Π΅Π»ΡŒΠ½Ρ‹ΠΌ ΠΏΠ°Π»ΡŒΡ†Π΅ΠΌ вдоль Π²Π΅ΠΊΡ‚ΠΎΡ€Π° a , Π° срСдним ΠΏΠ°Π»ΡŒΡ†Π΅ΠΌ — вдоль Π²Π΅ΠΊΡ‚ΠΎΡ€Π° b : пСрСкрСстноС ΠΏΡ€ΠΎΠΈΠ·Π²Π΅Π΄Π΅Π½ΠΈΠ΅ ΠΈΠ΄Π΅Ρ‚ Π² Π½Π°ΠΏΡ€Π°Π²Π»Π΅Π½ΠΈΠΈ большого ΠΏΠ°Π»ΡŒΡ†Π°.

    Π’ΠΎΡ‡Π΅Ρ‡Π½Ρ‹ΠΉ ΠΏΡ€ΠΎΠ΄ΡƒΠΊΡ‚

    ΠŸΠ΅Ρ€Π΅ΠΊΡ€Π΅ΡΡ‚Π½ΠΎΠ΅ ΠΏΡ€ΠΎΠΈΠ·Π²Π΅Π΄Π΅Π½ΠΈΠ΅ Π΄Π°Π΅Ρ‚ Π²Π΅ΠΊΡ‚ΠΎΡ€Π½Ρ‹ΠΉ ΠΎΡ‚Π²Π΅Ρ‚ ΠΈ ΠΈΠ½ΠΎΠ³Π΄Π° называСтся Π²Π΅ΠΊΡ‚ΠΎΡ€Π½Ρ‹ΠΌ ΠΏΡ€ΠΎΠΈΠ·Π²Π΅Π΄Π΅Π½ΠΈΠ΅ΠΌ .

    Но Π΅ΡΡ‚ΡŒ Ρ‚Π°ΠΊΠΆΠ΅ скалярноС ΠΏΡ€ΠΎΠΈΠ·Π²Π΅Π΄Π΅Π½ΠΈΠ΅, ΠΊΠΎΡ‚ΠΎΡ€ΠΎΠ΅ Π΄Π°Π΅Ρ‚ ΠΎΡ‚Π²Π΅Ρ‚ скаляр (ΠΎΠ±Ρ‹Ρ‡Π½ΠΎΠ΅ число), ΠΈ Π΅Π³ΠΎ ΠΈΠ½ΠΎΠ³Π΄Π° Π½Π°Π·Ρ‹Π²Π°ΡŽΡ‚ скалярным ΠΏΡ€ΠΎΠΈΠ·Π²Π΅Π΄Π΅Π½ΠΈΠ΅ΠΌ .

    Вопрос: Π§Ρ‚ΠΎ получится, Ссли ΡΠΊΡ€Π΅ΡΡ‚ΠΈΡ‚ΡŒ слона с Π±Π°Π½Π°Π½ΠΎΠΌ?

    ΠžΡ‚Π²Π΅Ρ‚: | слон | | Π±Π°Π½Π°Π½ | sin (ΞΈ) n

    12.4 ΠŸΠ΅Ρ€Π΅ΠΊΡ€Π΅ΡΡ‚Π½ΠΎΠ΅ ΠΏΡ€ΠΎΠΈΠ·Π²Π΅Π΄Π΅Π½ΠΈΠ΅

    Π•Ρ‰Π΅ ΠΎΠ΄Π½Π° полСзная опСрация: ΠΏΠΎ Π΄Π²ΡƒΠΌ Π²Π΅ΠΊΡ‚ΠΎΡ€Π°ΠΌ Π½Π°ΠΉΠ΄ΠΈΡ‚Π΅ Ρ‚Ρ€Π΅Ρ‚ΠΈΠΉ (Π½Π΅Π½ΡƒΠ»Π΅Π²ΠΎΠΉ!) Π’Π΅ΠΊΡ‚ΠΎΡ€. пСрпСндикулярно ΠΏΠ΅Ρ€Π²Ρ‹ΠΌ Π΄Π²ΡƒΠΌ. ΠšΠΎΠ½Π΅Ρ‡Π½ΠΎ, Π΅ΡΡ‚ΡŒ бСсконСчноС количСство Ρ‚Π°ΠΊΠΈΡ… Π²Π΅ΠΊΡ‚ΠΎΡ€ΠΎΠ² Ρ€Π°Π·Π½ΠΎΠΉ Π΄Π»ΠΈΠ½Ρ‹. Π’Π΅ΠΌ Π½Π΅ ΠΌΠ΅Π½Π΅Π΅, Π΄Π°Π²Π°ΠΉΡ‚Π΅ Π½Π°ΠΉΠ΄Π΅ΠΌ Π΅Π³ΠΎ. ΠŸΡ€Π΅Π΄ΠΏΠΎΠ»ΠΎΠΆΠΈΠΌ, $ \ ds {\ bf A} = \ langle a_1, a_2, a_3 \ rangle $ ΠΈ $ \ ds {\ bf B} = \ langle b_1, b_2, b_3 \ rangle $. ΠœΡ‹ Ρ…ΠΎΡ‚ΠΈΠΌ Π½Π°ΠΉΡ‚ΠΈ Π²Π΅ΠΊΡ‚ΠΎΡ€ $ \ ds {\ bf v} = \ langle v_1, v_2, v_3 \ rangle $ с $ {\ bf v} \ cdot {\ bf A} = {\ bf v} \ cdot {\ bf B} = 0 $, ΠΈΠ»ΠΈ $$ \ eqalign { a_1v_1 + a_2v_2 + a_3v_3 & = 0, \ cr b_1v_1 + b_2v_2 + b_3v_3 & = 0.\ cr } $$ Π£ΠΌΠ½ΠΎΠΆΡŒΡ‚Π΅ ΠΏΠ΅Ρ€Π²ΠΎΠ΅ ΡƒΡ€Π°Π²Π½Π΅Π½ΠΈΠ΅ Π½Π° $ \ ds b_3 $, Π° Π²Ρ‚ΠΎΡ€ΠΎΠ΅ — Π½Π° $ \ ds a_3 $ ΠΈ Π²Ρ‹Ρ‡Π΅ΡΡ‚ΡŒ, Ρ‡Ρ‚ΠΎΠ±Ρ‹ ΠΏΠΎΠ»ΡƒΡ‡ΠΈΡ‚ΡŒ $$ \ eqalign { b_3a_1v_1 + b_3a_2v_2 + b_3a_3v_3 & = 0 \ cr a_3b_1v_1 + a_3b_2v_2 + a_3b_3v_3 & = 0 \ cr (a_1b_3-b_1a_3) v_1 + (a_2b_3-b_2a_3) v_2 & = 0 \ cr } $$ ΠšΠΎΠ½Π΅Ρ‡Π½ΠΎ, это ΡƒΡ€Π°Π²Π½Π΅Π½ΠΈΠ΅ с двумя ΠΏΠ΅Ρ€Π΅ΠΌΠ΅Π½Π½Ρ‹ΠΌΠΈ ΠΈΠΌΠ΅Π΅Ρ‚ ΠΌΠ½ΠΎΠ³ΠΎ Ρ€Π΅ΡˆΠ΅Π½ΠΈΠΉ; Π° особСнно Π»Π΅Π³ΠΊΠΎ ΡƒΠ²ΠΈΠ΄Π΅Ρ‚ΡŒ $ \ ds v_1 = a_2b_3-b_2a_3 $, $ \ ds v_2 = b_1a_3-a_1b_3 $. ΠŸΠΎΠ΄ΡΡ‚Π°Π²Π»ΡΡ ΠΎΠ±Ρ€Π°Ρ‚Π½ΠΎ Π² любой ΠΈΠ· ΠΎΡ€ΠΈΠ³ΠΈΠ½Π°Π»ΡŒΠ½Ρ‹Ρ… уравнСния ΠΈ Ρ€Π΅ΡˆΠ΅Π½ΠΈΠ΅ для $ \ ds v_3 $ Π΄Π°Π΅Ρ‚ $ \ ds v_3 = a_1b_2-b_1a_2 $.

    Π­Ρ‚ΠΎΡ‚ ΠΊΠΎΠ½ΠΊΡ€Π΅Ρ‚Π½Ρ‹ΠΉ ΠΎΡ‚Π²Π΅Ρ‚ Π½Π° ΠΏΡ€ΠΎΠ±Π»Π΅ΠΌΡƒ оказываСтся приятным свойства, ΠΈ ΠΎΠ½ удостоСн ΠΈΠΌΠ΅Π½ΠΈ: пСрСкрСстноС ΠΏΡ€ΠΎΠΈΠ·Π²Π΅Π΄Π΅Π½ΠΈΠ΅ : $$ {\ bf A} \ times {\ bf B} = \ langle a_2b_3-b_2a_3, b_1a_3-a_1b_3, a_1b_2-b_1a_2 \ rangle. $$ Π₯отя Π² этом Π²Π΅ΠΊΡ‚ΠΎΡ€Π΅ Π΅ΡΡ‚ΡŒ Ρ…ΠΎΡ€ΠΎΡˆΠΈΠΉ ΡƒΠ·ΠΎΡ€, ΠΎΠ½ ΠΌΠΎΠΆΠ΅Ρ‚ Π±Ρ‹Ρ‚ΡŒ Π½Π΅ΠΌΠ½ΠΎΠ³ΠΎ Ρ‚Ρ€ΡƒΠ΄Π½ΠΎ Π·Π°ΠΏΠΎΠΌΠ½ΠΈΡ‚ΡŒ; Π²ΠΎΡ‚ удобная ΠΌΠ½Π΅ΠΌΠΎΠ½ΠΈΠΊΠ°. ΠžΠΏΡ€Π΅Π΄Π΅Π»ΠΈΡ‚Π΅Π»ΡŒ ΠΌΠ°Ρ‚Ρ€ΠΈΡ†Ρ‹ Π΄Π²Π° Π½Π° Π΄Π²Π° Ρ€Π°Π²Π΅Π½ $$ \ left | \ matrix {a & b \ cr c & d \ cr} \ right | = ad-cb. $$ Π­Ρ‚ΠΎ Ρ€Π°ΡΡˆΠΈΡ€ΡΠ΅Ρ‚ΡΡ Π΄ΠΎ опрСдСлитСля ΠΌΠ°Ρ‚Ρ€ΠΈΡ†Ρ‹ Ρ‚Ρ€ΠΈ Π½Π° Ρ‚Ρ€ΠΈ: $$ \ eqalign { \ left | \ ΠΌΠ°Ρ‚Ρ€ΠΈΡ†Π° {x & y & z \ cr Π°_1 ΠΈ Π°_2 ΠΈ Π°_3 \ cr b_1 & b_2 & b_3 \ cr} \ right | & = x \ left | \ matrix {a_2 & a_3 \ cr b_2 & b_3 \ cr} \ right | -y \ left | \ matrix {a_1 & a_3 \ cr b_1 & b_3 \ cr} \ right | + z \ left | \ matrix {a_1 & a_2 \ cr b_1 & b_2 \ cr} \ right | \ cr & = x (a_2b_3-b_2a_3) -y (a_1b_3-b_1a_3) + z (a_1b_2-b_1a_2) \ cr & = x (a_2b_3-b_2a_3) + y (b_1a_3-a_1b_3) + z (a_1b_2-b_1a_2).\ cr} $$ КаТдая ΠΈΠ· Π΄Π²ΡƒΡ… ΠΌΠ°Ρ‚Ρ€ΠΈΡ† формируСтся ΠΏΡƒΡ‚Π΅ΠΌ удалСния Π²Π΅Ρ€Ρ…Π½Π΅ΠΉ строки ΠΈ ΠΎΠ΄ΠΈΠ½ столбСц ΠΌΠ°Ρ‚Ρ€ΠΈΡ†Ρ‹ Ρ‚Ρ€ΠΈ Π½Π° Ρ‚Ρ€ΠΈ; Π²Ρ‹Ρ‡ΠΈΡ‚Π°Π½ΠΈΠ΅ сСрСдины Ρ‚Π΅Ρ€ΠΌΠΈΠ½ Ρ‚ΠΎΠΆΠ΅ Π½ΡƒΠΆΠ½ΠΎ Π·Π°ΠΏΠΎΠΌΠ½ΠΈΡ‚ΡŒ. Π­Ρ‚ΠΎ Π½Π΅ мСсто, Ρ‡Ρ‚ΠΎΠ±Ρ‹ ΠΏΡ€Π΅Π²ΠΎΠ·Π½ΠΎΡΠΈΡ‚ΡŒ использованиС опрСдСлитСля; достаточно ΡΠΊΠ°Π·Π°Ρ‚ΡŒ, Ρ‡Ρ‚ΠΎ Π΄Π΅Ρ‚Π΅Ρ€ΠΌΠΈΠ½Π°Π½Ρ‚Ρ‹ Π½Π΅ΠΎΠ±Ρ‹Ρ‡Π°ΠΉΠ½ΠΎ ΠΏΠΎΠ»Π΅Π·Π½ΠΎ ΠΈ Π²Π°ΠΆΠ½ΠΎ. Π—Π΄Π΅ΡΡŒ ΠΌΡ‹ Ρ…ΠΎΡ‚ΠΈΠΌ ΠΈΡΠΏΠΎΠ»ΡŒΠ·ΠΎΠ²Π°Ρ‚ΡŒ Π΅Π³ΠΎ просто ΠΊΠ°ΠΊ мнСмоничСский ΠΏΡ€ΠΈΠ΅ΠΌ. Π’Ρ‹ ΠΌΠΎΠ³Π»ΠΈ Π·Π°ΠΌΠ΅Ρ‚ΠΈΡ‚ΡŒ, Ρ‡Ρ‚ΠΎ Ρ‚Ρ€ΠΈ выраТСния Π² ΠΊΡ€ΡƒΠ³Π»Ρ‹Π΅ скобки Π² послСднСй строкС — это Π² точности Ρ‚Ρ€ΠΈ ΠΊΠΎΠΎΡ€Π΄ΠΈΠ½Π°Ρ‚Ρ‹ пСрСкрСстноС ΠΏΡ€ΠΎΠΈΠ·Π²Π΅Π΄Π΅Π½ΠΈΠ΅; замСняя $ x $, $ y $, $ z $ Π½Π° $ \ bf i $, $ \ bf j $, $ \ bf k $ Π΄Π°Π΅Ρ‚ Π½Π°ΠΌ $$ \ eqalign { \ left | \ matrix {{\ bf i} & {\ bf j} & {\ bf k} \ cr Π°_1 ΠΈ Π°_2 ΠΈ Π°_3 \ cr b_1 & b_2 & b_3 \ cr} \ right | & = (a_2b_3-b_2a_3) {\ bf i} — (a_1b_3-b_1a_3) {\ bf j} + (a_1b_2-b_1a_2) {\ bf k} \ cr & = (a_2b_3-b_2a_3) {\ bf i} + (b_1a_3-a_1b_3) {\ bf j} + (a_1b_2-b_1a_2) {\ bf k} \ cr & = \ langle a_2b_3-b_2a_3, b_1a_3-a_1b_3, a_1b_2-b_1a_2 \ rangle \ cr & = {\ bf A} \ times {\ bf B}.\ cr} $$

    ΠŸΡ€ΠΈΠΌΠ΅Ρ€ 12.4.1. ΠŸΡ€Π΅Π΄ΠΏΠΎΠ»ΠΎΠΆΠΈΠΌ, Ρ‡Ρ‚ΠΎ $ {\ bf A} = \ langle 1,2,3 \ rangle $, $ {\ bf B} = \ langle. 4,5,6 \ rangle $. ΠΏΠΎΡ‚ΠΎΠΌ $$ \ eqalign { {\ bf A} \ times {\ bf B} & = \ left | \ matrix {{\ bf i} & {\ bf j} & {\ bf k} \ cr 1 ΠΈ 2 ΠΈ 3 \ cr 4 ΠΈ 5 ΠΈ 6 \ cr} \ right | \ cr & = (2 \ cdot 6-5 \ cdot 3) {\ bf i} + (4 \ cdot 3-1 \ cdot 6) {\ bf j} + (1 \ cdot 5-4 \ cdot 2) {\ bf k} \ cr & = — 3 {\ bf i} +6 {\ bf j} -3 {\ bf k} \ cr & = \ langle -3, 6, -3 \ rangle \ cr } $$ ПослС нСбольшой ΠΏΡ€Π°ΠΊΡ‚ΠΈΠΊΠΈ Π²Ρ‹ ΠΎΠ±Π½Π°Ρ€ΡƒΠΆΠΈΡ‚Π΅, Ρ‡Ρ‚ΠΎ Π»Π΅Π³ΠΊΠΎ ΡƒΡΡ‚Ρ€Π°Π½ΠΈΡ‚ΡŒ ΠΏΡ€ΠΎΠΌΠ΅ΠΆΡƒΡ‚ΠΎΡ‡Π½Ρ‹Π΅ шаги, пСрСходящиС нСпосрСдствСнно ΠΎΡ‚ ΠΌΠ°Ρ‚Ρ€ΠΈΡ†Ρ‹ $ 3 \ times3 $ ΠΊ обычная вСкторная Ρ„ΠΎΡ€ΠΌΠ°.$ \ ΠΊΠ²Π°Π΄Ρ€Π°Ρ‚ $

    Для $ \ bf A $ ΠΈ $ \ bf B $ ΠΎΠ±Ρ‹Ρ‡Π½ΠΎ Π΅ΡΡ‚ΡŒ Π΄Π²Π° Π²ΠΎΠ·ΠΌΠΎΠΆΠ½Ρ‹Ρ… направлСния ΠΈ бСсконСчноС количСство Π²Π΅Π»ΠΈΡ‡ΠΈΠ½, ΠΊΠΎΡ‚ΠΎΡ€Ρ‹Π΅ Π΄Π°Π΄ΡƒΡ‚ Π²Π΅ΠΊΡ‚ΠΎΡ€, пСрпСндикулярный ΠΊΠ°ΠΊ $ \ bf A $, Ρ‚Π°ΠΊ ΠΈ $ \ bf B $. ΠŸΠΎΡΠΊΠΎΠ»ΡŒΠΊΡƒ ΠΌΡ‹ Π²Ρ‹Π±Ρ€Π°Π»ΠΈ ΠΊΠΎΠ½ΠΊΡ€Π΅Ρ‚Π½Ρ‹ΠΉ Π²Π°Ρ€ΠΈΠ°Π½Ρ‚, ΠΌΡ‹ слСдуСт ΠΈΡΡΠ»Π΅Π΄ΠΎΠ²Π°Ρ‚ΡŒ Π²Π΅Π»ΠΈΡ‡ΠΈΠ½Ρƒ ΠΈ Π½Π°ΠΏΡ€Π°Π²Π»Π΅Π½ΠΈΠ΅.

    ΠœΡ‹ Π·Π½Π°Π΅ΠΌ, ΠΊΠ°ΠΊ Π²Ρ‹Ρ‡ΠΈΡΠ»ΠΈΡ‚ΡŒ Π²Π΅Π»ΠΈΡ‡ΠΈΠ½Ρƒ $ {\ bf A} \ times {\ bf B} $; это Π½Π΅ΠΌΠ½ΠΎΠ³ΠΎ бСспорядочно, Π½ΠΎ Π½Π΅ слоТно. 2 \ theta \ cr | {\ bf A} \ times {\ bf B} | & = | {\ bf A} || {\ bf B} | \ sin \ theta \ cr } $$ Π’Π°ΠΊΠΈΠΌ ΠΎΠ±Ρ€Π°Π·ΠΎΠΌ, Π²Π΅Π»ΠΈΡ‡ΠΈΠ½Π° $ {\ bf A} \ times {\ bf B} $ ΠΎΡ‡Π΅Π½ΡŒ ΠΏΠΎΡ…ΠΎΠΆΠ° Π½Π° Ρ‚ΠΎΡ‡ΠΊΡƒ ΠΏΡ€ΠΎΠ΄ΡƒΠΊΡ‚.Π’ частности, ΠΎΠ±Ρ€Π°Ρ‚ΠΈΡ‚Π΅ Π²Π½ΠΈΠΌΠ°Π½ΠΈΠ΅, Ρ‡Ρ‚ΠΎ Ссли $ \ bf A $ ΠΏΠ°Ρ€Π°Π»Π»Π΅Π»ΡŒΠ½ΠΎ $ \ bf B $, ΡƒΠ³ΠΎΠ» ΠΌΠ΅ΠΆΠ΄Ρƒ Π½ΠΈΠΌΠΈ Ρ€Π°Π²Π΅Π½ Π½ΡƒΠ»ΡŽ, поэтому $ \ sin \ theta = 0 $, поэтому $ | {\ bf A} \ times {\ bf B} | = 0 $, ΠΈ Π°Π½Π°Π»ΠΎΠ³ΠΈΡ‡Π½ΠΎ, Ссли ΠΎΠ½ΠΈ Π°Π½Ρ‚ΠΈΠΏΠ°Ρ€Π°Π»Π»Π΅Π»ΡŒΠ½Ρ‹, $ \ sin \ theta = 0 $ ΠΈ $ | {\ bf A} \ times {\ bf B} | = 0 $. Наоборот, Ссли $ | {\ bf A} \ times {\ bf B} | = 0 $ ΠΈ $ | {\ bf A} | $ ΠΈ $ | {\ bf B} | $ Π½Π΅ Ρ€Π°Π²Π½Ρ‹ Π½ΡƒΠ»ΡŽ, это Π΄ΠΎΠ»ΠΆΠ½ΠΎ Π±Ρ‹Ρ‚ΡŒ $ \ sin \ theta = 0 $, поэтому $ \ bf A $ ΠΏΠ°Ρ€Π°Π»Π»Π΅Π»ΡŒΠ½ΠΎ ΠΈΠ»ΠΈ Π°Π½Ρ‚ΠΈΠΏΠ°Ρ€Π°Π»Π»Π΅Π»ΡŒΠ½ΠΎ $ \ bf B $.

    Π’ΠΎΡ‚ Π»ΡŽΠ±ΠΎΠΏΡ‹Ρ‚Π½Ρ‹ΠΉ Ρ„Π°ΠΊΡ‚ ΠΎΠ± этом количСство, ΠΊΠΎΡ‚ΠΎΡ€ΠΎΠ΅ ΠΏΠΎΠ·ΠΆΠ΅ окаТСтся вСсьма ΠΏΠΎΠ»Π΅Π·Π½Ρ‹ΠΌ: Учитывая Π΄Π²Π° Π²Π΅ΠΊΡ‚ΠΎΡ€ΠΎΠ², ΠΌΡ‹ ΠΌΠΎΠΆΠ΅ΠΌ ΡΠΎΠ΅Π΄ΠΈΠ½ΠΈΡ‚ΡŒ ΠΈΡ… хвост ΠΊ хвосту ΠΈ ΡΡ„ΠΎΡ€ΠΌΠΈΡ€ΠΎΠ²Π°Ρ‚ΡŒ ΠΏΠ°Ρ€Π°Π»Π»Π΅Π»ΠΎΠ³Ρ€Π°ΠΌΠΌ, ΠΊΠ°ΠΊ Π½Π° рисункС 12.4.1. Π’ высота ΠΏΠ°Ρ€Π°Π»Π»Π΅Π»ΠΎΠ³Ρ€Π°ΠΌΠΌΠ° $ h $ Ρ€Π°Π²Π½Π° $ | {\ bf A} | \ sin \ theta $, Π° Π±Π°Π·Π° Ρ€Π°Π²Π½Π° $ | {\ bf B} | $, поэтому ΠΏΠ»ΠΎΡ‰Π°Π΄ΡŒ ΠΏΠ°Ρ€Π°Π»Π»Π΅Π»ΠΎΠ³Ρ€Π°ΠΌΠΌ Ρ€Π°Π²Π΅Π½ $ | {\ bf A} || {\ bf B} | \ sin \ theta $, Π² точности Π²Π΅Π»ΠΈΡ‡ΠΈΠ½Π° $ | {\ bf A} \ times {\ bf B} | $.

    Рисунок 12.4.1. ΠŸΠ°Ρ€Π°Π»Π»Π΅Π»ΠΎΠ³Ρ€Π°ΠΌΠΌ.

    А ΠΊΠ°ΠΊ насчСт направлСния пСрСкрСстного произвСдСния? ΠŸΡ€ΠΈΠΌΠ΅Ρ‡Π°Ρ‚Π΅Π»ΡŒΠ½ΠΎ, Ρ‡Ρ‚ΠΎ Π΅ΡΡ‚ΡŒ простоС ΠΏΡ€Π°Π²ΠΈΠ»ΠΎ, ΠΎΠΏΠΈΡΡ‹Π²Π°ΡŽΡ‰Π΅Π΅ Π½Π°ΠΏΡ€Π°Π²Π»Π΅Π½ΠΈΠ΅. Π”Π°Π²Π°ΠΉΡ‚Π΅ посмотрим Π½Π° простой ΠΏΡ€ΠΈΠΌΠ΅Ρ€: ΠŸΡƒΡΡ‚ΡŒ $ {\ bf A} = \ langle a, 0,0 \ rangle $, $ {\ bf B} = \ langle b, c, 0 \ rangle $. Если Π²Π΅ΠΊΡ‚ΠΎΡ€Ρ‹ располоТСны хвостами Π² Π½Π°Ρ‡Π°Π»Π΅ ΠΊΠΎΠΎΡ€Π΄ΠΈΠ½Π°Ρ‚, $ \ bf A $ Π»Π΅ΠΆΠΈΡ‚ вдоль оси $ x $, Π° $ \ bf B $ Π»Π΅ΠΆΠΈΡ‚ Π² плоскости $ x $ — $ y $, поэтому ΠΌΡ‹ Π·Π½Π°Π΅ΠΌ, Ρ‡Ρ‚ΠΎ пСрСкрСстноС ΠΏΡ€ΠΎΠΈΠ·Π²Π΅Π΄Π΅Π½ΠΈΠ΅ Π±ΡƒΠ΄Π΅Ρ‚ ΡƒΠΊΠ°Π·Ρ‹Π²Π°Ρ‚ΡŒ Π²Π²Π΅Ρ€Ρ… ΠΈΠ»ΠΈ Π²Π½ΠΈΠ·.ΠšΡ€Π΅ΡΡ‚ ΠΏΡ€ΠΎΠ΄ΡƒΠΊΡ‚ $$ \ eqalign { {\ bf A} \ times {\ bf B} = \ left | \ matrix {{\ bf i} & {\ bf j} & {\ bf k} \ cr Π° & 0 & 0 \ cr b & c & 0 \ cr} \ right | & = \ langle 0,0, ac \ rangle. \ Cr} $$ Как ΠΈ ΠΏΡ€Π΅Π΄ΠΏΠΎΠ»Π°Π³Π°Π»ΠΎΡΡŒ, это Π²Π΅ΠΊΡ‚ΠΎΡ€, ΡƒΠΊΠ°Π·Ρ‹Π²Π°ΡŽΡ‰ΠΈΠΉ Π²Π²Π΅Ρ€Ρ… ΠΈΠ»ΠΈ Π²Π½ΠΈΠ·, Π² зависимости ΠΎΡ‚ Π·Π½Π°ΠΊ $ ac $. ΠŸΡ€Π΅Π΄ΠΏΠΎΠ»ΠΎΠΆΠΈΠΌ, Ρ‡Ρ‚ΠΎ $ a> 0 $, поэтому Π·Π½Π°ΠΊ зависит Ρ‚ΠΎΠ»ΡŒΠΊΠΎ ΠΎΡ‚ $ c $: Ссли $ c> 0 $, $ ac> 0 $ ΠΈ Π²Π΅ΠΊΡ‚ΠΎΡ€ Π½Π°ΠΏΡ€Π°Π²Π»Π΅Π½ Π²Π²Π΅Ρ€Ρ…; Ссли $ c0 $, Π²Π΅ΠΊΡ‚ΠΎΡ€ Π½Π°ΠΏΡ€Π°Π²Π»Π΅Π½ Π²Π½ΠΈΠ·, Π° Ссли $ a0 $ ΠΈ $ c> 0 $, ΠΈΠ»ΠΈ $ a

    Π₯отя с Π²Ρ‹Ρ‡ΠΈΡΠ»ΠΈΡ‚Π΅Π»ΡŒΠ½ΠΎΠΉ Ρ‚ΠΎΡ‡ΠΊΠΈ зрСния довольно слоТно ΡƒΠ²ΠΈΠ΄Π΅Ρ‚ΡŒ, ΠΊΠ°ΠΊ это для Π»ΡŽΠ±Ρ‹Ρ… Π΄Π²ΡƒΡ… Π½Π°Ρ‡Π°Π»ΡŒΠ½Ρ‹Ρ… Π²Π΅ΠΊΡ‚ΠΎΡ€ΠΎΠ², ΠΏΡ€Π°Π²ΠΈΠ»ΠΎ сводится ΠΊ ΡΠ»Π΅Π΄ΡƒΡŽΡ‰Π΅ΠΌΡƒ. ΠΎΠ΄Π½ΠΎ ΠΈ Ρ‚ΠΎΠΆΠ΅.ΠŸΠΎΠΌΠ΅ΡΡ‚ΠΈΡ‚Π΅ $ \ bf A $ ΠΈ $ \ bf B $ хвост Π² хвост. Π‘Π°ΠΌΠΎΠ»Π΅Ρ‚, Π² ΠΊΠΎΡ‚ΠΎΡ€ΠΎΠΌ $ \ bf На A $ ΠΈ $ \ bf B $ ΠΌΠΎΠΆΠ½ΠΎ ΡΠΌΠΎΡ‚Ρ€Π΅Ρ‚ΡŒ с Π΄Π²ΡƒΡ… сторон; посмотри Π½Π° это со стороны для ΠΊΠΎΡ‚ΠΎΡ€ΠΎΠ³ΠΎ $ \ bf A $ Π΄ΠΎΠ»ΠΆΠ΅Π½ ΠΏΠΎΠ²Π΅Ρ€Π½ΡƒΡ‚ΡŒΡΡ ΠΏΡ€ΠΎΡ‚ΠΈΠ² часовой стрСлки, Ρ‡Ρ‚ΠΎΠ±Ρ‹ Π΄ΠΎΡΡ‚ΠΈΡ‡ΡŒ $ \ bf B $; Ρ‚ΠΎΠ³Π΄Π° Π²Π΅ΠΊΡ‚ΠΎΡ€ $ {\ bf A} \ times {\ bf B} $ ΡƒΠΊΠ°Π·Ρ‹Π²Π°Π΅Ρ‚ Π½Π° вас.

    Π­Ρ‚ΠΎ ΠΏΡ€Π°Π²ΠΈΠ»ΠΎ ΠΎΠ±Ρ‹Ρ‡Π½ΠΎ Π½Π°Π·Ρ‹Π²Π°ΡŽΡ‚ Π»ΠΈΠ½Π΅ΠΉΠΊΠ° ΠΏΡ€Π°Π²ΠΎΠΉ Ρ€ΡƒΠΊΠΈ . ΠŸΡ€Π΅Π΄ΡΡ‚Π°Π²ΡŒΡ‚Π΅, Ρ‡Ρ‚ΠΎ Π²Ρ‹ помСстили пятку ΠΏΡ€Π°Π²ΠΎΠΉ Ρ€ΡƒΠΊΠΈ Π² Ρ‚ΠΎΡ‡ΠΊΡƒ, Π³Π΄Π΅ соСдинСны Ρ‚Π°ΠΊ, Ρ‡Ρ‚ΠΎΠ±Ρ‹ ваши слСгка согнутыС ΠΏΠ°Π»ΡŒΡ†Ρ‹ ΡƒΠΊΠ°Π·Ρ‹Π²Π°Π»ΠΈ Π½Π°ΠΏΡ€Π°Π²Π»Π΅Π½ΠΈΠ΅ ΠΏΠΎΠ²ΠΎΡ€ΠΎΡ‚ ΠΈΠ· $ \ bf A $ Π² $ \ bf B $.Π—Π°Ρ‚Π΅ΠΌ ваш большой ΠΏΠ°Π»Π΅Ρ† ΡƒΠΊΠ°Π·Ρ‹Π²Π°Π΅Ρ‚ Π½Π° Π½Π°ΠΏΡ€Π°Π²Π»Π΅Π½ΠΈΠ΅ Π²Π΅ΠΊΡ‚ΠΎΡ€Π½ΠΎΠ³ΠΎ произвСдСния $ {\ bf A} \ times {\ bf B} $.

    НСпосрСдствСнным слСдствиСм этих Ρ„Π°ΠΊΡ‚ΠΎΠ² являСтся Ρ‚ΠΎ, Ρ‡Ρ‚ΠΎ $ {\ bf A} \ times {\ bf B} \ not = {\ bf B} \ times {\ bf A} $, ΠΏΠΎΡ‚ΠΎΠΌΡƒ Ρ‡Ρ‚ΠΎ Π΄Π²Π° пСрСкрСстныС произвСдСния ΡƒΠΊΠ°Π·Ρ‹Π²Π°ΡŽΡ‚ Π² ΠΏΡ€ΠΎΡ‚ΠΈΠ²ΠΎΠΏΠΎΠ»ΠΎΠΆΠ½ΠΎΠΌ Π½Π°ΠΏΡ€Π°Π²Π»Π΅Π½ΠΈΠΈ. Π‘ Π΄Ρ€ΡƒΠ³ΠΎΠΉ стороны, ΠΏΠΎΡΠΊΠΎΠ»ΡŒΠΊΡƒ $$ | {\ bf A} \ times {\ bf B} | = | {\ bf A} || {\ bf B} | \ sin \ theta = | {\ bf B} || {\ bf A} | \ sin \ theta = | {\ bf B} \ times {\ bf A} |, $$ Π΄Π»ΠΈΠ½Ρ‹ Π΄Π²ΡƒΡ… ΠΏΠΎΠΏΠ΅Ρ€Π΅Ρ‡Π½Ρ‹Ρ… ΠΏΡ€ΠΎΠΈΠ·Π²Π΅Π΄Π΅Π½ΠΈΠΉ Ρ€Π°Π²Π½Ρ‹, поэтому ΠΌΡ‹ Π·Π½Π°Π΅ΠΌ, Ρ‡Ρ‚ΠΎ $ {\ bf A} \ times {\ bf B} = — ({\ bf B} \ times {\ bf A}) $.

    ΠŸΠ΅Ρ€Π΅ΠΊΡ€Π΅ΡΡ‚Π½ΠΎΠ΅ ΠΏΡ€ΠΎΠΈΠ·Π²Π΅Π΄Π΅Π½ΠΈΠ΅ ΠΈΠΌΠ΅Π΅Ρ‚ Π½Π΅ΠΊΠΎΡ‚ΠΎΡ€Ρ‹Π΅ Π·Π½Π°ΠΊΠΎΠΌΡ‹Π΅ свойства, ΠΊΠΎΡ‚ΠΎΡ€Ρ‹Π΅ Π±ΡƒΠ΄ΡƒΡ‚ пригодятся ΠΏΠΎΠ·ΠΆΠ΅, поэтому ΠΌΡ‹ пСрСчислим ΠΈΡ… здСсь. Как ΠΈ Π² случаС скалярного произвСдСния, ΠΎΠ½ΠΈ ΠΌΠΎΠ³ΡƒΡ‚ Π±Ρ‹Ρ‚ΡŒ Π΄ΠΎΠΊΠ°Π·Π°Π½ΠΎ Π²Ρ‹ΠΏΠΎΠ»Π½Π΅Π½ΠΈΠ΅ΠΌ ΡΠΎΠΎΡ‚Π²Π΅Ρ‚ΡΡ‚Π²ΡƒΡŽΡ‰ΠΈΡ… расчСтов ΠΏΠΎ ΠΊΠΎΠΎΡ€Π΄ΠΈΠ½Π°Ρ‚Π°ΠΌ, послС Ρ‡Π΅Π³ΠΎ ΠΌΡ‹ ΠΌΠΎΠΆΠ΅ΠΌ ΠΈΠ½ΠΎΠ³Π΄Π° ΠΈΠ·Π±Π΅ΠΆΠ°Ρ‚ΡŒ Ρ‚Π°ΠΊΠΈΡ… вычислСний, ΠΈΡΠΏΠΎΠ»ΡŒΠ·ΡƒΡ характСристики.

    Π’Π΅ΠΎΡ€Π΅ΠΌΠ° 12.4.2. Если $ {\ bf u} $, $ {\ bf v} $ ΠΈ $ {\ bf w} $ — Π²Π΅ΠΊΡ‚ΠΎΡ€Ρ‹, Π° $ a $ — вСщСствСнноС число, Ρ‚ΠΎΠ³Π΄Π°

      1. $ {\ bf u} \ times ({\ bf v} + {\ bf w}) = {\ bf u} \ times {\ bf v} + {\ bf u} \ times {\ bf w} $

      2.$ ({\ bf v} + {\ bf w}) \ times {\ bf u} = {\ bf v} \ times {\ bf u} + {\ bf w} \ times {\ bf u} $

      3. $ (a {\ bf u}) \ times {\ bf v} = a ({\ bf u} \ times {\ bf v}) = {\ bf u} \ times (a {\ bf v}) $

      4. $ {\ bf u} \ cdot ({\ bf v} \ times {\ bf w}) = ({\ bf u} \ times {\ bf v}) \ cdot {\ bf w} $

      5. $ {\ bf u} \ times ({\ bf v} \ times {\ bf w}) = ({\ bf u} \ cdot {\ bf w}) {\ bf v} — ({\ bf u} \ cdot {\ bf v}) {\ bf w} $ $ \ qed $

    УпраТнСния 12.4

    Π’Ρ‹ ΠΌΠΎΠΆΠ΅Ρ‚Π΅ ΠΈΡΠΏΠΎΠ»ΡŒΠ·ΠΎΠ²Π°Ρ‚ΡŒ Sage для вычислСния пСрСкрСстных ΠΏΡ€ΠΎΠΈΠ·Π²Π΅Π΄Π΅Π½ΠΈΠΉ.

    ΠŸΡ€. 12.4,1 НайдитС Π²Π΅ΠΊΡ‚ΠΎΡ€Π½ΠΎΠ΅ ΠΏΡ€ΠΎΠΈΠ·Π²Π΅Π΄Π΅Π½ΠΈΠ΅ $ \ langle 1,1,1 \ rangle $ ΠΈ $ \ langle 1,2,3 \ rangle $. (ΠΎΡ‚Π²Π΅Ρ‡Π°Ρ‚ΡŒ)

    ΠŸΡ€. 12.4.2 НайдитС Π²Π΅ΠΊΡ‚ΠΎΡ€Π½ΠΎΠ΅ ΠΏΡ€ΠΎΠΈΠ·Π²Π΅Π΄Π΅Π½ΠΈΠ΅ $ \ langle 1,0,2 \ rangle $ ΠΈ $ \ langle -1, -2,4 \ rangle $. (ΠΎΡ‚Π²Π΅Ρ‡Π°Ρ‚ΡŒ)

    ΠŸΡ€. 12.4.3 НайдитС пСрСкрСстноС ΠΏΡ€ΠΎΠΈΠ·Π²Π΅Π΄Π΅Π½ΠΈΠ΅ $ \ langle -2,1,3 \ rangle $ ΠΈ $ \ langle 5,2, -1 \ rangle $. (ΠΎΡ‚Π²Π΅Ρ‡Π°Ρ‚ΡŒ)

    ΠŸΡ€. 12.4.4 НайдитС Π²Π΅ΠΊΡ‚ΠΎΡ€Π½ΠΎΠ΅ ΠΏΡ€ΠΎΠΈΠ·Π²Π΅Π΄Π΅Π½ΠΈΠ΅ $ \ langle 1,0,0 \ rangle $ ΠΈ $ \ langle 0,0,1 \ rangle $. (ΠΎΡ‚Π²Π΅Ρ‡Π°Ρ‚ΡŒ)

    ΠŸΡ€. 12.4,5 Π”Π²Π° Π²Π΅ΠΊΡ‚ΠΎΡ€Π° $ {\ bf u} $ ΠΈ $ {\ bf v} $ Ρ€Π°Π·Π΄Π΅Π»Π΅Π½Ρ‹ ΡƒΠ³ΠΎΠ» $ \ pi / 6 $ ΠΈ $ | {\ bf u} | = 2 $ ΠΈ $ | {\ bf v} | = 3 $. ΠΠ°Ρ…ΠΎΠ΄ΠΈΡ‚ΡŒ $ | {\ bf u} \ times {\ bf v} | $. (ΠΎΡ‚Π²Π΅Ρ‡Π°Ρ‚ΡŒ)

    ΠŸΡ€. 12.4.6 Π”Π²Π° Π²Π΅ΠΊΡ‚ΠΎΡ€Π° $ {\ bf u} $ ΠΈ $ {\ bf v} $ Ρ€Π°Π·Π΄Π΅Π»Π΅Π½Ρ‹ ΡƒΠ³ΠΎΠ» $ \ pi / 4 $ ΠΈ $ | {\ bf u} | = 3 $ ΠΈ $ | {\ bf v} | = 7 $. ΠΠ°Ρ…ΠΎΠ΄ΠΈΡ‚ΡŒ $ | {\ bf u} \ times {\ bf v} | $. (ΠΎΡ‚Π²Π΅Ρ‡Π°Ρ‚ΡŒ)

    ΠŸΡ€. 12.4.7 НайдитС ΠΏΠ»ΠΎΡ‰Π°Π΄ΡŒ ΠΏΠ°Ρ€Π°Π»Π»Π΅Π»ΠΎΠ³Ρ€Π°ΠΌΠΌΠ° с Π²Π΅Ρ€ΡˆΠΈΠ½Π°ΠΌΠΈ $ (0,0) $, $ (1,2) $, (3,7) $ ΠΈ (2,5) $. (ΠΎΡ‚Π²Π΅Ρ‡Π°Ρ‚ΡŒ)

    ΠŸΡ€. 12.4,8 НайдитС ΠΏΠ»ΠΎΡ‰Π°Π΄ΡŒ ΠΏΠ°Ρ€Π°Π»Π»Π΅Π»ΠΎΠ³Ρ€Π°ΠΌΠΌΠ° с Π²Π΅Ρ€ΡˆΠΈΠ½Π°ΠΌΠΈ $ (0, -1) $, $ (3,4) $, $ (1,6) $ ΠΈ $ (- 2,1) $. (ΠΎΡ‚Π²Π΅Ρ‡Π°Ρ‚ΡŒ)

    ΠŸΡ€. 12.4.9 НайдитС ΠΏΠ»ΠΎΡ‰Π°Π΄ΡŒ Ρ‚Ρ€Π΅ΡƒΠ³ΠΎΠ»ΡŒΠ½ΠΈΠΊΠ° с Π²Π΅Ρ€ΡˆΠΈΠ½Π°ΠΌΠΈ $ (2,0,0) $, $ (1,3,4) $, ΠΈ $ (- 2, -1,1) $. (ΠΎΡ‚Π²Π΅Ρ‡Π°Ρ‚ΡŒ)

    ΠŸΡ€. 12.4.10 НайдитС ΠΏΠ»ΠΎΡ‰Π°Π΄ΡŒ Ρ‚Ρ€Π΅ΡƒΠ³ΠΎΠ»ΡŒΠ½ΠΈΠΊΠ° с Π²Π΅Ρ€ΡˆΠΈΠ½Π°ΠΌΠΈ $ (2, -2,1) $, $ (- 3,2,3) $, ΠΈ $ (3,3, -2) $. (ΠΎΡ‚Π²Π΅Ρ‡Π°Ρ‚ΡŒ)

    ΠŸΡ€. 12.4.11 НайдитС ΠΈ ΠΎΠ±ΡŠΡΡΠ½ΠΈΡ‚Π΅ Π·Π½Π°Ρ‡Π΅Π½ΠΈΠ΅ $ ({\ bf i} \ times {\ bf j}) \ times {\ bf k} $ ΠΈ $ ({\ bf i} + {\ bf j}) \ times ({\ bf i} — {\ bf j}) $.

    ΠŸΡ€. 12.4.12 Π”ΠΎΠΊΠ°ΠΆΠΈΡ‚Π΅, Ρ‡Ρ‚ΠΎ для всСх Π²Π΅ΠΊΡ‚ΠΎΡ€ΠΎΠ² $ {\ bf u} $ ΠΈ $ {\ bf v} $, $ ({\ bf u} \ times {\ bf v}) \ cdot {\ bf v} = 0 $.

    ΠŸΡ€. 12.4.13 Π”ΠΎΠΊΠ°ΠΆΠΈΡ‚Π΅ Ρ‚Π΅ΠΎΡ€Π΅ΠΌΡƒ 12.4.2.

    ΠŸΡ€. 12.4.14 ΠžΠΏΡ€Π΅Π΄Π΅Π»ΠΈΡ‚Π΅ Ρ‚Ρ€ΠΎΠΉΠ½ΠΎΠ΅ ΠΏΡ€ΠΎΠΈΠ·Π²Π΅Π΄Π΅Π½ΠΈΠ΅ Ρ‚Ρ€Π΅Ρ… Π²Π΅ΠΊΡ‚ΠΎΡ€ΠΎΠ², $ {\ bf x} $, $ {\ bf y} $ ΠΈ $ {\ bf z} $, Ρ‡Ρ‚ΠΎΠ±Ρ‹ Π±Ρ‹Ρ‚ΡŒ скаляром $ {\ bf x} \ cdot ({\ bf y} \ times {\ bf z}) $. ΠŸΠΎΠΊΠ°ΠΆΠΈΡ‚Π΅, Ρ‡Ρ‚ΠΎ Ρ‚Ρ€ΠΈ Π²Π΅ΠΊΡ‚ΠΎΡ€Π° Π»Π΅ΠΆΠ°Ρ‚ Π² ΠΎΠ΄Π½ΠΎΠΉ плоскости Ρ‚ΠΎΠ³Π΄Π° ΠΈ Ρ‚ΠΎΠ»ΡŒΠΊΠΎ Ρ‚ΠΎΠ³Π΄Π°, ΠΊΠΎΠ³Π΄Π° ΠΈΡ… Ρ‚Ρ€ΠΎΠΉΠ½ΠΎΠ΅ ΠΏΡ€ΠΎΠΈΠ·Π²Π΅Π΄Π΅Π½ΠΈΠ΅ Ρ€Π°Π²Π½ΠΎ Π½ΡƒΠ»ΡŽ. Π£Π±Π΅Π΄ΠΈΡ‚Π΅ΡΡŒ, Ρ‡Ρ‚ΠΎ $ \ langle 1, 5, -2 \ rangle $, $ \ langle 4, 3, 0 \ rangle $ ΠΈ $ \ langle 6, 13, -4 \ rangle $ — это ΠΊΠΎΠΏΠ»Π°Π½Π°Ρ€Π½Ρ‹ΠΉ.

    Как ΡƒΠΌΠ½ΠΎΠΆΠΈΡ‚ΡŒ Π²Π΅ΠΊΡ‚ΠΎΡ€Ρ‹ — БкалярноС (Ρ‚ΠΎΡ‡Π΅Ρ‡Π½ΠΎΠ΅) ΠΏΡ€ΠΎΠΈΠ·Π²Π΅Π΄Π΅Π½ΠΈΠ΅

    Как ΡƒΠΌΠ½ΠΎΠΆΠΈΡ‚ΡŒ Π²Π΅ΠΊΡ‚ΠΎΡ€Ρ‹

    ΠšΠ»ΡŽΡ‡Π΅Π²Ρ‹Π΅ Ρ‚Π΅Ρ€ΠΌΠΈΠ½Ρ‹

  2. Π•Π΄ΠΈΠ½ΠΈΡ‡Π½Ρ‹ΠΉ Π²Π΅ΠΊΡ‚ΠΎΡ€
  3. БкалярноС (Ρ‚ΠΎΡ‡Π΅Ρ‡Π½ΠΎΠ΅) ΠΏΡ€ΠΎΠΈΠ·Π²Π΅Π΄Π΅Π½ΠΈΠ΅
  4. Π¦Π΅Π»ΠΈ

  5. ΠžΠΏΡ€Π΅Π΄Π΅Π»ΠΈΡ‚Π΅ ΠΊΠΎΠ½Ρ†Π΅ΠΏΡ†ΠΈΡŽ Π΅Π΄ΠΈΠ½ΠΈΡ‡Π½Ρ‹Ρ… Π²Π΅ΠΊΡ‚ΠΎΡ€ΠΎΠ²
  6. Π˜ΡΠΏΠΎΠ»ΡŒΠ·ΠΎΠ²Π°Ρ‚ΡŒ Π΅Π΄ΠΈΠ½ΠΈΡ‡Π½Ρ‹Π΅ Π²Π΅ΠΊΡ‚ΠΎΡ€Ρ‹ для алгСбраичСского прСдставлСния Π²Π΅ΠΊΡ‚ΠΎΡ€ΠΎΠ²
  7. ΠžΠΏΡ€Π΅Π΄Π΅Π»ΠΈΡ‚ΡŒ ΡƒΠΌΠ½ΠΎΠΆΠ΅Π½ΠΈΠ΅ скаляра ΠΈ Π²Π΅ΠΊΡ‚ΠΎΡ€Π°
  8. Π˜ΡΠΏΠΎΠ»ΡŒΠ·ΡƒΠΉΡ‚Π΅ скалярноС ΠΏΡ€ΠΎΠΈΠ·Π²Π΅Π΄Π΅Π½ΠΈΠ΅, Ρ‡Ρ‚ΠΎΠ±Ρ‹ Π²Ρ‹Ρ‡ΠΈΡΠ»ΠΈΡ‚ΡŒ Π΄Π»ΠΈΠ½Ρƒ Π²Π΅ΠΊΡ‚ΠΎΡ€Π°

  9. Π’ этой ΡΡ‚Π°Ρ‚ΡŒΠ΅ ΠΌΡ‹ рассмотрим Π΄Ρ€ΡƒΠ³ΠΎΠ΅ прСдставлСниС Π²Π΅ΠΊΡ‚ΠΎΡ€ΠΎΠ², Π° Ρ‚Π°ΠΊΠΆΠ΅ основы умноТСния Π²Π΅ΠΊΡ‚ΠΎΡ€ΠΎΠ².

    Π•Π΄ΠΈΠ½ΠΈΡ‡Π½Ρ‹Π΅ Π²Π΅ΠΊΡ‚ΠΎΡ€Ρ‹

    Π₯отя координатная Ρ„ΠΎΡ€ΠΌΠ° для прСдставлСния Π²Π΅ΠΊΡ‚ΠΎΡ€ΠΎΠ² ясна, ΠΌΡ‹ Ρ‚Π°ΠΊΠΆΠ΅ ΠΌΠΎΠΆΠ΅ΠΌ ΠΏΡ€Π΅Π΄ΡΡ‚Π°Π²ΠΈΡ‚ΡŒ ΠΈΡ… Π² Π²ΠΈΠ΄Π΅ алгСбраичСских Π²Ρ‹Ρ€Π°ΠΆΠ΅Π½ΠΈΠΉ с использованиСм Π΅Π΄ΠΈΠ½ΠΈΡ‡Π½Ρ‹Ρ… Π²Π΅ΠΊΡ‚ΠΎΡ€ΠΎΠ². Π’ Π½Π°ΡˆΠΈΡ… стандартных ΠΏΡ€ΡΠΌΠΎΡƒΠ³ΠΎΠ»ΡŒΠ½Ρ‹Ρ… (ΠΈΠ»ΠΈ Π΅Π²ΠΊΠ»ΠΈΠ΄ΠΎΠ²Ρ‹Ρ…) ΠΊΠΎΠΎΡ€Π΄ΠΈΠ½Π°Ρ‚Π°Ρ… ( x, y, ΠΈ z ) Π΅Π΄ΠΈΠ½ΠΈΡ‡Π½Ρ‹ΠΉ Π²Π΅ΠΊΡ‚ΠΎΡ€ прСдставляСт собой Π²Π΅ΠΊΡ‚ΠΎΡ€ Π΄Π»ΠΈΠ½Ρ‹ 1, ΠΏΠ°Ρ€Π°Π»Π»Π΅Π»ΡŒΠ½Ρ‹ΠΉ ΠΎΠ΄Π½ΠΎΠΉ ΠΈΠ· осСй. Π’ Π΄Π²ΡƒΠΌΠ΅Ρ€Π½ΠΎΠΉ ΠΊΠΎΠΎΡ€Π΄ΠΈΠ½Π°Ρ‚Π½ΠΎΠΉ плоскости Π΅Π΄ΠΈΠ½ΠΈΡ‡Π½Ρ‹Π΅ Π²Π΅ΠΊΡ‚ΠΎΡ€Ρ‹ часто Π½Π°Π·Ρ‹Π²Π°ΡŽΡ‚ΡΡ i, ΠΈ j, , ΠΊΠ°ΠΊ ΠΏΠΎΠΊΠ°Π·Π°Π½ΠΎ Π½Π° Π³Ρ€Π°Ρ„ΠΈΠΊΠ΅ Π½ΠΈΠΆΠ΅.Для Ρ‚Ρ€Π΅Ρ… ΠΈΠ·ΠΌΠ΅Ρ€Π΅Π½ΠΈΠΉ ΠΌΡ‹ добавляСм Π΅Π΄ΠΈΠ½ΠΈΡ‡Π½Ρ‹ΠΉ Π²Π΅ΠΊΡ‚ΠΎΡ€ k , ΡΠΎΠΎΡ‚Π²Π΅Ρ‚ΡΡ‚Π²ΡƒΡŽΡ‰ΠΈΠΉ Π½Π°ΠΏΡ€Π°Π²Π»Π΅Π½ΠΈΡŽ оси z . Π­Ρ‚ΠΈ Π²Π΅ΠΊΡ‚ΠΎΡ€Ρ‹ ΠΎΠΏΡ€Π΅Π΄Π΅Π»ΡΡŽΡ‚ΡΡ алгСбраичСски ΡΠ»Π΅Π΄ΡƒΡŽΡ‰ΠΈΠΌ ΠΎΠ±Ρ€Π°Π·ΠΎΠΌ.

    i = (1, 0) ΠΈΠ»ΠΈ (1, 0, 0)

    j = (0, 1) ΠΈΠ»ΠΈ (0, 1, 0)

    ΠΊ = (0, 0, 1)

    ΠŸΡ€Π΅ΠΆΠ΄Π΅ Ρ‡Π΅ΠΌ ΠΌΡ‹ прСдставим алгСбраичСскоС прСдставлСниС Π²Π΅ΠΊΡ‚ΠΎΡ€ΠΎΠ² с ΠΏΠΎΠΌΠΎΡ‰ΡŒΡŽ Π΅Π΄ΠΈΠ½ΠΈΡ‡Π½Ρ‹Ρ… Π²Π΅ΠΊΡ‚ΠΎΡ€ΠΎΠ², ΠΌΡ‹ Π΄ΠΎΠ»ΠΆΠ½Ρ‹ сначала ввСсти ΡƒΠΌΠ½ΠΎΠΆΠ΅Π½ΠΈΠ΅ Π²Π΅ΠΊΡ‚ΠΎΡ€ΠΎΠ² — Π² Π΄Π°Π½Π½ΠΎΠΌ случаС Π½Π° скаляры.

    Π£ΠΌΠ½ΠΎΠΆΠ΅Π½ΠΈΠ΅ Π²Π΅ΠΊΡ‚ΠΎΡ€Π° Π½Π° скаляр

    Π£ΠΌΠ½ΠΎΠΆΠ΅Π½ΠΈΠ΅ Π²Π΅ΠΊΡ‚ΠΎΡ€ΠΎΠ² слоТнСС, Ρ‡Π΅ΠΌ ΡƒΠΌΠ½ΠΎΠΆΠ΅Π½ΠΈΠ΅ Ρ‚ΠΎΠ»ΡŒΠΊΠΎ скаляров, поэтому ΠΌΡ‹ Π΄ΠΎΠ»ΠΆΠ½Ρ‹ ΠΎΡ‚Π½ΠΎΡΠΈΡ‚ΡŒΡΡ ΠΊ ΠΏΡ€Π΅Π΄ΠΌΠ΅Ρ‚Ρƒ остороТно. НачнСм с ΠΏΡ€ΠΎΡΡ‚Π΅ΠΉΡˆΠ΅Π³ΠΎ случая: умноТСния Π²Π΅ΠΊΡ‚ΠΎΡ€Π° Π½Π° скаляр. НиТС ΠΏΡ€ΠΈΠ²Π΅Π΄Π΅Π½ΠΎ ΠΎΠΏΡ€Π΅Π΄Π΅Π»Π΅Π½ΠΈΠ΅ умноТСния скаляра c Π½Π° Π²Π΅ΠΊΡ‚ΠΎΡ€ a, , Π³Π΄Π΅ a = ( x, y ). (ΠžΠΏΡΡ‚ΡŒ ΠΆΠ΅, ΠΌΡ‹ ΠΌΠΎΠΆΠ΅ΠΌ Π»Π΅Π³ΠΊΠΎ Ρ€Π°ΡΡˆΠΈΡ€ΠΈΡ‚ΡŒ эти ΠΏΡ€ΠΈΠ½Ρ†ΠΈΠΏΡ‹ Π΄ΠΎ Ρ‚Ρ€Π΅Ρ… ΠΈΠ·ΠΌΠ΅Ρ€Π΅Π½ΠΈΠΉ.)

    БкалярноС ΡƒΠΌΠ½ΠΎΠΆΠ΅Π½ΠΈΠ΅ ΠΊΠΎΠΌΠΌΡƒΡ‚Π°Ρ‚ΠΈΠ²Π½ΠΎ, поэтому . Но Ρ‡Ρ‚ΠΎ ΠΎΠ·Π½Π°Ρ‡Π°Π΅Ρ‚ это ΡƒΠΌΠ½ΠΎΠΆΠ΅Π½ΠΈΠ΅? Как оказалось, ΡƒΠΌΠ½ΠΎΠΆΠ΅Π½ΠΈΠ΅ Π½Π° скаляр c ΡƒΠ²Π΅Π»ΠΈΡ‡ΠΈΠ²Π°Π΅Ρ‚ Π΄Π»ΠΈΠ½Ρƒ Π²Π΅ΠΊΡ‚ΠΎΡ€Π° Π² c. Π­Ρ‚ΠΎ Π½Π°ΠΈΠ±ΠΎΠ»Π΅Π΅ ясно Π²ΠΈΠ΄Π½ΠΎ с Π΅Π΄ΠΈΠ½ΠΈΡ‡Π½Ρ‹ΠΌΠΈ Π²Π΅ΠΊΡ‚ΠΎΡ€Π°ΠΌΠΈ, Π½ΠΎ это ΠΏΡ€ΠΈΠΌΠ΅Π½ΠΈΠΌΠΎ ΠΊ Π»ΡŽΠ±ΠΎΠΌΡƒ Π²Π΅ΠΊΡ‚ΠΎΡ€Ρƒ. (Однако ΡƒΠΌΠ½ΠΎΠΆΠ΅Π½ΠΈΠ΅ Π½Π° ΠΎΡ‚Ρ€ΠΈΡ†Π°Ρ‚Π΅Π»ΡŒΠ½Ρ‹ΠΉ скаляр мСняСт Π½Π°ΠΏΡ€Π°Π²Π»Π΅Π½ΠΈΠ΅ Π²Π΅ΠΊΡ‚ΠΎΡ€Π° Π½Π° ΠΏΡ€ΠΎΡ‚ΠΈΠ²ΠΎΠΏΠΎΠ»ΠΎΠΆΠ½ΠΎΠ΅.На ΠΏΡ€ΠΈΠ²Π΅Π΄Π΅Π½Π½ΠΎΠΌ Π½ΠΈΠΆΠ΅ Π³Ρ€Π°Ρ„ΠΈΠΊΠ΅ ΠΏΠΎΠΊΠ°Π·Π°Π½Ρ‹ Π½Π΅ΠΊΠΎΡ‚ΠΎΡ€Ρ‹Π΅ ΠΏΡ€ΠΈΠΌΠ΅Ρ€Ρ‹ с использованиСм c = 2. (Напомним, Ρ‡Ρ‚ΠΎ располоТСниС Π²Π΅ΠΊΡ‚ΠΎΡ€Π° Π½Π΅ влияСт Π½Π° Π΅Π³ΠΎ Π·Π½Π°Ρ‡Π΅Π½ΠΈΠ΅.)

    ΠŸΡ€Π°ΠΊΡ‚ΠΈΡ‡Π΅ΡΠΊΠ°Ρ Π·Π°Π΄Π°Ρ‡Π°: Для Π²Π΅ΠΊΡ‚ΠΎΡ€Π° a = (3, 1) Π½Π°ΠΉΠ΄ΠΈΡ‚Π΅ Π²Π΅ΠΊΡ‚ΠΎΡ€ Π² Ρ‚ΠΎΠΌ ΠΆΠ΅ Π½Π°ΠΏΡ€Π°Π²Π»Π΅Π½ΠΈΠΈ, Ρ‡Ρ‚ΠΎ ΠΈ a , Π½ΠΎ Π² Π΄Π²Π° Ρ€Π°Π·Π° большС Π΅Π³ΠΎ Π΄Π»ΠΈΠ½Ρ‹.

    РСшСниС: Когда ΠΌΡ‹ ΡƒΠΌΠ½ΠΎΠΆΠ°Π΅ΠΌ Π²Π΅ΠΊΡ‚ΠΎΡ€ Π½Π° скаляр, Π½Π°ΠΏΡ€Π°Π²Π»Π΅Π½ΠΈΠ΅ Π²Π΅ΠΊΡ‚ΠΎΡ€Π° произвСдСния совпадаСт с Π½Π°ΠΏΡ€Π°Π²Π»Π΅Π½ΠΈΠ΅ΠΌ мноТитСля.ЕдинствСнная Ρ€Π°Π·Π½ΠΈΡ†Π° Π² Ρ‚ΠΎΠΌ, Ρ‡Ρ‚ΠΎ Π΄Π»ΠΈΠ½Π° умноТаСтся Π½Π° скаляр. Π˜Ρ‚Π°ΠΊ, Ρ‡Ρ‚ΠΎΠ±Ρ‹ ΠΏΠΎΠ»ΡƒΡ‡ΠΈΡ‚ΡŒ Π²Π΅ΠΊΡ‚ΠΎΡ€, ΠΊΠΎΡ‚ΠΎΡ€Ρ‹ΠΉ Π² Π΄Π²Π° Ρ€Π°Π·Π° Π΄Π»ΠΈΠ½Π½Π΅Π΅ a , Π½ΠΎ Π² Ρ‚ΠΎΠΌ ΠΆΠ΅ Π½Π°ΠΏΡ€Π°Π²Π»Π΅Π½ΠΈΠΈ, Ρ‡Ρ‚ΠΎ ΠΈ a, просто ΡƒΠΌΠ½ΠΎΠΆΡŒΡ‚Π΅ Π½Π° 2.

    2 a = 2 β€’ (3, 1) = (2 β€’ 3, 2 β€’ 1) = (6, 2)

    АлгСбраичСскоС прСдставлСниС Π²Π΅ΠΊΡ‚ΠΎΡ€ΠΎΠ²

    ΠœΡ‹ ΠΌΠΎΠΆΠ΅ΠΌ ΠΈΡΠΏΠΎΠ»ΡŒΠ·ΠΎΠ²Π°Ρ‚ΡŒ скалярноС ΡƒΠΌΠ½ΠΎΠΆΠ΅Π½ΠΈΠ΅ Π½Π° Π²Π΅ΠΊΡ‚ΠΎΡ€Ρ‹ для алгСбраичСского прСдставлСния Π²Π΅ΠΊΡ‚ΠΎΡ€ΠΎΠ².ΠžΠ±Ρ€Π°Ρ‚ΠΈΡ‚Π΅ Π²Π½ΠΈΠΌΠ°Π½ΠΈΠ΅, Ρ‡Ρ‚ΠΎ любой Π΄Π²ΡƒΠΌΠ΅Ρ€Π½Ρ‹ΠΉ Π²Π΅ΠΊΡ‚ΠΎΡ€ v ΠΌΠΎΠΆΠ΅Ρ‚ Π±Ρ‹Ρ‚ΡŒ прСдставлСн ΠΊΠ°ΠΊ сумма Π΄Π»ΠΈΠ½Ρ‹, ΡƒΠΌΠ½ΠΎΠΆΠ΅Π½Π½ΠΎΠΉ Π½Π° Π΅Π΄ΠΈΠ½ΠΈΡ‡Π½Ρ‹ΠΉ Π²Π΅ΠΊΡ‚ΠΎΡ€ i , ΠΈ Π΄Ρ€ΡƒΠ³ΠΎΠΉ Π΄Π»ΠΈΠ½Ρ‹, ΡƒΠΌΠ½ΠΎΠΆΠ΅Π½Π½ΠΎΠΉ Π½Π° Π΅Π΄ΠΈΠ½ΠΈΡ‡Π½Ρ‹ΠΉ Π²Π΅ΠΊΡ‚ΠΎΡ€ j. НапримСр, рассмотрим Π²Π΅ΠΊΡ‚ΠΎΡ€ (2, 4). ΠŸΡ€ΠΈΠΌΠ΅Π½ΠΈΡ‚Π΅ ΠΏΡ€Π°Π²ΠΈΠ»Π° Π²Π΅ΠΊΡ‚ΠΎΡ€ΠΎΠ², ΠΊΠΎΡ‚ΠΎΡ€Ρ‹Π΅ ΠΌΡ‹ ΡƒΠ·Π½Π°Π»ΠΈ Π΄ΠΎ сих ΠΏΠΎΡ€:

    (2, 4) = (2, 0) + (0, 4) (ΠΏΡ€Π°Π²ΠΈΠ»ΠΎ слоТСния для Π²Π΅ΠΊΡ‚ΠΎΡ€ΠΎΠ²)

    (2, 4) = 2 β€’ (1, 0) + 4 β€’ (0, 1) (ΠΏΡ€Π°Π²ΠΈΠ»ΠΎ умноТСния для скаляров ΠΈ Π²Π΅ΠΊΡ‚ΠΎΡ€ΠΎΠ²)

    (2, 4) = 2 i + 4 j

    ГрафичСски ΠΌΡ‹ складываСм Π΄Π²Π° Π²Π΅ΠΊΡ‚ΠΎΡ€Π° Π² Π΅Π΄ΠΈΠ½ΠΈΡ‡Π½Ρ‹Ρ… направлСниях, Ρ‡Ρ‚ΠΎΠ±Ρ‹ ΠΏΠΎΠ»ΡƒΡ‡ΠΈΡ‚ΡŒ ΠΏΡ€ΠΎΠΈΠ·Π²ΠΎΠ»ΡŒΠ½Ρ‹ΠΉ Π²Π΅ΠΊΡ‚ΠΎΡ€.

    ΠžΠ±Ρ€Π°Ρ‚ΠΈΡ‚Π΅ Π²Π½ΠΈΠΌΠ°Π½ΠΈΠ΅, Ρ‡Ρ‚ΠΎ Π΅Π΄ΠΈΠ½ΠΈΡ‡Π½Ρ‹Π΅ Π²Π΅ΠΊΡ‚ΠΎΡ€Ρ‹ Π΄Π΅ΠΉΡΡ‚Π²ΡƒΡŽΡ‚ ΠΏΠΎΡ‡Ρ‚ΠΈ ΠΈΠ΄Π΅Π½Ρ‚ΠΈΡ‡Π½ΠΎ ΠΏΠ΅Ρ€Π΅ΠΌΠ΅Π½Π½Ρ‹ΠΌ. Π’Π°ΠΊΠΈΠΌ ΠΎΠ±Ρ€Π°Π·ΠΎΠΌ, ΠΌΡ‹ ΠΌΠΎΠΆΠ΅ΠΌ ΡΠ»ΠΎΠΆΠΈΡ‚ΡŒ Π΄Π²Π° Π²Π΅ΠΊΡ‚ΠΎΡ€Π° a ΠΈ b ΡΠ»Π΅Π΄ΡƒΡŽΡ‰ΠΈΠΌ ΠΎΠ±Ρ€Π°Π·ΠΎΠΌ.

    a = 3 i — 2 j b = i + 3 j

    a + b = (3 i -2 j ) + ( i + 3 j ) = 3 i + i -2 j + 3 j = 4 i + j

    Π­Ρ‚ΠΎ прСдставлСниС обСспСчиваСт Π±ΠΎΠ»ΡŒΡˆΡƒΡŽ Π³ΠΈΠ±ΠΊΠΎΡΡ‚ΡŒ, Ρ‡Π΅ΠΌ прСдставлСниС ΠΊΠΎΠΎΡ€Π΄ΠΈΠ½Π°Ρ‚, Π½ΠΎ эквивалСнтно.

    ΠŸΡ€Π°ΠΊΡ‚ΠΈΡ‡Π΅ΡΠΊΠ°Ρ Π·Π°Π΄Π°Ρ‡Π°: Π’Ρ‹Ρ‡ΠΈΡΠ»ΠΈΡ‚ΡŒ сумму ΠΈ Ρ€Π°Π·Π½ΠΎΡΡ‚ΡŒ ( t u ) Π²Π΅ΠΊΡ‚ΠΎΡ€ΠΎΠ² t = -2 i + 3 j ΠΈ u = 6 i — 4 j .

    РСшСниС: ΠœΡ‹ ΠΌΠΎΠΆΠ΅ΠΌ довольно Π»Π΅Π³ΠΊΠΎ Ρ€Π΅ΡˆΠΈΡ‚ΡŒ эту Π·Π°Π΄Π°Ρ‡Ρƒ алгСбраичСски.

    t + u = (-2 i + 3 j ) + (6 i — 4 j ) = 4 i j = (4, -1)

    t u = (-2 i + 3 j ) — (6 i -4 j ) = -2 i + 3 j -6 i + 4 j = -8 i + 7 j = (-8, 7)

    Π’Π΅ΠΊΡ‚ΠΎΡ€Π½ΠΎΠ΅ ΡƒΠΌΠ½ΠΎΠΆΠ΅Π½ΠΈΠ΅: скалярноС (Ρ‚ΠΎΡ‡Π΅Ρ‡Π½ΠΎΠ΅) ΠΏΡ€ΠΎΠΈΠ·Π²Π΅Π΄Π΅Π½ΠΈΠ΅

    Π£ΠΌΠ½ΠΎΠΆΠ΅Π½ΠΈΠ΅ Π΄Π²ΡƒΡ… Π²Π΅ΠΊΡ‚ΠΎΡ€ΠΎΠ² Π½Π΅ΠΌΠ½ΠΎΠ³ΠΎ слоТнСС, Ρ‡Π΅ΠΌ скалярноС ΡƒΠΌΠ½ΠΎΠΆΠ΅Π½ΠΈΠ΅.ΠžΠΏΡ€Π΅Π΄Π΅Π»Π΅Π½Ρ‹ Π΄Π²Π° Ρ‚ΠΈΠΏΠ° умноТСния с участиСм Π΄Π²ΡƒΡ… Π²Π΅ΠΊΡ‚ΠΎΡ€ΠΎΠ²: Ρ‚Π°ΠΊ Π½Π°Π·Ρ‹Π²Π°Π΅ΠΌΠΎΠ΅ скалярноС ΠΏΡ€ΠΎΠΈΠ·Π²Π΅Π΄Π΅Π½ΠΈΠ΅ (ΠΈΠ»ΠΈ «скалярноС ΠΏΡ€ΠΎΠΈΠ·Π²Π΅Π΄Π΅Π½ΠΈΠ΅Β») ΠΈ Ρ‚Π°ΠΊ Π½Π°Π·Ρ‹Π²Π°Π΅ΠΌΠΎΠ΅ Π²Π΅ΠΊΡ‚ΠΎΡ€Π½ΠΎΠ΅ ΠΏΡ€ΠΎΠΈΠ·Π²Π΅Π΄Π΅Π½ΠΈΠ΅ (ΠΈΠ»ΠΈ «пСрСкрСстноС ΠΏΡ€ΠΎΠΈΠ·Π²Π΅Π΄Π΅Π½ΠΈΠ΅Β»). Для простоты ΠΌΡ‹ обратимся Ρ‚ΠΎΠ»ΡŒΠΊΠΎ ΠΊ скалярному ΠΏΡ€ΠΎΠΈΠ·Π²Π΅Π΄Π΅Π½ΠΈΡŽ, Π½ΠΎ Π½Π° этом этапС Ρƒ вас Π΄ΠΎΠ»ΠΆΠ½Π° Π±Ρ‹Ρ‚ΡŒ достаточная матСматичСская Π±Π°Π·Π° для понимания Π²Π΅ΠΊΡ‚ΠΎΡ€Π½ΠΎΠ³ΠΎ произвСдСния. БкалярноС ΠΏΡ€ΠΎΠΈΠ·Π²Π΅Π΄Π΅Π½ΠΈΠ΅ (ΠΈΠ»ΠΈ скалярноС ΠΏΡ€ΠΎΠΈΠ·Π²Π΅Π΄Π΅Π½ΠΈΠ΅ ) Π΄Π²ΡƒΡ… Π²Π΅ΠΊΡ‚ΠΎΡ€ΠΎΠ² опрСдСляСтся ΡΠ»Π΅Π΄ΡƒΡŽΡ‰ΠΈΠΌ ΠΎΠ±Ρ€Π°Π·ΠΎΠΌ Π² Π΄Π²ΡƒΡ… измСрСниях. Как всСгда, это ΠΎΠΏΡ€Π΅Π΄Π΅Π»Π΅Π½ΠΈΠ΅ ΠΌΠΎΠΆΠ½ΠΎ Π»Π΅Π³ΠΊΠΎ Ρ€Π°ΡΡˆΠΈΡ€ΠΈΡ‚ΡŒ Π΄ΠΎ Ρ‚Ρ€Π΅Ρ… ΠΈΠ·ΠΌΠ΅Ρ€Π΅Π½ΠΈΠΉ — просто слСдуйтС ΡˆΠ°Π±Π»ΠΎΠ½Ρƒ.ΠžΠ±Ρ€Π°Ρ‚ΠΈΡ‚Π΅ Π²Π½ΠΈΠΌΠ°Π½ΠΈΠ΅, Ρ‡Ρ‚ΠΎ опСрация всСгда Π΄ΠΎΠ»ΠΆΠ½Π° ΠΎΠ±ΠΎΠ·Π½Π°Ρ‡Π°Ρ‚ΡŒΡΡ Ρ‚ΠΎΡ‡ΠΊΠΎΠΉ (β€’), Ρ‡Ρ‚ΠΎΠ±Ρ‹ ΠΎΡ‚Π»ΠΈΡ‡Π°Ρ‚ΡŒ ΠΎΡ‚ Π²Π΅ΠΊΡ‚ΠΎΡ€Π½ΠΎΠ³ΠΎ произвСдСния, Π² ΠΊΠΎΡ‚ΠΎΡ€ΠΎΠΌ ΠΈΡΠΏΠΎΠ»ΡŒΠ·ΡƒΠ΅Ρ‚ΡΡ символ Π²Ρ€Π΅ΠΌΠ΅Π½ΠΈ () — ΠΎΡ‚ΡΡŽΠ΄Π° ΠΈ названия скалярноС ΠΏΡ€ΠΎΠΈΠ·Π²Π΅Π΄Π΅Π½ΠΈΠ΅ ΠΈ пСрСкрСстноС ΠΏΡ€ΠΎΠΈΠ·Π²Π΅Π΄Π΅Π½ΠΈΠ΅.

    Однако Π·Π½Π°Ρ‡Π΅Π½ΠΈΠ΅ этого ΠΏΡ€ΠΎΠ΄ΡƒΠΊΡ‚Π° ΠΌΠΎΠΆΠ΅Ρ‚ Π±Ρ‹Ρ‚ΡŒ Π²Π°ΠΌ Π½Π΅ совсСм понятно Π½Π° Π΄Π°Π½Π½ΠΎΠΌ этапС. ΠœΡ‹ ΠΌΠΎΠΆΠ΅ΠΌ ΠΏΡ€ΠΎΠΈΠ»Π»ΡŽΡΡ‚Ρ€ΠΈΡ€ΠΎΠ²Π°Ρ‚ΡŒ это, посмотрСв Π½Π° простой случай: скалярноС ΠΏΡ€ΠΎΠΈΠ·Π²Π΅Π΄Π΅Π½ΠΈΠ΅ ΠΏΡ€ΠΎΠΈΠ·Π²ΠΎΠ»ΡŒΠ½ΠΎΠ³ΠΎ Π²Π΅ΠΊΡ‚ΠΎΡ€Π° v ΠΈ Π΅Π΄ΠΈΠ½ΠΈΡ‡Π½Ρ‹Ρ… Π²Π΅ΠΊΡ‚ΠΎΡ€ΠΎΠ² i ΠΈ j.

    Π’Π°ΠΊΠΈΠΌ ΠΎΠ±Ρ€Π°Π·ΠΎΠΌ, v β€’ i являСтся Β«Ρ‡Π°ΡΡ‚ΡŒΡŽΒ» Π²Π΅ΠΊΡ‚ΠΎΡ€Π° v Π² Π½Π°ΠΏΡ€Π°Π²Π»Π΅Π½ΠΈΠΈ I.

    Π­Ρ‚ΠΎ объяснСниС Ρ€Π°Π±ΠΎΡ‚Π°Π΅Ρ‚, ΠΎΠ΄Π½Π°ΠΊΠΎ, Ρ‚ΠΎΠ»ΡŒΠΊΠΎ для Π²Π΅ΠΊΡ‚ΠΎΡ€ΠΎΠ² Π΄Π»ΠΈΠ½Ρ‹ 1.Когда Π΄Π²Π° ΠΏΡ€ΠΎΠΈΠ·Π²ΠΎΠ»ΡŒΠ½Ρ‹Ρ… Π²Π΅ΠΊΡ‚ΠΎΡ€Π° ΡƒΠΌΠ½ΠΎΠΆΠ°ΡŽΡ‚ΡΡ, скалярноС ΠΏΡ€ΠΎΠΈΠ·Π²Π΅Π΄Π΅Π½ΠΈΠ΅ ΠΈΠΌΠ΅Π΅Ρ‚ Π°Π½Π°Π»ΠΎΠ³ΠΈΡ‡Π½ΠΎΠ΅ Π·Π½Π°Ρ‡Π΅Π½ΠΈΠ΅, Π½ΠΎ Π²Π΅Π»ΠΈΡ‡ΠΈΠ½Π° числа Π½Π΅ΠΌΠ½ΠΎΠ³ΠΎ отличаСтся. ΠœΡ‹ Π½Π΅ Π±ΡƒΠ΄Π΅ΠΌ ΡƒΠ³Π»ΡƒΠ±Π»ΡΡ‚ΡŒΡΡ Π² это, Π½ΠΎ ΠΌΡ‹ ΠΌΠΎΠΆΠ΅ΠΌ Ρ€Π°ΡΡΠΌΠΎΡ‚Ρ€Π΅Ρ‚ΡŒ особый случай, ΠΊΠΎΠ³Π΄Π° скалярноС ΠΏΡ€ΠΎΠΈΠ·Π²Π΅Π΄Π΅Π½ΠΈΠ΅ Π΄Π°Π΅Ρ‚ Ρ†Π΅Π½Π½ΡƒΡŽ ΠΈΠ½Ρ„ΠΎΡ€ΠΌΠ°Ρ†ΠΈΡŽ.

    Π”Π»ΠΈΠ½Π° Π²Π΅ΠΊΡ‚ΠΎΡ€Π°

    Рассмотрим случай скалярного произвСдСния Π²Π΅ΠΊΡ‚ΠΎΡ€Π° v Π½Π° сСбя.

    Π”Π°Π²Π°ΠΉΡ‚Π΅ посмотрим Π½Π° эту ΡΠΈΡ‚ΡƒΠ°Ρ†ΠΈΡŽ графичСски.

    Π’ Ρ€Π΅Π·ΡƒΠ»ΡŒΡ‚Π°Ρ‚Π΅ получаСтся ΠΏΡ€ΡΠΌΠΎΡƒΠ³ΠΎΠ»ΡŒΠ½Ρ‹ΠΉ Ρ‚Ρ€Π΅ΡƒΠ³ΠΎΠ»ΡŒΠ½ΠΈΠΊ с Π³ΠΎΡ€ΠΈΠ·ΠΎΠ½Ρ‚Π°Π»ΡŒΠ½Ρ‹ΠΌ участком Π΄Π»ΠΈΠ½ΠΎΠΉ x ΠΈ Π²Π΅Ρ€Ρ‚ΠΈΠΊΠ°Π»ΡŒΠ½Ρ‹ΠΌ участком Π΄Π»ΠΈΠ½ΠΎΠΉ y. Π­Ρ‚ΠΈ Π΄Π»ΠΈΠ½Ρ‹ ΡΠΎΠΎΡ‚Π²Π΅Ρ‚ΡΡ‚Π²ΡƒΡŽΡ‚ Π΄Π»ΠΈΠ½Π°ΠΌ ΡΠΎΡΡ‚Π°Π²Π»ΡΡŽΡ‰ΠΈΡ… Π²Π΅ΠΊΡ‚ΠΎΡ€ΠΎΠ² x i ΠΈ y j, соотвСтствСнно. Но ΠΌΡ‹ Π·Π½Π°Π΅ΠΌ ΠΈΠ· Ρ‚Π΅ΠΎΡ€Π΅ΠΌΡ‹ ΠŸΠΈΡ„Π°Π³ΠΎΡ€Π°, Ρ‡Ρ‚ΠΎ — это ΠΊΠ²Π°Π΄Ρ€Π°Ρ‚ Π΄Π»ΠΈΠ½Ρ‹ Π²Π΅ΠΊΡ‚ΠΎΡ€Π° v. НС случайно, это Ρ‚ΠΎ ΠΆΠ΅ самоС, Ρ‡Ρ‚ΠΎ скалярноС ΠΏΡ€ΠΎΠΈΠ·Π²Π΅Π΄Π΅Π½ΠΈΠ΅ v Π½Π° сСбя. Π’Π°ΠΊΠΈΠΌ ΠΎΠ±Ρ€Π°Π·ΠΎΠΌ, Π΄Π»ΠΈΠ½Π° любого Π²Π΅ΠΊΡ‚ΠΎΡ€Π° v, , записанного ΠΊΠ°ΠΊ (ΠΈΠ»ΠΈ ΠΈΠ½ΠΎΠ³Π΄Π° ), являСтся ΠΊΠ²Π°Π΄Ρ€Π°Ρ‚Π½Ρ‹ΠΌ ΠΊΠΎΡ€Π½Π΅ΠΌ ΠΈΠ· скалярного произвСдСния.

    Π’ простом случаС Π΅Π΄ΠΈΠ½ΠΈΡ‡Π½Ρ‹Ρ… Π²Π΅ΠΊΡ‚ΠΎΡ€ΠΎΠ²

    Π­Ρ‚ΠΈ простыС случаи ΠΏΠΎΠΌΠΎΠ³Π°ΡŽΡ‚ ΠΏΡ€ΠΎΠ²Π΅Ρ€ΠΈΡ‚ΡŒ эту ΠΈΠ½Ρ‚Π΅Ρ€ΠΏΡ€Π΅Ρ‚Π°Ρ†ΠΈΡŽ скалярного произвСдСния.

    ΠŸΡ€Π°ΠΊΡ‚ΠΈΡ‡Π΅ΡΠΊΠ°Ρ Π·Π°Π΄Π°Ρ‡Π°: РассчитайтС Π΄Π»ΠΈΠ½Ρ‹ ΡΠ»Π΅Π΄ΡƒΡŽΡ‰ΠΈΡ… Π²Π΅ΠΊΡ‚ΠΎΡ€ΠΎΠ².

    Π°. Π±. 3 i + 2 j k c. (2, –1) d. 5 j

    РСшСниС: Π’ ΠΊΠ°ΠΆΠ΄ΠΎΠΌ случаС просто Π²ΠΎΠ·ΡŒΠΌΠΈΡ‚Π΅ ΠΊΠ²Π°Π΄Ρ€Π°Ρ‚Π½Ρ‹ΠΉ ΠΊΠΎΡ€Π΅Π½ΡŒ ΠΈΠ· скалярного произвСдСния Π²Π΅ΠΊΡ‚ΠΎΡ€Π° Π½Π° сСбя. Π Π΅Π·ΡƒΠ»ΡŒΡ‚Π°Ρ‚ — Π΄Π»ΠΈΠ½Π° Π²Π΅ΠΊΡ‚ΠΎΡ€Π° Π² ΠΊΠ°ΠΆΠ΄ΠΎΠΌ случаС. Π§Ρ‚ΠΎ касаСтся части b, просто Ρ€Π°ΡΡˆΠΈΡ€ΠΈΡ‚Π΅ ΠΎΠΏΡ€Π΅Π΄Π΅Π»Π΅Π½ΠΈΠ΅ скалярного произвСдСния Π΄ΠΎ Ρ‚Ρ€Π΅Ρ… ΠΈΠ·ΠΌΠ΅Ρ€Π΅Π½ΠΈΠΉ.

    Π°.

    Π³.

    Π³.

    Π³.

    2.4 ΠŸΡ€ΠΎΠΈΠ·Π²Π΅Π΄Π΅Π½ΠΈΡ Π²Π΅ΠΊΡ‚ΠΎΡ€ΠΎΠ² | University Physics Volume 1

    Π’Π΅ΠΊΡ‚ΠΎΡ€ ΠΌΠΎΠΆΠ½ΠΎ ΡƒΠΌΠ½ΠΎΠΆΠΈΡ‚ΡŒ Π½Π° Π΄Ρ€ΡƒΠ³ΠΎΠΉ Π²Π΅ΠΊΡ‚ΠΎΡ€, Π½ΠΎ нСльзя Ρ€Π°Π·Π΄Π΅Π»ΠΈΡ‚ΡŒ Π½Π° Π΄Ρ€ΡƒΠ³ΠΎΠΉ Π²Π΅ΠΊΡ‚ΠΎΡ€. Π•ΡΡ‚ΡŒ Π΄Π²Π° Π²ΠΈΠ΄Π° ΠΏΡ€ΠΎΠΈΠ·Π²Π΅Π΄Π΅Π½ΠΈΠΉ Π²Π΅ΠΊΡ‚ΠΎΡ€ΠΎΠ², ΡˆΠΈΡ€ΠΎΠΊΠΎ ΠΈΡΠΏΠΎΠ»ΡŒΠ·ΡƒΠ΅ΠΌΡ‹Ρ… Π² Ρ„ΠΈΠ·ΠΈΠΊΠ΅ ΠΈ Ρ‚Π΅Ρ…Π½ΠΈΠΊΠ΅.Один ΠΈΠ· Π²ΠΈΠ΄ΠΎΠ² умноТСния — это скалярноС ΡƒΠΌΠ½ΠΎΠΆΠ΅Π½ΠΈΠ΅ Π΄Π²ΡƒΡ… Π²Π΅ΠΊΡ‚ΠΎΡ€ΠΎΠ² . Π’ Ρ€Π΅Π·ΡƒΠ»ΡŒΡ‚Π°Ρ‚Π΅ скалярного произвСдСния Π΄Π²ΡƒΡ… Π²Π΅ΠΊΡ‚ΠΎΡ€ΠΎΠ² получаСтся число (скаляр), ΠΊΠ°ΠΊ ΡƒΠΊΠ°Π·Ρ‹Π²Π°Π΅Ρ‚ Π΅Π³ΠΎ Π½Π°Π·Π²Π°Π½ΠΈΠ΅. БкалярныС произвСдСния ΠΈΡΠΏΠΎΠ»ΡŒΠ·ΡƒΡŽΡ‚ΡΡ для опрСдСлСния ΠΎΡ‚Π½ΠΎΡˆΠ΅Π½ΠΈΠΉ ΠΌΠ΅ΠΆΠ΄Ρƒ Ρ€Π°Π±ΠΎΡ‚ΠΎΠΉ ΠΈ энСргиСй. НапримСр, Ρ€Π°Π±ΠΎΡ‚Π°, ΠΊΠΎΡ‚ΠΎΡ€ΡƒΡŽ сила (Π²Π΅ΠΊΡ‚ΠΎΡ€) выполняСт с ΠΎΠ±ΡŠΠ΅ΠΊΡ‚ΠΎΠΌ, вызывая Π΅Π³ΠΎ смСщСниС (Π²Π΅ΠΊΡ‚ΠΎΡ€), опрСдСляСтся ΠΊΠ°ΠΊ скалярноС ΠΏΡ€ΠΎΠΈΠ·Π²Π΅Π΄Π΅Π½ΠΈΠ΅ Π²Π΅ΠΊΡ‚ΠΎΡ€Π° силы Π½Π° Π²Π΅ΠΊΡ‚ΠΎΡ€ смСщСния. БовсСм Π΄Ρ€ΡƒΠ³ΠΎΠΉ Π²ΠΈΠ΄ умноТСния — это ΡƒΠΌΠ½ΠΎΠΆΠ΅Π½ΠΈΠ΅ Π²Π΅ΠΊΡ‚ΠΎΡ€ΠΎΠ² Π²Π΅ΠΊΡ‚ΠΎΡ€ΠΎΠ² .ΠŸΠΎΠ»ΡƒΡ‡Π΅Π½ΠΈΠ΅ Π²Π΅ΠΊΡ‚ΠΎΡ€Π½ΠΎΠ³ΠΎ произвСдСния Π΄Π²ΡƒΡ… Π²Π΅ΠΊΡ‚ΠΎΡ€ΠΎΠ² Π²ΠΎΠ·Π²Ρ€Π°Ρ‰Π°Π΅Ρ‚ Π² Ρ€Π΅Π·ΡƒΠ»ΡŒΡ‚Π°Ρ‚Π΅ Π²Π΅ΠΊΡ‚ΠΎΡ€, ΠΊΠ°ΠΊ слСдуСт ΠΈΠ· Π΅Π³ΠΎ названия. Π’Π΅ΠΊΡ‚ΠΎΡ€Π½Ρ‹Π΅ произвСдСния ΠΈΡΠΏΠΎΠ»ΡŒΠ·ΡƒΡŽΡ‚ΡΡ для опрСдСлСния Π΄Ρ€ΡƒΠ³ΠΈΡ… ΠΏΡ€ΠΎΠΈΠ·Π²ΠΎΠ΄Π½Ρ‹Ρ… Π²Π΅ΠΊΡ‚ΠΎΡ€Π½Ρ‹Ρ… Π²Π΅Π»ΠΈΡ‡ΠΈΠ½. НапримСр, ΠΏΡ€ΠΈ описании Π²Ρ€Π°Ρ‰Π΅Π½ΠΈΠΉ вСкторная Π²Π΅Π»ΠΈΡ‡ΠΈΠ½Π°, называСмая крутящим ΠΌΠΎΠΌΠ΅Π½Ρ‚ΠΎΠΌ , опрСдСляСтся ΠΊΠ°ΠΊ Π²Π΅ΠΊΡ‚ΠΎΡ€Π½ΠΎΠ΅ ΠΏΡ€ΠΎΠΈΠ·Π²Π΅Π΄Π΅Π½ΠΈΠ΅ ΠΏΡ€ΠΈΠ»ΠΎΠΆΠ΅Π½Π½ΠΎΠΉ силы (Π²Π΅ΠΊΡ‚ΠΎΡ€Π°) ΠΈ Π΅Π΅ расстояния ΠΎΡ‚ Ρ‚ΠΎΡ‡ΠΊΠΈ ΠΏΠΎΠ²ΠΎΡ€ΠΎΡ‚Π° Π΄ΠΎ силы (Π²Π΅ΠΊΡ‚ΠΎΡ€). Π’Π°ΠΆΠ½ΠΎ Ρ€Π°Π·Π»ΠΈΡ‡Π°Ρ‚ΡŒ эти Π΄Π²Π° Π²ΠΈΠ΄Π° Π²Π΅ΠΊΡ‚ΠΎΡ€Π½Ρ‹Ρ… ΡƒΠΌΠ½ΠΎΠΆΠ΅Π½ΠΈΠΉ, ΠΏΠΎΡ‚ΠΎΠΌΡƒ Ρ‡Ρ‚ΠΎ скалярноС ΠΏΡ€ΠΎΠΈΠ·Π²Π΅Π΄Π΅Π½ΠΈΠ΅ — это скалярная Π²Π΅Π»ΠΈΡ‡ΠΈΠ½Π°, Π° Π²Π΅ΠΊΡ‚ΠΎΡ€Π½ΠΎΠ΅ ΠΏΡ€ΠΎΠΈΠ·Π²Π΅Π΄Π΅Π½ΠΈΠ΅ — это вСкторная Π²Π΅Π»ΠΈΡ‡ΠΈΠ½Π°.

    БкалярноС ΠΏΡ€ΠΎΠΈΠ·Π²Π΅Π΄Π΅Π½ΠΈΠ΅ Π΄Π²ΡƒΡ… Π²Π΅ΠΊΡ‚ΠΎΡ€ΠΎΠ² (скалярноС ΠΏΡ€ΠΎΠΈΠ·Π²Π΅Π΄Π΅Π½ΠΈΠ΅)

    БкалярноС ΡƒΠΌΠ½ΠΎΠΆΠ΅Π½ΠΈΠ΅ Π΄Π²ΡƒΡ… Π²Π΅ΠΊΡ‚ΠΎΡ€ΠΎΠ² Π΄Π°Π΅Ρ‚ скалярноС ΠΏΡ€ΠΎΠΈΠ·Π²Π΅Π΄Π΅Π½ΠΈΠ΅.

    БкалярноС ΠΏΡ€ΠΎΠΈΠ·Π²Π΅Π΄Π΅Π½ΠΈΠ΅ (скалярноС ΠΏΡ€ΠΎΠΈΠ·Π²Π΅Π΄Π΅Π½ΠΈΠ΅)

    БкалярноС ΠΏΡ€ΠΎΠΈΠ·Π²Π΅Π΄Π΅Π½ΠΈΠ΅ [латСкс] \ overset {\ to} {A} Β· \ overset {\ to} {B} [/ latex] Π΄Π²ΡƒΡ… Π²Π΅ΠΊΡ‚ΠΎΡ€ΠΎΠ² [латСкс] \ overset {\ to} {A} [/ latex ] ΠΈ [латСкс] \ overset {\ to} {B} [/ latex] — это число, опрСдСляСмоС ΡƒΡ€Π°Π²Π½Π΅Π½ΠΈΠ΅ΠΌ

    [латСкс] \ overset {\ to} {A} Β· \ overset {\ to} {B} = AB \, \ text {cos} \, \ phi, [/ latex]

    Π³Π΄Π΅ [latex] \ phi [/ latex] — ΡƒΠ³ΠΎΠ» ΠΌΠ΅ΠΆΠ΄Ρƒ Π²Π΅ΠΊΡ‚ΠΎΡ€Π°ΠΌΠΈ (ΠΏΠΎΠΊΠ°Π·Π°Π½ Π½Π° (Рисунок)).БкалярноС ΠΏΡ€ΠΎΠΈΠ·Π²Π΅Π΄Π΅Π½ΠΈΠ΅ Ρ‚Π°ΠΊΠΆΠ΅ называСтся скалярным ΠΏΡ€ΠΎΠΈΠ·Π²Π΅Π΄Π΅Π½ΠΈΠ΅ΠΌ ΠΈΠ·-Π·Π° Ρ‚ΠΎΡ‡Π΅Ρ‡Π½ΠΎΠΉ записи, которая Π΅Π³ΠΎ ΠΎΠ±ΠΎΠ·Π½Π°Ρ‡Π°Π΅Ρ‚.

    Π’ ΠΎΠΏΡ€Π΅Π΄Π΅Π»Π΅Π½ΠΈΠΈ скалярного произвСдСния Π½Π°ΠΏΡ€Π°Π²Π»Π΅Π½ΠΈΠ΅ ΡƒΠ³Π»Π° [латСкс] \ phi [/ latex] Π½Π΅ ΠΈΠΌΠ΅Π΅Ρ‚ значСния, ΠΈ [латСкс] \ phi [/ latex] ΠΌΠΎΠΆΠ΅Ρ‚ Π±Ρ‹Ρ‚ΡŒ ΠΈΠ·ΠΌΠ΅Ρ€Π΅Π½ ΠΎΡ‚ ΠΎΠ΄Π½ΠΎΠ³ΠΎ ΠΈΠ· Π΄Π²ΡƒΡ… Π²Π΅ΠΊΡ‚ΠΎΡ€ΠΎΠ² ΠΊ Π΄Ρ€ΡƒΠ³ΠΎΠΌΡƒ, ΠΏΠΎΡ‚ΠΎΠΌΡƒ Ρ‡Ρ‚ΠΎ [ latex] \ text {cos} \, \ phi = \ text {cos} \, (\ text {-} \ phi) = \ text {cos} \, (2 \ pi — \ phi) [/ latex]. {2}.[/ латСкс]

    Рисунок 2.27 БкалярноС ΠΏΡ€ΠΎΠΈΠ·Π²Π΅Π΄Π΅Π½ΠΈΠ΅ Π΄Π²ΡƒΡ… Π²Π΅ΠΊΡ‚ΠΎΡ€ΠΎΠ². (Π°) Π£Π³ΠΎΠ» ΠΌΠ΅ΠΆΠ΄Ρƒ двумя Π²Π΅ΠΊΡ‚ΠΎΡ€Π°ΠΌΠΈ. (b) ΠžΡ€Ρ‚ΠΎΠ³ΠΎΠ½Π°Π»ΡŒΠ½Π°Ρ проСкция [латСкс] {A} _ {\ perp} [/ latex] Π²Π΅ΠΊΡ‚ΠΎΡ€Π° [латСкс] \ overset {\ to} {A} [/ latex] Π½Π° Π½Π°ΠΏΡ€Π°Π²Π»Π΅Π½ΠΈΠ΅ Π²Π΅ΠΊΡ‚ΠΎΡ€Π° [латСкс] \ overset { \ to} {B} [/ латСкс]. (c) ΠžΡ€Ρ‚ΠΎΠ³ΠΎΠ½Π°Π»ΡŒΠ½Π°Ρ проСкция [латСкс] {B} _ {\ perp} [/ latex] Π²Π΅ΠΊΡ‚ΠΎΡ€Π° [латСкс] \ overset {\ to} {B} [/ latex] Π½Π° Π½Π°ΠΏΡ€Π°Π²Π»Π΅Π½ΠΈΠ΅ Π²Π΅ΠΊΡ‚ΠΎΡ€Π° [латСкс] \ overset { \ to} {A} [/ латСкс].

    ΠŸΡ€ΠΈΠΌΠ΅Ρ€

    БкалярноС ΠΏΡ€ΠΎΠΈΠ·Π²Π΅Π΄Π΅Π½ΠΈΠ΅

    Для Π²Π΅ΠΊΡ‚ΠΎΡ€ΠΎΠ², ΠΏΠΎΠΊΠ°Π·Π°Π½Π½Ρ‹Ρ… Π½Π° (Рисунок), Π½Π°ΠΉΠ΄ΠΈΡ‚Π΅ скалярноС ΠΏΡ€ΠΎΠΈΠ·Π²Π΅Π΄Π΅Π½ΠΈΠ΅ [latex] \ overset {\ to} {A} Β· \ overset {\ to} {F} [/ latex].

    БтратСгия

    Из (Рисунок), Π²Π΅Π»ΠΈΡ‡ΠΈΠ½Ρ‹ Π²Π΅ΠΊΡ‚ΠΎΡ€ΠΎΠ² [latex] \ overset {\ to} {A} [/ latex] ΠΈ [latex] \ overset {\ to} {F} [/ latex] Ρ€Π°Π²Π½Ρ‹ A, = 10,0 ΠΈ F = 20,0. Π£Π³ΠΎΠ» [latex] \ theta [/ latex] ΠΌΠ΅ΠΆΠ΄Ρƒ Π½ΠΈΠΌΠΈ являСтся Ρ€Π°Π·Π½ΠΈΡ†Π΅ΠΉ: [latex] \ theta = \ phi — \ alpha = 110 \ text {Β°} -35 \ text {Β°} = 75 \ text {Β°} [/латСкс]. ΠŸΠΎΠ΄ΡΡ‚Π°Π½ΠΎΠ²ΠΊΠ° этих Π·Π½Π°Ρ‡Π΅Π½ΠΈΠΉ Π² (рисунок) Π΄Π°Π΅Ρ‚ скалярноС ΠΏΡ€ΠΎΠΈΠ·Π²Π΅Π΄Π΅Π½ΠΈΠ΅.

    РСшСниС
    ПокаТи ΠΎΡ‚Π²Π΅Ρ‚ ΠŸΡ€ΠΎΡΡ‚ΠΎΠΉ расчСт Π΄Π°Π΅Ρ‚ Π½Π°ΠΌ

    [латСкс] \ overset {\ to} {A} Β· \ overset {\ to} {F} = AF \, \ text {cos} \, \ theta = (10.0) (20,0) \, \ text {cos} \, 75 \ text {Β°} = 51,76. [/ латСкс]

    ΠŸΡ€ΠΎΠ²Π΅Ρ€ΡŒΡ‚Π΅ своС ΠΏΠΎΠ½ΠΈΠΌΠ°Π½ΠΈΠ΅

    Для Π²Π΅ΠΊΡ‚ΠΎΡ€ΠΎΠ², ΡƒΠΊΠ°Π·Π°Π½Π½Ρ‹Ρ… Π½Π° (Рисунок), Π½Π°ΠΉΠ΄ΠΈΡ‚Π΅ скалярныС произвСдСния [latex] \ overset {\ to} {A} Β· \ overset {\ to} {B} [/ latex] ΠΈ [latex] \ overset {\ to} {F} Β· \ overset {\ to} {C} [/ latex].

    ΠŸΠΎΠΊΠ°Π·Π°Ρ‚ΡŒ Ρ€Π΅ΡˆΠ΅Π½ΠΈΠ΅

    [латСкс] \ overset {\ to} {A} Β· \ overset {\ to} {B} = — 57,3 [/ латСкс], [латСкс] \ overset {\ to} {F} Β· \ overset {\ to} {C} = 27,8 [/ латСкс]

    Π’ Π΄Π΅ΠΊΠ°Ρ€Ρ‚ΠΎΠ²ΠΎΠΉ систСмС ΠΊΠΎΠΎΡ€Π΄ΠΈΠ½Π°Ρ‚ скалярныС произвСдСния Π΅Π΄ΠΈΠ½ΠΈΡ‡Π½ΠΎΠ³ΠΎ Π²Π΅ΠΊΡ‚ΠΎΡ€Π° оси Π½Π° Π΄Ρ€ΡƒΠ³ΠΈΠ΅ Π΅Π΄ΠΈΠ½ΠΈΡ‡Π½Ρ‹Π΅ Π²Π΅ΠΊΡ‚ΠΎΡ€Ρ‹ осСй всСгда ΠΎΠ±Ρ€Π°Ρ‰Π°ΡŽΡ‚ΡΡ Π² Π½ΡƒΠ»ΡŒ, ΠΏΠΎΡ‚ΠΎΠΌΡƒ Ρ‡Ρ‚ΠΎ эти Π΅Π΄ΠΈΠ½ΠΈΡ‡Π½Ρ‹Π΅ Π²Π΅ΠΊΡ‚ΠΎΡ€Ρ‹ ΠΎΡ€Ρ‚ΠΎΠ³ΠΎΠ½Π°Π»ΡŒΠ½Ρ‹:

    [латСкс] \ begin {array} {c} \ hat {i} Β· \ hat {j} = | \ hat {i} || \ hat {j} | \, \ text {cos} \, 90 \ text {Β°} = (1) (1) (0) = 0, \ hfill \\ \ hat {i} Β· \ hat {k} = | \ hat {i} || \ hat {k} | \, \ text {cos} \, 90 \ text {Β°} = (1) (1) (0) = 0, \ hfill \\ \ hat {k} Β· \ hat {j} = | \ hat {k} || \ hat {j} | \, \ text {cos} \, 90 \ text {Β°} = (1) (1) (0) = 0.{2} = 1. [/ латСкс]

    БкалярноС ΠΏΡ€ΠΎΠΈΠ·Π²Π΅Π΄Π΅Π½ΠΈΠ΅ [латСкс] \ overset {\ to} {A} Β· \ overset {\ to} {B} [/ latex] Ρ‚Π°ΠΊΠΆΠ΅ ΠΌΠΎΠΆΠ½ΠΎ ΠΈΠ½Ρ‚Π΅Ρ€ΠΏΡ€Π΅Ρ‚ΠΈΡ€ΠΎΠ²Π°Ρ‚ΡŒ ΠΊΠ°ΠΊ ΠΏΡ€ΠΎΠΈΠ·Π²Π΅Π΄Π΅Π½ΠΈΠ΅ B с ΠΎΡ€Ρ‚ΠΎΠ³ΠΎΠ½Π°Π»ΡŒΠ½ΠΎΠΉ ΠΏΡ€ΠΎΠ΅ΠΊΡ†ΠΈΠ΅ΠΉ [латСкс] { A} _ {\ perp} [/ latex] Π²Π΅ΠΊΡ‚ΠΎΡ€Π° [latex] \ overset {\ to} {A} [/ latex] Π½Π° Π½Π°ΠΏΡ€Π°Π²Π»Π΅Π½ΠΈΠ΅ Π²Π΅ΠΊΡ‚ΠΎΡ€Π° [latex] \ overset {\ to} {B} [/ latex] ((Рисунок) (b)) ΠΈΠ»ΠΈ ΠΏΡ€ΠΎΠΈΠ·Π²Π΅Π΄Π΅Π½ΠΈΠ΅ A с ΠΎΡ€Ρ‚ΠΎΠ³ΠΎΠ½Π°Π»ΡŒΠ½ΠΎΠΉ ΠΏΡ€ΠΎΠ΅ΠΊΡ†ΠΈΠ΅ΠΉ [латСкс] {B} _ {\ perp} [/ latex] Π²Π΅ΠΊΡ‚ΠΎΡ€Π° [латСкс] \ overset {\ to} {B} [/ латСкс] Π² Π½Π°ΠΏΡ€Π°Π²Π»Π΅Π½ΠΈΠΈ Π²Π΅ΠΊΡ‚ΠΎΡ€Π° [латСкс] \ overset {\ to} {A} [/ latex] ((Рисунок) (c)):

    [латСкс] \ begin {array} {ll} \ hfill \ overset {\ to} {A} Β· \ overset {\ to} {B} & = AB \, \ text {cos} \, \ phi \ hfill \ \ & = B (A \, \ text {cos} \, \ phi) = B {A} _ {\ perp} \ hfill \\ & = A (B \, \ text {cos} \, \ phi) = A {B} _ {\ perp}.\ hfill \ end {array} [/ latex]

    НапримСр, Π² ΠΏΡ€ΡΠΌΠΎΡƒΠ³ΠΎΠ»ΡŒΠ½ΠΎΠΉ систСмС ΠΊΠΎΠΎΡ€Π΄ΠΈΠ½Π°Ρ‚ Π½Π° плоскости скалярная ΠΊΠΎΠΌΠΏΠΎΠ½Π΅Π½Ρ‚Π° Π²Π΅ΠΊΡ‚ΠΎΡ€Π° Ρ€Π°Π·ΠΌΠ΅Ρ€ΠΎΠΌ x являСтся Π΅Π³ΠΎ скалярным ΠΏΡ€ΠΎΠΈΠ·Π²Π΅Π΄Π΅Π½ΠΈΠ΅ΠΌ Π½Π° Π΅Π΄ΠΈΠ½ΠΈΡ‡Π½Ρ‹ΠΉ Π²Π΅ΠΊΡ‚ΠΎΡ€ [latex] \ hat {i} [/ latex] ΠΈ скаляр y -ΠΊΠΎΠΌΠΏΠΎΠ½Π΅Π½Ρ‚ Π²Π΅ΠΊΡ‚ΠΎΡ€Π° — это Π΅Π³ΠΎ скалярноС ΠΏΡ€ΠΎΠΈΠ·Π²Π΅Π΄Π΅Π½ΠΈΠ΅ с Π΅Π΄ΠΈΠ½ΠΈΡ‡Π½Ρ‹ΠΌ Π²Π΅ΠΊΡ‚ΠΎΡ€ΠΎΠΌ [latex] \ hat {j} [/ latex]:

    [латСкс] \ {\ begin {array} {l} \ overset {\ to} {A} Β· \ hat {i} = | \ overset {\ to} {A} || \ hat {i} | \, \ text {cos} \, {\ theta} _ {A} = A \, \ text {cos} \, {\ theta} _ {A} = {A} _ {x} \\ \ overset {\ to} {A} Β· \ hat {j} = | \ overset {\ to} {A} || \ hat {j} | \, \ text {cos} \, (90 \ text {Β°} — {\ theta} _ {A}) = A \, \ text {sin} \, {\ theta} _ {A} = {A} _ {y} \ end {array}.[/ латСкс]

    БкалярноС ΡƒΠΌΠ½ΠΎΠΆΠ΅Π½ΠΈΠ΅ Π²Π΅ΠΊΡ‚ΠΎΡ€ΠΎΠ² ΠΊΠΎΠΌΠΌΡƒΡ‚Π°Ρ‚ΠΈΠ²Π½ΠΎ,

    [латСкс] \ overset {\ to} {A} Β· \ overset {\ to} {B} = \ overset {\ to} {B} Β· \ overset {\ to} {A}, [/ latex]

    ΠΈ подчиняСтся Π·Π°ΠΊΠΎΠ½Ρƒ ΠΎ распрСдСлСнии Π΄ΠΎΡ…ΠΎΠ΄ΠΎΠ²:

    [латСкс] \ overset {\ to} {A} Β· (\ overset {\ to} {B} + \ overset {\ to} {C}) = \ overset {\ to} {A} Β· \ overset {\ to} {B} + \ overset {\ to} {A} Β· \ overset {\ to} {C}. [/ латСкс]

    ΠœΡ‹ ΠΌΠΎΠΆΠ΅ΠΌ ΠΈΡΠΏΠΎΠ»ΡŒΠ·ΠΎΠ²Π°Ρ‚ΡŒ Π·Π°ΠΊΠΎΠ½Ρ‹ коммутативности ΠΈ распрСдСлСния для Π²Ρ‹Π²ΠΎΠ΄Π° Ρ€Π°Π·Π»ΠΈΡ‡Π½Ρ‹Ρ… ΡΠΎΠΎΡ‚Π½ΠΎΡˆΠ΅Π½ΠΈΠΉ для Π²Π΅ΠΊΡ‚ΠΎΡ€ΠΎΠ², Ρ‚Π°ΠΊΠΈΡ… ΠΊΠ°ΠΊ Π²Ρ‹Ρ€Π°ΠΆΠ΅Π½ΠΈΠ΅ скалярного произвСдСния Π΄Π²ΡƒΡ… Π²Π΅ΠΊΡ‚ΠΎΡ€ΠΎΠ² Ρ‡Π΅Ρ€Π΅Π· ΠΈΡ… скалярныС ΠΊΠΎΠΌΠΏΠΎΠ½Π΅Π½Ρ‚Ρ‹.

    ΠŸΡ€ΠΎΠ²Π΅Ρ€ΡŒΡ‚Π΅ своС ΠΏΠΎΠ½ΠΈΠΌΠ°Π½ΠΈΠ΅

    Для Π²Π΅ΠΊΡ‚ΠΎΡ€Π° [латСкс] \ overset {\ to} {A} = {A} _ {x} \ hat {i} + {A} _ {y} \ hat {j} + {A} _ {z} \ hat {k} [/ latex] Π² ΠΏΡ€ΡΠΌΠΎΡƒΠ³ΠΎΠ»ΡŒΠ½ΠΎΠΉ систСмС ΠΊΠΎΠΎΡ€Π΄ΠΈΠ½Π°Ρ‚, ΠΈΡΠΏΠΎΠ»ΡŒΠ·ΡƒΠΉΡ‚Π΅ (Рисунок) — (Рисунок), Ρ‡Ρ‚ΠΎΠ±Ρ‹ ΠΏΠΎΠΊΠ°Π·Π°Ρ‚ΡŒ, Ρ‡Ρ‚ΠΎ [latex] \ overset {\ to} {A} Β· \ hat {i} = {A} _ {x } [/ латСкс] [латСкс] \ overset {\ to} {A} Β· \ hat {j} = {A} _ {y} [/ latex] ΠΈ [латСкс] \ overset {\ to} {A} Β· \ шляпа {k} = {A} _ {z} [/ latex].

    Когда Π²Π΅ΠΊΡ‚ΠΎΡ€Ρ‹ Π½Π° (Рисунок) Π΄Π°Π½Ρ‹ Π² Ρ„ΠΎΡ€ΠΌΠ΅ ΠΈΡ… Π²Π΅ΠΊΡ‚ΠΎΡ€Π½Ρ‹Ρ… ΠΊΠΎΠΌΠΏΠΎΠ½Π΅Π½Ρ‚ΠΎΠ²,

    [латСкс] \ overset {\ to} {A} = {A} _ {x} \ hat {i} + {A} _ {y} \ hat {j} + {A} _ {z} \ hat { k} \, \ text {ΠΈ} \, \ overset {\ to} {B} = {B} _ {x} \ hat {i} + {B} _ {y} \ hat {j} + {B} _ {z} \ hat {k}, [/ latex]

    , ΠΌΡ‹ ΠΌΠΎΠΆΠ΅ΠΌ Π²Ρ‹Ρ‡ΠΈΡΠ»ΠΈΡ‚ΡŒ ΠΈΡ… скалярноС ΠΏΡ€ΠΎΠΈΠ·Π²Π΅Π΄Π΅Π½ΠΈΠ΅ ΡΠ»Π΅Π΄ΡƒΡŽΡ‰ΠΈΠΌ ΠΎΠ±Ρ€Π°Π·ΠΎΠΌ:

    [латСкс] \ begin {array} {lll} \ hfill \ overset {\ to} {A} Β· \ overset {\ to} {B} & = \ hfill & ({A} _ {x} \ hat {i } + {A} _ {y} \ hat {j} + {A} _ {z} \ hat {k}) Β· ({B} _ {x} \ hat {i} + {B} _ {y} \ hat {j} + {B} _ {z} \ hat {k}) \ hfill \\ & = \ hfill & \ enspace {A} _ {x} {B} _ {x} \ hat {i} Β· \ hat {i} + {A} _ {x} {B} _ {y} \ hat {i} Β· \ hat {j} + {A} _ {x} {B} _ {z} \ hat {i } Β· \ Hat {k} \ hfill \\ & & + {A} _ {y} {B} _ {x} \ hat {j} Β· \ hat {i} + {A} _ {y} {B} _ {y} \ hat {j} Β· \ hat {j} + {A} _ {y} {B} _ {z} \ hat {j} Β· \ hat {k} \ hfill \\ & & + {A } _ {z} {B} _ {x} \, \ hat {k} Β· \ hat {i} + {A} _ {z} {B} _ {y} \ hat {k} Β· \ hat {j } + {A} _ {z} {B} _ {z} \, \ hat {k} Β· \ hat {k}.\ hfill \ end {array} [/ latex]

    ΠŸΠΎΡΠΊΠΎΠ»ΡŒΠΊΡƒ скалярныС произвСдСния Π΄Π²ΡƒΡ… Ρ€Π°Π·Π»ΠΈΡ‡Π½Ρ‹Ρ… Π΅Π΄ΠΈΠ½ΠΈΡ‡Π½Ρ‹Ρ… Π²Π΅ΠΊΡ‚ΠΎΡ€ΠΎΠ² осСй Π΄Π°ΡŽΡ‚ ноль, Π° скалярныС произвСдСния Π΅Π΄ΠΈΠ½ΠΈΡ‡Π½Ρ‹Ρ… Π²Π΅ΠΊΡ‚ΠΎΡ€ΠΎΠ² сами Π½Π° сСбя Π΄Π°ΡŽΡ‚ Π΅Π΄ΠΈΠ½ΠΈΡ†Ρƒ (см. (Рисунок) ΠΈ (рисунок)), Π² этом Π²Ρ‹Ρ€Π°ΠΆΠ΅Π½ΠΈΠΈ Π΅ΡΡ‚ΡŒ Ρ‚ΠΎΠ»ΡŒΠΊΠΎ Ρ‚Ρ€ΠΈ Π½Π΅Π½ΡƒΠ»Π΅Π²Ρ‹Ρ… Ρ‡Π»Π΅Π½Π°. Π’Π°ΠΊΠΈΠΌ ΠΎΠ±Ρ€Π°Π·ΠΎΠΌ, скалярноС ΠΏΡ€ΠΎΠΈΠ·Π²Π΅Π΄Π΅Π½ΠΈΠ΅ упрощаСтся Π΄ΠΎ

    .

    [латСкс] \ overset {\ to} {A} Β· \ overset {\ to} {B} = {A} _ {x} {B} _ {x} + {A} _ {y} {B} _ {y} + {A} _ {z} {B} _ {z}. [/ латСкс]

    ΠœΡ‹ ΠΌΠΎΠΆΠ΅ΠΌ ΠΈΡΠΏΠΎΠ»ΡŒΠ·ΠΎΠ²Π°Ρ‚ΡŒ (рисунок) для скалярного произвСдСния Π² Ρ‚Π΅Ρ€ΠΌΠΈΠ½Π°Ρ… скалярных ΠΊΠΎΠΌΠΏΠΎΠ½Π΅Π½Ρ‚ΠΎΠ² Π²Π΅ΠΊΡ‚ΠΎΡ€ΠΎΠ², Ρ‡Ρ‚ΠΎΠ±Ρ‹ Π½Π°ΠΉΡ‚ΠΈ ΡƒΠ³ΠΎΠ» ΠΌΠ΅ΠΆΠ΄Ρƒ двумя Π²Π΅ΠΊΡ‚ΠΎΡ€Π°ΠΌΠΈ .Когда ΠΌΡ‹ Ρ€Π°Π·Π΄Π΅Π»ΠΈΠΌ (рисунок) Π½Π° AB , ΠΌΡ‹ ΠΏΠΎΠ»ΡƒΡ‡ΠΈΠΌ ΡƒΡ€Π°Π²Π½Π΅Π½ΠΈΠ΅ для [latex] \ text {cos} \, \ phi [/ latex], Π² ΠΊΠΎΡ‚ΠΎΡ€ΠΎΠ΅ подставим (рисунок):

    [латСкс] \ text {cos} \, \ phi = \ frac {\ overset {\ to} {A} Β· \ overset {\ to} {B}} {AB} = \ frac {{A} _ {x } {B} _ {x} + {A} _ {y} {B} _ {y} + {A} _ {z} {B} _ {z}} {AB}. [/ латСкс]

    Π£Π³ΠΎΠ» [латСкс] \ phi [/ latex] ΠΌΠ΅ΠΆΠ΄Ρƒ Π²Π΅ΠΊΡ‚ΠΎΡ€Π°ΠΌΠΈ [latex] \ overset {\ to} {A} [/ latex] ΠΈ [latex] \ overset {\ to} {B} [/ latex] получаСтся ΠΏΡƒΡ‚Π΅ΠΌ взятия ΠΎΠ±Ρ€Π°Ρ‚Π½Ρ‹ΠΉ косинус выраТСния Π½Π° (рисунок).

    ΠŸΡ€ΠΈΠΌΠ΅Ρ€

    Π£Π³ΠΎΠ» ΠΌΠ΅ΠΆΠ΄Ρƒ двумя силами

    Π’Ρ€ΠΈ собаки тянут ΠΏΠ°Π»ΠΊΡƒ Π² Ρ€Π°Π·Π½Ρ‹Π΅ стороны, ΠΊΠ°ΠΊ ΠΏΠΎΠΊΠ°Π·Π°Π½ΠΎ Π½Π° (Рисунок). ΠŸΠ΅Ρ€Π²Π°Ρ собака тянСт с силой [латСкс] {\ overset {\ to} {F}} _ {1} = (10.0 \ hat {i} -20.4 \ hat {j} +2.0 \ hat {k}) \ text { N} [/ latex], вторая собака тянСт с силой [latex] {\ overset {\ to} {F}} _ {2} = (- 15.0 \ hat {i} -6.2 \ hat {k}) \ text {N} [/ latex], Π° Ρ‚Ρ€Π΅Ρ‚ΡŒΡ собака тянСт с силой [латСкс] {\ overset {\ to} {F}} _ {3} = (5.0 \ hat {i} +12.5 \ hat {j}) \ тСкст {N} [/ latex]. Каков ΡƒΠ³ΠΎΠ» ΠΌΠ΅ΠΆΠ΄Ρƒ силами [латСкс] {\ overset {\ to} {F}} _ {1} [/ latex] ΠΈ [latex] {\ overset {\ to} {F}} _ {2} [/ latex ]?

    Рисунок 2.28 Π’Ρ€ΠΈ собаки ΠΈΠ³Ρ€Π°ΡŽΡ‚ с ΠΏΠ°Π»ΠΊΠΎΠΉ.

    БтратСгия

    ΠšΠΎΠΌΠΏΠΎΠ½Π΅Π½Ρ‚Ρ‹ Π²Π΅ΠΊΡ‚ΠΎΡ€Π° силы [латСкс] {\ overset {\ to} {F}} _ {1} [/ latex] Ρ€Π°Π²Π½Ρ‹ [латСкс] {F} _ {1x} = 10.0 \, \ text {N} [/ латСкс], [латСкс] {F} _ {1y} = — 20,4 \, \ text {N} [/ latex] ΠΈ [латСкс] {F} _ {1z} = 2,0 \, \ text {N} [/ латСкс], Π° Π²Π΅ΠΊΡ‚ΠΎΡ€ силы [латСкс] {\ overset {\ to} {F}} _ {2} [/ latex] — [латСкс] {F} _ {2x} = — 15.0 \, \ text {N } [/ latex], [latex] {F} _ {2y} = 0.0 \, \ text {N} [/ latex] ΠΈ [latex] {F} _ {2z} = — 6.2 \, \ text {N } [/ латСкс].{-1} (- 0,439) = 116,0 \ text {Β°}. [/ латСкс]

    Π—Π½Π°Ρ‡Π΅Π½ΠΈΠ΅

    ΠžΠ±Ρ€Π°Ρ‚ΠΈΡ‚Π΅ Π²Π½ΠΈΠΌΠ°Π½ΠΈΠ΅, Ρ‡Ρ‚ΠΎ ΠΊΠΎΠ³Π΄Π° Π²Π΅ΠΊΡ‚ΠΎΡ€Ρ‹ Π·Π°Π΄Π°ΡŽΡ‚ΡΡ Π² Ρ‚Π΅Ρ€ΠΌΠΈΠ½Π°Ρ… Π΅Π΄ΠΈΠ½ΠΈΡ‡Π½Ρ‹Ρ… Π²Π΅ΠΊΡ‚ΠΎΡ€ΠΎΠ² осСй, ΠΌΡ‹ ΠΌΠΎΠΆΠ΅ΠΌ Π½Π°ΠΉΡ‚ΠΈ ΡƒΠ³ΠΎΠ» ΠΌΠ΅ΠΆΠ΄Ρƒ Π½ΠΈΠΌΠΈ, Π½Π΅ зная спСцифики гСографичСских Π½Π°ΠΏΡ€Π°Π²Π»Π΅Π½ΠΈΠΉ, ΠΊΠΎΡ‚ΠΎΡ€Ρ‹Π΅ ΠΏΡ€Π΅Π΄ΡΡ‚Π°Π²Π»ΡΡŽΡ‚ Π΅Π΄ΠΈΠ½ΠΈΡ‡Π½Ρ‹Π΅ Π²Π΅ΠΊΡ‚ΠΎΡ€Ρ‹. Π—Π΄Π΅ΡΡŒ, Π½Π°ΠΏΡ€ΠΈΠΌΠ΅Ρ€, Π½Π°ΠΏΡ€Π°Π²Π»Π΅Π½ΠΈΠ΅ + x ΠΌΠΎΠΆΠ΅Ρ‚ Π±Ρ‹Ρ‚ΡŒ Π½Π° восток, Π° Π½Π°ΠΏΡ€Π°Π²Π»Π΅Π½ΠΈΠ΅ + y — Π½Π° сСвСр. Но ΡƒΠ³ΠΎΠ» ΠΌΠ΅ΠΆΠ΄Ρƒ силами Π² Π·Π°Π΄Π°Ρ‡Π΅ Π±ΡƒΠ΄Π΅Ρ‚ Ρ‚Π°ΠΊΠΈΠΌ ΠΆΠ΅, Ссли Π½Π°ΠΏΡ€Π°Π²Π»Π΅Π½ΠΈΠ΅ + x Π½Π°ΠΏΡ€Π°Π²Π»Π΅Π½ΠΎ Π½Π° Π·Π°ΠΏΠ°Π΄, Π° Π½Π°ΠΏΡ€Π°Π²Π»Π΅Π½ΠΈΠ΅ + y — Π½Π° юг.

    ΠŸΡ€ΠΎΠ²Π΅Ρ€ΡŒΡ‚Π΅ своС ΠΏΠΎΠ½ΠΈΠΌΠ°Π½ΠΈΠ΅

    НайдитС ΡƒΠ³ΠΎΠ» ΠΌΠ΅ΠΆΠ΄Ρƒ силами [латСкс] {\ overset {\ to} {F}} _ {1} [/ latex] ΠΈ [латСкс] {\ overset {\ to} {F}} _ {3} [/ latex ] Π² (Рисунок).

    ΠŸΠΎΠΊΠ°Π·Π°Ρ‚ΡŒ Ρ€Π΅ΡˆΠ΅Π½ΠΈΠ΅

    [латСкс] 131.9 \ text {Β°} [/ латСкс]

    ΠŸΡ€ΠΈΠΌΠ΅Ρ€

    Π Π°Π±ΠΎΡ‚Π° силы

    Когда сила [латСкс] \ overset {\ to} {F} [/ latex] тянСт ΠΎΠ±ΡŠΠ΅ΠΊΡ‚ ΠΈ Π²Ρ‹Π·Ρ‹Π²Π°Π΅Ρ‚ Π΅Π³ΠΎ смСщСниС [латСкс] \ overset {\ to} {D} [/ latex], ΠΌΡ‹ Π³ΠΎΠ²ΠΎΡ€ΠΈΠΌ, Ρ‡Ρ‚ΠΎ сила выполняСт Ρ€Π°Π±ΠΎΡ‚Ρƒ. ΠšΠΎΠ»ΠΈΡ‡Π΅ΡΡ‚Π²ΠΎ Ρ€Π°Π±ΠΎΡ‚Ρ‹, ΡΠΎΠ²Π΅Ρ€ΡˆΠ°Π΅ΠΌΠΎΠΉ силой, — это скалярноС ΠΏΡ€ΠΎΠΈΠ·Π²Π΅Π΄Π΅Π½ΠΈΠ΅ [латСкс] \ overset {\ to} {F} Β· \ overset {\ to} {D} [/ latex].Если ΠΏΠ°Π»ΠΊΠ° Π½Π° (рис.) На ΠΌΠ³Π½ΠΎΠ²Π΅Π½ΠΈΠ΅ сдвинСтся ΠΈ смСщаСтся Π²Π΅ΠΊΡ‚ΠΎΡ€ΠΎΠΌ [латСкс] \ overset {\ to} {D} = (- 7.9 \ hat {j} -4.2 \ hat {k}) \, \ text {cm} [/ latex], сколько Ρ€Π°Π±ΠΎΡ‚Ρ‹ ΠΏΡ€ΠΎΠ΄Π΅Π»Ρ‹Π²Π°Π΅Ρ‚ Ρ‚Ρ€Π΅Ρ‚ΡŒΡ собака Π½Π° (Рисунок)?

    БтратСгия

    ΠœΡ‹ вычисляСм скалярноС ΠΏΡ€ΠΎΠΈΠ·Π²Π΅Π΄Π΅Π½ΠΈΠ΅ Π²Π΅ΠΊΡ‚ΠΎΡ€Π° смСщСния [latex] \ overset {\ to} {D} [/ latex] Π½Π° Π²Π΅ΠΊΡ‚ΠΎΡ€ силы [latex] {\ overset {\ to} {F}} _ {3} = (5.0 \ hat {i} +12.5 \ hat {j}) \ text {N} [/ latex], ΠΊΠΎΡ‚ΠΎΡ€Ρ‹ΠΉ являСтся натяТСниСм Ρ‚Ρ€Π΅Ρ‚ΡŒΠ΅ΠΉ собаки. Π”Π°Π²Π°ΠΉΡ‚Π΅ ΠΈΡΠΏΠΎΠ»ΡŒΠ·ΠΎΠ²Π°Ρ‚ΡŒ [latex] {W} _ {3} [/ latex] для обозначСния Ρ€Π°Π±ΠΎΡ‚Ρ‹, выполняСмой силой [latex] {\ overset {\ to} {F}} _ {3} [/ latex] ΠΏΡ€ΠΈ ΠΏΠ΅Ρ€Π΅ΠΌΠ΅Ρ‰Π΅Π½ΠΈΠΈ [латСкс] \ overset {\ to} {D} [/ латСкс].

    РСшСниС
    ПокаТи ΠΎΡ‚Π²Π΅Ρ‚ РасчСт Ρ€Π°Π±ΠΎΡ‚Ρ‹ — это простоС ΠΏΡ€ΠΈΠΌΠ΅Π½Π΅Π½ΠΈΠ΅ скалярного произвСдСния:

    [латСкс] \ begin {array} {cc} \ hfill {W} _ {3} & = {\ overset {\ to} {F}} _ {3} Β· \ overset {\ to} {D} = { F} _ {3x} {D} _ {x} + {F} _ {3y} {D} _ {y} + {F} _ {3z} {D} _ {z} \ hfill \\ & = ( 5.0 \, \ text {N}) (0.0 \, \ text {cm}) + (12.5 \, \ text {N}) (- 7.9 \, \ text {cm}) + (0.0 \, \ text {N }) (- 4.2 \, \ text {cm}) \ hfill \\ & = -98.7 \, \ text {N} Β· \ text {cm}. \ Hfill \ end {array} [/ latex]

    Π—Π½Π°Ρ‡Π΅Π½ΠΈΠ΅

    Π•Π΄ΠΈΠ½ΠΈΡ†Π° Ρ€Π°Π±ΠΎΡ‚Ρ‹ Π² систСмС БИ называСтся Π΄ΠΆΠΎΡƒΠ»ΡŒ [латСкс] (\ text {J}) [/ latex], Π³Π΄Π΅ 1 Π”ΠΆ = 1 [латСкс] \ text {N} Β· \ text {m} [/ latex].{-2} \ text {J} [/ latex], поэтому ΠΎΡ‚Π²Π΅Ρ‚ ΠΌΠΎΠΆΠ½ΠΎ Π²Ρ‹Ρ€Π°Π·ΠΈΡ‚ΡŒ ΠΊΠ°ΠΊ [latex] {W} _ {3} = — 0,9875 \, \ text {J} \ ΠΏΡ€ΠΈΠ±Π»ΠΈΠ·ΠΈΡ‚Π΅Π»ΡŒΠ½ΠΎ -1,0 \, \ text { J} [/ латСкс].

    ΠŸΡ€ΠΎΠ²Π΅Ρ€ΡŒΡ‚Π΅ своС ΠΏΠΎΠ½ΠΈΠΌΠ°Π½ΠΈΠ΅

    Бколько Ρ€Π°Π±ΠΎΡ‚Ρ‹ выполняСт пСрвая собака ΠΈ вторая собака Π½Π° (Рис.) Над смСщСниСм Π½Π° (Рис.)?

    ΠŸΠΎΠΊΠ°Π·Π°Ρ‚ΡŒ Ρ€Π΅ΡˆΠ΅Π½ΠΈΠ΅

    [латСкс] {W} _ {1} = 1,5 \, \ text {J} [/ latex], [латСкс] {W} _ {2} = 0,3 \, \ text {J} [/ latex]

    Π’Π΅ΠΊΡ‚ΠΎΡ€Π½ΠΎΠ΅ ΠΏΡ€ΠΎΠΈΠ·Π²Π΅Π΄Π΅Π½ΠΈΠ΅ Π΄Π²ΡƒΡ… Π²Π΅ΠΊΡ‚ΠΎΡ€ΠΎΠ² (пСрСкрСстноС ΠΏΡ€ΠΎΠΈΠ·Π²Π΅Π΄Π΅Π½ΠΈΠ΅)

    Π’Π΅ΠΊΡ‚ΠΎΡ€Π½ΠΎΠ΅ ΡƒΠΌΠ½ΠΎΠΆΠ΅Π½ΠΈΠ΅ Π΄Π²ΡƒΡ… Π²Π΅ΠΊΡ‚ΠΎΡ€ΠΎΠ² Π΄Π°Π΅Ρ‚ Π²Π΅ΠΊΡ‚ΠΎΡ€Π½ΠΎΠ΅ ΠΏΡ€ΠΎΠΈΠ·Π²Π΅Π΄Π΅Π½ΠΈΠ΅.

    Π’Π΅ΠΊΡ‚ΠΎΡ€Π½ΠΎΠ΅ ΠΏΡ€ΠΎΠΈΠ·Π²Π΅Π΄Π΅Π½ΠΈΠ΅ (пСрСкрСстноС ΠΏΡ€ΠΎΠΈΠ·Π²Π΅Π΄Π΅Π½ΠΈΠ΅)

    Π’Π΅ΠΊΡ‚ΠΎΡ€Π½ΠΎΠ΅ ΠΏΡ€ΠΎΠΈΠ·Π²Π΅Π΄Π΅Π½ΠΈΠ΅ Π΄Π²ΡƒΡ… Π²Π΅ΠΊΡ‚ΠΎΡ€ΠΎΠ² [латСкс] \ overset {\ to} {A} [/ latex] ΠΈ [latex] \ overset {\ to} {B} [/ latex] обозначаСтся [латСкс] \ overset {\ to} {A} \, Γ— \, \ overset {\ to} {B} [/ latex] ΠΈ часто упоминаСтся ΠΊΠ°ΠΊ пСрСкрСстноС ΠΏΡ€ΠΎΠΈΠ·Π²Π΅Π΄Π΅Π½ΠΈΠ΅ . Π’Π΅ΠΊΡ‚ΠΎΡ€Π½ΠΎΠ΅ ΠΏΡ€ΠΎΠΈΠ·Π²Π΅Π΄Π΅Π½ΠΈΠ΅ — это Π²Π΅ΠΊΡ‚ΠΎΡ€, Π½Π°ΠΏΡ€Π°Π²Π»Π΅Π½ΠΈΠ΅ ΠΊΠΎΡ‚ΠΎΡ€ΠΎΠ³ΠΎ пСрпСндикулярно ΠΎΠ±ΠΎΠΈΠΌ Π²Π΅ΠΊΡ‚ΠΎΡ€Π°ΠΌ [latex] \ overset {\ to} {A} [/ latex] ΠΈ [latex] \ overset {\ to} {B} [/ latex]. Π”Ρ€ΡƒΠ³ΠΈΠΌΠΈ словами, Π²Π΅ΠΊΡ‚ΠΎΡ€ [латСкс] \ overset {\ to} {A} \, Γ— \, \ overset {\ to} {B} [/ latex] пСрпСндикулярСн плоскости, содСрТащСй Π²Π΅ΠΊΡ‚ΠΎΡ€Ρ‹ [латСкс] \ overset {\ to} {A} [/ latex] ΠΈ [latex] \ overset {\ to} {B} [/ latex], ΠΊΠ°ΠΊ ΠΏΠΎΠΊΠ°Π·Π°Π½ΠΎ Π½Π° (Рисунок).Π’Π΅Π»ΠΈΡ‡ΠΈΠ½Π° Π²Π΅ΠΊΡ‚ΠΎΡ€Π½ΠΎΠ³ΠΎ произвСдСния опрСдСляСтся ΠΊΠ°ΠΊ

    .

    [латСкс] | \ overset {\ to} {A} \, Γ— \, \ overset {\ to} {B} | = \, AB \, \ text {sin} \, \ phi, [/ latex]

    Π³Π΄Π΅ ΡƒΠ³ΠΎΠ» [латСкс] \ phi [/ latex] ΠΌΠ΅ΠΆΠ΄Ρƒ двумя Π²Π΅ΠΊΡ‚ΠΎΡ€Π°ΠΌΠΈ измСряСтся ΠΎΡ‚ Π²Π΅ΠΊΡ‚ΠΎΡ€Π° [latex] \ overset {\ to} {A} [/ latex] (ΠΏΠ΅Ρ€Π²Ρ‹ΠΉ Π²Π΅ΠΊΡ‚ΠΎΡ€ Π² ΠΏΡ€ΠΎΠ΄ΡƒΠΊΡ‚Π΅) Π΄ΠΎ Π²Π΅ΠΊΡ‚ΠΎΡ€Π° [латСкс] \ overset {\ to} {B} [/ latex] (Π²Ρ‚ΠΎΡ€ΠΎΠΉ Π²Π΅ΠΊΡ‚ΠΎΡ€ Π² ΠΏΡ€ΠΎΠ΄ΡƒΠΊΡ‚Π΅), ΠΊΠ°ΠΊ ΠΏΠΎΠΊΠ°Π·Π°Π½ΠΎ Π½Π° (Рисунок), ΠΈ находится ΠΌΠ΅ΠΆΠ΄Ρƒ [latex] 0 \ text {Β°} [/ latex] ΠΈ [latex] 180 \ тСкст {Β°} [/ латСкс].

    Богласно (Рисунок), Π²Π΅ΠΊΡ‚ΠΎΡ€Π½ΠΎΠ΅ ΠΏΡ€ΠΎΠΈΠ·Π²Π΅Π΄Π΅Π½ΠΈΠ΅ исчСзаСт для ΠΏΠ°Ρ€ Π²Π΅ΠΊΡ‚ΠΎΡ€ΠΎΠ², ΠΊΠΎΡ‚ΠΎΡ€Ρ‹Π΅ Π»ΠΈΠ±ΠΎ ΠΏΠ°Ρ€Π°Π»Π»Π΅Π»ΡŒΠ½Ρ‹ [латСкс] (\ phi = 0 \ text {Β°}) [/ latex], Π»ΠΈΠ±ΠΎ Π°Π½Ρ‚ΠΈΠΏΠ°Ρ€Π°Π»Π»Π΅Π»ΡŒΠ½Ρ‹ [латСкс] (\ phi = 180 \ text { Β°}) [/ latex], ΠΏΠΎΡ‚ΠΎΠΌΡƒ Ρ‡Ρ‚ΠΎ [латСкс] \ text {sin} \, 0 \ text {Β°} = \ text {sin} \, 180 \ text {Β°} = 0 [/ latex].

    Рисунок 2.29 Π’Π΅ΠΊΡ‚ΠΎΡ€Π½ΠΎΠ΅ ΠΏΡ€ΠΎΠΈΠ·Π²Π΅Π΄Π΅Π½ΠΈΠ΅ Π΄Π²ΡƒΡ… Π²Π΅ΠΊΡ‚ΠΎΡ€ΠΎΠ² нарисовано Π² Ρ‚Ρ€Π΅Ρ…ΠΌΠ΅Ρ€Π½ΠΎΠΌ пространствС. (a) Π’Π΅ΠΊΡ‚ΠΎΡ€Π½ΠΎΠ΅ ΠΏΡ€ΠΎΠΈΠ·Π²Π΅Π΄Π΅Π½ΠΈΠ΅ [латСкс] \ overset {\ to} {A} \, Γ— \, \ overset {\ to} {B} [/ latex] — это Π²Π΅ΠΊΡ‚ΠΎΡ€, пСрпСндикулярный плоскости, ΠΊΠΎΡ‚ΠΎΡ€Ρ‹ΠΉ содСрТит Π²Π΅ΠΊΡ‚ΠΎΡ€Ρ‹ [латСкс] \ overset {\ to} {A} [/ latex] ΠΈ [latex] \ overset {\ to} {B} [/ latex]. МалСнькиС ΠΊΠ²Π°Π΄Ρ€Π°Ρ‚Ρ‹, нарисованныС Π² пСрспСктивС, ΠΎΠ±ΠΎΠ·Π½Π°Ρ‡Π°ΡŽΡ‚ прямыС ΡƒΠ³Π»Ρ‹ ΠΌΠ΅ΠΆΠ΄Ρƒ [латСксом] \ overset {\ to} {A} [/ latex] ΠΈ [latex] \ overset {\ to} {C} [/ latex], Π° Ρ‚Π°ΠΊΠΆΠ΅ ΠΌΠ΅ΠΆΠ΄Ρƒ [latex] \ overset {\ to} {B} [/ latex] ΠΈ [latex] \ overset {\ to} {C} [/ latex], Ρ‚Π°ΠΊ Ρ‡Ρ‚ΠΎ Ссли [latex] \ overset {\ to} {A} [/ latex] ΠΈ [latex ] \ overset {\ to} {B} [/ latex] Π»Π΅ΠΆΠ°Ρ‚ΡŒ Π½Π° ΠΏΠΎΠ»Ρƒ, Π²Π΅ΠΊΡ‚ΠΎΡ€ [latex] \ overset {\ to} {C} [/ latex] Π½Π°ΠΏΡ€Π°Π²Π»Π΅Π½ Π²Π΅Ρ€Ρ‚ΠΈΠΊΠ°Π»ΡŒΠ½ΠΎ Π²Π²Π΅Ρ€Ρ… ΠΊ ΠΏΠΎΡ‚ΠΎΠ»ΠΊΡƒ.(b) Π’Π΅ΠΊΡ‚ΠΎΡ€Π½ΠΎΠ΅ ΠΏΡ€ΠΎΠΈΠ·Π²Π΅Π΄Π΅Π½ΠΈΠ΅ [латСкс] \ overset {\ to} {B} \, Γ— \, \ overset {\ to} {A} [/ latex] являСтся Π²Π΅ΠΊΡ‚ΠΎΡ€ΠΎΠΌ, Π°Π½Ρ‚ΠΈΠΏΠ°Ρ€Π°Π»Π»Π΅Π»ΡŒΠ½Ρ‹ΠΌ Π²Π΅ΠΊΡ‚ΠΎΡ€Ρƒ [латСкс] \ overset {\ to } {A} \, Γ— \, \ overset {\ to} {B} [/ latex].

    На Π»ΠΈΠ½ΠΈΠΈ, пСрпСндикулярной плоскости, которая содСрТит Π²Π΅ΠΊΡ‚ΠΎΡ€Ρ‹ [латСкс] \ overset {\ to} {A} [/ latex] ΠΈ [latex] \ overset {\ to} {B} [/ latex], Π΅ΡΡ‚ΡŒ Π΄Π²Π° Π°Π»ΡŒΡ‚Π΅Ρ€Π½Π°Ρ‚ΠΈΠ²Π½Ρ‹Ρ… направлСния — Π»ΠΈΠ±ΠΎ Π²Π²Π΅Ρ€Ρ…, Π»ΠΈΠ±ΠΎ Π²Π½ΠΈΠ·, ΠΊΠ°ΠΊ ΠΏΠΎΠΊΠ°Π·Π°Π½ΠΎ Π½Π° (Рисунок) — ΠΈ Π½Π°ΠΏΡ€Π°Π²Π»Π΅Π½ΠΈΠ΅ Π²Π΅ΠΊΡ‚ΠΎΡ€Π½ΠΎΠ³ΠΎ произвСдСния ΠΌΠΎΠΆΠ΅Ρ‚ Π±Ρ‹Ρ‚ΡŒ ΠΎΠ΄Π½ΠΈΠΌ ΠΈΠ· Π½ΠΈΡ…. Π’ стандартной ΠΏΡ€Π°Π²ΠΎΠΉ ΠΎΡ€ΠΈΠ΅Π½Ρ‚Π°Ρ†ΠΈΠΈ, ΠΊΠΎΠ³Π΄Π° ΡƒΠ³ΠΎΠ» ΠΌΠ΅ΠΆΠ΄Ρƒ Π²Π΅ΠΊΡ‚ΠΎΡ€Π°ΠΌΠΈ измСряСтся ΠΏΡ€ΠΎΡ‚ΠΈΠ² часовой стрСлки ΠΎΡ‚ ΠΏΠ΅Ρ€Π²ΠΎΠ³ΠΎ Π²Π΅ΠΊΡ‚ΠΎΡ€Π°, vector [latex] \ overset {\ to} {A} \, Γ— \, \ overset {\ to} {B} [/ латСкс] ΡƒΠΊΠ°Π·Ρ‹Π²Π°Π΅Ρ‚ Π²Π²Π΅Ρ€Ρ… Π½Π° , ΠΊΠ°ΠΊ ΠΏΠΎΠΊΠ°Π·Π°Π½ΠΎ Π½Π° (Рисунок) (Π°).Если ΠΌΡ‹ ΠΈΠ·ΠΌΠ΅Π½ΠΈΠΌ порядок умноТСния Π½Π° ΠΎΠ±Ρ€Π°Ρ‚Π½Ρ‹ΠΉ, Ρ‚Π°ΠΊ Ρ‡Ρ‚ΠΎ Ρ‚Π΅ΠΏΠ΅Ρ€ΡŒ [latex] \ overset {\ to} {B} [/ latex] ΠΈΠ΄Π΅Ρ‚ ΠΏΠ΅Ρ€Π²Ρ‹ΠΌ Π² ΠΏΡ€ΠΎΠ΄ΡƒΠΊΡ‚Π΅, Π·Π°Ρ‚Π΅ΠΌ vector [latex] \ overset {\ to} {B} \, Γ— \, \ overset {\ to} {A} [/ latex] Π΄ΠΎΠ»ΠΆΠ½Ρ‹ ΡƒΠΊΠ°Π·Ρ‹Π²Π°Ρ‚ΡŒ Π½Π° Π²Π½ΠΈΠ· Π½Π° , ΠΊΠ°ΠΊ ΠΏΠΎΠΊΠ°Π·Π°Π½ΠΎ Π½Π° (Рисунок) (b). Π­Ρ‚ΠΎ ΠΎΠ·Π½Π°Ρ‡Π°Π΅Ρ‚, Ρ‡Ρ‚ΠΎ Π²Π΅ΠΊΡ‚ΠΎΡ€Ρ‹ [латСкс] \ overset {\ to} {A} \, Γ— \, \ overset {\ to} {B} [/ latex] ΠΈ [latex] \ overset {\ to} {B} \, Γ— \, \ overset {\ to} {A} [/ latex] Π°Π½Ρ‚ΠΈΠΏΠ°Ρ€Π°Π»Π»Π΅Π»ΡŒΠ½Ρ‹ Π΄Ρ€ΡƒΠ³ Π΄Ρ€ΡƒΠ³Ρƒ, ΠΈ это ΡƒΠΌΠ½ΠΎΠΆΠ΅Π½ΠΈΠ΅ Π²Π΅ΠΊΡ‚ΠΎΡ€ΠΎΠ² Π½Π΅ ΠΊΠΎΠΌΠΌΡƒΡ‚Π°Ρ‚ΠΈΠ²Π½ΠΎ , Π° Π°Π½Ρ‚ΠΈΠΊΠΎΠΌΠΌΡƒΡ‚Π°Ρ‚ΠΈΠ²Π½ΠΎ .АнтикоммутативноС свойство ΠΎΠ·Π½Π°Ρ‡Π°Π΅Ρ‚, Ρ‡Ρ‚ΠΎ Π²Π΅ΠΊΡ‚ΠΎΡ€Π½ΠΎΠ΅ ΠΏΡ€ΠΎΠΈΠ·Π²Π΅Π΄Π΅Π½ΠΈΠ΅ мСняСт Π·Π½Π°ΠΊ ΠΏΡ€ΠΈ ΠΎΠ±Ρ€Π°Ρ‚Π½ΠΎΠΌ порядкС умноТСния:

    [латСкс] \ overset {\ to} {A} \, Γ— \, \ overset {\ to} {B} = \ text {-} \ overset {\ to} {B} \, Γ— \, \ overset { \ to} {A}. [/ латСкс]

    ΠŸΡ€Π°Π²Ρ‹ΠΉ ΡˆΡ‚ΠΎΠΏΠΎΡ€ ΠŸΡ€Π°Π²ΠΈΠ»ΠΎ — это обычная ΠΌΠ½Π΅ΠΌΠΎΠ½ΠΈΠΊΠ°, ΠΈΡΠΏΠΎΠ»ΡŒΠ·ΡƒΠ΅ΠΌΠ°Ρ для опрСдСлСния направлСния Π²Π΅ΠΊΡ‚ΠΎΡ€Π½ΠΎΠ³ΠΎ произвСдСния. Как ΠΏΠΎΠΊΠ°Π·Π°Π½ΠΎ Π½Π° (Рисунок), ΡˆΡ‚ΠΎΠΏΠΎΡ€ помСщаСтся Π² Π½Π°ΠΏΡ€Π°Π²Π»Π΅Π½ΠΈΠΈ, пСрпСндикулярном плоскости, которая содСрТит Π²Π΅ΠΊΡ‚ΠΎΡ€Ρ‹ [латСкс] \ overset {\ to} {A} [/ latex] ΠΈ [латСкс] \ overset {\ to} {B} [/ latex], Π° Π΅Π³ΠΎ Ρ€ΡƒΡ‡ΠΊΠ° ΠΏΠΎΠ²Π΅Ρ€Π½ΡƒΡ‚Π° Π² Π½Π°ΠΏΡ€Π°Π²Π»Π΅Π½ΠΈΠΈ ΠΎΡ‚ ΠΏΠ΅Ρ€Π²ΠΎΠ³ΠΎ Π²Π΅ΠΊΡ‚ΠΎΡ€Π° ΠΊΠΎ Π²Ρ‚ΠΎΡ€ΠΎΠΌΡƒ Π² ΠΈΠ·Π΄Π΅Π»ΠΈΠΈ.НаправлСниС ΠΏΠΎΠΏΠ΅Ρ€Π΅Ρ‡Π½ΠΎΠ³ΠΎ произвСдСния задаСтся Π΄Π²ΠΈΠΆΠ΅Π½ΠΈΠ΅ΠΌ ΡˆΡ‚ΠΎΠΏΠΎΡ€Π°.

    Рис. 2.30 ΠŸΡ€Π°Π²ΠΈΠ»ΠΎ ΠΏΡ€Π°Π²ΠΎΠΉ Ρ€ΡƒΠΊΠΈ со ΡˆΡ‚ΠΎΠΏΠΎΡ€ΠΎΠΌ ΠΌΠΎΠΆΠ½ΠΎ ΠΈΡΠΏΠΎΠ»ΡŒΠ·ΠΎΠ²Π°Ρ‚ΡŒ для опрСдСлСния направлСния пСрСкрСстного произвСдСния [латСкс] \ overset {\ to} {A} \, Γ— \, \ overset {\ to} {B} [/ латСкс]. ΠŸΠΎΠΌΠ΅ΡΡ‚ΠΈΡ‚Π΅ ΡˆΡ‚ΠΎΠΏΠΎΡ€ Π² Π½Π°ΠΏΡ€Π°Π²Π»Π΅Π½ΠΈΠΈ, пСрпСндикулярном плоскости, содСрТащСй Π²Π΅ΠΊΡ‚ΠΎΡ€Ρ‹ [латСкс] \ overset {\ to} {A} [/ latex] ΠΈ [latex] \ overset {\ to} {B} [/ latex], ΠΈ ΠΏΠΎΠ²Π΅Ρ€Π½ΠΈΡ‚Π΅ Π΅Π³ΠΎ Π² Π½Π°ΠΏΡ€Π°Π²Π»Π΅Π½ΠΈΠΈ ΠΎΡ‚ ΠΏΠ΅Ρ€Π²ΠΎΠ³ΠΎ Π²Π΅ΠΊΡ‚ΠΎΡ€Π° ΠΊΠΎ Π²Ρ‚ΠΎΡ€ΠΎΠΌΡƒ Π² ΠΏΡ€ΠΎΠΈΠ·Π²Π΅Π΄Π΅Π½ΠΈΠΈ.НаправлСниС ΠΏΠΎΠΏΠ΅Ρ€Π΅Ρ‡Π½ΠΎΠ³ΠΎ произвСдСния задаСтся Π΄Π²ΠΈΠΆΠ΅Π½ΠΈΠ΅ΠΌ ΡˆΡ‚ΠΎΠΏΠΎΡ€Π°. (Π°) Π”Π²ΠΈΠΆΠ΅Π½ΠΈΠ΅ Π²Π²Π΅Ρ€Ρ… ΠΎΠ·Π½Π°Ρ‡Π°Π΅Ρ‚, Ρ‡Ρ‚ΠΎ Π²Π΅ΠΊΡ‚ΠΎΡ€ пСрСкрСстного произвСдСния Π½Π°ΠΏΡ€Π°Π²Π»Π΅Π½ Π²Π²Π΅Ρ€Ρ…. (b) Π”Π²ΠΈΠΆΠ΅Π½ΠΈΠ΅ Π²Π½ΠΈΠ· ΠΎΠ·Π½Π°Ρ‡Π°Π΅Ρ‚, Ρ‡Ρ‚ΠΎ Π²Π΅ΠΊΡ‚ΠΎΡ€ пСрСкрСстного произвСдСния Π½Π°ΠΏΡ€Π°Π²Π»Π΅Π½ Π²Π½ΠΈΠ·.

    ΠŸΡ€ΠΈΠΌΠ΅Ρ€

    ΠšΡ€ΡƒΡ‚ΡΡ‰ΠΈΠΉ ΠΌΠΎΠΌΠ΅Π½Ρ‚ силы

    ΠœΠ΅Ρ…Π°Π½ΠΈΡ‡Π΅ΡΠΊΠΎΠ΅ прСимущСство, ΠΊΠΎΡ‚ΠΎΡ€ΠΎΠ΅ Π΄Π°Π΅Ρ‚ Π·Π½Π°ΠΊΠΎΠΌΡ‹ΠΉ инструмСнт, Π½Π°Π·Ρ‹Π²Π°Π΅ΠΌΡ‹ΠΉ Π³Π°Π΅Ρ‡Π½Ρ‹ΠΌ ΠΊΠ»ΡŽΡ‡ΠΎΠΌ , ((Рисунок)), зависит ΠΎΡ‚ Π²Π΅Π»ΠΈΡ‡ΠΈΠ½Ρ‹ F ΠΏΡ€ΠΈΠ»ΠΎΠΆΠ΅Π½Π½ΠΎΠΉ силы, Π΅Π΅ направлСния ΠΏΠΎ ΠΎΡ‚Π½ΠΎΡˆΠ΅Π½ΠΈΡŽ ΠΊ рукояткС Π³Π°Π΅Ρ‡Π½ΠΎΠ³ΠΎ ΠΊΠ»ΡŽΡ‡Π° ΠΈ ΠΎΡ‚ Ρ‚ΠΎΠ³ΠΎ, насколько Π΄Π°Π»Π΅ΠΊΠΎ ΠΎΡ‚ Π³Π°ΠΉΠΊΠΈ это усилиС. примСняСмый.РасстояниС R ΠΎΡ‚ Π³Π°ΠΉΠΊΠΈ Π΄ΠΎ Ρ‚ΠΎΡ‡ΠΊΠΈ, Π³Π΄Π΅ ΠΏΡ€ΠΈΠ»ΠΎΠΆΠ΅Π½ Π²Π΅ΠΊΡ‚ΠΎΡ€ силы [latex] \ overset {\ to} {F} [/ latex], ΠΈ прСдставлСно Ρ€Π°Π΄ΠΈΠ°Π»ΡŒΠ½Ρ‹ΠΌ Π²Π΅ΠΊΡ‚ΠΎΡ€ΠΎΠΌ [latex] \ overset {\ to} {R } [/ латСкс]. ЀизичСская вСкторная Π²Π΅Π»ΠΈΡ‡ΠΈΠ½Π°, которая заставляСт Π³Π°ΠΉΠΊΡƒ ΠΏΠΎΠ²ΠΎΡ€Π°Ρ‡ΠΈΠ²Π°Ρ‚ΡŒΡΡ, называСтся крутящим ΠΌΠΎΠΌΠ΅Π½Ρ‚ΠΎΠΌ (обозначаСтся [latex] \ overset {\ to} {\ tau}) [/ latex], ΠΈ это Π²Π΅ΠΊΡ‚ΠΎΡ€Π½ΠΎΠ΅ ΠΏΡ€ΠΎΠΈΠ·Π²Π΅Π΄Π΅Π½ΠΈΠ΅ расстояния ΠΌΠ΅ΠΆΠ΄Ρƒ стСрТнСм ΠΈ силой с силой: [латСкс] \ overset {\ to} {\ tau} = \ overset {\ to} {R} \, Γ— \, \ overset {\ to} {F} [/ latex].

    Π§Ρ‚ΠΎΠ±Ρ‹ ΠΎΡΠ»Π°Π±ΠΈΡ‚ΡŒ Ρ€ΠΆΠ°Π²ΡƒΡŽ Π³Π°ΠΉΠΊΡƒ, ΠΊ рукояткС Π³Π°Π΅Ρ‡Π½ΠΎΠ³ΠΎ ΠΊΠ»ΡŽΡ‡Π° ΠΏΡ€ΠΈΠΊΠ»Π°Π΄Ρ‹Π²Π°ΡŽΡ‚ усилиС 20,00 Н ΠΏΠΎΠ΄ ΡƒΠ³Π»ΠΎΠΌ [латСкс] \ phi = 40 \ text {Β°} [/ latex] ΠΈ Π½Π° расстоянии 0,25 ΠΌ ΠΎΡ‚ Π³Π°ΠΉΠΊΠΈ, ΠΊΠ°ΠΊ ΠΏΠΎΠΊΠ°Π·Π°Π½ΠΎ Π½Π° (Рисунок) (Π°). НайдитС Π²Π΅Π»ΠΈΡ‡ΠΈΠ½Ρƒ ΠΈ Π½Π°ΠΏΡ€Π°Π²Π»Π΅Π½ΠΈΠ΅ крутящСго ΠΌΠΎΠΌΠ΅Π½Ρ‚Π°, ΠΏΡ€ΠΈΠ»Π°Π³Π°Π΅ΠΌΠΎΠ³ΠΎ ΠΊ Π³Π°ΠΉΠΊΠ΅. ΠšΠ°ΠΊΠΎΠ²Ρ‹ Π±Ρ‹Π»ΠΈ Π±Ρ‹ Π²Π΅Π»ΠΈΡ‡ΠΈΠ½Π° ΠΈ Π½Π°ΠΏΡ€Π°Π²Π»Π΅Π½ΠΈΠ΅ крутящСго ΠΌΠΎΠΌΠ΅Π½Ρ‚Π°, Ссли Π±Ρ‹ сила Π±Ρ‹Π»Π° ΠΏΡ€ΠΈΠ»ΠΎΠΆΠ΅Π½Π° ΠΏΠΎΠ΄ ΡƒΠ³Π»ΠΎΠΌ [латСкс] \ phi = 45 \ text {Β°} [/ latex], ΠΊΠ°ΠΊ ΠΏΠΎΠΊΠ°Π·Π°Π½ΠΎ Π½Π° (Рисунок) (b)? Для ΠΊΠ°ΠΊΠΎΠ³ΠΎ значСния ΡƒΠ³Π»Π° [латСкс] \ Ρ„ΠΈ [/ латСкс] крутящий ΠΌΠΎΠΌΠ΅Π½Ρ‚ ΠΈΠΌΠ΅Π΅Ρ‚ Π½Π°ΠΈΠ±ΠΎΠ»ΡŒΡˆΡƒΡŽ Π²Π΅Π»ΠΈΡ‡ΠΈΠ½Ρƒ?

    Рисунок 2.31 Π“Π°Π΅Ρ‡Π½Ρ‹ΠΉ ΠΊΠ»ΡŽΡ‡ обСспСчиваСт Π·Π°Ρ…Π²Π°Ρ‚ ΠΈ мСханичСскоС прСимущСство ΠΏΡ€ΠΈ ΠΏΡ€ΠΈΠ»ΠΎΠΆΠ΅Π½ΠΈΠΈ крутящСго ΠΌΠΎΠΌΠ΅Π½Ρ‚Π° для ΠΏΠΎΠ²ΠΎΡ€ΠΎΡ‚Π° Π³Π°ΠΉΠΊΠΈ. (a) ΠŸΠΎΠ²Π΅Ρ€Π½ΠΈΡ‚Π΅ ΠΏΡ€ΠΎΡ‚ΠΈΠ² часовой стрСлки, Ρ‡Ρ‚ΠΎΠ±Ρ‹ ΠΎΡΠ»Π°Π±ΠΈΡ‚ΡŒ Π³Π°ΠΉΠΊΡƒ. (b) ΠŸΠΎΠ²Π΅Ρ€Π½ΠΈΡ‚Π΅ ΠΏΠΎ часовой стрСлкС, Ρ‡Ρ‚ΠΎΠ±Ρ‹ Π·Π°Ρ‚ΡΠ½ΡƒΡ‚ΡŒ Π³Π°ΠΉΠΊΡƒ.

    БтратСгия

    ΠœΡ‹ ΠΏΡ€ΠΈΠ½ΠΈΠΌΠ°Π΅ΠΌ систСму отсчСта, ΠΏΠΎΠΊΠ°Π·Π°Π½Π½ΡƒΡŽ Π½Π° (Рисунок), Π³Π΄Π΅ Π²Π΅ΠΊΡ‚ΠΎΡ€Ρ‹ [латСкс] \ overset {\ to} {R} [/ latex] ΠΈ [latex] \ overset {\ to} {F} [/ latex] Π»Π΅ΠΆΠ°Ρ‚ Π² ΠΏΠ»ΠΎΡΠΊΠΎΡΡ‚ΡŒ xy , Π° Π½Π°Ρ‡Π°Π»ΠΎ ΠΊΠΎΠΎΡ€Π΄ΠΈΠ½Π°Ρ‚ находится Π² ΠΏΠΎΠ»ΠΎΠΆΠ΅Π½ΠΈΠΈ Π³Π°ΠΉΠΊΠΈ. РадиальноС Π½Π°ΠΏΡ€Π°Π²Π»Π΅Π½ΠΈΠ΅ вдоль Π²Π΅ΠΊΡ‚ΠΎΡ€Π° [латСкс] \ overset {\ to} {R} [/ latex] (ΡƒΠΊΠ°Π·Ρ‹Π²Π°ΡŽΡ‰Π΅Π΅ ΠΎΡ‚ Π½Π°Ρ‡Π°Π»Π° ΠΊΠΎΠΎΡ€Π΄ΠΈΠ½Π°Ρ‚) являСтся ΠΎΠΏΠΎΡ€Π½Ρ‹ΠΌ Π½Π°ΠΏΡ€Π°Π²Π»Π΅Π½ΠΈΠ΅ΠΌ для измСрСния ΡƒΠ³Π»Π° [латСкс] \ phi [/ latex], ΠΏΠΎΡ‚ΠΎΠΌΡƒ Ρ‡Ρ‚ΠΎ [латСкс] \ overset {\ to} {R} [/ latex] — это ΠΏΠ΅Ρ€Π²Ρ‹ΠΉ Π²Π΅ΠΊΡ‚ΠΎΡ€ Π² Π²Π΅ΠΊΡ‚ΠΎΡ€Π½ΠΎΠΌ ΠΏΡ€ΠΎΠΈΠ·Π²Π΅Π΄Π΅Π½ΠΈΠΈ [latex] \ overset {\ to} {\ tau} = \ overset {\ to} {R} \, Γ— \, \ overset {\ to} {F} [/ латСкс].Π’Π΅ΠΊΡ‚ΠΎΡ€ [latex] \ overset {\ to} {\ tau} [/ latex] Π΄ΠΎΠ»ΠΆΠ΅Π½ Π»Π΅ΠΆΠ°Ρ‚ΡŒ вдоль оси z , ΠΏΠΎΡ‚ΠΎΠΌΡƒ Ρ‡Ρ‚ΠΎ это ось, пСрпСндикулярная плоскости xy , Π³Π΄Π΅ ΠΎΠ±Π° [latex] \ overset {\ to} {R} [/ latex] ΠΈ [latex] \ overset {\ to} {F} [/ latex] Π»Π³ΡƒΡ‚. Π§Ρ‚ΠΎΠ±Ρ‹ Π²Ρ‹Ρ‡ΠΈΡΠ»ΠΈΡ‚ΡŒ Π²Π΅Π»ΠΈΡ‡ΠΈΠ½Ρƒ [латСкс] \ Ρ‚Π°Ρƒ [/ латСкс], ΠΌΡ‹ ΠΈΡΠΏΠΎΠ»ΡŒΠ·ΡƒΠ΅ΠΌ (рисунок). Π§Ρ‚ΠΎΠ±Ρ‹ Π½Π°ΠΉΡ‚ΠΈ Π½Π°ΠΏΡ€Π°Π²Π»Π΅Π½ΠΈΠ΅ [latex] \ overset {\ to} {\ tau} [/ latex], ΠΌΡ‹ ΠΈΡΠΏΠΎΠ»ΡŒΠ·ΡƒΠ΅ΠΌ ΠΏΡ€Π°Π²ΠΈΠ»ΠΎ ΠΏΡ€Π°Π²ΠΎΠΉ Ρ€ΡƒΠΊΠΈ со ΡˆΡ‚ΠΎΠΏΠΎΡ€ΠΎΠΌ ((Рисунок)).

    РСшСниС
    ПокаТи ΠΎΡ‚Π²Π΅Ρ‚ Для ситуации Π² (Π°) ΠΏΡ€Π°Π²ΠΈΠ»ΠΎ ΡˆΡ‚ΠΎΠΏΠΎΡ€Π° Π΄Π°Π΅Ρ‚ Π½Π°ΠΏΡ€Π°Π²Π»Π΅Π½ΠΈΠ΅ [латСкс] \ overset {\ to} {R} \, Γ— \, \ overset {\ to} {F} [/ latex] Π² ΠΏΠΎΠ»ΠΎΠΆΠΈΡ‚Π΅Π»ΡŒΠ½ΠΎΠΌ Π½Π°ΠΏΡ€Π°Π²Π»Π΅Π½ΠΈΠΈ ось z.ЀизичСски это ΠΎΠ·Π½Π°Ρ‡Π°Π΅Ρ‚, Ρ‡Ρ‚ΠΎ Π²Π΅ΠΊΡ‚ΠΎΡ€ крутящСго ΠΌΠΎΠΌΠ΅Π½Ρ‚Π° [latex] \ overset {\ to} {\ tau} [/ latex] Π½Π°ΠΏΡ€Π°Π²Π»Π΅Π½ Π·Π° ΠΏΡ€Π΅Π΄Π΅Π»Ρ‹ страницы пСрпСндикулярно рукояткС Π³Π°Π΅Ρ‡Π½ΠΎΠ³ΠΎ ΠΊΠ»ΡŽΡ‡Π°. ΠœΡ‹ опрСдСляСм F = 20,00 N ΠΈ R = 0,25 ΠΌ ΠΈ вычисляСм Π²Π΅Π»ΠΈΡ‡ΠΈΠ½Ρƒ, ΠΈΡΠΏΠΎΠ»ΡŒΠ·ΡƒΡ (Рисунок):

    [латСкс] \ tau \, = | \ overset {\ to} {R} \, Γ— \, \ overset {\ to} {F} | = \, RF \, \ text {sin} \, \ phi = (0,25 \, \ text {m}) (20,00 \, \ text {N}) \, \ text {sin} \, 40 \ text {Β°} = 3,21 \, \ text {N} Β· \ text {m} . [/ latex] Для ситуации Π² (b) ΠΏΡ€Π°Π²ΠΈΠ»ΠΎ ΡˆΡ‚ΠΎΠΏΠΎΡ€Π° Π΄Π°Π΅Ρ‚ Π½Π°ΠΏΡ€Π°Π²Π»Π΅Π½ΠΈΠ΅ [latex] \ overset {\ to} {R} \, Γ— \, \ overset {\ to} {F} [/ latex] Π² ΠΎΡ‚Ρ€ΠΈΡ†Π°Ρ‚Π΅Π»ΡŒΠ½ΠΎΠ΅ Π½Π°ΠΏΡ€Π°Π²Π»Π΅Π½ΠΈΠ΅ оси z.ЀизичСски это ΠΎΠ·Π½Π°Ρ‡Π°Π΅Ρ‚, Ρ‡Ρ‚ΠΎ Π²Π΅ΠΊΡ‚ΠΎΡ€ [латСкс] \ overset {\ to} {\ tau} [/ latex] ΡƒΠΊΠ°Π·Ρ‹Π²Π°Π΅Ρ‚ Π½Π° страницу пСрпСндикулярно рукояткС Π³Π°Π΅Ρ‡Π½ΠΎΠ³ΠΎ ΠΊΠ»ΡŽΡ‡Π°. Π’Π΅Π»ΠΈΡ‡ΠΈΠ½Π° этого крутящСго ΠΌΠΎΠΌΠ΅Π½Ρ‚Π°

    [латСкс] \ tau \, = | \ overset {\ to} {R} \, Γ— \, \ overset {\ to} {F} | = \, RF \, \ text {sin} \, \ phi = (0,25 \, \ text {m}) (20,00 \, \ text {N}) \, \ text {sin} \, 45 \ text {Β°} = 3,53 \, \ text {N} Β· \ text {m} . [/ latex] ΠšΡ€ΡƒΡ‚ΡΡ‰ΠΈΠΉ ΠΌΠΎΠΌΠ΅Π½Ρ‚ ΠΈΠΌΠ΅Π΅Ρ‚ наибольшСС Π·Π½Π°Ρ‡Π΅Π½ΠΈΠ΅, ΠΊΠΎΠ³Π΄Π° [latex] \ text {sin} \, \ phi = 1 [/ latex], Ρ‡Ρ‚ΠΎ происходит, ΠΊΠΎΠ³Π΄Π° [latex] \ phi = 90 \ text {Β°} [/ latex]. ЀизичСски это ΠΎΠ·Π½Π°Ρ‡Π°Π΅Ρ‚, Ρ‡Ρ‚ΠΎ Π³Π°Π΅Ρ‡Π½Ρ‹ΠΉ ΠΊΠ»ΡŽΡ‡ Π½Π°ΠΈΠ±ΠΎΠ»Π΅Π΅ эффСктивСн — Ρ‡Ρ‚ΠΎ Π΄Π°Π΅Ρ‚ Π½Π°ΠΌ Π»ΡƒΡ‡ΡˆΠ΅Π΅ мСханичСскоС прСимущСство — ΠΊΠΎΠ³Π΄Π° ΠΌΡ‹ ΠΏΡ€ΠΈΠΊΠ»Π°Π΄Ρ‹Π²Π°Π΅ΠΌ силу пСрпСндикулярно рукояткС Π³Π°Π΅Ρ‡Π½ΠΎΠ³ΠΎ ΠΊΠ»ΡŽΡ‡Π°.Для ситуации Π² этом ΠΏΡ€ΠΈΠΌΠ΅Ρ€Π΅ это Π·Π½Π°Ρ‡Π΅Π½ΠΈΠ΅ Π½Π°ΠΈΠ»ΡƒΡ‡ΡˆΠ΅Π³ΠΎ крутящСго ΠΌΠΎΠΌΠ΅Π½Ρ‚Π° [latex] {\ tau} _ {\ text {best}} = RF = (0.25 \, \ text {m}) (20.00 \, \ text {N} ) = 5,00 \, \ text {N} Β· \ text {m} [/ latex].

    Π—Π½Π°Ρ‡Π΅Π½ΠΈΠ΅

    ΠŸΡ€ΠΈ Ρ€Π΅ΡˆΠ΅Π½ΠΈΠΈ Π·Π°Π΄Π°Ρ‡ ΠΌΠ΅Ρ…Π°Π½ΠΈΠΊΠΈ Π½Π°ΠΌ часто Π²ΠΎΠΎΠ±Ρ‰Π΅ Π½Π΅ Π½ΡƒΠΆΠ½ΠΎ ΠΈΡΠΏΠΎΠ»ΡŒΠ·ΠΎΠ²Π°Ρ‚ΡŒ ΠΏΡ€Π°Π²ΠΈΠ»ΠΎ ΡˆΡ‚ΠΎΠΏΠΎΡ€Π°, ΠΊΠ°ΠΊ ΠΌΡ‹ сСйчас ΡƒΠ²ΠΈΠ΄ΠΈΠΌ Π² ΡΠ»Π΅Π΄ΡƒΡŽΡ‰Π΅ΠΌ эквивалСнтном Ρ€Π΅ΡˆΠ΅Π½ΠΈΠΈ. ΠžΠ±Ρ€Π°Ρ‚ΠΈΡ‚Π΅ Π²Π½ΠΈΠΌΠ°Π½ΠΈΠ΅: ΠΊΠ°ΠΊ Ρ‚ΠΎΠ»ΡŒΠΊΠΎ ΠΌΡ‹ ΠΎΠΏΡ€Π΅Π΄Π΅Π»ΠΈΠ»ΠΈ, Ρ‡Ρ‚ΠΎ Π²Π΅ΠΊΡ‚ΠΎΡ€ [латСкс] \ overset {\ to} {R} \, Γ— \, \ overset {\ to} {F} [/ latex] Π»Π΅ΠΆΠΈΡ‚ вдоль оси z , ΠΌΡ‹ ΠΌΠΎΠΆΠ΅ΠΌ Π½Π°ΠΏΠΈΡΠ°Ρ‚ΡŒ этот Π²Π΅ΠΊΡ‚ΠΎΡ€ Π² Ρ‚Π΅Ρ€ΠΌΠΈΠ½Π°Ρ… Π΅Π΄ΠΈΠ½ΠΈΡ‡Π½ΠΎΠ³ΠΎ Π²Π΅ΠΊΡ‚ΠΎΡ€Π° [latex] \ hat {k} [/ latex] оси z :

    [латСкс] \ overset {\ to} {R} \, Γ— \, \ overset {\ to} {F} = RF \, \ text {sin} \, \ phi \ hat {k}.[/ латСкс]

    Π’ этом ΡƒΡ€Π°Π²Π½Π΅Π½ΠΈΠΈ число, ΡƒΠΌΠ½ΠΎΠΆΠ°ΡŽΡ‰Π΅Π΅ [latex] \ hat {k} [/ latex], являСтся скалярной z -ΠΊΠΎΠΌΠΏΠΎΠ½Π΅Π½Ρ‚ΠΎΠΉ Π²Π΅ΠΊΡ‚ΠΎΡ€Π° [latex] \ overset {\ to} {R} \, Γ— \, \ overset {\ to} {F} [/ latex]. ΠŸΡ€ΠΈ вычислСнии этого ΠΊΠΎΠΌΠΏΠΎΠ½Π΅Π½Ρ‚Π° Π½Π΅ΠΎΠ±Ρ…ΠΎΠ΄ΠΈΠΌΠΎ ΡΠ»Π΅Π΄ΠΈΡ‚ΡŒ Π·Π° Ρ‚Π΅ΠΌ, Ρ‡Ρ‚ΠΎΠ±Ρ‹ ΡƒΠ³ΠΎΠ» [латСкс] \ phi [/ latex] измСрялся ΠΏΡ€ΠΎΡ‚ΠΈΠ² часовой стрСлки ΠΎΡ‚ [latex] \ overset {\ to} {R} [/ latex] (ΠΏΠ΅Ρ€Π²Ρ‹ΠΉ Π²Π΅ΠΊΡ‚ΠΎΡ€) Π΄ΠΎ [латСкс] \ overset {\ to} {F} [/ latex] (Π²Ρ‚ΠΎΡ€ΠΎΠΉ Π²Π΅ΠΊΡ‚ΠΎΡ€). БлСдуя этому ΠΏΡ€ΠΈΠ½Ρ†ΠΈΠΏΡƒ для ΡƒΠ³Π»ΠΎΠ², ΠΌΡ‹ ΠΏΠΎΠ»ΡƒΡ‡Π°Π΅ΠΌ [latex] RF \, \ text {sin} \, (+ 40 \ text {Β°}) = + 3.2 \, \ text {N} Β· \ text {m} [/ latex] для ситуации Π² (a), ΠΈ ΠΌΡ‹ ΠΏΠΎΠ»ΡƒΡ‡Π°Π΅ΠΌ [latex] RF \, \ text {sin} \, (- 45 \ text {Β°} ) = — 3.5 \, \ text {N} Β· \ text {m} [/ latex] для ситуации Π² (b). Π’ послСднСм случаС ΡƒΠ³ΠΎΠ» ΠΎΡ‚Ρ€ΠΈΡ†Π°Ρ‚Π΅Π»ΡŒΠ½Ρ‹ΠΉ, ΠΏΠΎΡ‚ΠΎΠΌΡƒ Ρ‡Ρ‚ΠΎ Π³Ρ€Π°Ρ„ΠΈΠΊ Π½Π° (Рисунок) ΠΏΠΎΠΊΠ°Π·Ρ‹Π²Π°Π΅Ρ‚, Ρ‡Ρ‚ΠΎ ΡƒΠ³ΠΎΠ» измСряСтся ΠΏΠΎ часовой стрСлкС; Π½ΠΎ Ρ‚ΠΎΡ‚ ΠΆΠ΅ Ρ€Π΅Π·ΡƒΠ»ΡŒΡ‚Π°Ρ‚ получаСтся, ΠΊΠΎΠ³Π΄Π° этот ΡƒΠ³ΠΎΠ» измСряСтся ΠΏΡ€ΠΎΡ‚ΠΈΠ² часовой стрСлки, ΠΏΠΎΡ‚ΠΎΠΌΡƒ Ρ‡Ρ‚ΠΎ [латСкс] + (360 \ text {Β°} -45 \ text {Β°}) = + 315 \ text {Β°} [/ latex] ΠΈ [latex] \ тСкст {Π³Ρ€Π΅Ρ…} \, (+ 315 \ тСкст {Β°}) = \ тСкст {Π³Ρ€Π΅Ρ…} \, (- 45 \ тСкст {Β°}) [/ латСкс].Π’Π°ΠΊΠΈΠΌ ΠΎΠ±Ρ€Π°Π·ΠΎΠΌ, ΠΌΡ‹ ΠΏΠΎΠ»ΡƒΡ‡Π°Π΅ΠΌ Ρ€Π΅ΡˆΠ΅Π½ΠΈΠ΅ Π±Π΅Π· привязки ΠΊ ΠΏΡ€Π°Π²ΠΈΠ»Ρƒ ΡˆΡ‚ΠΎΠΏΠΎΡ€Π°. Для ситуации Π² (a) Ρ€Π΅ΡˆΠ΅Π½ΠΈΠ΅ΠΌ являСтся [latex] \ overset {\ to} {R} \, Γ— \, \ overset {\ to} {F} = + 3.2 \, \ text {N} Β· \ text {ΠΌ} \ шляпа {ΠΊ} [/ латСкс]; для ситуации Π² (b) Ρ€Π΅ΡˆΠ΅Π½ΠΈΠ΅ΠΌ являСтся [latex] \ overset {\ to} {R} \, Γ— \, \ overset {\ to} {F} = — 3.5 \, \ text {N} Β· \ text {m} \ hat {k} [/ латСкс].

    ΠŸΡ€ΠΎΠ²Π΅Ρ€ΡŒΡ‚Π΅ своС ΠΏΠΎΠ½ΠΈΠΌΠ°Π½ΠΈΠ΅

    Для Π²Π΅ΠΊΡ‚ΠΎΡ€ΠΎΠ², ΡƒΠΊΠ°Π·Π°Π½Π½Ρ‹Ρ… Π½Π° (Рисунок), Π½Π°ΠΉΠ΄ΠΈΡ‚Π΅ Π²Π΅ΠΊΡ‚ΠΎΡ€Π½Ρ‹Π΅ произвСдСния [latex] \ overset {\ to} {A} \, Γ— \, \ overset {\ to} {B} [/ latex] ΠΈ [latex] \ overset {\ to} {C} \, Γ— \, \ overset {\ to} {F} [/ latex].

    ΠŸΠΎΠΊΠ°Π·Π°Ρ‚ΡŒ Ρ€Π΅ΡˆΠ΅Π½ΠΈΠ΅

    [латСкс] \ overset {\ to} {A} \, Γ— \, \ overset {\ to} {B} = — 40.1 \ hat {k} [/ latex] ΠΈΠ»ΠΈ, Ρ‡Ρ‚ΠΎ Ρ‚ΠΎ ΠΆΠ΅ самоС, [латСкс] | \ overset { \ to} {A} \, Γ— \, \ overset {\ to} {B} | = 40,1 [/ latex], ΠΈ Π½Π°ΠΏΡ€Π°Π²Π»Π΅Π½ΠΈΠ΅ — Π²Π½ΡƒΡ‚Ρ€ΡŒ страницы; [латСкс] \ overset {\ to} {C} \, Γ— \, \ overset {\ to} {F} = + 157.6 \ hat {k} [/ latex] ΠΈΠ»ΠΈ, Ρ‡Ρ‚ΠΎ Ρ‚ΠΎ ΠΆΠ΅ самоС, [латСкс] | \ overset {\ to} {C} \, Γ— \, \ overset {\ to} {F} | = 157,6 [/ latex], ΠΈ Π½Π°ΠΏΡ€Π°Π²Π»Π΅Π½ΠΈΠ΅ — Π²Π½Π΅ страницы.

    Подобно скалярному ΠΏΡ€ΠΎΠΈΠ·Π²Π΅Π΄Π΅Π½ΠΈΡŽ ((Рисунок)), пСрСкрСстноС ΠΏΡ€ΠΎΠΈΠ·Π²Π΅Π΄Π΅Π½ΠΈΠ΅ ΠΈΠΌΠ΅Π΅Ρ‚ ΡΠ»Π΅Π΄ΡƒΡŽΡ‰Π΅Π΅ свойство распрСдСлСния:

    [латСкс] \ overset {\ to} {A} \, Γ— \, (\ overset {\ to} {B} + \ overset {\ to} {C}) = \ overset {\ to} {A} \ , Γ— \, \ overset {\ to} {B} + \ overset {\ to} {A} \, Γ— \, \ overset {\ to} {C}.[/ латСкс]

    Бвойство распрСдСлСния часто примСняСтся, ΠΊΠΎΠ³Π΄Π° Π²Π΅ΠΊΡ‚ΠΎΡ€Ρ‹ Π²Ρ‹Ρ€Π°ΠΆΠ°ΡŽΡ‚ΡΡ Π² ΠΈΡ… составных Ρ„ΠΎΡ€ΠΌΠ°Ρ… Π² Ρ‚Π΅Ρ€ΠΌΠΈΠ½Π°Ρ… Π΅Π΄ΠΈΠ½ΠΈΡ‡Π½Ρ‹Ρ… Π²Π΅ΠΊΡ‚ΠΎΡ€ΠΎΠ² Π΄Π΅ΠΊΠ°Ρ€Ρ‚ΠΎΠ²Ρ‹Ρ… осСй.

    Когда ΠΌΡ‹ примСняСм ΠΎΠΏΡ€Π΅Π΄Π΅Π»Π΅Π½ΠΈΠ΅ пСрСкрСстного произвСдСния (рисунок) ΠΊ Π΅Π΄ΠΈΠ½ΠΈΡ‡Π½Ρ‹ΠΌ Π²Π΅ΠΊΡ‚ΠΎΡ€Π°ΠΌ [latex] \ hat {i} [/ latex], [latex] \ hat {j} [/ latex] ΠΈ [latex] \ hat {k} [/ latex], ΠΊΠΎΡ‚ΠΎΡ€Ρ‹Π΅ ΠΎΠΏΡ€Π΅Π΄Π΅Π»ΡΡŽΡ‚ ΠΏΠΎΠ»ΠΎΠΆΠΈΡ‚Π΅Π»ΡŒΠ½Ρ‹Π΅ x -, y — ΠΈ z — направлСния Π² пространствС, ΠΌΡ‹ Π½Π°Ρ…ΠΎΠ΄ΠΈΠΌ, Ρ‡Ρ‚ΠΎ

    [латСкс] \ hat {i} \, Γ— \, \ hat {i} = \ hat {j} \, Γ— \, \ hat {j} = \ hat {k} \, Γ— \, \ hat {k } = 0.[/ латСкс]

    ВсС Π΄Ρ€ΡƒΠ³ΠΈΠ΅ пСрСкрСстныС произвСдСния этих Ρ‚Ρ€Π΅Ρ… Π΅Π΄ΠΈΠ½ΠΈΡ‡Π½Ρ‹Ρ… Π²Π΅ΠΊΡ‚ΠΎΡ€ΠΎΠ² Π΄ΠΎΠ»ΠΆΠ½Ρ‹ Π±Ρ‹Ρ‚ΡŒ Π²Π΅ΠΊΡ‚ΠΎΡ€Π°ΠΌΠΈ Π΅Π΄ΠΈΠ½ΠΈΡ‡Π½ΠΎΠΉ Π²Π΅Π»ΠΈΡ‡ΠΈΠ½Ρ‹, ΠΏΠΎΡ‚ΠΎΠΌΡƒ Ρ‡Ρ‚ΠΎ [latex] \ hat {i} [/ latex], [latex] \ hat {j} [/ latex] ΠΈ [latex] \ hat { k} [/ latex] ΠΎΡ€Ρ‚ΠΎΠ³ΠΎΠ½Π°Π»ΡŒΠ½Ρ‹. НапримСр, для ΠΏΠ°Ρ€Ρ‹ [латСкс] \ hat {i} [/ latex] ΠΈ [latex] \ hat {j} [/ latex] Π²Π΅Π»ΠΈΡ‡ΠΈΠ½Π° Π±ΡƒΠ΄Π΅Ρ‚ [latex] | \ hat {i} \, Γ— \, \ шляпа {j} | = ij \, \ text {sin} \, 90 \ text {Β°} = (1) (1) (1) = 1 [/ latex]. НаправлСниС Π²Π΅ΠΊΡ‚ΠΎΡ€Π½ΠΎΠ³ΠΎ произвСдСния [латСкс] \ hat {i} \, Γ— \, \ hat {j} [/ latex] Π΄ΠΎΠ»ΠΆΠ½ΠΎ Π±Ρ‹Ρ‚ΡŒ ΠΎΡ€Ρ‚ΠΎΠ³ΠΎΠ½Π°Π»ΡŒΠ½ΠΎ плоскости xy , Ρ‡Ρ‚ΠΎ ΠΎΠ·Π½Π°Ρ‡Π°Π΅Ρ‚, Ρ‡Ρ‚ΠΎ ΠΎΠ½ΠΎ Π΄ΠΎΠ»ΠΆΠ½ΠΎ Π±Ρ‹Ρ‚ΡŒ вдоль z — ось.ЕдинствСнныС Π΅Π΄ΠΈΠ½ΠΈΡ‡Π½Ρ‹Π΅ Π²Π΅ΠΊΡ‚ΠΎΡ€Ρ‹ вдоль оси z — это [latex] \ text {-} \ hat {k} [/ latex] ΠΈΠ»ΠΈ [latex] + \ hat {k} [/ latex]. По ΠΏΡ€Π°Π²ΠΈΠ»Ρƒ ΡˆΡ‚ΠΎΠΏΠΎΡ€Π° Π½Π°ΠΏΡ€Π°Π²Π»Π΅Π½ΠΈΠ΅ Π²Π΅ΠΊΡ‚ΠΎΡ€Π° [латСкс] \ hat {i} \, Γ— \, \ hat {j} [/ latex] Π΄ΠΎΠ»ΠΆΠ½ΠΎ Π±Ρ‹Ρ‚ΡŒ ΠΏΠ°Ρ€Π°Π»Π»Π΅Π»ΡŒΠ½ΠΎ ΠΏΠΎΠ»ΠΎΠΆΠΈΡ‚Π΅Π»ΡŒΠ½ΠΎΠΉ оси z . Π‘Π»Π΅Π΄ΠΎΠ²Π°Ρ‚Π΅Π»ΡŒΠ½ΠΎ, Ρ€Π΅Π·ΡƒΠ»ΡŒΡ‚Π°Ρ‚ умноТСния [latex] \ hat {i} \, Γ— \, \ hat {j} [/ latex] ΠΈΠ΄Π΅Π½Ρ‚ΠΈΡ‡Π΅Π½ [latex] + \ hat {k} [/ latex]. ΠœΡ‹ ΠΌΠΎΠΆΠ΅ΠΌ ΠΏΠΎΠ²Ρ‚ΠΎΡ€ΠΈΡ‚ΡŒ Π°Π½Π°Π»ΠΎΠ³ΠΈΡ‡Π½Ρ‹Π΅ рассуТдСния для ΠΎΡΡ‚Π°Π»ΡŒΠ½Ρ‹Ρ… ΠΏΠ°Ρ€ Π΅Π΄ΠΈΠ½ΠΈΡ‡Π½Ρ‹Ρ… Π²Π΅ΠΊΡ‚ΠΎΡ€ΠΎΠ². Π Π΅Π·ΡƒΠ»ΡŒΡ‚Π°Ρ‚ΠΎΠΌ умноТСния Π±ΡƒΠ΄Π΅Ρ‚

    .

    [латСкс] \ {\ begin {array} {l} \ hat {i} \, Γ— \, \ hat {j} = + \ hat {k}, \\ \ hat {j} \, Γ— \, \ hat {k} = + \ hat {i}, \\ \ hat {k} \, Γ— \, \ hat {i} = + \ hat {j}.\ end {array} [/ latex]

    ΠžΠ±Ρ€Π°Ρ‚ΠΈΡ‚Π΅ Π²Π½ΠΈΠΌΠ°Π½ΠΈΠ΅, Ρ‡Ρ‚ΠΎ Π½Π° (Рисунок) Ρ‚Ρ€ΠΈ Π΅Π΄ΠΈΠ½ΠΈΡ‡Π½Ρ‹Ρ… Π²Π΅ΠΊΡ‚ΠΎΡ€Π° [latex] \ hat {i} [/ latex], [latex] \ hat {j} [/ latex] ΠΈ [latex] \ hat {k} [/ latex ] ΠΏΠΎΡΠ²Π»ΡΡŽΡ‚ΡΡ Π² цикличСском порядкС , ΠΏΠΎΠΊΠ°Π·Π°Π½Π½ΠΎΠΌ Π½Π° Π΄ΠΈΠ°Π³Ρ€Π°ΠΌΠΌΠ΅ (Рисунок) (a). ЦикличСский порядок ΠΎΠ·Π½Π°Ρ‡Π°Π΅Ρ‚, Ρ‡Ρ‚ΠΎ Π² Ρ„ΠΎΡ€ΠΌΡƒΠ»Π΅ ΠΏΡ€ΠΎΠ΄ΡƒΠΊΡ‚Π° [latex] \ hat {i} [/ latex] слСдуСт Π·Π° [latex] \ hat {k} [/ latex] ΠΈ ΠΏΡ€Π΅Π΄ΡˆΠ΅ΡΡ‚Π²ΡƒΠ΅Ρ‚ [latex] \ hat {j} [/ latex] , ΠΈΠ»ΠΈ [латСкс] \ hat {k} [/ latex] слСдуСт Π·Π° [latex] \ hat {j} [/ latex] ΠΈ ΠΏΡ€Π΅Π΄ΡˆΠ΅ΡΡ‚Π²ΡƒΠ΅Ρ‚ [latex] \ hat {i} [/ latex] ΠΈΠ»ΠΈ [latex] \ hat { j} [/ latex] слСдуСт Π·Π° [latex] \ hat {i} [/ latex] ΠΈ ΠΏΡ€Π΅Π΄ΡˆΠ΅ΡΡ‚Π²ΡƒΠ΅Ρ‚ [latex] \ hat {k} [/ latex].ΠŸΠ΅Ρ€Π΅ΠΊΡ€Π΅ΡΡ‚Π½ΠΎΠ΅ ΠΏΡ€ΠΎΠΈΠ·Π²Π΅Π΄Π΅Π½ΠΈΠ΅ Π΄Π²ΡƒΡ… Ρ€Π°Π·Π½Ρ‹Ρ… Π΅Π΄ΠΈΠ½ΠΈΡ‡Π½Ρ‹Ρ… Π²Π΅ΠΊΡ‚ΠΎΡ€ΠΎΠ² всСгда являСтся Ρ‚Ρ€Π΅Ρ‚ΡŒΠΈΠΌ Π΅Π΄ΠΈΠ½ΠΈΡ‡Π½Ρ‹ΠΌ Π²Π΅ΠΊΡ‚ΠΎΡ€ΠΎΠΌ. Когда Π΄Π²Π° Π΅Π΄ΠΈΠ½ΠΈΡ‡Π½Ρ‹Ρ… Π²Π΅ΠΊΡ‚ΠΎΡ€Π° Π² пСрСкрСстном ΠΏΡ€ΠΎΠΈΠ·Π²Π΅Π΄Π΅Π½ΠΈΠΈ ΠΏΠΎΡΠ²Π»ΡΡŽΡ‚ΡΡ Π² цикличСском порядкС, Ρ€Π΅Π·ΡƒΠ»ΡŒΡ‚Π°Ρ‚ΠΎΠΌ Ρ‚Π°ΠΊΠΎΠ³ΠΎ умноТСния являСтся ΠΎΡΡ‚Π°Π²ΡˆΠΈΠΉΡΡ Π΅Π΄ΠΈΠ½ΠΈΡ‡Π½Ρ‹ΠΉ Π²Π΅ΠΊΡ‚ΠΎΡ€, ΠΊΠ°ΠΊ ΠΏΠΎΠΊΠ°Π·Π°Π½ΠΎ Π½Π° (Рисунок) (b). Когда Π΅Π΄ΠΈΠ½ΠΈΡ‡Π½Ρ‹Π΅ Π²Π΅ΠΊΡ‚ΠΎΡ€Ρ‹ Π² пСрСкрСстном ΠΏΡ€ΠΎΠΈΠ·Π²Π΅Π΄Π΅Π½ΠΈΠΈ ΠΏΠΎΡΠ²Π»ΡΡŽΡ‚ΡΡ Π² Π΄Ρ€ΡƒΠ³ΠΎΠΌ порядкС, Ρ€Π΅Π·ΡƒΠ»ΡŒΡ‚Π°Ρ‚ΠΎΠΌ являСтся Π΅Π΄ΠΈΠ½ΠΈΡ‡Π½Ρ‹ΠΉ Π²Π΅ΠΊΡ‚ΠΎΡ€, Π°Π½Ρ‚ΠΈΠΏΠ°Ρ€Π°Π»Π»Π΅Π»ΡŒΠ½Ρ‹ΠΉ ΠΎΡΡ‚Π°Π²ΡˆΠ΅ΠΌΡƒΡΡ Π΅Π΄ΠΈΠ½ΠΈΡ‡Π½ΠΎΠΌΡƒ Π²Π΅ΠΊΡ‚ΠΎΡ€Ρƒ (Ρ‚. Π•. Π Π΅Π·ΡƒΠ»ΡŒΡ‚Π°Ρ‚ со Π·Π½Π°ΠΊΠΎΠΌ минус, ΠΊΠ°ΠΊ ΠΏΠΎΠΊΠ°Π·Π°Π½ΠΎ Π² ΠΏΡ€ΠΈΠΌΠ΅Ρ€Π°Ρ… Π½Π° (Рисунок) (c). ΠΈ (Рисунок) (d). На ΠΏΡ€Π°ΠΊΡ‚ΠΈΠΊΠ΅, ΠΊΠΎΠ³Π΄Π° Π·Π°Π΄Π°Ρ‡Π° состоит Π² Ρ‚ΠΎΠΌ, Ρ‡Ρ‚ΠΎΠ±Ρ‹ Π½Π°ΠΉΡ‚ΠΈ пСрСкрСстныС произвСдСния Π²Π΅ΠΊΡ‚ΠΎΡ€ΠΎΠ², ΠΊΠΎΡ‚ΠΎΡ€Ρ‹Π΅ Π΄Π°Π½Ρ‹ Π² Ρ„ΠΎΡ€ΠΌΠ΅ ΠΊΠΎΠΌΠΏΠΎΠ½Π΅Π½Ρ‚ΠΎΠ² Π²Π΅ΠΊΡ‚ΠΎΡ€Π°, это ΠΏΡ€Π°Π²ΠΈΠ»ΠΎ пСрСкрСстного умноТСния Π΅Π΄ΠΈΠ½ΠΈΡ‡Π½Ρ‹Ρ… Π²Π΅ΠΊΡ‚ΠΎΡ€ΠΎΠ² ΠΎΡ‡Π΅Π½ΡŒ ΠΏΠΎΠ»Π΅Π·Π½ΠΎ.

    Рисунок 2.32 (a) Π”ΠΈΠ°Π³Ρ€Π°ΠΌΠΌΠ° цикличСского порядка Π΅Π΄ΠΈΠ½ΠΈΡ‡Π½Ρ‹Ρ… Π²Π΅ΠΊΡ‚ΠΎΡ€ΠΎΠ² осСй. (b) ЕдинствСнныС пСрСкрСстныС произвСдСния, Π² ΠΊΠΎΡ‚ΠΎΡ€Ρ‹Ρ… Π΅Π΄ΠΈΠ½ΠΈΡ‡Π½Ρ‹Π΅ Π²Π΅ΠΊΡ‚ΠΎΡ€Ρ‹ ΠΏΠΎΡΠ²Π»ΡΡŽΡ‚ΡΡ Π² цикличСском порядкС. Π­Ρ‚ΠΈ ΠΏΡ€ΠΎΠ΄ΡƒΠΊΡ‚Ρ‹ ΠΈΠΌΠ΅ΡŽΡ‚ ΠΏΠΎΠ»ΠΎΠΆΠΈΡ‚Π΅Π»ΡŒΠ½Ρ‹ΠΉ Π·Π½Π°ΠΊ. (c, d) Π”Π²Π° ΠΏΡ€ΠΈΠΌΠ΅Ρ€Π° пСрСкрСстных ΠΏΡ€ΠΎΠΈΠ·Π²Π΅Π΄Π΅Π½ΠΈΠΉ, Π³Π΄Π΅ Π΅Π΄ΠΈΠ½ΠΈΡ‡Π½Ρ‹Π΅ Π²Π΅ΠΊΡ‚ΠΎΡ€Ρ‹ Π½Π΅ ΠΏΠΎΡΠ²Π»ΡΡŽΡ‚ΡΡ Π² цикличСском порядкС. Π­Ρ‚ΠΈ ΠΏΡ€ΠΎΠ΄ΡƒΠΊΡ‚Ρ‹ ΠΈΠΌΠ΅ΡŽΡ‚ ΠΎΡ‚Ρ€ΠΈΡ†Π°Ρ‚Π΅Π»ΡŒΠ½Ρ‹ΠΉ Π·Π½Π°ΠΊ.

    ΠŸΡ€Π΅Π΄ΠΏΠΎΠ»ΠΎΠΆΠΈΠΌ, ΠΌΡ‹ Ρ…ΠΎΡ‚ΠΈΠΌ Π½Π°ΠΉΡ‚ΠΈ пСрСкрСстноС ΠΏΡ€ΠΎΠΈΠ·Π²Π΅Π΄Π΅Π½ΠΈΠ΅ [латСкс] \ overset {\ to} {A} \, Γ— \, \ overset {\ to} {B} [/ latex] для Π²Π΅ΠΊΡ‚ΠΎΡ€ΠΎΠ² [latex] \ overset {\ to} {A} = {A} _ {x} \ hat {i} + {A} _ {y} \ hat {j} + {A} _ {z} \ hat {k} [/ латСкс] ΠΈ [латСкс] \ overset {\ to} {B} = {B} _ {x} \ hat {i} + {B} _ {y} \ hat {j} + {B} _ {z} \ hat {k} [/ латСкс].ΠœΡ‹ ΠΌΠΎΠΆΠ΅ΠΌ ΠΈΡΠΏΠΎΠ»ΡŒΠ·ΠΎΠ²Π°Ρ‚ΡŒ свойство распрСдСлСния ((рисунок)), Π°Π½Ρ‚ΠΈΠΊΠΎΠΌΠΌΡƒΡ‚Π°Ρ‚ΠΈΠ²Π½ΠΎΠ΅ свойство ((рисунок)) ΠΈ Ρ€Π΅Π·ΡƒΠ»ΡŒΡ‚Π°Ρ‚Ρ‹ (рисунок) ΠΈ (рисунок) для Π΅Π΄ΠΈΠ½ΠΈΡ‡Π½Ρ‹Ρ… Π²Π΅ΠΊΡ‚ΠΎΡ€ΠΎΠ², Ρ‡Ρ‚ΠΎΠ±Ρ‹ Π²Ρ‹ΠΏΠΎΠ»Π½ΠΈΡ‚ΡŒ ΡΠ»Π΅Π΄ΡƒΡŽΡ‰ΡƒΡŽ Π°Π»Π³Π΅Π±Ρ€Ρƒ:

    [латСкс] \ begin {array} {ccc} \ hfill \ overset {\ to} {A} \, Γ— \, \ overset {\ to} {B} & = \ hfill & ({A} _ {x} \ hat {i} + {A} _ {y} \ hat {j} + {A} _ {z} \ hat {k}) \, Γ— \, ({B} _ {x} \ hat {i} + {B} _ {y} \ hat {j} + {B} _ {z} \ hat {k}) \ hfill \\ & = \ hfill & {A} _ {x} \ hat {i} \, Γ— \, ({B} _ {x} \ hat {i} + {B} _ {y} \ hat {j} + {B} _ {z} \ hat {k}) + {A} _ {y } \ hat {j} \, Γ— \, ({B} _ {x} \ hat {i} + {B} _ {y} \ hat {j} + {B} _ {z} \ hat {k} ) + {A} _ {z} \ hat {k} \, Γ— \, ({B} _ {x} \ hat {i} + {B} _ {y} \ hat {j} + {B} _ {z} \ hat {k}) \ hfill \\ & = \ hfill & \ enspace {A} _ {x} {B} _ {x} \ hat {i} \, Γ— \, \ hat {i} + {A} _ {x} {B} _ {y} \ hat {i} \, Γ— \, \ hat {j} + {A} _ {x} {B} _ {z} \ hat {i} \ , Γ— \, \ hat {k} \ hfill \\ & & + {A} _ {y} {B} _ {x} \ hat {j} \, Γ— \, \ hat {i} + {A} _ {y} {B} _ {y} \ hat {j} \, Γ— \, \ hat {j} + {A} _ {y} {B} _ {z} \ hat {j} \, Γ— \, \ hat {k} \ hfill \\ & & + {A} _ {z} {B} _ {x} \ hat {k} \, Γ— \, \ hat {i} + {A} _ {z} { B} _ {y} \ hat {k} \, Γ— \, \ hat {j} + {A} _ {z} {B} _ {z} \ hat {k} \, Γ— \, \ hat {k } \ hfill \\ & = \ hfill & \ enspace {A} _ {x} {B} _ {x} (0) + {A} _ {x} {B} _ {y} (+ \ hat {k }) + {A} _ {x} {B} _ {z} (\ text {-} \ hat {j}) \ hfill \\ & & + {A} _ {y} {B} _ {x} (\ text {-} \ hat {k}) + {A} _ {y} {B} _ {y} (0) + {A} _ {y} {B} _ {z} (+ \ hat { i}) \ hfill \\ & & + {A} _ {z} {B} _ {x} (+ \ hat {j} ) + {A} _ {z} {B} _ {y} (\ text {-} \ hat {i}) + {A} _ {z} {B} _ {z} (0).\ hfill \ end {array} [/ latex]

    ΠŸΡ€ΠΈ Π²Ρ‹ΠΏΠΎΠ»Π½Π΅Π½ΠΈΠΈ алгСбраичСских ΠΎΠΏΠ΅Ρ€Π°Ρ†ΠΈΠΉ с пСрСкрСстным ΠΏΡ€ΠΎΠΈΠ·Π²Π΅Π΄Π΅Π½ΠΈΠ΅ΠΌ Π±ΡƒΠ΄ΡŒΡ‚Π΅ ΠΎΡ‡Π΅Π½ΡŒ остороТны с соблюдСниСм ΠΏΡ€Π°Π²ΠΈΠ»ΡŒΠ½ΠΎΠ³ΠΎ порядка умноТСния, ΠΏΠΎΡ‚ΠΎΠΌΡƒ Ρ‡Ρ‚ΠΎ пСрСкрСстноС ΠΏΡ€ΠΎΠΈΠ·Π²Π΅Π΄Π΅Π½ΠΈΠ΅ Π°Π½Ρ‚ΠΈΠΊΠΎΠΌΠΌΡƒΡ‚Π°Ρ‚ΠΈΠ²Π½ΠΎ. ПослСдниС Π΄Π²Π° шага, ΠΊΠΎΡ‚ΠΎΡ€Ρ‹Π΅ Π½Π°ΠΌ Π΅Ρ‰Π΅ прСдстоит ΡΠ΄Π΅Π»Π°Ρ‚ΡŒ для выполнСния нашСй Π·Π°Π΄Π°Ρ‡ΠΈ, — это, Π²ΠΎ-ΠΏΠ΅Ρ€Π²Ρ‹Ρ…, Π³Ρ€ΡƒΠΏΠΏΠΈΡ€ΠΎΠ²ΠΊΠ° Ρ‚Π΅Ρ€ΠΌΠΈΠ½ΠΎΠ², содСрТащих ΠΎΠ±Ρ‰ΠΈΠΉ Π΅Π΄ΠΈΠ½ΠΈΡ‡Π½Ρ‹ΠΉ Π²Π΅ΠΊΡ‚ΠΎΡ€, ΠΈ, Π²ΠΎ-Π²Ρ‚ΠΎΡ€Ρ‹Ρ…, Ρ€Π°Π·Π»ΠΎΠΆΠ΅Π½ΠΈΠ΅ Π½Π° ΠΌΠ½ΠΎΠΆΠΈΡ‚Π΅Π»ΠΈ. Π’Π°ΠΊΠΈΠΌ ΠΎΠ±Ρ€Π°Π·ΠΎΠΌ, ΠΌΡ‹ ΠΏΠΎΠ»ΡƒΡ‡Π°Π΅ΠΌ ΡΠ»Π΅Π΄ΡƒΡŽΡ‰Π΅Π΅ ΠΎΡ‡Π΅Π½ΡŒ ΠΏΠΎΠ»Π΅Π·Π½ΠΎΠ΅ Π²Ρ‹Ρ€Π°ΠΆΠ΅Π½ΠΈΠ΅ для вычислСния пСрСкрСстного произвСдСния:

    [латСкс] \ overset {\ to} {C} = \ overset {\ to} {A} \, Γ— \, \ overset {\ to} {B} = ({A} _ {y} {B} _ {z} — {A} _ {z} {B} _ {y}) \ hat {i} + ({A} _ {z} {B} _ {x} — {A} _ {x} {B } _ {z}) \ hat {j} + ({A} _ {x} {B} _ {y} — {A} _ {y} {B} _ {x}) \ hat {k}.[/ латСкс]

    Π’ этом Π²Ρ‹Ρ€Π°ΠΆΠ΅Π½ΠΈΠΈ скалярныС ΠΊΠΎΠΌΠΏΠΎΠ½Π΅Π½Ρ‚Ρ‹ Π²Π΅ΠΊΡ‚ΠΎΡ€Π° пСрСкрСстного произвСдСния Ρ€Π°Π²Π½Ρ‹

    [латСкс] \ {\ begin {array} {c} {C} _ {x} = {A} _ {y} {B} _ {z} — {A} _ {z} {B} _ {y }, \\ {C} _ {y} = {A} _ {z} {B} _ {x} — {A} _ {x} {B} _ {z}, \\ {C} _ {z } = {A} _ {x} {B} _ {y} — {A} _ {y} {B} _ {x}. \ End {array} [/ latex]

    На ΠΏΡ€Π°ΠΊΡ‚ΠΈΠΊΠ΅ ΠΏΡ€ΠΈ Π½Π°Ρ…ΠΎΠΆΠ΄Π΅Π½ΠΈΠΈ пСрСкрСстного произвСдСния ΠΌΡ‹ ΠΌΠΎΠΆΠ΅ΠΌ ΠΈΡΠΏΠΎΠ»ΡŒΠ·ΠΎΠ²Π°Ρ‚ΡŒ Π»ΠΈΠ±ΠΎ (рисунок), Π»ΠΈΠ±ΠΎ (рисунок), Π² зависимости ΠΎΡ‚ Ρ‚ΠΎΠ³ΠΎ, ΠΊΠ°ΠΊΠΎΠ΅ ΠΈΠ· Π½ΠΈΡ… каТСтся ΠΌΠ΅Π½Π΅Π΅ слоТным Π² Π²Ρ‹Ρ‡ΠΈΡΠ»ΠΈΡ‚Π΅Π»ΡŒΠ½ΠΎΠΌ ΠΎΡ‚Π½ΠΎΡˆΠ΅Π½ΠΈΠΈ. Оба ΠΎΠ½ΠΈ приводят ΠΊ ΠΎΠ΄Π½ΠΎΠΌΡƒ ΠΈ Ρ‚ΠΎΠΌΡƒ ΠΆΠ΅ ΠΊΠΎΠ½Π΅Ρ‡Π½ΠΎΠΌΡƒ Ρ€Π΅Π·ΡƒΠ»ΡŒΡ‚Π°Ρ‚Ρƒ. Один ΠΈΠ· способов ΡƒΠ±Π΅Π΄ΠΈΡ‚ΡŒΡΡ, Ρ‡Ρ‚ΠΎ ΠΎΠΊΠΎΠ½Ρ‡Π°Ρ‚Π΅Π»ΡŒΠ½Ρ‹ΠΉ Ρ€Π΅Π·ΡƒΠ»ΡŒΡ‚Π°Ρ‚ Π²Π΅Ρ€Π΅Π½, — ΠΈΡΠΏΠΎΠ»ΡŒΠ·ΠΎΠ²Π°Ρ‚ΡŒ ΠΈΡ… ΠΎΠ±Π°.

    ΠŸΡ€ΠΈΠΌΠ΅Ρ€

    Частица Π² ΠΌΠ°Π³Π½ΠΈΡ‚Π½ΠΎΠΌ ΠΏΠΎΠ»Π΅

    ΠŸΡ€ΠΈ Π΄Π²ΠΈΠΆΠ΅Π½ΠΈΠΈ Π² ΠΌΠ°Π³Π½ΠΈΡ‚Π½ΠΎΠΌ ΠΏΠΎΠ»Π΅ Π½Π΅ΠΊΠΎΡ‚ΠΎΡ€Ρ‹Π΅ частицы ΠΌΠΎΠ³ΡƒΡ‚ ΠΈΡΠΏΡ‹Ρ‚Ρ‹Π²Π°Ρ‚ΡŒ ΠΌΠ°Π³Π½ΠΈΡ‚Π½ΡƒΡŽ силу. НС вдаваясь Π² подробности — ΠΏΠΎΠ΄Ρ€ΠΎΠ±Π½ΠΎΠ΅ ΠΈΠ·ΡƒΡ‡Π΅Π½ΠΈΠ΅ ΠΌΠ°Π³Π½ΠΈΡ‚Π½Ρ‹Ρ… явлСний Π±ΡƒΠ΄Π΅Ρ‚ Π² ΡΠ»Π΅Π΄ΡƒΡŽΡ‰ΠΈΡ… Π³Π»Π°Π²Π°Ρ… — Π΄Π°Π²Π°ΠΉΡ‚Π΅ ΠΏΡ€ΠΈΠ·Π½Π°Π΅ΠΌ, Ρ‡Ρ‚ΠΎ ΠΌΠ°Π³Π½ΠΈΡ‚Π½ΠΎΠ΅ ΠΏΠΎΠ»Π΅ [латСкс] \ overset {\ to} {B} [/ latex] являСтся Π²Π΅ΠΊΡ‚ΠΎΡ€ΠΎΠΌ, магнитная сила [латСкс] \ overset {\ to} {F} [/ latex] — это Π²Π΅ΠΊΡ‚ΠΎΡ€, Π° ΡΠΊΠΎΡ€ΠΎΡΡ‚ΡŒ [latex] \ overset {\ to} {u} [/ latex] частицы — это Π²Π΅ΠΊΡ‚ΠΎΡ€. Π’Π΅ΠΊΡ‚ΠΎΡ€ ΠΌΠ°Π³Π½ΠΈΡ‚Π½ΠΎΠΉ силы ΠΏΡ€ΠΎΠΏΠΎΡ€Ρ†ΠΈΠΎΠ½Π°Π»Π΅Π½ Π²Π΅ΠΊΡ‚ΠΎΡ€Π½ΠΎΠΌΡƒ ΠΏΡ€ΠΎΠΈΠ·Π²Π΅Π΄Π΅Π½ΠΈΡŽ Π²Π΅ΠΊΡ‚ΠΎΡ€Π° скорости Π½Π° Π²Π΅ΠΊΡ‚ΠΎΡ€ ΠΌΠ°Π³Π½ΠΈΡ‚Π½ΠΎΠ³ΠΎ поля, ΠΊΠΎΡ‚ΠΎΡ€ΠΎΠ΅ ΠΌΡ‹ Π²Ρ‹Ρ€Π°ΠΆΠ°Π΅ΠΌ ΠΊΠ°ΠΊ [latex] \ overset {\ to} {F} = \ zeta \ overset {\ to} {u} \, Γ— \, \ overset {\ to} {B} [/ латСкс].Π’ этом ΡƒΡ€Π°Π²Π½Π΅Π½ΠΈΠΈ константа [latex] \ zeta [/ latex] заботится ΠΎ согласованности физичСских Π΅Π΄ΠΈΠ½ΠΈΡ†, поэтому ΠΌΡ‹ ΠΌΠΎΠΆΠ΅ΠΌ ΠΎΠΏΡƒΡΠΊΠ°Ρ‚ΡŒ физичСскиС Π΅Π΄ΠΈΠ½ΠΈΡ†Ρ‹ Π² Π²Π΅ΠΊΡ‚ΠΎΡ€Π°Ρ… [latex] \ overset {\ to} {u} [/ latex] ΠΈ [ латСкс] \ overset {\ to} {B} [/ латСкс]. Π’ этом ΠΏΡ€ΠΈΠΌΠ΅Ρ€Π΅ ΠΏΡ€Π΅Π΄ΠΏΠΎΠ»ΠΎΠΆΠΈΠΌ, Ρ‡Ρ‚ΠΎ константа [latex] \ zeta [/ latex] ΠΏΠΎΠ»ΠΎΠΆΠΈΡ‚Π΅Π»ΡŒΠ½Π°.

    Частица, двиТущаяся Π² пространствС с Π²Π΅ΠΊΡ‚ΠΎΡ€ΠΎΠΌ скорости [latex] \ overset {\ to} {u} = — 5.0 \ hat {i} -2.0 \ hat {j} +3.5 \ hat {k} [/ latex], Π²Ρ…ΠΎΠ΄ΠΈΡ‚ Π² ΠΎΠ±Π»Π°ΡΡ‚ΡŒ с ΠΌΠ°Π³Π½ΠΈΡ‚Π½Ρ‹ΠΌ ΠΏΠΎΠ»Π΅ΠΌ ΠΈ испытываСт ΠΌΠ°Π³Π½ΠΈΡ‚Π½ΡƒΡŽ силу.НайдитС ΠΌΠ°Π³Π½ΠΈΡ‚Π½ΡƒΡŽ силу [латСкс] \ overset {\ to} {F} [/ latex] Π½Π° этой частицС Π² Ρ‚ΠΎΡ‡ΠΊΠ΅ Π²Ρ…ΠΎΠ΄Π° Π² ΠΎΠ±Π»Π°ΡΡ‚ΡŒ, Π³Π΄Π΅ Π²Π΅ΠΊΡ‚ΠΎΡ€ ΠΌΠ°Π³Π½ΠΈΡ‚Π½ΠΎΠ³ΠΎ поля Ρ€Π°Π²Π΅Π½ (a) [latex] \ overset {\ to} {B } = 7.2 \ hat {i} — \ hat {j} -2.4 \ hat {k} [/ latex] ΠΈ (b) [latex] \ overset {\ to} {B} = 4.5 \ hat {k} [/ латСкс]. Π’ ΠΊΠ°ΠΆΠ΄ΠΎΠΌ случаС Π½Π°ΠΉΠ΄ΠΈΡ‚Π΅ Π²Π΅Π»ΠΈΡ‡ΠΈΠ½Ρƒ F ΠΌΠ°Π³Π½ΠΈΡ‚Π½ΠΎΠΉ силы ΠΈ ΡƒΠ³ΠΎΠ» [latex] \ theta [/ latex], ΠΊΠΎΡ‚ΠΎΡ€Ρ‹ΠΉ Π²Π΅ΠΊΡ‚ΠΎΡ€ силы [latex] \ overset {\ to} {F} [/ latex] создаСт с Π·Π°Π΄Π°Π½Π½Ρ‹ΠΌ Π²Π΅ΠΊΡ‚ΠΎΡ€ΠΎΠΌ ΠΌΠ°Π³Π½ΠΈΡ‚Π½ΠΎΠ³ΠΎ поля. [латСкс] \ overset {\ to} {B} [/ латСкс].{2}} [/ latex], ΠΈΠ»ΠΈ вычислСниСм Π²Π΅Π»ΠΈΡ‡ΠΈΠ½Ρ‹ [latex] | \ overset {\ to} {u} \, Γ— \, \ overset {\ to} {B} | [/ latex] Π½Π°ΠΏΡ€ΡΠΌΡƒΡŽ с ΠΏΠΎΠΌΠΎΡ‰ΡŒΡŽ (рисунок). Π’ послСднСм ΠΏΠΎΠ΄Ρ…ΠΎΠ΄Π΅ Π½Π°ΠΌ Π½ΡƒΠΆΠ½ΠΎ Π±ΡƒΠ΄Π΅Ρ‚ Π½Π°ΠΉΡ‚ΠΈ ΡƒΠ³ΠΎΠ» ΠΌΠ΅ΠΆΠ΄Ρƒ Π²Π΅ΠΊΡ‚ΠΎΡ€Π°ΠΌΠΈ [latex] \ overset {\ to} {u} [/ latex] ΠΈ [latex] \ overset {\ to} {B} [/ latex]. Когда Ρƒ нас Π΅ΡΡ‚ΡŒ [latex] \ overset {\ to} {F} [/ latex], ΠΎΠ±Ρ‰ΠΈΠΉ ΠΌΠ΅Ρ‚ΠΎΠ΄ опрСдСлСния ΡƒΠ³Π»Π° направлСния [latex] \ theta [/ latex] Π²ΠΊΠ»ΡŽΡ‡Π°Π΅Ρ‚ вычислСниС скалярного произвСдСния [latex] \ overset { \ to} {F} Β· \ overset {\ to} {B} [/ latex] ΠΈ подставляСм Π² (рисунок).Для вычислСния Π²Π΅ΠΊΡ‚ΠΎΡ€Π½ΠΎΠ³ΠΎ произвСдСния ΠΌΡ‹ ΠΌΠΎΠΆΠ΅ΠΌ Π»ΠΈΠ±ΠΎ ΠΈΡΠΏΠΎΠ»ΡŒΠ·ΠΎΠ²Π°Ρ‚ΡŒ (рисунок), Π»ΠΈΠ±ΠΎ Π²Ρ‹Ρ‡ΠΈΡΠ»ΠΈΡ‚ΡŒ ΠΏΡ€ΠΎΠΈΠ·Π²Π΅Π΄Π΅Π½ΠΈΠ΅ Π½Π°ΠΏΡ€ΡΠΌΡƒΡŽ, Π² зависимости ΠΎΡ‚ Ρ‚ΠΎΠ³ΠΎ, Ρ‡Ρ‚ΠΎ Π±ΡƒΠ΄Π΅Ρ‚ ΠΏΡ€ΠΎΡ‰Π΅.

    РСшСниС
    ПокаТи ΠΎΡ‚Π²Π΅Ρ‚ ΠšΠΎΠΌΠΏΠΎΠ½Π΅Π½Ρ‚Ρ‹ Π²Π΅ΠΊΡ‚ΠΎΡ€Π° скорости: [латСкс] {u} _ {x} = — 5.0 [/ latex], [latex] {u} _ {y} = — 2.0 [/ latex] ΠΈ [latex] {u} _ {z} = 3,5 [/ латСкс].

    (a) ΠšΠΎΠΌΠΏΠΎΠ½Π΅Π½Ρ‚Π°ΠΌΠΈ Π²Π΅ΠΊΡ‚ΠΎΡ€Π° ΠΌΠ°Π³Π½ΠΈΡ‚Π½ΠΎΠ³ΠΎ поля ΡΠ²Π»ΡΡŽΡ‚ΡΡ [латСкс] {B} _ {x} = 7,2 [/ латСкс], [латСкс] {B} _ {y} = — 1,0 [/ латСкс] ΠΈ [латСкс ] {B} _ {z} = — 2,4 [/ латСкс]. ΠŸΠΎΠ΄ΡΡ‚Π°Π½ΠΎΠ²ΠΊΠ° ΠΈΡ… Π² (рисунок) Π΄Π°Π΅Ρ‚ скалярныС ΠΊΠΎΠΌΠΏΠΎΠ½Π΅Π½Ρ‚Ρ‹ Π²Π΅ΠΊΡ‚ΠΎΡ€Π° [latex] \ overset {\ to} {F} = \ zeta \ overset {\ to} {u} \, Γ— \, \ overset {\ to} {B} [/ latex]:

    [латСкс] \ {\ begin {array} {l} {F} _ {x} = \ zeta ({u} _ {y} {B} _ {z} — {u} _ {z} {B} _ {y}) = \ zeta [(-2.{2}} = 7.6, [/ latex] ΠΈ скалярноС ΠΏΡ€ΠΎΠΈΠ·Π²Π΅Π΄Π΅Π½ΠΈΠ΅ [latex] \ overset {\ to} {F} Β· \ overset {\ to} {B} [/ latex]:

    [латСкс] \ overset {\ to} {F} Β· \ overset {\ to} {B} = {F} _ {x} {B} _ {x} + {F} _ {y} {B} _ {y} + {F} _ {z} {B} _ {z} = (8.3 \ zeta) (7.2) + (13.2 \ zeta) (- 1.0) + (19.4 \ zeta) (- 2.4) = 0. [/ latex] Π’Π΅ΠΏΠ΅Ρ€ΡŒ Π·Π°ΠΌΠ΅Π½Π° Π² (Рисунок) Π΄Π°Π΅Ρ‚ ΡƒΠ³ΠΎΠ» [латСкс] \ theta [/ latex]:

    [латСкс] \ text {cos} \, \ theta = \ frac {\ overset {\ to} {F} Β· \ overset {\ to} {B}} {FB} = \ frac {0} {(18.2 \ zeta) (7.6)} = 0 \, β‡’ \ enspace \ theta = 90 \ text {Β°}. [/ латСкс]

    Π‘Π»Π΅Π΄ΠΎΠ²Π°Ρ‚Π΅Π»ΡŒΠ½ΠΎ, Π²Π΅ΠΊΡ‚ΠΎΡ€ ΠΌΠ°Π³Π½ΠΈΡ‚Π½ΠΎΠΉ силы пСрпСндикулярСн Π²Π΅ΠΊΡ‚ΠΎΡ€Ρƒ ΠΌΠ°Π³Π½ΠΈΡ‚Π½ΠΎΠ³ΠΎ поля.(ΠœΡ‹ ΠΌΠΎΠ³Π»ΠΈ Π±Ρ‹ ΡΡΠΊΠΎΠ½ΠΎΠΌΠΈΡ‚ΡŒ врСмя, Ссли Π±Ρ‹ вычислили скалярноС ΠΏΡ€ΠΎΠΈΠ·Π²Π΅Π΄Π΅Π½ΠΈΠ΅ Ρ€Π°Π½ΡŒΡˆΠ΅.)

    (b) ΠŸΠΎΡΠΊΠΎΠ»ΡŒΠΊΡƒ Π²Π΅ΠΊΡ‚ΠΎΡ€ [latex] \ overset {\ to} {B} = 4.5 \ hat {k} [/ latex] ΠΈΠΌΠ΅Π΅Ρ‚ Ρ‚ΠΎΠ»ΡŒΠΊΠΎ ΠΎΠ΄ΠΈΠ½ ΠΊΠΎΠΌΠΏΠΎΠ½Π΅Π½Ρ‚, ΠΌΡ‹ ΠΌΠΎΠΆΠ΅ΠΌ быстро Π²Ρ‹ΠΏΠΎΠ»Π½ΠΈΡ‚ΡŒ Π°Π»Π³Π΅Π±Ρ€Ρƒ ΠΈ Π½Π°ΠΉΡ‚ΠΈ Π²Π΅ΠΊΡ‚ΠΎΡ€Π½ΠΎΠ΅ ΠΏΡ€ΠΎΠΈΠ·Π²Π΅Π΄Π΅Π½ΠΈΠ΅ Π½Π°ΠΏΡ€ΡΠΌΡƒΡŽ:

    [латСкс] \ begin {array} {ll} \ hfill \ overset {\ to} {F} & = \ zeta \ overset {\ to} {u} \, Γ— \, \ overset {\ to} {B} = \ zeta (-5.0 \ hat {i} -2.0 \ hat {j} +3.5 \ hat {k}) \, Γ— \, (4.5 \ hat {k}) \ hfill \\ & = \ zeta [(- 5.0) (4.5) \ hat {i} \, Γ— \, \ hat {k} + (- 2.0) (4.5) \ hat {j} \, Γ— \, \ hat {k} + (3.{2}} = 24,2 \ Π΄Π·Π΅Ρ‚Π°. [/ latex] ΠŸΠΎΡΠΊΠΎΠ»ΡŒΠΊΡƒ скалярноС ΠΏΡ€ΠΎΠΈΠ·Π²Π΅Π΄Π΅Π½ΠΈΠ΅ Ρ€Π°Π²Π½ΠΎ

    [латСкс] \ overset {\ to} {F} Β· \ overset {\ to} {B} = {F} _ {x} {B} _ {x} + {F} _ {y} {B} _ {y} + {F} _ {z} {B} _ {z} = (- 9.0 \ zeta) (0) + (22.5 \ zeta) (0) + (0) (4.5) = 0, [/ латСкс ] Π²Π΅ΠΊΡ‚ΠΎΡ€ ΠΌΠ°Π³Π½ΠΈΡ‚Π½ΠΎΠΉ силы [латСкс] \ overset {\ to} {F} [/ latex] пСрпСндикулярСн Π²Π΅ΠΊΡ‚ΠΎΡ€Ρƒ ΠΌΠ°Π³Π½ΠΈΡ‚Π½ΠΎΠ³ΠΎ поля [латСкс] \ overset {\ to} {B} [/ latex].

    Π—Π½Π°Ρ‡Π΅Π½ΠΈΠ΅

    Π”Π°ΠΆΠ΅ Π±Π΅Π· фактичСского вычислСния скалярного произвСдСния ΠΌΡ‹ ΠΌΠΎΠΆΠ΅ΠΌ ΠΏΡ€Π΅Π΄ΡΠΊΠ°Π·Π°Ρ‚ΡŒ, Ρ‡Ρ‚ΠΎ Π²Π΅ΠΊΡ‚ΠΎΡ€ ΠΌΠ°Π³Π½ΠΈΡ‚Π½ΠΎΠΉ силы всСгда Π΄ΠΎΠ»ΠΆΠ΅Π½ Π±Ρ‹Ρ‚ΡŒ пСрпСндикулярСн Π²Π΅ΠΊΡ‚ΠΎΡ€Ρƒ ΠΌΠ°Π³Π½ΠΈΡ‚Π½ΠΎΠ³ΠΎ поля ΠΈΠ·-Π·Π° способа построСния этого Π²Π΅ΠΊΡ‚ΠΎΡ€Π°.А ΠΈΠΌΠ΅Π½Π½ΠΎ, Π²Π΅ΠΊΡ‚ΠΎΡ€ ΠΌΠ°Π³Π½ΠΈΡ‚Π½ΠΎΠΉ силы — это Π²Π΅ΠΊΡ‚ΠΎΡ€Π½ΠΎΠ΅ ΠΏΡ€ΠΎΠΈΠ·Π²Π΅Π΄Π΅Π½ΠΈΠ΅ [латСкс] \ overset {\ to} {F} = \ zeta \ overset {\ to} {u} \, Γ— \, \ overset {\ to} {B} [/ латСкс] ΠΈ, ΠΏΠΎ ΠΎΠΏΡ€Π΅Π΄Π΅Π»Π΅Π½ΠΈΡŽ Π²Π΅ΠΊΡ‚ΠΎΡ€Π½ΠΎΠ³ΠΎ произвСдСния (см. (рисунок)), Π²Π΅ΠΊΡ‚ΠΎΡ€ [латСкс] \ overset {\ to} {F} [/ latex] Π΄ΠΎΠ»ΠΆΠ΅Π½ Π±Ρ‹Ρ‚ΡŒ пСрпСндикулярСн ΠΎΠ±ΠΎΠΈΠΌ Π²Π΅ΠΊΡ‚ΠΎΡ€Π°ΠΌ [латСкс] \ overset {\ to} {u} [/ latex] ΠΈ [latex] \ overset {\ to} {B} [/ latex].

    ΠŸΡ€ΠΎΠ²Π΅Ρ€ΡŒΡ‚Π΅ своС ΠΏΠΎΠ½ΠΈΠΌΠ°Π½ΠΈΠ΅

    Π”Π°Π½Ρ‹ Π΄Π²Π° Π²Π΅ΠΊΡ‚ΠΎΡ€Π° [латСкс] \ overset {\ to} {A} = \ text {-} \ hat {i} + \ hat {j} [/ latex] ΠΈ [latex] \ overset {\ to} {B} = 3 \ hat {i} — \ hat {j} [/ latex], Π½Π°ΠΉΠ΄ΠΈΡ‚Π΅ (a) [latex] \ overset {\ to} {A} \, Γ— \, \ overset {\ to} {B} [/ латСкс], (b) [латСкс] | \ overset {\ to} {A} \, Γ— \, \ overset {\ to} {B} | [/ latex], (c) ΡƒΠ³ΠΎΠ» ΠΌΠ΅ΠΆΠ΄Ρƒ [латСксом] \ overset {\ to} {A} [/ latex] ΠΈ [latex] \ overset {\ to} {B} [/ latex], ΠΈ (d) ΡƒΠ³ΠΎΠ» ΠΌΠ΅ΠΆΠ΄Ρƒ [латСксом] \ overset {\ to} {A} \, Γ— \, \ overset {\ to} {B} [/ latex] ΠΈ Π²Π΅ΠΊΡ‚ΠΎΡ€ΠΎΠΌ [латСксом] \ overset {\ to} {C} = \ hat { i} + \ hat {k} [/ латСкс].

    ΠŸΠΎΠΊΠ°Π·Π°Ρ‚ΡŒ Ρ€Π΅ΡˆΠ΅Π½ΠΈΠ΅

    Π°. [латСкс] -2 \ шляпа {k} [/ латСкс], Π±. 2, Π². [латСкс] 153.4 \ text {Β°} [/ латСкс], d. [латСкс] 135 \ text {Β°} [/ латСкс]

    Π’ Π·Π°ΠΊΠ»ΡŽΡ‡Π΅Π½ΠΈΠ΅ этого Ρ€Π°Π·Π΄Π΅Π»Π° ΠΌΡ‹ Ρ…ΠΎΡ‚ΠΈΠΌ ΠΏΠΎΠ΄Ρ‡Π΅Ρ€ΠΊΠ½ΡƒΡ‚ΡŒ, Ρ‡Ρ‚ΠΎ «скалярноС ΠΏΡ€ΠΎΠΈΠ·Π²Π΅Π΄Π΅Π½ΠΈΠ΅Β» ΠΈ «пСрСкрСстноС ΠΏΡ€ΠΎΠΈΠ·Π²Π΅Π΄Π΅Π½ΠΈΠ΅Β» — это ΡΠΎΠ²Π΅Ρ€ΡˆΠ΅Π½Π½ΠΎ Ρ€Π°Π·Π½Ρ‹Π΅ матСматичСскиС ΠΎΠ±ΡŠΠ΅ΠΊΡ‚Ρ‹, ΠΊΠΎΡ‚ΠΎΡ€Ρ‹Π΅ ΠΈΠΌΠ΅ΡŽΡ‚ Ρ€Π°Π·Π½ΠΎΠ΅ Π·Π½Π°Ρ‡Π΅Π½ΠΈΠ΅. БкалярноС ΠΏΡ€ΠΎΠΈΠ·Π²Π΅Π΄Π΅Π½ΠΈΠ΅ — это скаляр; пСрСкрСстноС ΠΏΡ€ΠΎΠΈΠ·Π²Π΅Π΄Π΅Π½ΠΈΠ΅ — это Π²Π΅ΠΊΡ‚ΠΎΡ€. Π’ ΠΏΠΎΡΠ»Π΅Π΄ΡƒΡŽΡ‰ΠΈΡ… Π³Π»Π°Π²Π°Ρ… Ρ‚Π΅Ρ€ΠΌΠΈΠ½Ρ‹ скалярноС ΠΏΡ€ΠΎΠΈΠ·Π²Π΅Π΄Π΅Π½ΠΈΠ΅ ΠΈ скалярноС ΠΏΡ€ΠΎΠΈΠ·Π²Π΅Π΄Π΅Π½ΠΈΠ΅ ΠΈΡΠΏΠΎΠ»ΡŒΠ·ΡƒΡŽΡ‚ΡΡ ΠΊΠ°ΠΊ взаимозамСняСмыС. Π’ΠΎΡ‡Π½ΠΎ Ρ‚Π°ΠΊ ΠΆΠ΅ Ρ‚Π΅Ρ€ΠΌΠΈΠ½Ρ‹ пСрСкрСстноС ΠΏΡ€ΠΎΠΈΠ·Π²Π΅Π΄Π΅Π½ΠΈΠ΅ ΠΈ Π²Π΅ΠΊΡ‚ΠΎΡ€Π½ΠΎΠ΅ ΠΏΡ€ΠΎΠΈΠ·Π²Π΅Π΄Π΅Π½ΠΈΠ΅ ΠΈΡΠΏΠΎΠ»ΡŒΠ·ΡƒΡŽΡ‚ΡΡ взаимозамСняСмо.

    Π’Π•ΠšΠ’ΠžΠ ΠΠ«Π• ΠœΠ•Π’ΠžΠ”Π«

    Π’Π•ΠšΠ’ΠžΠ ΠΠ«Π• ΠœΠ•Π’ΠžΠ”Π«

    ΠŸΡ€ΠΈΠΎΡ€ΠΈΡ‚Π΅Ρ‚Π½Ρ‹Π΅ направлСния:

    1. Π’Π΅ΠΊΡ‚ΠΎΡ€Ρ‹ ΠΈ слоТСниС Π²Π΅ΠΊΡ‚ΠΎΡ€ΠΎΠ²
    2. Π•Π΄ΠΈΠ½ΠΈΡ‡Π½Ρ‹Π΅ Π²Π΅ΠΊΡ‚ΠΎΡ€Ρ‹
    3. Π‘Π°Π·ΠΎΠ²Ρ‹Π΅ Π²Π΅ΠΊΡ‚ΠΎΡ€Ρ‹ ΠΈ ΠΊΠΎΠΌΠΏΠΎΠ½Π΅Π½Ρ‚Ρ‹ Π²Π΅ΠΊΡ‚ΠΎΡ€Π°
    4. ΠŸΡ€ΡΠΌΠΎΡƒΠ³ΠΎΠ»ΡŒΠ½Ρ‹Π΅ ΠΊΠΎΠΎΡ€Π΄ΠΈΠ½Π°Ρ‚Ρ‹ Π² 2-D
    5. ΠŸΡ€ΡΠΌΠΎΡƒΠ³ΠΎΠ»ΡŒΠ½Ρ‹Π΅ ΠΊΠΎΠΎΡ€Π΄ΠΈΠ½Π°Ρ‚Ρ‹ Π² 3-D
    6. А Π²Π΅ΠΊΡ‚ΠΎΡ€, ΡΠΎΠ΅Π΄ΠΈΠ½ΡΡŽΡ‰ΠΈΠΉ Π΄Π²Π΅ Ρ‚ΠΎΡ‡ΠΊΠΈ
    7. Π’ΠΎΡ‡Π΅Ρ‡Π½Ρ‹ΠΉ ΠΏΡ€ΠΎΠ΄ΡƒΠΊΡ‚
    8. ΠŸΠ΅Ρ€Π΅ΠΊΡ€Π΅ΡΡ‚Π½ΠΎΠ΅ ΠΏΡ€ΠΎΠΈΠ·Π²Π΅Π΄Π΅Π½ΠΈΠ΅
    9. Π’Ρ€ΠΎΠΉΠ½ΠΎΠΉ ΠΏΡ€ΠΎΠ΄ΡƒΠΊΡ‚
    10. Π’Ρ€ΠΎΠΉΠ½ΠΎΠ΅ Π²Π΅ΠΊΡ‚ΠΎΡ€Π½ΠΎΠ΅ ΠΏΡ€ΠΎΠΈΠ·Π²Π΅Π΄Π΅Π½ΠΈΠ΅

    Π’Π΅ΠΊΡ‚ΠΎΡ€Ρ‹ ΠΈ слоТСниС Π²Π΅ΠΊΡ‚ΠΎΡ€Π°:

    Бкаляр — это Π²Π΅Π»ΠΈΡ‡ΠΈΠ½Π°, подобная массС ΠΈΠ»ΠΈ Ρ‚Π΅ΠΌΠΏΠ΅Ρ€Π°Ρ‚ΡƒΡ€Π΅, ΠΈΠΌΠ΅ΡŽΡ‰Π°Ρ Ρ‚ΠΎΠ»ΡŒΠΊΠΎ Π²Π΅Π»ΠΈΡ‡ΠΈΠ½Π°.» Π½Π° ΠΆΠΈΡ€Π½ΠΎΠΌ символС (Ρ‚.Π΅., ). Π‘Π»Π΅Π΄ΠΎΠ²Π°Ρ‚Π΅Π»ΡŒΠ½ΠΎ,

    Π›ΡŽΠ±ΠΎΠΉ Π²Π΅ΠΊΡ‚ΠΎΡ€ ΠΌΠΎΠΆΠ½ΠΎ ΠΏΡ€Π΅Π²Ρ€Π°Ρ‚ΠΈΡ‚ΡŒ Π² Π΅Π΄ΠΈΠ½ΠΈΡ‡Π½Ρ‹ΠΉ Π²Π΅ΠΊΡ‚ΠΎΡ€, Ρ€Π°Π·Π΄Π΅Π»ΠΈΠ² Π΅Π³ΠΎ Π½Π° Π΄Π»ΠΈΠ½Ρƒ.

    Π›ΡŽΠ±ΠΎΠΉ Π²Π΅ΠΊΡ‚ΠΎΡ€ ΠΌΠΎΠΆΠ½ΠΎ ΠΏΠΎΠ»Π½ΠΎΡΡ‚ΡŒΡŽ ΠΏΡ€Π΅Π΄ΡΡ‚Π°Π²ΠΈΡ‚ΡŒ, ΡƒΠΊΠ°Π·Π°Π² Π΅Π³ΠΎ Π²Π΅Π»ΠΈΡ‡ΠΈΠ½Ρƒ ΠΈ Π΅Π΄ΠΈΠ½ΠΈΡ†Ρƒ. Π²Π΅ΠΊΡ‚ΠΎΡ€ ΠΏΠΎ Π΅Π³ΠΎ Π½Π°ΠΏΡ€Π°Π²Π»Π΅Π½ΠΈΡŽ.

    Π‘Π°Π·Π° Π²Π΅ΠΊΡ‚ΠΎΡ€Ρ‹ ΠΈ ΠΊΠΎΠΌΠΏΠΎΠ½Π΅Π½Ρ‚Ρ‹ Π²Π΅ΠΊΡ‚ΠΎΡ€Π°:

    Π‘Π°Π·ΠΎΠ²Ρ‹Π΅ Π²Π΅ΠΊΡ‚ΠΎΡ€Ρ‹ — это Π½Π°Π±ΠΎΡ€ Π²Π΅ΠΊΡ‚ΠΎΡ€ΠΎΠ², Π²Ρ‹Π±Ρ€Π°Π½Π½Ρ‹Ρ… Π² качСствС Π±Π°Π·ΠΎΠ²Ρ‹Ρ… для прСдставлСния всСх Π΄Ρ€ΡƒΠ³ΠΈΠ΅ Π²Π΅ΠΊΡ‚ΠΎΡ€Ρ‹.ИдСя состоит Π² Ρ‚ΠΎΠΌ, Ρ‡Ρ‚ΠΎΠ±Ρ‹ ΠΏΠΎΡΡ‚Ρ€ΠΎΠΈΡ‚ΡŒ ΠΊΠ°ΠΆΠ΄Ρ‹ΠΉ Π²Π΅ΠΊΡ‚ΠΎΡ€ ΠΈΠ· добавлСния Π²Π΅ΠΊΡ‚ΠΎΡ€Ρ‹ ΠΏΠΎ Π±Π°Π·ΠΎΠ²Ρ‹ΠΌ направлСниям. НапримСр, Π²Π΅ΠΊΡ‚ΠΎΡ€ Π½Π° рисункС ΠΌΠΎΠΆΠ΅Ρ‚ ΠΌΠΎΠΆΠ½ΠΎ Π·Π°ΠΏΠΈΡΠ°Ρ‚ΡŒ ΠΊΠ°ΠΊ сумму Ρ‚Ρ€Π΅Ρ… Π²Π΅ΠΊΡ‚ΠΎΡ€ΠΎΠ² u 1 , u 2 , ΠΈ u 3 , ΠΊΠ°ΠΆΠ΄Ρ‹ΠΉ ΠΏΠΎ Π½Π°ΠΏΡ€Π°Π²Π»Π΅Π½ΠΈΡŽ ΠΎΠ΄Π½ΠΎΠ³ΠΎ ΠΈΠ· оснований Π²Π΅ΠΊΡ‚ΠΎΡ€Ρ‹ e 1 , e 2 ΠΈ e 3 , Ρ‚Π°ΠΊ Ρ‡Ρ‚ΠΎ

    ΠšΠ°ΠΆΠ΄Ρ‹ΠΉ ΠΈΠ· Π²Π΅ΠΊΡ‚ΠΎΡ€ΠΎΠ² u 1 , u 2 , ΠΈ u 3 ΠΏΠ°Ρ€Π°Π»Π»Π΅Π»ΡŒΠ½ΠΎ ΠΎΠ΄Π½ΠΎΠΌΡƒ ΠΈΠ· Π±Π°Π·ΠΎΠ²Ρ‹Ρ… Π²Π΅ΠΊΡ‚ΠΎΡ€ΠΎΠ² ΠΈ ΠΌΠΎΠΆΠ΅Ρ‚ Π±Ρ‹Ρ‚ΡŒ записываСтся ΠΊΠ°ΠΊ скалярноС ΠΊΡ€Π°Ρ‚Π½ΠΎΠ΅ этой Π±Π°Π·Π΅.ΠŸΡƒΡΡ‚ΡŒ u 1 , u 2 , ΠΈ u 3 ΠΎΠ±ΠΎΠ·Π½Π°Ρ‡Π°ΡŽΡ‚ эти скалярныС ΠΌΠ½ΠΎΠΆΠΈΡ‚Π΅Π»ΠΈ, Ρ‚Π°ΠΊ Ρ‡Ρ‚ΠΎ получаСтся

    ΠžΡ€ΠΈΠ³ΠΈΠ½Π°Π»ΡŒΠ½Ρ‹ΠΉ Π²Π΅ΠΊΡ‚ΠΎΡ€ u Π±Π°Π½ΠΊΠ° Ρ‚Π΅ΠΏΠ΅Ρ€ΡŒ записываСтся ΠΊΠ°ΠΊ

    БкалярныС ΠΌΠ½ΠΎΠΆΠΈΡ‚Π΅Π»ΠΈ u 1 , u 2 ΠΈ u 3 извСстны ΠΊΠ°ΠΊ ΠΊΠΎΠΌΠΏΠΎΠ½Π΅Π½Ρ‚Ρ‹ u Π² Π±Π°Π·Π΅ описываСтся Π±Π°Π·ΠΎΠ²Ρ‹ΠΌΠΈ Π²Π΅ΠΊΡ‚ΠΎΡ€Π°ΠΌΠΈ e 1 , e 2 , ΠΈ e 3 .Если Π±Π°Π·ΠΎΠ²Ρ‹Π΅ Π²Π΅ΠΊΡ‚ΠΎΡ€Ρ‹ ΡΠ²Π»ΡΡŽΡ‚ΡΡ Π΅Π΄ΠΈΠ½ΠΈΡ‡Π½Ρ‹ΠΌΠΈ Π²Π΅ΠΊΡ‚ΠΎΡ€Π°ΠΌΠΈ, Ρ‚ΠΎ ΠΊΠΎΠΌΠΏΠΎΠ½Π΅Π½Ρ‚Ρ‹ ΠΏΡ€Π΅Π΄ΡΡ‚Π°Π²Π»ΡΡŽΡ‚ собой соотвСтствСнно Π΄Π»ΠΈΠ½Ρ‹ Ρ‚Ρ€Π΅Ρ… Π²Π΅ΠΊΡ‚ΠΎΡ€ΠΎΠ² u 1 , u 2 ΠΈ u 3 . Если Π±Π°Π·ΠΎΠ²Ρ‹Π΅ Π²Π΅ΠΊΡ‚ΠΎΡ€Ρ‹ Π΅Π΄ΠΈΠ½ΠΈΡ‡Π½Ρ‹ Π²Π΅ΠΊΡ‚ΠΎΡ€ΠΎΠ² ΠΈ Π²Π·Π°ΠΈΠΌΠ½ΠΎ ΠΎΡ€Ρ‚ΠΎΠ³ΠΎΠ½Π°Π»ΡŒΠ½Ρ‹, Ρ‚ΠΎ основаниС извСстно ΠΊΠ°ΠΊ ΠΎΡ€Ρ‚ΠΎΠ½ΠΎΡ€ΠΌΠΈΡ€ΠΎΠ²Π°Π½Π½ΠΎΠ΅, Π΅Π²ΠΊΠ»ΠΈΠ΄ΠΎΠ²ΠΎ ΠΈΠ»ΠΈ Π΄Π΅ΠΊΠ°Ρ€Ρ‚ΠΎΠ²ΠΎ основаниС.

    Π’Π΅ΠΊΡ‚ΠΎΡ€ ΠΌΠΎΠΆΠ΅Ρ‚ Π±Ρ‹Ρ‚ΡŒ Ρ€Π°Π·Ρ€Π΅ΡˆΠ΅Π½ ΠΏΠΎ Π»ΡŽΠ±Ρ‹ΠΌ Π΄Π²ΡƒΠΌ направлСниям Π² плоскости, содСрТащСй Π­Ρ‚ΠΎ. На рисункС ΠΏΠΎΠΊΠ°Π·Π°Π½ΠΎ, ΠΊΠ°ΠΊ ΠΏΡ€Π°Π²ΠΈΠ»ΠΎ ΠΏΠ°Ρ€Π°Π»Π»Π΅Π»ΠΎΠ³Ρ€Π°ΠΌΠΌΠ° ΠΈΡΠΏΠΎΠ»ΡŒΠ·ΡƒΠ΅Ρ‚ΡΡ для построСния Π²Π΅ΠΊΡ‚ΠΎΡ€ΠΎΠ². a ΠΈ b , Ρ‡Ρ‚ΠΎ Π² суммС Π΄Π°Π΅Ρ‚ c .

    Π’ Ρ‚Ρ€Π΅Ρ… измСрСниях Π²Π΅ΠΊΡ‚ΠΎΡ€ ΠΌΠΎΠΆΠ΅Ρ‚ Π±Ρ‹Ρ‚ΡŒ Ρ€Π°Π·Ρ€Π΅ΡˆΠ΅Π½ ΠΏΠΎ Π»ΡŽΠ±Ρ‹ΠΌ Ρ‚Ρ€Π΅ΠΌ направлСниям. Π½Π΅ΠΊΠΎΠΏΠ»Π°Π½Π°Ρ€Π½Ρ‹Π΅ Π»ΠΈΠ½ΠΈΠΈ. На рисункС ΠΏΠΎΠΊΠ°Π·Π°Π½ΠΎ, ΠΊΠ°ΠΊ ΠΌΠΎΠΆΠ½ΠΎ Ρ€Π°Π·Ρ€Π΅ΡˆΠΈΡ‚ΡŒ Π²Π΅ΠΊΡ‚ΠΎΡ€ вдоль Π² Ρ‚Ρ€Π΅Ρ… направлСниях, сначала найдя Π²Π΅ΠΊΡ‚ΠΎΡ€ Π² плоскости Π΄Π²ΡƒΡ… ΠΈΠ· направлСниях, Π° Π·Π°Ρ‚Π΅ΠΌ Ρ€Π°Π·Ρ€Π΅ΡˆΠΈΠ² этот Π½ΠΎΠ²Ρ‹ΠΉ Π²Π΅ΠΊΡ‚ΠΎΡ€ ΠΏΠΎ Π΄Π²ΡƒΠΌ направлСниям Π² самолСт.

    Когда Π²Π΅ΠΊΡ‚ΠΎΡ€Ρ‹ прСдставлСны Π² Ρ‚Π΅Ρ€ΠΌΠΈΠ½Π°Ρ… Π±Π°Π·ΠΎΠ²Ρ‹Ρ… Π²Π΅ΠΊΡ‚ΠΎΡ€ΠΎΠ² ΠΈ ΠΊΠΎΠΌΠΏΠΎΠ½Π΅Π½Ρ‚ΠΎΠ², слоТСниС Π΄Π²ΡƒΡ… Π²Π΅ΠΊΡ‚ΠΎΡ€ΠΎΠ² ΠΏΡ€ΠΈΠ²ΠΎΠ΄ΠΈΡ‚ ΠΊ слоТСнию ΠΊΠΎΠΌΠΏΠΎΠ½Π΅Π½Ρ‚ΠΎΠ² Π²Π΅ΠΊΡ‚ΠΎΡ€Ρ‹.Π‘Π»Π΅Π΄ΠΎΠ²Π°Ρ‚Π΅Π»ΡŒΠ½ΠΎ, Ссли прСдставлСны Π΄Π²Π° Π²Π΅ΠΊΡ‚ΠΎΡ€Π° A ΠΈ B ΠΏΠΎ

    Ρ‚ΠΎΠ³Π΄Π°,

    ΠŸΡ€ΡΠΌΠΎΡƒΠ³ΠΎΠ»ΡŒΠ½Ρ‹Π΅ ΠΊΠΎΠΌΠΏΠΎΠ½Π΅Π½Ρ‚Ρ‹ Π² 2-D:

    Π”Π°Π½Ρ‹ Π±Π°Π·ΠΎΠ²Ρ‹Π΅ Π²Π΅ΠΊΡ‚ΠΎΡ€Ρ‹ ΠΏΡ€ΡΠΌΠΎΡƒΠ³ΠΎΠ»ΡŒΠ½ΠΎΠΉ систСмы ΠΊΠΎΠΎΡ€Π΄ΠΈΠ½Π°Ρ‚ x-y . Π΅Π΄ΠΈΠ½ΠΈΡ‡Π½Ρ‹ΠΌΠΈ Π²Π΅ΠΊΡ‚ΠΎΡ€Π°ΠΌΠΈ Π° Ρ‚Π°ΠΊΠΆΠ΅ вдоль направлСниях x ΠΈ y соотвСтствСнно.

    Π˜ΡΠΏΠΎΠ»ΡŒΠ·ΡƒΡ Π±Π°Π·ΠΎΠ²Ρ‹Π΅ Π²Π΅ΠΊΡ‚ΠΎΡ€Ρ‹, ΠΌΠΎΠΆΠ½ΠΎ ΠΏΡ€Π΅Π΄ΡΡ‚Π°Π²ΠΈΡ‚ΡŒ любой Π²Π΅ΠΊΡ‚ΠΎΡ€ F ΠΊΠ°ΠΊ

    Π’ силу ΠΎΡ€Ρ‚ΠΎΠ³ΠΎΠ½Π°Π»ΡŒΠ½ΠΎΡΡ‚ΠΈ базисов ΠΈΠΌΠ΅ΡŽΡ‚ мСсто ΡΠ»Π΅Π΄ΡƒΡŽΡ‰ΠΈΠ΅ ΡΠΎΠΎΡ‚Π½ΠΎΡˆΠ΅Π½ΠΈΡ.

    ΠŸΡ€ΡΠΌΠΎΡƒΠ³ΠΎΠ»ΡŒΠ½Ρ‹Π΅ ΠΊΠΎΠΎΡ€Π΄ΠΈΠ½Π°Ρ‚Ρ‹ Π² 3-D:

    Π‘Π°Π·ΠΎΠ²Ρ‹Π΅ Π²Π΅ΠΊΡ‚ΠΎΡ€Ρ‹ ΠΏΡ€ΡΠΌΠΎΡƒΠ³ΠΎΠ»ΡŒΠ½ΠΎΠΉ систСмы ΠΊΠΎΠΎΡ€Π΄ΠΈΠ½Π°Ρ‚ Π·Π°Π΄Π°ΡŽΡ‚ΡΡ Π½Π°Π±ΠΎΡ€ΠΎΠΌ Ρ‚Ρ€ΠΈ Π²Π·Π°ΠΈΠΌΠ½ΠΎ ΠΎΡ€Ρ‚ΠΎΠ³ΠΎΠ½Π°Π»ΡŒΠ½Ρ‹Ρ… Π΅Π΄ΠΈΠ½ΠΈΡ‡Π½Ρ‹Ρ… Π²Π΅ΠΊΡ‚ΠΎΡ€Π°, ΠΎΠ±ΠΎΠ·Π½Π°Ρ‡Π°Π΅ΠΌΡ‹Ρ… , , Π° Ρ‚Π°ΠΊΠΆΠ΅ Ρ‡Ρ‚ΠΎ располоТСны вдоль ΠΊΠΎΠΎΡ€Π΄ΠΈΠ½Π°Ρ‚Π½Ρ‹Ρ… Π½Π°ΠΏΡ€Π°Π²Π»Π΅Π½ΠΈΠΉ x , y ΠΈ z , соотвСтствСнно, ΠΊΠ°ΠΊ ΠΏΠΎΠΊΠ°Π·Π°Π½ΠΎ Π½Π° рисункС.

    Показанная систСма являСтся систСмой для ΠΏΡ€Π°Π²ΠΎΠΉ Ρ€ΡƒΠΊΠΈ, ΠΏΠΎΡΠΊΠΎΠ»ΡŒΠΊΡƒ большой ΠΏΠ°Π»Π΅Ρ† ΠΏΡ€Π°Π²ΠΎΠΉ Ρ€ΡƒΠΊΠΈ Ρ€ΡƒΠΊΠ° ΡƒΠΊΠ°Π·Ρ‹Π²Π°Π΅Ρ‚ Π² Π½Π°ΠΏΡ€Π°Π²Π»Π΅Π½ΠΈΠΈ z , Ссли ΠΏΠ°Π»ΡŒΡ†Ρ‹ Ρ‚Π°ΠΊΠΎΠ²Ρ‹, Ρ‡Ρ‚ΠΎ ΠΎΠ½ΠΈ ΠΏΡ€Π΅Π΄ΡΡ‚Π°Π²Π»ΡΡŽΡ‚ Π²Ρ€Π°Ρ‰Π΅Π½ΠΈΠ΅ Π²ΠΎΠΊΡ€ΡƒΠ³ оси z ΠΎΡ‚ x Π΄ΠΎ y . Π­Ρ‚Ρƒ систСму ΠΌΠΎΠΆΠ½ΠΎ ΠΏΡ€Π΅Π²Ρ€Π°Ρ‚ΠΈΡ‚ΡŒ Π² Π»Π΅Π²ΡƒΡŽ, ΠΏΠ΅Ρ€Π΅Π²Π΅Ρ€Π½ΡƒΠ² Π½Π°ΠΏΡ€Π°Π²Π»Π΅Π½ΠΈΠ΅ любой ΠΈΠ· ΠΊΠΎΠΎΡ€Π΄ΠΈΠ½Π°Ρ‚Π½Ρ‹Ρ… Π»ΠΈΠ½ΠΈΠΉ ΠΈ связанный с Π½Π΅ΠΉ Π±Π°Π·ΠΎΠ²Ρ‹ΠΉ Π²Π΅ΠΊΡ‚ΠΎΡ€.

    Π’ ΠΏΡ€ΡΠΌΠΎΡƒΠ³ΠΎΠ»ΡŒΠ½ΠΎΠΉ систСмС ΠΊΠΎΠΎΡ€Π΄ΠΈΠ½Π°Ρ‚ ΠΊΠΎΠΌΠΏΠΎΠ½Π΅Π½Ρ‚Π°ΠΌΠΈ Π²Π΅ΠΊΡ‚ΠΎΡ€Π° ΡΠ²Π»ΡΡŽΡ‚ΡΡ ΠΏΡ€ΠΎΠ΅ΠΊΡ†ΠΈΠΈ Π²Π΅ΠΊΡ‚ΠΎΡ€Π° вдоль x , y ΠΈ z направлСния. НапримСр, Π½Π° рисункС ΠΏΡ€ΠΎΠ΅ΠΊΡ†ΠΈΠΈ Π²Π΅ΠΊΡ‚ΠΎΡ€Π° A вдоль Π½Π°ΠΏΡ€Π°Π²Π»Π΅Π½ΠΈΠΉ x, y, ΠΈ z Π·Π°Π΄Π°ΡŽΡ‚ΡΡ ΠΊΠ°ΠΊ A x , A y , ΠΈ A z соотвСтствСнно.

    Π’ Ρ€Π΅Π·ΡƒΠ»ΡŒΡ‚Π°Ρ‚Π΅ Ρ‚Π΅ΠΎΡ€Π΅ΠΌΡ‹ ΠŸΠΈΡ„Π°Π³ΠΎΡ€Π° ΠΈ ΠΎΡ€Ρ‚ΠΎΠ³ΠΎΠ½Π°Π»ΡŒΠ½ΠΎΡΡ‚ΠΈ Π±Π°Π·Ρ‹ Π²Π΅ΠΊΡ‚ΠΎΡ€ΠΎΠ², Π²Π΅Π»ΠΈΡ‡ΠΈΠ½Π° Π²Π΅ΠΊΡ‚ΠΎΡ€Π° Π² ΠΏΡ€ΡΠΌΠΎΡƒΠ³ΠΎΠ»ΡŒΠ½ΠΎΠΉ систСмС ΠΊΠΎΠΎΡ€Π΄ΠΈΠ½Π°Ρ‚ ΠΌΠΎΠΆΠ΅Ρ‚ Π±Ρ‹Ρ‚ΡŒ рассчитано ΠΏΠΎ

    ΠΠ°ΠΏΡ€Π°Π²Π»ΡΡŽΡ‰ΠΈΠΉ косинус:

    ΠΠ°ΠΏΡ€Π°Π²Π»ΡΡŽΡ‰ΠΈΠ΅ косинусы ΠΎΠΏΡ€Π΅Π΄Π΅Π»Π΅Π½Ρ‹ ΠΊΠ°ΠΊ

    .

    Π³Π΄Π΅ ΡƒΠ³Π»Ρ‹ , , Π° Ρ‚Π°ΠΊΠΆΠ΅ находятся ΡƒΠ³Π»Ρ‹, ΠΏΠΎΠΊΠ°Π·Π°Π½Π½Ρ‹Π΅ Π½Π° рисункС.Как ΠΏΠΎΠΊΠ°Π·Π°Π½ΠΎ Π½Π° рисункС, Π½Π°ΠΏΡ€Π°Π²Π»Π΅Π½ΠΈΠ΅ косинусы ΠΏΡ€Π΅Π΄ΡΡ‚Π°Π²Π»ΡΡŽΡ‚ собой косинусы ΡƒΠ³Π»ΠΎΠ² ΠΌΠ΅ΠΆΠ΄Ρƒ Π²Π΅ΠΊΡ‚ΠΎΡ€ΠΎΠΌ ΠΈ Ρ‚Ρ€ΠΈ ΠΊΠΎΠΎΡ€Π΄ΠΈΠ½Π°Ρ‚Π½Ρ‹Ρ… направлСния.

    ΠΠ°ΠΏΡ€Π°Π²Π»ΡΡŽΡ‰ΠΈΠ΅ косинусы ΠΌΠΎΠ³ΡƒΡ‚ Π±Ρ‹Ρ‚ΡŒ вычислСны ΠΈΠ· ΠΊΠΎΠΌΠΏΠΎΠ½Π΅Π½Ρ‚Ρ‹ Π²Π΅ΠΊΡ‚ΠΎΡ€Π° ΠΈ Π΅Π³ΠΎ Π²Π΅Π»ΠΈΡ‡ΠΈΠ½Π° Ρ‡Π΅Ρ€Π΅Π· ΡΠΎΠΎΡ‚Π½ΠΎΡˆΠ΅Π½ΠΈΡ

    Π’Ρ€ΠΈ Π½Π°ΠΏΡ€Π°Π²Π»ΡΡŽΡ‰ΠΈΡ… косинуса Π½Π΅ ΡΠ²Π»ΡΡŽΡ‚ΡΡ нСзависимыми ΠΈ Π΄ΠΎΠ»ΠΆΠ΅Π½ ΡƒΠ΄ΠΎΠ²Π»Π΅Ρ‚Π²ΠΎΡ€ΡΡ‚ΡŒ ΡΠΎΠΎΡ‚Π½ΠΎΡˆΠ΅Π½ΠΈΡŽ

    Π­Ρ‚ΠΈ Ρ€Π΅Π·ΡƒΠ»ΡŒΡ‚Π°Ρ‚Ρ‹ Ρ„ΠΎΡ€ΠΌΠΈΡ€ΡƒΡŽΡ‚ Ρ‚ΠΎΡ‚ Ρ„Π°ΠΊΡ‚, Ρ‡Ρ‚ΠΎ

    Π•Π΄ΠΈΠ½ΠΈΡ‡Π½Ρ‹ΠΉ Π²Π΅ΠΊΡ‚ΠΎΡ€ ΠΌΠΎΠΆΠ΅Ρ‚ Π±Ρ‹Ρ‚ΡŒ построСн вдоль Π²Π΅ΠΊΡ‚ΠΎΡ€Π° ΠΈΡΠΏΠΎΠ»ΡŒΠ·ΡƒΡ Π½Π°ΠΏΡ€Π°Π²Π»ΡΡŽΡ‰ΠΈΠ΅ косинусы Π² качСствС ΠΊΠΎΠΌΠΏΠΎΠ½Π΅Π½Ρ‚ΠΎΠ² вдоль x , y , ΠΈ z Π½Π°ΠΏΡ€Π°Π²Π»Π΅Π½ΠΈΠΉ.НапримСр, Π΅Π΄ΠΈΠ½ΠΈΡ‡Π½Ρ‹ΠΉ Π²Π΅ΠΊΡ‚ΠΎΡ€ вдоль Π²Π΅ΠΊΡ‚ΠΎΡ€ A получаСтся ΠΈΠ·

    Π‘Π»Π΅Π΄ΠΎΠ²Π°Ρ‚Π΅Π»ΡŒΠ½ΠΎ,

    Π’Π΅ΠΊΡ‚ΠΎΡ€ соСдинСниС Π΄Π²ΡƒΡ… Ρ‚ΠΎΡ‡Π΅ΠΊ:

    Π’Π΅ΠΊΡ‚ΠΎΡ€, ΡΠΎΠ΅Π΄ΠΈΠ½ΡΡŽΡ‰ΠΈΠΉ Ρ‚ΠΎΡ‡ΠΊΡƒ A с Ρ‚ΠΎΡ‡ΠΊΠΎΠΉ B выдаСтся

    Π•Π΄ΠΈΠ½ΠΈΡ‡Π½Ρ‹ΠΉ Π²Π΅ΠΊΡ‚ΠΎΡ€ вдоль Π»ΠΈΠ½ΠΈΠΈ A-B ΠΌΠΎΠΆΠ΅Ρ‚ Π±Ρ‹Ρ‚ΡŒ ΠΏΠΎΠ»ΡƒΡ‡Π΅Π½ ΠΈΠ·

    A Π²Π΅ΠΊΡ‚ΠΎΡ€ F ΠΏΠΎ Π»ΠΈΠ½ΠΈΠΈ A-B ΠΈ Π²Π΅Π»ΠΈΡ‡ΠΈΠ½ΠΎΠΉ F ​​ ΠΌΠΎΠΆΠ΅Ρ‚ Ρ‚Π°ΠΊΠΈΠΌ ΠΎΠ±Ρ€Π°Π·ΠΎΠΌ получаСтся ΠΈΠ· ΡΠΎΠΎΡ‚Π½ΠΎΡˆΠ΅Π½ΠΈΡ

    Ρ‚ΠΎΡ‡ΠΊΠ° Ρ‚ΠΎΠ²Π°Ρ€:

    БкалярноС ΠΏΡ€ΠΎΠΈΠ·Π²Π΅Π΄Π΅Π½ΠΈΠ΅ обозначаСтся «» ΠΌΠ΅ΠΆΠ΄Ρƒ двумя Π²Π΅ΠΊΡ‚ΠΎΡ€Π°ΠΌΠΈ.БкалярноС ΠΏΡ€ΠΎΠΈΠ·Π²Π΅Π΄Π΅Π½ΠΈΠ΅ Π²Π΅ΠΊΡ‚ΠΎΡ€ΠΎΠ² A ΠΈ B ΠΏΡ€ΠΈΠ²ΠΎΠ΄ΠΈΡ‚ ΠΊ скаляру, Π·Π°Π΄Π°Π²Π°Π΅ΠΌΠΎΠΌΡƒ ΡΠΎΠΎΡ‚Π½ΠΎΡˆΠ΅Π½ΠΈΠ΅ΠΌ

    Π³Π΄Π΅ являСтся ΡƒΠ³ΠΎΠ» ΠΌΠ΅ΠΆΠ΄Ρƒ двумя Π²Π΅ΠΊΡ‚ΠΎΡ€Π°ΠΌΠΈ. ΠŸΠΎΡ€ΡΠ΄ΠΎΠΊ Π½Π΅ Π²Π°ΠΆΠ΅Π½ Π² скалярном ΠΏΡ€ΠΎΠΈΠ·Π²Π΅Π΄Π΅Π½ΠΈΠΈ ΠΊΠ°ΠΊ Π²ΠΈΠ΄Π½ΠΎ ΠΈΠ· опрСдСлСния скалярных ΠΏΡ€ΠΎΠΈΠ·Π²Π΅Π΄Π΅Π½ΠΈΠΉ. Π’ ΠΈΡ‚ΠΎΠ³Π΅ получаСтся

    БкалярноС ΠΏΡ€ΠΎΠΈΠ·Π²Π΅Π΄Π΅Π½ΠΈΠ΅ ΠΈΠΌΠ΅Π΅Ρ‚ ΡΠ»Π΅Π΄ΡƒΡŽΡ‰ΠΈΠ΅ свойства.

    ΠŸΠΎΡΠΊΠΎΠ»ΡŒΠΊΡƒ косинус 90 o Ρ€Π°Π²Π΅Π½ Π½ΡƒΠ»ΡŽ, скалярноС ΠΏΡ€ΠΎΠΈΠ·Π²Π΅Π΄Π΅Π½ΠΈΠ΅ Π΄Π²ΡƒΡ… ΠΎΡ€Ρ‚ΠΎΠ³ΠΎΠ½Π°Π»ΡŒΠ½Ρ‹Π΅ Π²Π΅ΠΊΡ‚ΠΎΡ€Ρ‹ ΠΏΡ€ΠΈΠ²Π΅Π΄ΡƒΡ‚ ΠΊ Π½ΡƒΠ»ΡŽ.

    Π’Π°ΠΊ ΠΊΠ°ΠΊ ΡƒΠ³ΠΎΠ» ΠΌΠ΅ΠΆΠ΄Ρƒ Π²Π΅ΠΊΡ‚ΠΎΡ€ΠΎΠΌ ΠΈ самим собой Ρ€Π°Π²Π΅Π½ Π½ΡƒΠ»ΡŽ, Π° косинус ноль Ρ€Π°Π²Π΅Π½ Π΅Π΄ΠΈΠ½ΠΈΡ†Π΅, Π²Π΅Π»ΠΈΡ‡ΠΈΠ½Π° Π²Π΅ΠΊΡ‚ΠΎΡ€Π° ΠΌΠΎΠΆΠ΅Ρ‚ Π±Ρ‹Ρ‚ΡŒ записана Ρ‡Π΅Ρ€Π΅Π· Ρ‚ΠΎΡ‡ΠΊΡƒ ΠΏΡ€ΠΎΠ΄ΡƒΠΊΡ‚ ΠΏΠΎ ΠΏΡ€Π°Π²ΠΈΠ»Ρƒ

    ΠŸΡ€ΡΠΌΠΎΡƒΠ³ΠΎΠ»ΡŒΠ½Ρ‹Π΅ ΠΊΠΎΠΎΡ€Π΄ΠΈΠ½Π°Ρ‚Ρ‹:

    ΠŸΡ€ΠΈ Ρ€Π°Π±ΠΎΡ‚Π΅ с Π²Π΅ΠΊΡ‚ΠΎΡ€Π°ΠΌΠΈ, прСдставлСнными Π² ΠΏΡ€ΡΠΌΠΎΡƒΠ³ΠΎΠ»ΡŒΠ½Π°Ρ систСма ΠΊΠΎΠΎΡ€Π΄ΠΈΠ½Π°Ρ‚ ΠΏΠΎ ΠΊΠΎΠΌΠΏΠΎΠ½Π΅Π½Ρ‚Π°ΠΌ

    , Ρ‚ΠΎ скалярноС ΠΏΡ€ΠΎΠΈΠ·Π²Π΅Π΄Π΅Π½ΠΈΠ΅ ΠΌΠΎΠΆΠ΅Ρ‚ Π±Ρ‹Ρ‚ΡŒ ΠΎΡ†Π΅Π½Π΅Π½ΠΎ ΠΈΠ· ΠΎΡ‚Π½ΠΎΡˆΠ΅Π½ΠΈΠ΅

    Π­Ρ‚ΠΎ ΠΌΠΎΠΆΠ½ΠΎ ΠΏΡ€ΠΎΠ²Π΅Ρ€ΠΈΡ‚ΡŒ прямым ΡƒΠΌΠ½ΠΎΠΆΠ΅Π½ΠΈΠ΅ΠΌ Π²Π΅ΠΊΡ‚ΠΎΡ€ΠΎΠ² ΠΈ отмСчая, Ρ‡Ρ‚ΠΎ ΠΈΠ·-Π·Π° ΠΎΡ€Ρ‚ΠΎΠ³ΠΎΠ½Π°Π»ΡŒΠ½ΠΎΡΡ‚ΠΈ Π±Π°Π·ΠΎΠ²Ρ‹Ρ… Π²Π΅ΠΊΡ‚ΠΎΡ€ΠΎΠ² ΠΏΡ€ΡΠΌΠΎΡƒΠ³ΠΎΠ»ΡŒΠ½Π°Ρ систСма

    ΠŸΡ€ΠΎΠ΅ΠΊΡ†ΠΈΡ Π²Π΅ΠΊΡ‚ΠΎΡ€Π° Π½Π° линию:

    ΠžΡ€Ρ‚ΠΎΠ³ΠΎΠ½Π°Π»ΡŒΠ½Π°Ρ проСкция Π²Π΅ΠΊΡ‚ΠΎΡ€Π° вдоль линия получаСтся ΠΏΠ΅Ρ€Π΅ΠΌΠ΅Ρ‰Π΅Π½ΠΈΠ΅ΠΌ ΠΎΠ΄Π½ΠΎΠ³ΠΎ ΠΊΠΎΠ½Ρ†Π° Π²Π΅ΠΊΡ‚ΠΎΡ€Π° Π½Π° линию ΠΈ опусканиСм пСрпСндикуляр Π½Π° линию ΠΎΡ‚ Π΄Ρ€ΡƒΠ³ΠΎΠ³ΠΎ ΠΊΠΎΠ½Ρ†Π° Π²Π΅ΠΊΡ‚ΠΎΡ€Π°.Π’ Π Π΅Π·ΡƒΠ»ΡŒΡ‚ΠΈΡ€ΡƒΡŽΡ‰ΠΈΠΉ ΠΎΡ‚Ρ€Π΅Π·ΠΎΠΊ Π½Π° прямой являСтся ΠΎΡ€Ρ‚ΠΎΠ³ΠΎΠ½Π°Π»ΡŒΠ½ΠΎΠΉ ΠΏΡ€ΠΎΠ΅ΠΊΡ†ΠΈΠ΅ΠΉ Π²Π΅ΠΊΡ‚ΠΎΡ€Π° ΠΈΠ»ΠΈ просто Π΅Π³ΠΎ проСкция.

    Бкалярная проСкция Π²Π΅ΠΊΡ‚ΠΎΡ€Π° A вдоль Π΅Π΄ΠΈΠ½ΠΈΡ‡Π½Ρ‹ΠΉ Π²Π΅ΠΊΡ‚ΠΎΡ€ являСтся Π΄Π»ΠΈΠ½Π° ΠΎΡ€Ρ‚ΠΎΠ³ΠΎΠ½Π°Π»ΡŒΠ½ΠΎΠ³ΠΎ выступа А ΠΏΠΎ Π»ΠΈΠ½ΠΈΠΈ, ΠΏΠ°Ρ€Π°Π»Π»Π΅Π»ΡŒΠ½ΠΎΠΉ , ΠΈ ΠΌΠΎΠΆΠ΅Ρ‚ Π±Ρ‹Ρ‚ΡŒ ΠΎΡ†Π΅Π½Π΅Π½ с ΠΏΠΎΠΌΠΎΡ‰ΡŒΡŽ скалярного произвСдСния. Π‘ΠΎΠΎΡ‚Π½ΠΎΡˆΠ΅Π½ΠΈΠ΅ для ΠΏΡ€ΠΎΠ΅ΠΊΡ†ΠΈΠΈ это

    ВСкторная проСкция А вдоль Π°Π³Ρ€Π΅Π³Π°Ρ‚Π°. Π²Π΅ΠΊΡ‚ΠΎΡ€ просто ΡƒΠΌΠ½ΠΎΠΆΠ°Π΅Ρ‚ ΡΠΊΠ°Π»ΡΡ€Π½ΡƒΡŽ ΠΏΡ€ΠΎΠ΅ΠΊΡ†ΠΈΡŽ Π½Π° Π΅Π΄ΠΈΠ½ΠΈΡ‡Π½Ρ‹ΠΉ Π²Π΅ΠΊΡ‚ΠΎΡ€ ΠΊ ΠΏΠΎΠ»ΡƒΡ‡ΠΈΡ‚ΡŒ Π²Π΅ΠΊΡ‚ΠΎΡ€ .Π­Ρ‚ΠΎ Π΄Π°Π΅Ρ‚ ΡΠΎΠΎΡ‚Π½ΠΎΡˆΠ΅Π½ΠΈΠ΅

    пСрСкрСстноС ΠΏΡ€ΠΎΠΈΠ·Π²Π΅Π΄Π΅Π½ΠΈΠ΅:

    ΠŸΠ΅Ρ€Π΅ΠΊΡ€Π΅ΡΡ‚Π½ΠΎΠ΅ ΠΏΡ€ΠΎΠΈΠ·Π²Π΅Π΄Π΅Π½ΠΈΠ΅ Π²Π΅ΠΊΡ‚ΠΎΡ€ΠΎΠ² a ΠΈ b являСтся Π²Π΅ΠΊΡ‚ΠΎΡ€ΠΎΠΌ пСрпСндикулярно ΠΎΠ±ΠΎΠΈΠΌ a ΠΈ b ΠΈ ΠΈΠΌΠ΅Π΅Ρ‚ Π²Π΅Π»ΠΈΡ‡ΠΈΠ½Ρƒ, Ρ€Π°Π²Π½ΡƒΡŽ ΠŸΠ»ΠΎΡ‰Π°Π΄ΡŒ ΠΏΠ°Ρ€Π°Π»Π»Π΅Π»ΠΎΠ³Ρ€Π°ΠΌΠΌΠ°, получСнная ΠΈΠ· a ΠΈ b . Π’ Π½Π°ΠΏΡ€Π°Π²Π»Π΅Π½ΠΈΠ΅ пСрСкрСстного произвСдСния задаСтся ΠΏΡ€Π°Π²ΠΈΠ»ΠΎΠΌ ΠΏΡ€Π°Π²ΠΎΠΉ Ρ€ΡƒΠΊΠΈ.ΠšΡ€Π΅ΡΡ‚ ΠΏΡ€ΠΎΠ΄ΡƒΠΊΡ‚ обозначаСтся Π·Π½Π°ΠΊΠΎΠΌ «» ΠΌΠ΅ΠΆΠ΄Ρƒ Π²Π΅ΠΊΡ‚ΠΎΡ€Π°ΠΌΠΈ

    ΠŸΠΎΡ€ΡΠ΄ΠΎΠΊ Π²Π°ΠΆΠ΅Π½ Π² пСрСкрСстном ΠΏΡ€ΠΎΠΈΠ·Π²Π΅Π΄Π΅Π½ΠΈΠΈ. Если порядок ΠΎΠΏΠ΅Ρ€Π°Ρ†ΠΈΠΉ измСнСния Π² пСрСкрСстном ΠΏΡ€ΠΎΠΈΠ·Π²Π΅Π΄Π΅Π½ΠΈΠΈ Π½Π°ΠΏΡ€Π°Π²Π»Π΅Π½ΠΈΠ΅ Ρ€Π΅Π·ΡƒΠ»ΡŒΡ‚ΠΈΡ€ΡƒΡŽΡ‰Π΅Π³ΠΎ Π²Π΅ΠΊΡ‚ΠΎΡ€Π° Π½Π°ΠΎΠ±ΠΎΡ€ΠΎΡ‚. Π’ΠΎ Π΅ΡΡ‚ΡŒ

    ΠŸΠ΅Ρ€Π΅ΠΊΡ€Π΅ΡΡ‚Π½ΠΎΠ΅ ΠΏΡ€ΠΎΠΈΠ·Π²Π΅Π΄Π΅Π½ΠΈΠ΅ ΠΈΠΌΠ΅Π΅Ρ‚ ΡΠ»Π΅Π΄ΡƒΡŽΡ‰ΠΈΠ΅ свойства.

    ΠŸΡ€ΡΠΌΠΎΡƒΠ³ΠΎΠ»ΡŒΠ½Ρ‹Π΅ ΠΊΠΎΠΎΡ€Π΄ΠΈΠ½Π°Ρ‚Ρ‹:

    ΠŸΡ€ΠΈ Ρ€Π°Π±ΠΎΡ‚Π΅ Π² ΠΏΡ€ΡΠΌΠΎΡƒΠ³ΠΎΠ»ΡŒΠ½Ρ‹Ρ… систСмах ΠΊΠΎΠΎΡ€Π΄ΠΈΠ½Π°Ρ‚, пСрСкрСстноС ΠΏΡ€ΠΎΠΈΠ·Π²Π΅Π΄Π΅Π½ΠΈΠ΅ Π²Π΅ΠΊΡ‚ΠΎΡ€ΠΎΠ² a ΠΈ b , Π·Π°Π΄Π°Π½Π½ΠΎΠ΅

    ΠΌΠΎΠΆΠ½ΠΎ ΠΎΡ†Π΅Π½ΠΈΡ‚ΡŒ с ΠΏΠΎΠΌΠΎΡ‰ΡŒΡŽ ΠΏΡ€Π°Π²ΠΈΠ»Π°

    МоТно Ρ‚Π°ΠΊΠΆΠ΅ ΠΈΡΠΏΠΎΠ»ΡŒΠ·ΠΎΠ²Π°Ρ‚ΡŒ прямоС ΡƒΠΌΠ½ΠΎΠΆΠ΅Π½ΠΈΠ΅ Π±Π°Π·ΠΎΠ²Ρ‹Π΅ Π²Π΅ΠΊΡ‚ΠΎΡ€Ρ‹ с использованиСм ΡΠΎΠΎΡ‚Π½ΠΎΡˆΠ΅Π½ΠΈΠΉ

    Ρ‚Ρ€ΠΎΠΉΠ½ΠΎΠ΅ ΠΏΡ€ΠΎΠΈΠ·Π²Π΅Π΄Π΅Π½ΠΈΠ΅:

    Π”Π°Π½ΠΎ Ρ‚Ρ€ΠΎΠΉΠ½ΠΎΠ΅ ΠΏΡ€ΠΎΠΈΠ·Π²Π΅Π΄Π΅Π½ΠΈΠ΅ Π²Π΅ΠΊΡ‚ΠΎΡ€ΠΎΠ² a , b ΠΈ c . ΠΏΠΎ

    Π‘Ρ‚ΠΎΠΈΠΌΠΎΡΡ‚ΡŒ Ρ‚Ρ€ΠΎΠΉΠ½ΠΎΠ³ΠΎ произвСдСния Ρ€Π°Π²Π½Π° ΠΎΠ±ΡŠΠ΅ΠΌΡƒ ΠΏΠ°Ρ€Π°Π»Π»Π΅Π»Π΅ΠΏΠΈΠΏΠ΅Π΄, построСнный ΠΈΠ· Π²Π΅ΠΊΡ‚ΠΎΡ€ΠΎΠ².Π­Ρ‚ΠΎ Π²ΠΈΠ΄Π½ΠΎ ΠΈΠ· Ρ†ΠΈΡ„Ρ€Π° с

    Π’Ρ€ΠΎΠΉΠ½ΠΎΠΉ ΠΏΡ€ΠΎΠ΄ΡƒΠΊΡ‚ ΠΈΠΌΠ΅Π΅Ρ‚ ΡΠ»Π΅Π΄ΡƒΡŽΡ‰ΠΈΠ΅ свойства

    ΠŸΡ€ΡΠΌΠΎΡƒΠ³ΠΎΠ»ΡŒΠ½Ρ‹Π΅ ΠΊΠΎΠΎΡ€Π΄ΠΈΠ½Π°Ρ‚Ρ‹:

    Рассмотрим Π²Π΅ΠΊΡ‚ΠΎΡ€Ρ‹, описанныС Π² ΠΏΡ€ΡΠΌΠΎΡƒΠ³ΠΎΠ»ΡŒΠ½ΠΈΠΊΠ΅. систСма ΠΊΠΎΠΎΡ€Π΄ΠΈΠ½Π°Ρ‚ ΠΊΠ°ΠΊ

    Π’Ρ€ΠΎΠΉΠ½ΠΎΠ΅ ΠΏΡ€ΠΎΠΈΠ·Π²Π΅Π΄Π΅Π½ΠΈΠ΅ ΠΌΠΎΠΆΠ½ΠΎ ΠΎΡ†Π΅Π½ΠΈΡ‚ΡŒ с ΠΏΠΎΠΌΠΎΡ‰ΡŒΡŽ ΠΎΡ‚Π½ΠΎΡˆΠ΅Π½ΠΈΠ΅

    ВрСхмСстный Π²Π΅ΠΊΡ‚ΠΎΡ€Π½Ρ‹ΠΉ ΠΏΡ€ΠΎΠ΄ΡƒΠΊΡ‚:

    ΠŸΡ€ΠΎΠΈΠ·Π²Π΅Π΄Π΅Π½ΠΈΠ΅ Ρ‚Ρ€ΠΎΠΉΠ½ΠΎΠ³ΠΎ Π²Π΅ΠΊΡ‚ΠΎΡ€Π° ΠΈΠΌΠ΅Π΅Ρ‚ свойства

    ПониманиС пСрСкрСстного ΠΏΡ€ΠΎΠ΄ΡƒΠΊΡ‚Π° — Π»ΡƒΡ‡ΡˆΠ΅Π΅ объяснСниС

    Взяв Π΄Π²Π° Π²Π΅ΠΊΡ‚ΠΎΡ€Π°, ΠΌΡ‹ ΠΌΠΎΠΆΠ΅ΠΌ Π·Π°ΠΏΠΈΡΠ°Ρ‚ΡŒ ΠΊΠ°ΠΆΠ΄ΡƒΡŽ ΠΊΠΎΠΌΠ±ΠΈΠ½Π°Ρ†ΠΈΡŽ ΠΊΠΎΠΌΠΏΠΎΠ½Π΅Π½Ρ‚ΠΎΠ² Π² сСтку:

    Π­Ρ‚Π° заполнСнная сСтка прСдставляСт собой внСшний ΠΏΡ€ΠΎΠ΄ΡƒΠΊΡ‚ , ΠΊΠΎΡ‚ΠΎΡ€Ρ‹ΠΉ ΠΌΠΎΠΆΠ½ΠΎ Ρ€Π°Π·Π΄Π΅Π»ΠΈΡ‚ΡŒ Π½Π°:

    • Π’ΠΎΡ‡Π΅Ρ‡Π½ΠΎΠ΅ ΠΏΡ€ΠΎΠΈΠ·Π²Π΅Π΄Π΅Π½ΠΈΠ΅ , взаимодСйствия ΠΌΠ΅ΠΆΠ΄Ρƒ схоТими Ρ€Π°Π·ΠΌΠ΅Ρ€Π°ΠΌΠΈ ( x * x , y * y , z * z )

    • ΠŸΠ΅Ρ€Π΅ΠΊΡ€Π΅ΡΡ‚Π½ΠΎΠ΅ ΠΏΡ€ΠΎΠΈΠ·Π²Π΅Π΄Π΅Π½ΠΈΠ΅ , взаимодСйствия ΠΌΠ΅ΠΆΠ΄Ρƒ Ρ€Π°Π·Π»ΠΈΡ‡Π½Ρ‹ΠΌΠΈ измСрСниями ( x * y , y * z , z * x ΠΈ Ρ‚. Π”.)

    Π’ΠΎΡ‡Π΅Ρ‡Π½ΠΎΠ΅ ΠΏΡ€ΠΎΠΈΠ·Π²Π΅Π΄Π΅Π½ΠΈΠ΅ ($ \ vec {a} \ cdot \ vec {b} $) измСряСт сходство, ΠΏΠΎΡ‚ΠΎΠΌΡƒ Ρ‡Ρ‚ΠΎ ΠΎΠ½ΠΎ Π½Π°ΠΊΠ°ΠΏΠ»ΠΈΠ²Π°Π΅Ρ‚ взаимодСйствия Ρ‚ΠΎΠ»ΡŒΠΊΠΎ Π² ΡΠΎΠ²ΠΏΠ°Π΄Π°ΡŽΡ‰ΠΈΡ… измСрСниях. Π­Ρ‚ΠΎ простой расчСт, состоящий ΠΈΠ· Ρ‚Ρ€Π΅Ρ… ΠΊΠΎΠΌΠΏΠΎΠ½Π΅Π½Ρ‚ΠΎΠ².

    ΠŸΠ΅Ρ€Π΅ΠΊΡ€Π΅ΡΡ‚Π½ΠΎΠ΅ ΠΏΡ€ΠΎΠΈΠ·Π²Π΅Π΄Π΅Π½ΠΈΠ΅ (ΠΎΠ±ΠΎΠ·Π½Π°Ρ‡Π°Π΅ΠΌΠΎΠ΅ ΠΊΠ°ΠΊ $ \ vec {a} \ times \ vec {b} $) Π΄ΠΎΠ»ΠΆΠ½ΠΎ ΠΈΠ·ΠΌΠ΅Ρ€ΡΡ‚ΡŒ ΠΏΠΎΠ»Π΄ΡŽΠΆΠΈΠ½Ρ‹ «пСрСкрСстных взаимодСйствий». РасчСт выглядит слоТным, Π½ΠΎ концСпция проста: Π½Π°ΠΊΠΎΠΏΠΈΡ‚Π΅ 6 ΠΈΠ½Π΄ΠΈΠ²ΠΈΠ΄ΡƒΠ°Π»ΡŒΠ½Ρ‹Ρ… Ρ€Π°Π·Π»ΠΈΡ‡ΠΈΠΉ для получСния ΠΎΠ±Ρ‰Π΅ΠΉ Ρ€Π°Π·Π½ΠΈΡ†Ρ‹.

    ВмСсто Ρ‚ΠΎΠ³ΠΎ, Ρ‡Ρ‚ΠΎΠ±Ρ‹ Π΄ΡƒΠΌΠ°Ρ‚ΡŒ: «Когда ΠΌΠ½Π΅ Π½ΡƒΠΆΠ½ΠΎ кросс-ΠΏΡ€ΠΎΠΈΠ·Π²Π΅Π΄Π΅Π½ΠΈΠ΅?Β» ΠΏΠΎΠ΄ΡƒΠΌΠ°ΠΉΡ‚Π΅: «Когда ΠΌΠ½Π΅ Π½ΡƒΠΆΠ½ΠΎ взаимодСйствиС ΠΌΠ΅ΠΆΠ΄Ρƒ Ρ€Π°Π·Π½Ρ‹ΠΌΠΈ измСрСниями?Β».

    ΠŸΠ»ΠΎΡ‰Π°Π΄ΡŒ, Π½Π°ΠΏΡ€ΠΈΠΌΠ΅Ρ€, ΠΎΠ±Ρ€Π°Π·ΠΎΠ²Π°Π½Π° Π²Π΅ΠΊΡ‚ΠΎΡ€Π°ΠΌΠΈ, ΡƒΠΊΠ°Π·Ρ‹Π²Π°ΡŽΡ‰ΠΈΠΌΠΈ Π² Ρ€Π°Π·Π½Ρ‹Ρ… направлСниях (Ρ‡Π΅ΠΌ большС ΠΎΡ€Ρ‚ΠΎΠ³ΠΎΠ½Π°Π»ΡŒΠ½ΠΎΡΡ‚ΡŒ, Ρ‚Π΅ΠΌ Π»ΡƒΡ‡ΡˆΠ΅). Π”Π΅ΠΉΡΡ‚Π²ΠΈΡ‚Π΅Π»ΡŒΠ½ΠΎ, пСрСкрСстноС ΠΏΡ€ΠΎΠΈΠ·Π²Π΅Π΄Π΅Π½ΠΈΠ΅ измСряСт ΠΏΠ»ΠΎΡ‰Π°Π΄ΡŒ, ΠΎΡ…Π²Π°Ρ‚Ρ‹Π²Π°Π΅ΠΌΡƒΡŽ двумя Ρ‚Ρ€Π΅Ρ…ΠΌΠ΅Ρ€Π½Ρ‹ΠΌΠΈ Π²Π΅ΠΊΡ‚ΠΎΡ€Π°ΠΌΠΈ (источник):

    (Β«ΠŸΠ΅Ρ€Π΅ΠΊΡ€Π΅ΡΡ‚Π½ΠΎΠ΅ ΠΏΡ€ΠΎΠΈΠ·Π²Π΅Π΄Π΅Π½ΠΈΠ΅Β» ΠΏΡ€Π΅Π΄ΠΏΠΎΠ»Π°Π³Π°Π΅Ρ‚ Ρ‚Ρ€Π΅Ρ…ΠΌΠ΅Ρ€Π½Ρ‹Π΅ Π²Π΅ΠΊΡ‚ΠΎΡ€Ρ‹, Π½ΠΎ концСпция распространяСтся Π½Π° Π±ΠΎΠ»Π΅Π΅ высокиС измСрСния.)

    Π©Π΅Π»ΠΊΠ½ΡƒΠ»Π° Π»ΠΈ ΠΊΠ»ΡŽΡ‡Π΅Π²Π°Ρ интуиция? Π”Π°Π²Π°ΠΉΡ‚Π΅ углубимся Π² Π΄Π΅Ρ‚Π°Π»ΠΈ.

    ΠžΠΏΡ€Π΅Π΄Π΅Π»Π΅Π½ΠΈΠ΅ пСрСкрСстного произвСдСния

    БкалярноС ΠΏΡ€ΠΎΠΈΠ·Π²Π΅Π΄Π΅Π½ΠΈΠ΅ прСдставляСт сходство ΠΌΠ΅ΠΆΠ΄Ρƒ Π²Π΅ΠΊΡ‚ΠΎΡ€Π°ΠΌΠΈ ΠΊΠ°ΠΊ ΠΎΠ΄Π½ΠΎ число:

    НапримСр, ΠΌΡ‹ ΠΌΠΎΠΆΠ΅ΠΌ ΡΠΊΠ°Π·Π°Ρ‚ΡŒ, Ρ‡Ρ‚ΠΎ сСвСр ΠΈ восток Π½Π° 0% ΠΏΠΎΡ…ΠΎΠΆΠΈ, ΠΏΠΎΡΠΊΠΎΠ»ΡŒΠΊΡƒ $ (0, 1) \ cdot (1, 0) = 0 $.Или Ρ‡Ρ‚ΠΎ сСвСр ΠΈ сСвСро-восток Π½Π° 70% ΠΏΠΎΡ…ΠΎΠΆΠΈ ($ \ cos (45) = 0,707 $, ΠΏΠΎΠΌΠ½ΠΈΡ‚Π΅, Ρ‡Ρ‚ΠΎ Ρ‚Ρ€ΠΈΠ³Π³Π΅Ρ€Π½Ρ‹Π΅ Ρ„ΡƒΠ½ΠΊΡ†ΠΈΠΈ — это ΠΏΡ€ΠΎΡ†Π΅Π½Ρ‚Ρ‹). Бходство ΠΏΠΎΠΊΠ°Π·Ρ‹Π²Π°Π΅Ρ‚ количСство ΠΎΠ΄Π½ΠΎΠ³ΠΎ Π²Π΅ΠΊΡ‚ΠΎΡ€Π°, ΠΊΠΎΡ‚ΠΎΡ€Ρ‹ΠΉ «проявляСтся» Π² Π΄Ρ€ΡƒΠ³ΠΎΠΌ.

    Π”ΠΎΠ»ΠΆΠ½ΠΎ Π»ΠΈ пСрСкрСстноС ΠΏΡ€ΠΎΠΈΠ·Π²Π΅Π΄Π΅Π½ΠΈΠ΅, Ρ€Π°Π·Π½ΠΎΡΡ‚ΡŒ Π²Π΅ΠΊΡ‚ΠΎΡ€ΠΎΠ², Ρ‚ΠΎΠΆΠ΅ Π±Ρ‹Ρ‚ΡŒ ΠΎΠ΄Π½ΠΈΠΌ числом?

    Π”Π°Π²Π°ΠΉ ΠΏΠΎΠΏΡ€ΠΎΠ±ΡƒΠ΅ΠΌ. Бинус — это процСнтная Ρ€Π°Π·Π½ΠΈΡ†Π°, поэтому ΠΌΡ‹ ΠΌΠΎΠ³Π»ΠΈ Π±Ρ‹ Π½Π°ΠΏΠΈΡΠ°Ρ‚ΡŒ:

    К соТалСнию, ΠΌΡ‹ упускаСм Π½Π΅ΠΊΠΎΡ‚ΠΎΡ€Ρ‹Π΅ Π΄Π΅Ρ‚Π°Π»ΠΈ. Допустим, ΠΌΡ‹ смотрим Π²Π½ΠΈΠ· ΠΏΠΎ оси x: ΠΎΠ±Π΅ Ρ‚ΠΎΡ‡ΠΊΠΈ — y ΠΈ z — Π½Π° 100% ΠΎΡ‚ нас.Число Π²Ρ€ΠΎΠ΄Π΅ Β«100%Β» Π³ΠΎΠ²ΠΎΡ€ΠΈΡ‚ Π½Π°ΠΌ ΠΎ большой Ρ€Π°Π·Π½ΠΈΡ†Π΅, Π½ΠΎ ΠΌΡ‹ Π½Π΅ Π·Π½Π°Π΅ΠΌ, Ρ‡Ρ‚ΠΎ это Ρ‚Π°ΠΊΠΎΠ΅! Нам Π½ΡƒΠΆΠ½Π° Π΄ΠΎΠΏΠΎΠ»Π½ΠΈΡ‚Π΅Π»ΡŒΠ½Π°Ρ информация, Ρ‡Ρ‚ΠΎΠ±Ρ‹ ΡΠΊΠ°Π·Π°Ρ‚ΡŒ Π½Π°ΠΌ Β«Ρ€Π°Π·Π½ΠΈΡ†Π° ΠΌΠ΅ΠΆΠ΄Ρƒ $ \ vec {x} $ ΠΈ $ \ vec {y} $ составляСт , Π° Π½Π΅ Β» ΠΈ Β«Ρ€Π°Π·Π½ΠΈΡ†Π° ΠΌΠ΅ΠΆΠ΄Ρƒ $ \ vec {x} $ ΠΈ $ \ vec {z} $ , Ρ‡Ρ‚ΠΎ β€œ.

    Π˜Ρ‚Π°ΠΊ, прСдставим кросс-ΠΏΡ€ΠΎΠΈΠ·Π²Π΅Π΄Π΅Π½ΠΈΠ΅ Π² Π²ΠΈΠ΄Π΅ Π²Π΅ΠΊΡ‚ΠΎΡ€Π°:

    • Π Π°Π·ΠΌΠ΅Ρ€ пСрСкрСстного произвСдСния прСдставляСт собой Ρ‡ΠΈΡΠ»ΠΎΠ²ΡƒΡŽ Β«Π²Π΅Π»ΠΈΡ‡ΠΈΠ½Ρƒ Ρ€Π°Π·Π½ΠΈΡ†Ρ‹Β» (с $ \ sin (\ theta) $ Π² ΠΏΡ€ΠΎΡ†Π΅Π½Ρ‚Π°Ρ…). Π‘Π°ΠΌΠΎ ΠΏΠΎ сСбС это Π½Π΅ ΠΎΡ‚Π»ΠΈΡ‡Π°Π΅Ρ‚ $ \ vec {x} \ times \ vec {y} $ ΠΎΡ‚ $ \ vec {x} \ times \ vec {z} $.

    • НаправлСниС пСрСкрСстного произвСдСния основано Π½Π° ΠΎΠ±ΠΎΠΈΡ… Π²Ρ…ΠΎΠ΄Π½Ρ‹Ρ… Π΄Π°Π½Π½Ρ‹Ρ…: это Π½Π°ΠΏΡ€Π°Π²Π»Π΅Π½ΠΈΠ΅, ΠΎΡ€Ρ‚ΠΎΠ³ΠΎΠ½Π°Π»ΡŒΠ½ΠΎΠ΅ ΠΎΠ±ΠΎΠΈΠΌ (Ρ‚.Π΅. Π½Π΅ благоприятноС Π½ΠΈ для ΠΎΠ΄Π½ΠΎΠ³ΠΎ).

    Π’Π΅ΠΏΠ΅Ρ€ΡŒ $ \ vec {x} \ times \ vec {y} $ ΠΈ $ \ vec {x} \ times \ vec {z} $ Π΄Π°ΡŽΡ‚ Ρ€Π°Π·Π½Ρ‹Π΅ Ρ€Π΅Π·ΡƒΠ»ΡŒΡ‚Π°Ρ‚Ρ‹, ΠΊΠ°ΠΆΠ΄Ρ‹ΠΉ с Π²Π΅Π»ΠΈΡ‡ΠΈΠ½ΠΎΠΉ, ΡƒΠΊΠ°Π·Ρ‹Π²Π°ΡŽΡ‰Π΅ΠΉ, Ρ‡Ρ‚ΠΎ ΠΎΠ½ΠΈ Π½Π° Β«100%Β» ΠΎΡ‚Π»ΠΈΡ‡Π°ΡŽΡ‚ΡΡ ΠΎΡ‚ $ \ vec {x} $.

    (Π”ΠΎΠ»ΠΆΠ½ΠΎ Π»ΠΈ скалярноС ΠΏΡ€ΠΎΠΈΠ·Π²Π΅Π΄Π΅Π½ΠΈΠ΅ Π±Ρ‹Ρ‚ΡŒ Π²Π΅ΠΊΡ‚ΠΎΡ€Π½Ρ‹ΠΌ Ρ€Π΅Π·ΡƒΠ»ΡŒΡ‚Π°Ρ‚ΠΎΠΌ? Ну, ΠΌΡ‹ отслСТиваСм сходство ΠΌΠ΅ΠΆΠ΄Ρƒ $ \ vec {a} $ ΠΈ $ \ vec {b} $. Бходство измСряСт ΠΏΠ΅Ρ€Π΅ΠΊΡ€Ρ‹Ρ‚ΠΈΠ΅ ΠΌΠ΅ΠΆΠ΄Ρƒ исходными Π²Π΅ΠΊΡ‚ΠΎΡ€Π½Ρ‹ΠΌΠΈ направлСниями, ΠΊΠΎΡ‚ΠΎΡ€ΠΎΠ΅ ΠΌΡ‹ ΡƒΠΆΠ΅ Π΅ΡΡ‚ΡŒ.)

    ГСомСтричСская интСрпрСтация

    Π”Π²Π° Π²Π΅ΠΊΡ‚ΠΎΡ€Π° ΠΎΠΏΡ€Π΅Π΄Π΅Π»ΡΡŽΡ‚ ΠΏΠ»ΠΎΡΠΊΠΎΡΡ‚ΡŒ, ΠΈ Π²Π΅ΠΊΡ‚ΠΎΡ€Π½ΠΎΠ΅ ΠΏΡ€ΠΎΠΈΠ·Π²Π΅Π΄Π΅Π½ΠΈΠ΅ ΡƒΠΊΠ°Π·Ρ‹Π²Π°Π΅Ρ‚ Π² Π½Π°ΠΏΡ€Π°Π²Π»Π΅Π½ΠΈΠΈ, ΠΎΡ‚Π»ΠΈΡ‡Π½ΠΎΠΌ ΠΎΡ‚ ΠΎΠ±ΠΎΠΈΡ… (источник):

    Π’ΠΎΡ‚ ΠΏΡ€ΠΎΠ±Π»Π΅ΠΌΠ°: Π΅ΡΡ‚ΡŒ Π΄Π²Π° пСрпСндикулярных направлСния. Условно ΠΌΡ‹ ΠΏΡ€Π΅Π΄ΠΏΠΎΠ»Π°Π³Π°Π΅ΠΌ Β«ΠΏΡ€Π°Π²ΠΎΡΡ‚ΠΎΡ€ΠΎΠ½Π½ΡŽΡŽ систСму» ​​(источник):

    Если Π²Ρ‹ Π΄Π΅Ρ€ΠΆΠΈΡ‚Π΅ ΠΏΠ΅Ρ€Π²Ρ‹Π΅ Π΄Π²Π° ΠΏΠ°Π»ΡŒΡ†Π°, ΠΊΠ°ΠΊ ΠΏΠΎΠΊΠ°Π·Π°Π½ΠΎ Π½Π° схСмС, большой ΠΏΠ°Π»Π΅Ρ† Π±ΡƒΠ΄Π΅Ρ‚ ΡƒΠΊΠ°Π·Ρ‹Π²Π°Ρ‚ΡŒ Π² Π½Π°ΠΏΡ€Π°Π²Π»Π΅Π½ΠΈΠΈ пСрСкрСстного произвСдСния. Π― ΠΏΡ€ΠΎΠ²Π΅Ρ€ΡΡŽ ΠΏΡ€Π°Π²ΠΈΠ»ΡŒΠ½ΠΎΡΡ‚ΡŒ ΠΎΡ€ΠΈΠ΅Π½Ρ‚Π°Ρ†ΠΈΠΈ, проводя ΡƒΠΊΠ°Π·Π°Ρ‚Π΅Π»ΡŒΠ½Ρ‹ΠΌ ΠΏΠ°Π»ΡŒΡ†Π΅ΠΌ ΠΎΡ‚ $ \ vec {a} $ Π΄ΠΎ $ \ vec {b} $.Когда Π½Π°ΠΏΡ€Π°Π²Π»Π΅Π½ΠΈΠ΅ выяснСно, Π²Π΅Π»ΠΈΡ‡ΠΈΠ½Π° пСрСкрСстного произвСдСния Ρ€Π°Π²Π½Π° $ | a | | Π± | \ sin (\ theta) $, ΠΊΠΎΡ‚ΠΎΡ€Ρ‹ΠΉ ΠΏΡ€ΠΎΠΏΠΎΡ€Ρ†ΠΈΠΎΠ½Π°Π»Π΅Π½ Π²Π΅Π»ΠΈΡ‡ΠΈΠ½Π΅ ΠΊΠ°ΠΆΠ΄ΠΎΠ³ΠΎ Π²Π΅ΠΊΡ‚ΠΎΡ€Π° ΠΈ Β«ΠΏΡ€ΠΎΡ†Π΅Π½Ρ‚Ρƒ Ρ€Π°Π·Π½ΠΈΡ†Ρ‹Β» (синус).

    ΠšΡ€Π΅ΡΡ‚Π½ΠΎΠ΅ ΠΏΡ€ΠΎΠΈΠ·Π²Π΅Π΄Π΅Π½ΠΈΠ΅ для ΠΎΡ€Ρ‚ΠΎΠ³ΠΎΠ½Π°Π»ΡŒΠ½Ρ‹Ρ… Π²Π΅ΠΊΡ‚ΠΎΡ€ΠΎΠ²

    Π§Ρ‚ΠΎΠ±Ρ‹ Π·Π°ΠΏΠΎΠΌΠ½ΠΈΡ‚ΡŒ ΠΏΡ€Π°Π²ΠΈΠ»ΠΎ ΠΏΡ€Π°Π²ΠΎΠΉ Ρ€ΡƒΠΊΠΈ, Π½Π°ΠΏΠΈΡˆΠΈΡ‚Π΅ порядок xyz Π΄Π²Π°ΠΆΠ΄Ρ‹: xyzxyz . Π—Π°Ρ‚Π΅ΠΌ Π½Π°ΠΉΠ΄ΠΈΡ‚Π΅ ΡƒΠ·ΠΎΡ€, ΠΊΠΎΡ‚ΠΎΡ€Ρ‹ΠΉ Π²Ρ‹ ΠΈΡ‰Π΅Ρ‚Π΅:

    • xy => z ( x крСст y is z )
    • yz => x ( y cross z is x ; ΠΌΡ‹ Π·Π°Ρ†ΠΈΠΊΠ»ΠΈΠ»ΠΈΡΡŒ Π½Π°: y to z to x )
    • zx => y

    Π’Π΅ΠΏΠ΅Ρ€ΡŒ, xy ΠΈ yx ΠΈΠΌΠ΅ΡŽΡ‚ ΠΏΡ€ΠΎΡ‚ΠΈΠ²ΠΎΠΏΠΎΠ»ΠΎΠΆΠ½Ρ‹Π΅ Π·Π½Π°ΠΊΠΈ, ΠΏΠΎΡ‚ΠΎΠΌΡƒ Ρ‡Ρ‚ΠΎ Π² нашСй настройкС xyzxyz ΠΎΠ½ΠΈ ΠΈΠ΄ΡƒΡ‚ Π²ΠΏΠ΅Ρ€Π΅Π΄ ΠΈ Π½Π°Π·Π°Π΄.

    Π˜Ρ‚Π°ΠΊ, Π±Π΅Π· Ρ„ΠΎΡ€ΠΌΡƒΠ»Ρ‹ Π²Ρ‹ смоТСтС Π²Ρ‹Ρ‡ΠΈΡΠ»ΠΈΡ‚ΡŒ:

    ΠžΠΏΡΡ‚ΡŒ ΠΆΠ΅, это ΠΏΠΎΡ‚ΠΎΠΌΡƒ, Ρ‡Ρ‚ΠΎ пСрСсСчСниС x y ΠΏΠΎΠ»ΠΎΠΆΠΈΡ‚Π΅Π»ΡŒΠ½ΠΎ z Π² ΠΏΡ€Π°Π²ΠΎΠΉ систСмС ΠΊΠΎΠΎΡ€Π΄ΠΈΠ½Π°Ρ‚. Π― использовал Π΅Π΄ΠΈΠ½ΠΈΡ‡Π½Ρ‹Π΅ Π²Π΅ΠΊΡ‚ΠΎΡ€Ρ‹, Π½ΠΎ ΠΌΡ‹ ΠΌΠΎΠ³Π»ΠΈ ΠΌΠ°ΡΡˆΡ‚Π°Π±ΠΈΡ€ΠΎΠ²Π°Ρ‚ΡŒ Ρ‚Π΅Ρ€ΠΌΠΈΠ½Ρ‹:

    ВычислСниС пСрСкрСстного произвСдСния

    Один Π²Π΅ΠΊΡ‚ΠΎΡ€ ΠΌΠΎΠΆΠ½ΠΎ Ρ€Π°Π·Π»ΠΎΠΆΠΈΡ‚ΡŒ Π½Π° 3 ΠΎΡ€Ρ‚ΠΎΠ³ΠΎΠ½Π°Π»ΡŒΠ½Ρ‹Π΅ части:

    Когда Π²Π΅ΠΊΡ‚ΠΎΡ€Ρ‹ ΠΏΠ΅Ρ€Π΅ΡΠ΅ΠΊΠ°ΡŽΡ‚ΡΡ, каТдая ΠΏΠ°Ρ€Π° ΠΎΡ€Ρ‚ΠΎΠ³ΠΎΠ½Π°Π»ΡŒΠ½Ρ‹Ρ… ΠΊΠΎΠΌΠΏΠΎΠ½Π΅Π½Ρ‚ΠΎΠ² (Π½Π°ΠΏΡ€ΠΈΠΌΠ΅Ρ€, $ a_x \ times b_y $) голосуСт Π·Π° Ρ‚ΠΎ, ΠΊΡƒΠ΄Π° Π΄ΠΎΠ»ΠΆΠ΅Π½ ΡƒΠΊΠ°Π·Ρ‹Π²Π°Ρ‚ΡŒ ΠΎΡ€Ρ‚ΠΎΠ³ΠΎΠ½Π°Π»ΡŒΠ½Ρ‹ΠΉ Π²Π΅ΠΊΡ‚ΠΎΡ€.6 ΠΊΠΎΠΌΠΏΠΎΠ½Π΅Π½Ρ‚ΠΎΠ², 6 голосов, ΠΈ ΠΈΡ… сумма являСтся пСрСкрСстным ΠΏΡ€ΠΎΠΈΠ·Π²Π΅Π΄Π΅Π½ΠΈΠ΅ΠΌ. (Подобно Π³Ρ€Π°Π΄ΠΈΠ΅Π½Ρ‚Ρƒ, Π³Π΄Π΅ каТдая ось голосуСт Π·Π° Π½Π°ΠΏΡ€Π°Π²Π»Π΅Π½ΠΈΠ΅ наибольшСго увСличСния.)

    • xy => z ΠΈ yx => -z (ΠΏΡ€Π΅Π΄ΠΏΠΎΠ»ΠΎΠΆΠΈΠΌ, Ρ‡Ρ‚ΠΎ $ \ vec {a} $ стоит ΠΏΠ΅Ρ€Π²Ρ‹ΠΌ, поэтому xy ΠΎΠ·Π½Π°Ρ‡Π°Π΅Ρ‚ $ a_x b_y $)
    • yz => x ΠΈ zy => -x
    • zx => y ΠΈ xz => -y

    xy ΠΈ yx ΡΡ€Π°ΠΆΠ°ΡŽΡ‚ΡΡ Π² Π½Π°ΠΏΡ€Π°Π²Π»Π΅Π½ΠΈΠΈ z .Если эти Ρ‡Π»Π΅Π½Ρ‹ Ρ€Π°Π²Π½Ρ‹, Π½Π°ΠΏΡ€ΠΈΠΌΠ΅Ρ€, Π² $ (2, 1, 0) \ times (2, 1, 1) $, Ρ‚ΠΎ Π² Π½Π°ΠΏΡ€Π°Π²Π»Π΅Π½ΠΈΠΈ z отсутствуСт ΠΊΠΎΠΌΠΏΠΎΠ½Π΅Π½Ρ‚ пСрСкрСстного произвСдСния (2 — 2 = 0).

    ΠžΠΊΠΎΠ½Ρ‡Π°Ρ‚Π΅Π»ΡŒΠ½Π°Ρ комбинация:

    , Π³Π΄Π΅ $ \ vec {n} $ — Π΅Π΄ΠΈΠ½ΠΈΡ‡Π½Ρ‹ΠΉ Π²Π΅ΠΊΡ‚ΠΎΡ€, Π½ΠΎΡ€ΠΌΠ°Π»ΡŒΠ½Ρ‹ΠΉ ΠΊ $ \ vec {a} $ ΠΈ $ \ vec {b} $.

    ΠŸΡƒΡΡ‚ΡŒ вас это Π½Π΅ ΠΏΡƒΠ³Π°Π΅Ρ‚:

    • Π•ΡΡ‚ΡŒ 6 Ρ‡Π»Π΅Π½ΠΎΠ², 3 ΠΏΠΎΠ»ΠΎΠΆΠΈΡ‚Π΅Π»ΡŒΠ½Ρ‹Ρ… ΠΈ 3 ΠΎΡ‚Ρ€ΠΈΡ†Π°Ρ‚Π΅Π»ΡŒΠ½Ρ‹Ρ…
    • Π”Π²Π° измСрСния Π³ΠΎΠ»ΠΎΡΡƒΡŽΡ‚ Π·Π° Ρ‚Ρ€Π΅Ρ‚ΡŒΠ΅ (поэтому Ρ‚Π΅Ρ€ΠΌΠΈΠ½ z Π΄ΠΎΠ»ΠΆΠ΅Π½ ΡΠΎΠ΄Π΅Ρ€ΠΆΠ°Ρ‚ΡŒ Ρ‚ΠΎΠ»ΡŒΠΊΠΎ ΠΊΠΎΠΌΠΏΠΎΠ½Π΅Π½Ρ‚Ρ‹ y ΠΈ x )
    • ΠŸΠΎΠ»ΠΎΠΆΠΈΡ‚Π΅Π»ΡŒΠ½Ρ‹ΠΉ / ΠΎΡ‚Ρ€ΠΈΡ†Π°Ρ‚Π΅Π»ΡŒΠ½Ρ‹ΠΉ порядок основан Π½Π° шаблонС xyzxyz

    Если Ρ…ΠΎΡ‚ΠΈΡ‚Π΅, сущСствуСт алгСбраичСскоС Π΄ΠΎΠΊΠ°Π·Π°Ρ‚Π΅Π»ΡŒΡΡ‚Π²ΠΎ Ρ‚ΠΎΠ³ΠΎ, Ρ‡Ρ‚ΠΎ Ρ„ΠΎΡ€ΠΌΡƒΠ»Π° ΠΎΠ΄Π½ΠΎΠ²Ρ€Π΅ΠΌΠ΅Π½Π½ΠΎ ΠΎΡ€Ρ‚ΠΎΠ³ΠΎΠ½Π°Π»ΡŒΠ½Π° ΠΈ ΠΈΠΌΠ΅Π΅Ρ‚ Ρ€Π°Π·ΠΌΠ΅Ρ€ $ | a | | Π± | \ sin (\ theta) $, Π½ΠΎ ΠΌΠ½Π΅ нравится интуиция Β«ΠΏΡ€ΠΎΠΏΠΎΡ€Ρ†ΠΈΠΎΠ½Π°Π»ΡŒΠ½ΠΎΠ³ΠΎ голосования».

    ВрСмя ΠΏΡ€ΠΈΠΌΠ΅Ρ€Π°

    ΠžΠΏΡΡ‚ΡŒ ΠΆΠ΅, ΠΌΡ‹ Π΄ΠΎΠ»ΠΆΠ½Ρ‹ Π΄Π΅Π»Π°Ρ‚ΡŒ простыС пСрСкрСстныС произвСдСния Π² ΡƒΠΌΠ΅:

    ΠŸΠΎΡ‡Π΅ΠΌΡƒ? ΠœΡ‹ пСрСсСкли оси x ΠΈ y , ΠΏΠΎΠ»ΡƒΡ‡ΠΈΠ² z (ΠΈΠ»ΠΈ $ \ vec {i} \ times \ vec {j} = \ vec {k} $, ΠΈΡΠΏΠΎΠ»ΡŒΠ·ΡƒΡ эти Π΅Π΄ΠΈΠ½ΠΈΡ‡Π½Ρ‹Π΅ Π²Π΅ΠΊΡ‚ΠΎΡ€Ρ‹). ΠŸΠ΅Ρ€Π΅Ρ…ΠΎΠ΄ Π² Π΄Ρ€ΡƒΠ³ΡƒΡŽ сторону Π΄Π°Π΅Ρ‚ $ — \ vec {k} $.

    Π’ΠΎΡ‚ ΠΊΠ°ΠΊ я Ρ€Π°ΡΡΠΌΠΎΡ‚Ρ€ΡŽ Π±ΠΎΠ»Π΅Π΅ слоТныС ΠΏΡ€ΠΈΠΌΠ΅Ρ€Ρ‹:

    • Π”Π°Π²Π°ΠΉΡ‚Π΅ займСмся послСдним Ρ‡Π»Π΅Π½ΠΎΠΌ, z-ΠΊΠΎΠΌΠΏΠΎΠ½Π΅Π½Ρ‚ΠΎΠΌ. Π­Ρ‚ΠΎ (1) (5) минус (4) (2), ΠΈΠ»ΠΈ 5-8 = -3.Π― сдСлал z ΠΏΠ΅Ρ€Π²Ρ‹ΠΌ, ΠΏΠΎΡ‚ΠΎΠΌΡƒ Ρ‡Ρ‚ΠΎ ΠΎΠ½ ΠΈΡΠΏΠΎΠ»ΡŒΠ·ΡƒΠ΅Ρ‚ x ΠΈ y , ΠΏΠ΅Ρ€Π²Ρ‹Π΅ Π΄Π²Π° Ρ‡Π»Π΅Π½Π°. ΠŸΠΎΠΏΡ€ΠΎΠ±ΡƒΠΉΡ‚Π΅ Ρ€Π°ΡΡΠΌΠ°Ρ‚Ρ€ΠΈΠ²Π°Ρ‚ΡŒ (1) (5) ΠΊΠ°ΠΊ Β«Π²ΠΏΠ΅Ρ€Π΅Π΄Β» ΠΏΡ€ΠΈ сканировании ΠΎΡ‚ ΠΏΠ΅Ρ€Π²ΠΎΠ³ΠΎ Π²Π΅ΠΊΡ‚ΠΎΡ€Π° ΠΊΠΎ Π²Ρ‚ΠΎΡ€ΠΎΠΌΡƒ, ΠΈ (4) (2) ΠΊΠ°ΠΊ Β«Π½Π°Π·Π°Π΄Β» ΠΏΡ€ΠΈ ΠΏΠ΅Ρ€Π΅ΠΌΠ΅Ρ‰Π΅Π½ΠΈΠΈ ΠΎΡ‚ Π²Ρ‚ΠΎΡ€ΠΎΠ³ΠΎ Π²Π΅ΠΊΡ‚ΠΎΡ€Π° ΠΊ ΠΏΠ΅Ρ€Π²ΠΎΠΌΡƒ.
    • Π’Π΅ΠΏΠ΅Ρ€ΡŒ ΠΊΠΎΠΌΠΏΠΎΠ½Π΅Π½Ρ‚ y : (3) (4) — (6) (1) = 12-6 = 6
    • Π’Π΅ΠΏΠ΅Ρ€ΡŒ ΠΊΠΎΠΌΠΏΠΎΠ½Π΅Π½Ρ‚ x : (2) (6) — (5) (3) = 12-15 = -3

    Π˜Ρ‚Π°ΠΊ, общая сумма $ (- 3, 6, -3) $, ΠΊΠΎΡ‚ΠΎΡ€ΡƒΡŽ ΠΌΡ‹ ΠΌΠΎΠΆΠ΅ΠΌ ΠΏΡ€ΠΎΠ²Π΅Ρ€ΠΈΡ‚ΡŒ с ΠΏΠΎΠΌΠΎΡ‰ΡŒΡŽ Wolfram Alpha.

    Π’ΠΊΡ€Π°Ρ‚Ρ†Π΅:

    • ΠŸΠ΅Ρ€Π΅ΠΊΡ€Π΅ΡΡ‚Π½ΠΎΠ΅ ΠΏΡ€ΠΎΠΈΠ·Π²Π΅Π΄Π΅Π½ΠΈΠ΅ отслСТиваСт всС «пСрСкрСстныС взаимодСйствия» ΠΌΠ΅ΠΆΠ΄Ρƒ измСрСниями
    • Π˜ΠΌΠ΅Π΅Ρ‚ΡΡ 6 взаимодСйствий (ΠΏΠΎ 2 Π² ΠΊΠ°ΠΆΠ΄ΠΎΠΌ ΠΈΠ·ΠΌΠ΅Ρ€Π΅Π½ΠΈΠΈ) со Π·Π½Π°ΠΊΠ°ΠΌΠΈ Π½Π° основС порядка xyzxyz

    ΠŸΡ€ΠΈΠ»ΠΎΠΆΠ΅Π½ΠΈΠ΅

    Бвязь с ΠΎΠΏΡ€Π΅Π΄Π΅Π»ΠΈΡ‚Π΅Π»Π΅ΠΌ

    Π’Ρ‹ ΠΌΠΎΠΆΠ΅Ρ‚Π΅ Π²Ρ‹Ρ‡ΠΈΡΠ»ΠΈΡ‚ΡŒ пСрСкрСстноС ΠΏΡ€ΠΎΠΈΠ·Π²Π΅Π΄Π΅Π½ΠΈΠ΅, ΠΈΡΠΏΠΎΠ»ΡŒΠ·ΡƒΡ ΠΎΠΏΡ€Π΅Π΄Π΅Π»ΠΈΡ‚Π΅Π»ΡŒ этой ΠΌΠ°Ρ‚Ρ€ΠΈΡ†Ρ‹:

    Π—Π΄Π΅ΡΡŒ Π΅ΡΡ‚ΡŒ чСткая связь, ΠΏΠΎΡΠΊΠΎΠ»ΡŒΠΊΡƒ ΠΎΠΏΡ€Π΅Π΄Π΅Π»ΠΈΡ‚Π΅Π»ΡŒ («подписанная ΠΏΠ»ΠΎΡ‰Π°Π΄ΡŒ / объСм») отслСТиваСт Π²ΠΊΠ»Π°Π΄Ρ‹ ΠΎΡ‚ ΠΎΡ€Ρ‚ΠΎΠ³ΠΎΠ½Π°Π»ΡŒΠ½Ρ‹Ρ… ΠΊΠΎΠΌΠΏΠΎΠ½Π΅Π½Ρ‚ΠΎΠ².

    Π‘ΡƒΡ‰Π΅ΡΡ‚Π²ΡƒΡŽΡ‚ тСорСтичСскиС ΠΏΡ€ΠΈΡ‡ΠΈΠ½Ρ‹, ΠΏΠΎ ΠΊΠΎΡ‚ΠΎΡ€Ρ‹ΠΌ Π²Π΅ΠΊΡ‚ΠΎΡ€Π½ΠΎΠ΅ ΠΏΡ€ΠΎΠΈΠ·Π²Π΅Π΄Π΅Π½ΠΈΠ΅ (ΠΊΠ°ΠΊ ΠΎΡ€Ρ‚ΠΎΠ³ΠΎΠ½Π°Π»ΡŒΠ½Ρ‹ΠΉ Π²Π΅ΠΊΡ‚ΠΎΡ€) доступно Ρ‚ΠΎΠ»ΡŒΠΊΠΎ Π² 0, 1, 3 ΠΈΠ»ΠΈ 7 измСрСниях. Однако пСрСкрСстноС ΠΏΡ€ΠΎΠΈΠ·Π²Π΅Π΄Π΅Π½ΠΈΠ΅ Π² Π²ΠΈΠ΄Π΅ СдинствСнного числа ΠΏΠΎ сути являСтся ΠΎΠΏΡ€Π΅Π΄Π΅Π»ΡΡŽΡ‰ΠΈΠΌ Ρ„Π°ΠΊΡ‚ΠΎΡ€ΠΎΠΌ (ΠΏΠ»ΠΎΡ‰Π°Π΄ΡŒ со Π·Π½Π°ΠΊΠΎΠΌ, объСм ΠΈΠ»ΠΈ Π³ΠΈΠΏΠ΅Ρ€ΠΎΠ±ΡŠΠ΅ΠΌ Π² Π²ΠΈΠ΄Π΅ скаляра).

    БоСдинСниС с Curl

    Curl измСряСт ΡΠΊΡ€ΡƒΡ‡ΠΈΠ²Π°ΡŽΡ‰ΡƒΡŽ силу, ΠΊΠΎΡ‚ΠΎΡ€ΡƒΡŽ Π²Π΅ΠΊΡ‚ΠΎΡ€Π½ΠΎΠ΅ ΠΏΠΎΠ»Π΅ ΠΏΡ€ΠΈΠΊΠ»Π°Π΄Ρ‹Π²Π°Π΅Ρ‚ ΠΊ Ρ‚ΠΎΡ‡ΠΊΠ΅, ΠΈ измСряСтся Π²Π΅ΠΊΡ‚ΠΎΡ€ΠΎΠΌ, пСрпСндикулярным повСрхности. Всякий Ρ€Π°Π·, ΠΊΠΎΠ³Π΄Π° Π²Ρ‹ ΡΠ»Ρ‹ΡˆΠΈΡ‚Π΅ «пСрпСндикулярный Π²Π΅ΠΊΡ‚ΠΎΡ€Β», Π½Π°Ρ‡ΠΈΠ½Π°ΠΉΡ‚Π΅ Π΄ΡƒΠΌΠ°Ρ‚ΡŒ «кросс-ΠΏΡ€ΠΎΠΈΠ·Π²Π΅Π΄Π΅Π½ΠΈΠ΅Β».

    Π‘Π΅Ρ€Π΅ΠΌ Β«ΠΎΠΏΡ€Π΅Π΄Π΅Π»ΠΈΡ‚Π΅Π»ΡŒΒ» этой ΠΌΠ°Ρ‚Ρ€ΠΈΡ†Ρ‹:

    ВмСсто умноТСния взаимодСйствиС ΠΏΡ€ΠΈΠ½ΠΈΠΌΠ°Π΅Ρ‚ Ρ‡Π°ΡΡ‚Π½ΡƒΡŽ ΠΏΡ€ΠΎΠΈΠ·Π²ΠΎΠ΄Π½ΡƒΡŽ. Как ΠΈ ΠΏΡ€Π΅ΠΆΠ΄Π΅, $ \ vec {i} $ ΠΊΠΎΠΌΠΏΠΎΠ½Π΅Π½Ρ‚ curl основан Π½Π° Π²Π΅ΠΊΡ‚ΠΎΡ€Π°Ρ… ΠΈ ΠΏΡ€ΠΎΠΈΠ·Π²ΠΎΠ΄Π½Ρ‹Ρ… Π² направлСниях $ \ vec {j} $ ΠΈ $ \ vec {k} $.

    Бвязь с Ρ‚Π΅ΠΎΡ€Π΅ΠΌΠΎΠΉ ΠŸΠΈΡ„Π°Π³ΠΎΡ€Π°

    ΠšΡ€Π΅ΡΡ‚ ΠΈ скалярноС ΠΏΡ€ΠΎΠΈΠ·Π²Π΅Π΄Π΅Π½ΠΈΠ΅ ΠΏΠΎΠ΄ΠΎΠ±Π½Ρ‹ ΠΎΡ€Ρ‚ΠΎΠ³ΠΎΠ½Π°Π»ΡŒΠ½Ρ‹ΠΌ сторонам Ρ‚Ρ€Π΅ΡƒΠ³ΠΎΠ»ΡŒΠ½ΠΈΠΊΠ°:

    Для Π΅Π΄ΠΈΠ½ΠΈΡ‡Π½Ρ‹Ρ… Π²Π΅ΠΊΡ‚ΠΎΡ€ΠΎΠ², Π³Π΄Π΅ $ | a | = | b | = 1 $, ΠΈΠΌΠ΅Π΅ΠΌ:

    Π― Π½Π΅ΠΌΠ½ΠΎΠ³ΠΎ схитрил Π² сСточной Π΄ΠΈΠ°Π³Ρ€Π°ΠΌΠΌΠ΅, Ρ‚Π°ΠΊ ΠΊΠ°ΠΊ ΠΌΡ‹ Π΄ΠΎΠ»ΠΆΠ½Ρ‹ ΠΎΡ‚ΡΠ»Π΅ΠΆΠΈΠ²Π°Ρ‚ΡŒ ΠΊΠ²Π°Π΄Ρ€Π°Ρ‚Ρ‹ Π²Π΅Π»ΠΈΡ‡ΠΈΠ½ (ΠΊΠ°ΠΊ это сдСлано Π² Ρ‚Π΅ΠΎΡ€Π΅ΠΌΠ΅ ΠŸΠΈΡ„Π°Π³ΠΎΡ€Π°).

    ΠŸΡ€ΠΎΠ΄Π²ΠΈΠ½ΡƒΡ‚Π°Ρ ΠΌΠ°Ρ‚Π΅ΠΌΠ°Ρ‚ΠΈΠΊΠ°

    ΠŸΠ΅Ρ€Π΅ΠΊΡ€Π΅ΡΡ‚Π½ΠΎΠ΅ ΠΏΡ€ΠΎΠΈΠ·Π²Π΅Π΄Π΅Π½ΠΈΠ΅ ΠΈ Π΄Ρ€ΡƒΠ·ΡŒΡ Ρ€Π°ΡΡˆΠΈΡ€ΡΡŽΡ‚ΡΡ Π² Π°Π»Π³Π΅Π±Ρ€Π΅ ΠšΠ»ΠΈΡ„Ρ„ΠΎΡ€Π΄Π° ΠΈ гСомСтричСской Π°Π»Π³Π΅Π±Ρ€Π΅. Π― всС Π΅Ρ‰Π΅ ΠΈΠ·ΡƒΡ‡Π°ΡŽ это.

    ΠŸΠ΅Ρ€Π΅ΠΊΡ€Π΅ΡΡ‚Π½Ρ‹Π΅ произвСдСния пСрСкрСстных ΠΏΡ€ΠΎΠ΄ΡƒΠΊΡ‚ΠΎΠ²

    Иногда Π±Ρ‹Π²Π°Π΅Ρ‚ Ρ‚Π°ΠΊΠΎΠΉ сцСнарий:

    Π’ΠΎ-ΠΏΠ΅Ρ€Π²Ρ‹Ρ…, пСрСкрСстноС ΠΏΡ€ΠΎΠΈΠ·Π²Π΅Π΄Π΅Π½ΠΈΠ΅ Π½Π΅ ассоциативно: порядок ΠΈΠΌΠ΅Π΅Ρ‚ Π·Π½Π°Ρ‡Π΅Π½ΠΈΠ΅.

    Π—Π°Ρ‚Π΅ΠΌ вспомнитС, Ρ‡Ρ‚ΠΎ Π΄Π΅Π»Π°Π΅Ρ‚ кросс-ΠΏΡ€ΠΎΠΈΠ·Π²Π΅Π΄Π΅Π½ΠΈΠ΅: Π½Π°Ρ…ΠΎΠΆΠ΄Π΅Π½ΠΈΠ΅ ΠΎΡ€Ρ‚ΠΎΠ³ΠΎΠ½Π°Π»ΡŒΠ½Ρ‹Ρ… Π²Π΅ΠΊΡ‚ΠΎΡ€ΠΎΠ². Если Π»ΡŽΠ±Ρ‹Π΅ Π΄Π²Π° ΠΊΠΎΠΌΠΏΠΎΠ½Π΅Π½Ρ‚Π° ΠΏΠ°Ρ€Π°Π»Π»Π΅Π»ΡŒΠ½Ρ‹ ($ \ vec {a} $ ΠΏΠ°Ρ€Π°Π»Π»Π΅Π»ΡŒΠ½ΠΎ $ \ vec {b} $), Ρ‚ΠΎ Π½ΠΈΠΊΠ°ΠΊΠΈΠ΅ измСрСния Π½Π΅ Π²Π»ΠΈΡΡŽΡ‚ Π΄Ρ€ΡƒΠ³ Π½Π° Π΄Ρ€ΡƒΠ³Π°, ΠΈ пСрСкрСстноС ΠΏΡ€ΠΎΠΈΠ·Π²Π΅Π΄Π΅Π½ΠΈΠ΅ Ρ€Π°Π²Π½ΠΎ Π½ΡƒΠ»ΡŽ (Ρ‡Ρ‚ΠΎ пСрСносится Π½Π° $ 0 \ times \ vec {c} $).

    Но это Π½ΠΎΡ€ΠΌΠ°Π»ΡŒΠ½ΠΎ для $ \ vec {a} $ ΠΈ $ \ vec {c} $ Π±Ρ‹Ρ‚ΡŒ ΠΏΠ°Ρ€Π°Π»Π»Π΅Π»ΡŒΠ½Ρ‹ΠΌΠΈ, ΠΏΠΎΡΠΊΠΎΠ»ΡŒΠΊΡƒ ΠΎΠ½ΠΈ Π½ΠΈΠΊΠΎΠ³Π΄Π° Π½Π΅ ΡƒΡ‡Π°ΡΡ‚Π²ΡƒΡŽΡ‚ Π½Π°ΠΏΡ€ΡΠΌΡƒΡŽ Π² пСрСкрСстном ΠΏΡ€ΠΎΠΈΠ·Π²Π΅Π΄Π΅Π½ΠΈΠΈ, Π½Π°ΠΏΡ€ΠΈΠΌΠ΅Ρ€:

    Π£Ρ… Ρ‚Ρ‹! Как Π½Π°ΠΌ Π²Π΅Ρ€Π½ΡƒΡ‚ΡŒΡΡ ΠΊ $ \ vec {j} $? ΠœΡ‹ запросили Π½Π°ΠΏΡ€Π°Π²Π»Π΅Π½ΠΈΠ΅, пСрпСндикулярноС ΠΊΠ°ΠΊ $ \ vec {i} $, Ρ‚Π°ΠΊ ΠΈ $ \ vec {j} $, ΠΈ снова сдСлали это Π½Π°ΠΏΡ€Π°Π²Π»Π΅Π½ΠΈΠ΅ пСрпСндикулярным $ \ vec {i} $. «Двойная ΠΏΠ΅Ρ€ΠΏΠ΅Π½Π΄ΠΈΠΊΡƒΠ»ΡΡ€Π½ΠΎΡΡ‚ΡŒΒ» ΠΎΠ·Π½Π°Ρ‡Π°Π΅Ρ‚, Ρ‡Ρ‚ΠΎ Π²Ρ‹ Π²Π΅Ρ€Π½ΡƒΠ»ΠΈΡΡŒ Π½Π° ΠΈΡΡ…ΠΎΠ΄Π½ΡƒΡŽ ось.

    Π’ΠΎΡ‡Π΅Ρ‡Π½ΠΎΠ΅ ΠΏΡ€ΠΎΠΈΠ·Π²Π΅Π΄Π΅Π½ΠΈΠ΅ пСрСкрСстных ΠΏΡ€ΠΎΠΈΠ·Π²Π΅Π΄Π΅Π½ΠΈΠΉ

    Π’Π΅ΠΏΠ΅Ρ€ΡŒ возьмСм

    Ρ‡Ρ‚ΠΎ происходит? ΠœΡ‹ Π²Ρ‹Π½ΡƒΠΆΠ΄Π΅Π½Ρ‹ сначала Π²Ρ‹ΠΏΠΎΠ»Π½ΠΈΡ‚ΡŒ $ \ vec {a} \ times \ vec {b} $, ΠΏΠΎΡ‚ΠΎΠΌΡƒ Ρ‡Ρ‚ΠΎ $ \ vec {b} \ cdot \ vec {c} $ Π²ΠΎΠ·Π²Ρ€Π°Ρ‰Π°Π΅Ρ‚ скаляр (ΠΎΠ΄Π½ΠΎ число), ΠΊΠΎΡ‚ΠΎΡ€ΠΎΠ΅ нСльзя ΠΈΡΠΏΠΎΠ»ΡŒΠ·ΠΎΠ²Π°Ρ‚ΡŒ Π² пСрСкрСстном ΠΏΡ€ΠΎΠΈΠ·Π²Π΅Π΄Π΅Π½ΠΈΠΈ.

    Если $ \ vec {a} $ ΠΈ $ \ vec {c} $ ΠΏΠ°Ρ€Π°Π»Π»Π΅Π»ΡŒΠ½Ρ‹, Ρ‡Ρ‚ΠΎ ΠΏΡ€ΠΎΠΈΠ·ΠΎΠΉΠ΄Π΅Ρ‚? Π˜Ρ‚Π°ΠΊ, $ \ vec {a} \ times \ vec {b} $ пСрпСндикулярно $ \ vec {a} $, Ρ‡Ρ‚ΠΎ ΠΎΠ·Π½Π°Ρ‡Π°Π΅Ρ‚, Ρ‡Ρ‚ΠΎ ΠΎΠ½ΠΎ пСрпСндикулярно $ \ vec {c} $, поэтому скалярноС ΠΏΡ€ΠΎΠΈΠ·Π²Π΅Π΄Π΅Π½ΠΈΠ΅ с $ \ vec {c } $ Π±ΡƒΠ΄Π΅Ρ‚ Π½ΡƒΠ»Π΅ΠΌ.

    Π― Π½ΠΈΠΊΠΎΠ³Π΄Π° особо Π½Π΅ Π·Π°ΠΏΠΎΠΌΠΈΠ½Π°Π» эти ΠΏΡ€Π°Π²ΠΈΠ»Π°, я Π΄ΠΎΠ»ΠΆΠ΅Π½ ΠΏΡ€ΠΎΠ΄ΡƒΠΌΡ‹Π²Π°Ρ‚ΡŒ взаимодСйствия.

    Π”Ρ€ΡƒΠ³ΠΈΠ΅ систСмы ΠΊΠΎΠΎΡ€Π΄ΠΈΠ½Π°Ρ‚

    Π”Π²ΠΈΠΆΠΎΠΊ Unity — лСвша, OpenGL (ΠΈ Π±ΠΎΠ»ΡŒΡˆΠΈΠ½ΡΡ‚Π²ΠΎ матСматичСских / физичСских инструмСнтов) — ΠΏΡ€Π°Π²ΡˆΠ°. ΠŸΠΎΡ‡Π΅ΠΌΡƒ?

    Π’ ΠΊΠΎΠΌΠΏΡŒΡŽΡ‚Π΅Ρ€Π½ΠΎΠΉ ΠΈΠ³Ρ€Π΅ x ΠΈΠ΄Π΅Ρ‚ Π³ΠΎΡ€ΠΈΠ·ΠΎΠ½Ρ‚Π°Π»ΡŒΠ½ΠΎ, y ΠΈΠ΄Π΅Ρ‚ Π²Π΅Ρ€Ρ‚ΠΈΠΊΠ°Π»ΡŒΠ½ΠΎ ΠΈ z ΠΈΠ΄Π΅Ρ‚ Β«Π² экран».Π­Ρ‚ΠΎ ΠΏΡ€ΠΈΠ²ΠΎΠ΄ΠΈΡ‚ ΠΊ лСвостороннСй систСмС. (ΠŸΠΎΠΏΡ€ΠΎΠ±ΡƒΠΉΡ‚Π΅: ΠΏΡ€Π°Π²ΠΎΠΉ Ρ€ΡƒΠΊΠΎΠΉ Π²Ρ‹ ΡƒΠ²ΠΈΠ΄ΠΈΡ‚Π΅ крСст Ρ€Π°Π·ΠΌΠ΅Ρ€ΠΎΠΌ x , y Π΄ΠΎΠ»ΠΆΠ΅Π½ ΡƒΠΊΠ°Π·Ρ‹Π²Π°Ρ‚ΡŒ Π·Π° ΠΏΡ€Π΅Π΄Π΅Π»Ρ‹ экрана).

    ΠŸΡ€ΠΈΠ»ΠΎΠΆΠ΅Π½ΠΈΡ пСрСкрСстного ΠΏΡ€ΠΎΠ΄ΡƒΠΊΡ‚Π°

    • НайдитС Π½Π°ΠΏΡ€Π°Π²Π»Π΅Π½ΠΈΠ΅, пСрпСндикулярноС Π΄Π²ΡƒΠΌ Π·Π°Π΄Π°Π½Π½Ρ‹ΠΌ Π²Π΅ΠΊΡ‚ΠΎΡ€Π°ΠΌ.
    • НайдитС ΠΏΠ»ΠΎΡ‰Π°Π΄ΡŒ со Π·Π½Π°ΠΊΠΎΠΌ, ΠΎΡ…Π²Π°Ρ‚Ρ‹Π²Π°Π΅ΠΌΡƒΡŽ двумя Π²Π΅ΠΊΡ‚ΠΎΡ€Π°ΠΌΠΈ.
    • ΠžΠΏΡ€Π΅Π΄Π΅Π»ΠΈΡ‚Π΅, ΠΎΡ€Ρ‚ΠΎΠ³ΠΎΠ½Π°Π»ΡŒΠ½Ρ‹ Π»ΠΈ Π΄Π²Π° Π²Π΅ΠΊΡ‚ΠΎΡ€Π° (хотя ΠΏΡ€ΠΎΠ²Π΅Ρ€ΠΊΠ° скалярного произвСдСния 0, вСроятно, быстрСС).
    • Β«Π£ΠΌΠ½ΠΎΠΆΡŒΡ‚Π΅Β» Π΄Π²Π° Π²Π΅ΠΊΡ‚ΠΎΡ€Π°, ΠΊΠΎΠ³Π΄Π° Π²ΠΊΠ»Π°Π΄ вносят Ρ‚ΠΎΠ»ΡŒΠΊΠΎ пСрпСндикулярныС ΠΏΠΎΠΏΠ΅Ρ€Π΅Ρ‡Π½Ρ‹Π΅ Ρ‡Π»Π΅Π½Ρ‹ (Π½Π°ΠΏΡ€ΠΈΠΌΠ΅Ρ€, ΠΎΠΏΡ€Π΅Π΄Π΅Π»Π΅Π½ΠΈΠ΅ крутящСго ΠΌΠΎΠΌΠ΅Π½Ρ‚Π°).
    • Π‘ ΠΊΠ²Π°Ρ‚Π΅Ρ€Π½ΠΈΠΎΠ½Π°ΠΌΠΈ (4d комплСксныС числа) пСрСкрСстноС ΠΏΡ€ΠΎΠΈΠ·Π²Π΅Π΄Π΅Π½ΠΈΠ΅ выполняСт Ρ€Π°Π±ΠΎΡ‚Ρƒ ΠΏΠΎ Π²Ρ€Π°Ρ‰Π΅Π½ΠΈΡŽ ΠΎΠ΄Π½ΠΎΠ³ΠΎ Π²Π΅ΠΊΡ‚ΠΎΡ€Π° Π²ΠΎΠΊΡ€ΡƒΠ³ Π΄Ρ€ΡƒΠ³ΠΎΠ³ΠΎ (Π΅Ρ‰Π΅ ΠΎΠ΄Π½Π° ΡΡ‚Π°Ρ‚ΡŒΡ Π² Ρ€Π°Π·Ρ€Π°Π±ΠΎΡ‚ΠΊΠ΅!).

    Бчастливая ΠΌΠ°Ρ‚Π΅ΠΌΠ°Ρ‚ΠΈΠΊΠ°.

    Π”Ρ€ΡƒΠ³ΠΈΠ΅ сообщСния этой сСрии

    1. Π’Π΅ΠΊΡ‚ΠΎΡ€Π½ΠΎΠ΅ исчислСниС: ΠΏΠΎΠ½ΠΈΠΌΠ°Π½ΠΈΠ΅ Ρ‚ΠΎΡ‡Π΅Ρ‡Π½ΠΎΠ³ΠΎ произвСдСния
    2. Π’Π΅ΠΊΡ‚ΠΎΡ€Π½ΠΎΠ΅ исчислСниС: ΠΏΠΎΠ½ΠΈΠΌΠ°Π½ΠΈΠ΅ кросс-произвСдСния
    3. Π’Π΅ΠΊΡ‚ΠΎΡ€Π½ΠΎΠ΅ исчислСниС: ΠΏΠΎΠ½ΠΈΠΌΠ°Π½ΠΈΠ΅ ΠΏΠΎΡ‚ΠΎΠΊΠ°
    4. Π’Π΅ΠΊΡ‚ΠΎΡ€Π½ΠΎΠ΅ исчислСниС: ΠΏΠΎΠ½ΠΈΠΌΠ°Π½ΠΈΠ΅ расходимости
    5. Π’Π΅ΠΊΡ‚ΠΎΡ€Π½ΠΎΠ΅ исчислСниС: ΠΏΠΎΠ½ΠΈΠΌΠ°Π½ΠΈΠ΅ циркуляции ΠΈ ΠΈΠ·Π³ΠΈΠ±Π°
    6. Π’Π΅ΠΊΡ‚ΠΎΡ€Π½ΠΎΠ΅ исчислСниС: ΠΏΠΎΠ½ΠΈΠΌΠ°Π½ΠΈΠ΅ Π³Ρ€Π°Π΄ΠΈΠ΅Π½Ρ‚Π°
    7. ΠŸΠΈΡ„Π°Π³ΠΎΡ€Π΅ΠΉΡΠΊΠΎΠ΅ расстояниС ΠΈ Π³Ρ€Π°Π΄ΠΈΠ΅Π½Ρ‚

    17.2: Π’Π΅ΠΊΡ‚ΠΎΡ€Π½ΠΎΠ΅ ΠΏΡ€ΠΎΠΈΠ·Π²Π΅Π΄Π΅Π½ΠΈΠ΅ (пСрСкрСстноС ΠΏΡ€ΠΎΠΈΠ·Π²Π΅Π΄Π΅Π½ΠΈΠ΅)

    ΠŸΡƒΡΡ‚ΡŒ \ (\ overrightarrow {\ mathbf {A}} \) ΠΈ \ (\ overrightarrow {\ mathbf {B}} \) Π±ΡƒΠ΄ΡƒΡ‚ двумя Π²Π΅ΠΊΡ‚ΠΎΡ€Π°ΠΌΠΈ. ΠŸΠΎΡΠΊΠΎΠ»ΡŒΠΊΡƒ Π»ΡŽΠ±Ρ‹Π΅ Π΄Π²Π° Π½Π΅ΠΏΠ°Ρ€Π°Π»Π»Π΅Π»ΡŒΠ½Ρ‹Ρ… Π²Π΅ΠΊΡ‚ΠΎΡ€Π° ΠΎΠ±Ρ€Π°Π·ΡƒΡŽΡ‚ ΠΏΠ»ΠΎΡΠΊΠΎΡΡ‚ΡŒ, ΠΌΡ‹ ΠΎΠ±ΠΎΠ·Π½Π°Ρ‡Π°Π΅ΠΌ ΡƒΠ³ΠΎΠ» ΞΈ ΠΊΠ°ΠΊ ΡƒΠ³ΠΎΠ» ΠΌΠ΅ΠΆΠ΄Ρƒ Π²Π΅ΠΊΡ‚ΠΎΡ€Π°ΠΌΠΈ \ (\ overrightarrow {\ mathbf {A}} \) ΠΈ \ (\ overrightarrow {\ mathbf {B}} \) ΠΊΠ°ΠΊ ΠΏΠΎΠΊΠ°Π·Π°Π½ΠΎ Π½Π° рисункС 17.2. Π’Π΅Π»ΠΈΡ‡ΠΈΠ½Π° Π²Π΅ΠΊΡ‚ΠΎΡ€Π½ΠΎΠ³ΠΎ произвСдСния \ (\ overrightarrow {\ mathbf {A}} \ times \ overrightarrow {\ mathbf {B}} \) Π²Π΅ΠΊΡ‚ΠΎΡ€ΠΎΠ² \ (\ overrightarrow {\ mathbf {A}} \) ΠΈ \ (\ overrightarrow {\ mathbf {B}} \) опрСдСляСтся ΠΊΠ°ΠΊ ΠΏΡ€ΠΎΠΈΠ·Π²Π΅Π΄Π΅Π½ΠΈΠ΅ Π²Π΅Π»ΠΈΡ‡ΠΈΠ½Ρ‹ Π²Π΅ΠΊΡ‚ΠΎΡ€ΠΎΠ² \ (\ overrightarrow {\ mathbf {A}} \) ΠΈ \ (\ overrightarrow {\ mathbf {B}} \) с синусом ΡƒΠ³Π»Π° ΞΈ ΠΌΠ΅ΠΆΠ΄Ρƒ двумя Π²Π΅ΠΊΡ‚ΠΎΡ€Π°ΠΌΠΈ,

    \ [| \ overrightarrow {\ mathbf {A}} \ times \ overrightarrow {\ mathbf {B}} | = | \ overrightarrow {\ mathbf {A}} || \ overrightarrow {\ mathbf {B}} | \ sin (\ theta) \]

    Π£Π³ΠΎΠ» ΞΈ ΠΌΠ΅ΠΆΠ΄Ρƒ Π²Π΅ΠΊΡ‚ΠΎΡ€Π°ΠΌΠΈ ΠΎΠ³Ρ€Π°Π½ΠΈΡ‡Π΅Π½ значСниями \ (0 \ leq \ theta \ leq \ pi \), гарантируя, Ρ‡Ρ‚ΠΎ \ (\ sin (\ theta) \ geq 0 \).

    Рисунок 17.2 ГСомСтрия Π²Π΅ΠΊΡ‚ΠΎΡ€Π½ΠΎΠ³ΠΎ произвСдСния.

    НаправлСниС Π²Π΅ΠΊΡ‚ΠΎΡ€Π½ΠΎΠ³ΠΎ произвСдСния опрСдСляСтся ΡΠ»Π΅Π΄ΡƒΡŽΡ‰ΠΈΠΌ ΠΎΠ±Ρ€Π°Π·ΠΎΠΌ. Π’Π΅ΠΊΡ‚ΠΎΡ€Ρ‹ \ (\ overrightarrow {\ mathbf {A}} \) ΠΈ \ (\ overrightarrow {\ mathbf {B}} \) ΠΎΠ±Ρ€Π°Π·ΡƒΡŽΡ‚ ΠΏΠ»ΠΎΡΠΊΠΎΡΡ‚ΡŒ. Рассмотрим Π½Π°ΠΏΡ€Π°Π²Π»Π΅Π½ΠΈΠ΅, пСрпСндикулярноС этой плоскости. Π•ΡΡ‚ΡŒ Π΄Π²Π΅ возмоТности: ΠΌΡ‹ Π²Ρ‹Π±Π΅Ρ€Π΅ΠΌ ΠΎΠ΄Π½Ρƒ ΠΈΠ· этих Π΄Π²ΡƒΡ… (ΠΏΠΎΠΊΠ°Π·Π°Π½Π½ΡƒΡŽ Π½Π° рисункС 17.2) для направлСния Π²Π΅ΠΊΡ‚ΠΎΡ€Π½ΠΎΠ³ΠΎ произвСдСния \ (\ overrightarrow {\ mathbf {A}} \ times \ overrightarrow {\ mathbf {B}} \ ) с использованиСм соглашСния, ΠΊΠΎΡ‚ΠΎΡ€ΠΎΠ΅ ΠΎΠ±Ρ‹Ρ‡Π½ΠΎ Π½Π°Π·Ρ‹Π²Π°ΡŽΡ‚ Β«ΠΏΡ€Π°Π²ΠΈΠ»ΠΎΠΌ ΠΏΡ€Π°Π²ΠΎΠΉ Ρ€ΡƒΠΊΠΈΒ».

    ΠŸΡ€Π°Π²ΠΈΠ»ΠΎ ΠΏΡ€Π°Π²ΠΎΠΉ Ρ€ΡƒΠΊΠΈ для направлСния Π²Π΅ΠΊΡ‚ΠΎΡ€Π½ΠΎΠ³ΠΎ произвСдСния

    ΠŸΠ΅Ρ€Π²Ρ‹ΠΉ шаг — ΠΏΠ΅Ρ€Π΅Ρ€ΠΈΡΠΎΠ²Π°Ρ‚ΡŒ Π²Π΅ΠΊΡ‚ΠΎΡ€Ρ‹ \ (\ overrightarrow {\ mathbf {A}} \ text {ΠΈ} \ overrightarrow {\ mathbf {B}} \) Ρ‚Π°ΠΊ, Ρ‡Ρ‚ΠΎΠ±Ρ‹ хвосты ΡΠΎΠΏΡ€ΠΈΠΊΠ°ΡΠ°Π»ΠΈΡΡŒ. Π—Π°Ρ‚Π΅ΠΌ нарисуйтС Π΄ΡƒΠ³Ρƒ, начиная с Π²Π΅ΠΊΡ‚ΠΎΡ€Π° \ (\ overrightarrow {\ mathbf {A}} \) ΠΈ заканчивая Π²Π΅ΠΊΡ‚ΠΎΡ€ΠΎΠΌ \ (\ overrightarrow {\ mathbf {B}} \). Π‘ΠΎΠ³Π½ΠΈΡ‚Π΅ ΠΏΠ°Π»ΡŒΡ†Ρ‹ ΠΏΡ€Π°Π²ΠΎΠΉ Ρ€ΡƒΠΊΠΈ Ρ‚Π°ΠΊ ΠΆΠ΅, ΠΊΠ°ΠΊ Π΄ΡƒΠ³Ρƒ. Π’Π°Ρˆ большой ΠΏΠ°Π»Π΅Ρ† ΠΏΡ€Π°Π²ΠΎΠΉ Ρ€ΡƒΠΊΠΈ ΡƒΠΊΠ°Π·Ρ‹Π²Π°Π΅Ρ‚ Π² Π½Π°ΠΏΡ€Π°Π²Π»Π΅Π½ΠΈΠΈ Π²Π΅ΠΊΡ‚ΠΎΡ€Π½ΠΎΠ³ΠΎ произвСдСния \ (\ overrightarrow {\ mathbf {A}} \ times \ overrightarrow {\ mathbf {B}} \) (рис.17.3).

    Рисунок 17.3 ΠŸΡ€Π°Π²ΠΈΠ»ΠΎ ΠΏΡ€Π°Π²ΠΎΠΉ Ρ€ΡƒΠΊΠΈ.

    Π’Ρ‹ Π΄ΠΎΠ»ΠΆΠ½Ρ‹ ΠΏΠΎΠΌΠ½ΠΈΡ‚ΡŒ, Ρ‡Ρ‚ΠΎ Π½Π°ΠΏΡ€Π°Π²Π»Π΅Π½ΠΈΠ΅ Π²Π΅ΠΊΡ‚ΠΎΡ€Π½ΠΎΠ³ΠΎ произвСдСния \ (\ overrightarrow {\ mathbf {A}} \ times \ overrightarrow {\ mathbf {B}} \) пСрпСндикулярно плоскости, ΠΎΠ±Ρ€Π°Π·ΠΎΠ²Π°Π½Π½ΠΎΠΉ \ (\ overrightarrow {\ mathbf { A}} \ text {ΠΈ} \ overrightarrow {\ mathbf {B}} \). ΠœΡ‹ ΠΌΠΎΠΆΠ΅ΠΌ Π΄Π°Ρ‚ΡŒ Π³Π΅ΠΎΠΌΠ΅Ρ‚Ρ€ΠΈΡ‡Π΅ΡΠΊΡƒΡŽ ΠΈΠ½Ρ‚Π΅Ρ€ΠΏΡ€Π΅Ρ‚Π°Ρ†ΠΈΡŽ Π²Π΅Π»ΠΈΡ‡ΠΈΠ½Π΅ Π²Π΅ΠΊΡ‚ΠΎΡ€Π½ΠΎΠ³ΠΎ произвСдСния, записав Π²Π΅Π»ΠΈΡ‡ΠΈΠ½Ρƒ ΠΊΠ°ΠΊ

    .

    \ [| \ overrightarrow {\ mathbf {A}} \ times \ overrightarrow {\ mathbf {B}} | = | \ overrightarrow {\ mathbf {A}} | (| \ overrightarrow {\ mathbf {B}} | \ sin \ theta) \]

    Π’Π΅ΠΊΡ‚ΠΎΡ€Ρ‹ \ (\ overrightarrow {\ mathbf {A}} \ text {ΠΈ} \ overrightarrow {\ mathbf {B}} \) ΠΎΠ±Ρ€Π°Π·ΡƒΡŽΡ‚ ΠΏΠ°Ρ€Π°Π»Π»Π΅Π»ΠΎΠ³Ρ€Π°ΠΌΠΌ.ΠŸΠ»ΠΎΡ‰Π°Π΄ΡŒ ΠΏΠ°Ρ€Π°Π»Π»Π΅Π»ΠΎΠ³Ρ€Π°ΠΌΠΌΠ° Ρ€Π°Π²Π½Π° высотС, ΡƒΠΌΠ½ΠΎΠΆΠ΅Π½Π½ΠΎΠΉ Π½Π° основаниС, ΠΊΠΎΡ‚ΠΎΡ€ΠΎΠ΅ являСтся Π²Π΅Π»ΠΈΡ‡ΠΈΠ½ΠΎΠΉ Π²Π΅ΠΊΡ‚ΠΎΡ€Π½ΠΎΠ³ΠΎ произвСдСния. На рис. 17.4 ΠΏΠΎΠΊΠ°Π·Π°Π½Ρ‹ Π΄Π²Π° Ρ€Π°Π·Π½Ρ‹Ρ… прСдставлСния высоты ΠΈ основания ΠΏΠ°Ρ€Π°Π»Π»Π΅Π»ΠΎΠ³Ρ€Π°ΠΌΠΌΠ°. Как ΠΏΠΎΠΊΠ°Π·Π°Π½ΠΎ Π½Π° рис. 17.4a, Ρ‡Π»Π΅Π½ \ (| \ overrightarrow {\ mathbf {B}} | \ sin \ theta \) — это проСкция Π²Π΅ΠΊΡ‚ΠΎΡ€Π° \ (\ overrightarrow {\ mathbf {B}} \) Π² Π½Π°ΠΏΡ€Π°Π²Π»Π΅Π½ΠΈΠΈ пСрпСндикулярно Π²Π΅ΠΊΡ‚ΠΎΡ€Ρƒ \ (\ overrightarrow {\ mathbf {B}} \) ΠœΡ‹ Ρ‚Π°ΠΊΠΆΠ΅ ΠΌΠΎΠ³Π»ΠΈ Π±Ρ‹ Π·Π°ΠΏΠΈΡΠ°Ρ‚ΡŒ Π²Π΅Π»ΠΈΡ‡ΠΈΠ½Ρƒ Π²Π΅ΠΊΡ‚ΠΎΡ€Π½ΠΎΠ³ΠΎ произвСдСния ΠΊΠ°ΠΊ

    \ [| \ overrightarrow {\ mathbf {A}} \ times \ overrightarrow {\ mathbf {B}} | = (| \ overrightarrow {\ mathbf {A}} | \ sin \ theta) | \ overrightarrow {\ mathbf { B}} | \]

    Π§Π»Π΅Π½ \ (| \ overrightarrow {\ mathbf {A}} | \ sin \ theta \) — это проСкция Π²Π΅ΠΊΡ‚ΠΎΡ€Π° \ (\ overrightarrow {\ mathbf {A}} \) Π² Π½Π°ΠΏΡ€Π°Π²Π»Π΅Π½ΠΈΠΈ, пСрпСндикулярном Π²Π΅ΠΊΡ‚ΠΎΡ€Ρƒ \ ( \ overrightarrow {\ mathbf {B}} \), ΠΊΠ°ΠΊ ΠΏΠΎΠΊΠ°Π·Π°Π½ΠΎ Π½Π° рисункС 17.4 (Π±). Π’Π΅ΠΊΡ‚ΠΎΡ€Π½ΠΎΠ΅ ΠΏΡ€ΠΎΠΈΠ·Π²Π΅Π΄Π΅Π½ΠΈΠ΅ Π΄Π²ΡƒΡ… Π²Π΅ΠΊΡ‚ΠΎΡ€ΠΎΠ², ΠΏΠ°Ρ€Π°Π»Π»Π΅Π»ΡŒΠ½Ρ‹Ρ… (ΠΈΠ»ΠΈ Π°Π½Ρ‚ΠΈΠΏΠ°Ρ€Π°Π»Π»Π΅Π»ΡŒΠ½Ρ‹Ρ…) Π΄Ρ€ΡƒΠ³ Π΄Ρ€ΡƒΠ³Ρƒ, Ρ€Π°Π²Π½ΠΎ Π½ΡƒΠ»ΡŽ, ΠΏΠΎΡ‚ΠΎΠΌΡƒ Ρ‡Ρ‚ΠΎ ΡƒΠ³ΠΎΠ» ΠΌΠ΅ΠΆΠ΄Ρƒ Π²Π΅ΠΊΡ‚ΠΎΡ€Π°ΠΌΠΈ Ρ€Π°Π²Π΅Π½ 0 (ΠΈΠ»ΠΈ \ (\ pi \)) ΠΈ \ (\ sin (0) = 0 \) ( ΠΈΠ»ΠΈ \ (\ sin (\ pi) = 0 \)). ГСомСтричСски Π΄Π²Π° ΠΏΠ°Ρ€Π°Π»Π»Π΅Π»ΡŒΠ½Ρ‹Ρ… Π²Π΅ΠΊΡ‚ΠΎΡ€Π° Π½Π΅ ΠΈΠΌΠ΅ΡŽΡ‚ СдинствСнного ΠΊΠΎΠΌΠΏΠΎΠ½Π΅Π½Ρ‚Π°, пСрпСндикулярного ΠΈΡ… ΠΎΠ±Ρ‰Π΅ΠΌΡƒ Π½Π°ΠΏΡ€Π°Π²Π»Π΅Π½ΠΈΡŽ.

    Рисунок 17.4 ΠŸΡ€ΠΎΠ΅ΠΊΡ†ΠΈΡ (a) \ (\ overrightarrow {\ mathbf {B}} \) пСрпСндикулярно \ (\ overrightarrow {\ mathbf {A}} \), (b) ΠΈΠ· \ (\ overrightarrow {\ mathbf {A}) } \) пСрпСндикулярно \ (\ overrightarrow {\ mathbf {B}} \)

    Бвойства Π²Π΅ΠΊΡ‚ΠΎΡ€Π½ΠΎΠ³ΠΎ произвСдСния

    (1) Π’Π΅ΠΊΡ‚ΠΎΡ€Π½ΠΎΠ΅ ΠΏΡ€ΠΎΠΈΠ·Π²Π΅Π΄Π΅Π½ΠΈΠ΅ Π°Π½Ρ‚ΠΈΠΊΠΎΠΌΠΌΡƒΡ‚Π°Ρ‚ΠΈΠ²Π½ΠΎ, ΠΏΠΎΡ‚ΠΎΠΌΡƒ Ρ‡Ρ‚ΠΎ ΠΈΠ·ΠΌΠ΅Π½Π΅Π½ΠΈΠ΅ порядка Π²Π΅ΠΊΡ‚ΠΎΡ€ΠΎΠ² мСняСт Π½Π°ΠΏΡ€Π°Π²Π»Π΅Π½ΠΈΠ΅ Π²Π΅ΠΊΡ‚ΠΎΡ€Π½ΠΎΠ³ΠΎ произвСдСния ΠΏΠΎ ΠΏΡ€Π°Π²ΠΈΠ»Ρƒ ΠΏΡ€Π°Π²ΠΎΠΉ Ρ€ΡƒΠΊΠΈ:

    \ [\ overrightarrow {\ mathbf {A}} \ times \ overrightarrow {\ mathbf {B}} = — \ overrightarrow {\ mathbf {B}} \ times \ overrightarrow {\ mathbf {A}} \]

    (2) Π’Π΅ΠΊΡ‚ΠΎΡ€Π½ΠΎΠ΅ ΠΏΡ€ΠΎΠΈΠ·Π²Π΅Π΄Π΅Π½ΠΈΠ΅ ΠΌΠ΅ΠΆΠ΄Ρƒ Π²Π΅ΠΊΡ‚ΠΎΡ€ΠΎΠΌ \ (c \ overrightarrow {\ mathbf {A}} \), Π³Π΄Π΅ \ (c \) — скаляр, ΠΈ Π²Π΅ΠΊΡ‚ΠΎΡ€ΠΎΠΌ \ (\ overrightarrow {\ mathbf {B}} \) —

    \ [c \ overrightarrow {\ mathbf {A}} \ times \ overrightarrow {\ mathbf {B}} = c (\ overrightarrow {\ mathbf {A}} \ times \ overrightarrow {\ mathbf {B}}) \]

    Аналогично

    \ [\ overrightarrow {\ mathbf {A}} \ times c \ overrightarrow {\ mathbf {B}} = c (\ overrightarrow {\ mathbf {A}} \ times \ overrightarrow {\ mathbf {B}}) \]

    (3) Π’Π΅ΠΊΡ‚ΠΎΡ€Π½ΠΎΠ΅ ΠΏΡ€ΠΎΠΈΠ·Π²Π΅Π΄Π΅Π½ΠΈΠ΅ суммы Π΄Π²ΡƒΡ… Π²Π΅ΠΊΡ‚ΠΎΡ€ΠΎΠ² \ (\ overrightarrow {\ mathbf {A}} \ text {ΠΈ} \ overrightarrow {\ mathbf {B}} \) Π½Π° Π²Π΅ΠΊΡ‚ΠΎΡ€ \ (\ overrightarrow {\ mathbf {C}} \) Ρ€Π°Π²Π½ΠΎ

    \ [(\ overrightarrow {\ mathbf {A}} + \ overrightarrow {\ mathbf {B}}) \ times \ overrightarrow {\ mathbf {C}} = \ overrightarrow {\ mathbf {A}} \ times \ overrightarrow { \ mathbf {C}} + \ overrightarrow {\ mathbf {B}} \ times \ overrightarrow {\ mathbf {C}} \]

    Аналогично

    \ [\ overrightarrow {\ mathbf {A}} \ times (\ overrightarrow {\ mathbf {B}} + \ overrightarrow {\ mathbf {C}}) = \ overrightarrow {\ mathbf {A}} \ times \ overrightarrow { \ mathbf {B}} + \ overrightarrow {\ mathbf {A}} \ times \ overrightarrow {\ mathbf {C}} \]

    Π’Π΅ΠΊΡ‚ΠΎΡ€Π½ΠΎΠ΅ Ρ€Π°Π·Π»ΠΎΠΆΠ΅Π½ΠΈΠ΅ ΠΈ Π²Π΅ΠΊΡ‚ΠΎΡ€Π½ΠΎΠ΅ ΠΏΡ€ΠΎΠΈΠ·Π²Π΅Π΄Π΅Π½ΠΈΠ΅: Π΄Π΅ΠΊΠ°Ρ€Ρ‚ΠΎΠ²Ρ‹ ΠΊΠΎΠΎΡ€Π΄ΠΈΠ½Π°Ρ‚Ρ‹

    Π‘Π½Π°Ρ‡Π°Π»Π° ΠΌΡ‹ вычисляСм, Ρ‡Ρ‚ΠΎ Π²Π΅Π»ΠΈΡ‡ΠΈΠ½Π° Π²Π΅ΠΊΡ‚ΠΎΡ€Π½ΠΎΠ³ΠΎ произвСдСния Π΅Π΄ΠΈΠ½ΠΈΡ‡Π½Ρ‹Ρ… Π²Π΅ΠΊΡ‚ΠΎΡ€ΠΎΠ² \ (\ hat {\ mathbf {i}} \) ΠΈ \ (\ hat {\ mathbf {j}} \):

    \ [| \ hat {\ mathbf {i}} \ times \ hat {\ mathbf {j}} | = | \ hat {\ mathbf {i}} \ | \ hat {\ mathbf {j}} | \ sin (\ pi / 2) = 1 \]

    , ΠΏΠΎΡΠΊΠΎΠ»ΡŒΠΊΡƒ Π΅Π΄ΠΈΠ½ΠΈΡ‡Π½Ρ‹Π΅ Π²Π΅ΠΊΡ‚ΠΎΡ€Ρ‹ ΠΈΠΌΠ΅ΡŽΡ‚ Π²Π΅Π»ΠΈΡ‡ΠΈΠ½Ρƒ \ (| \ hat {\ mathbf {i}} | = | \ hat {\ mathbf {j}} | = 1 \) ΠΈ \ (\ sin (\ pi / 2) = 1 \ ).По ΠΏΡ€Π°Π²ΠΈΠ»Ρƒ ΠΏΡ€Π°Π²ΠΎΠΉ Ρ€ΡƒΠΊΠΈ Π½Π°ΠΏΡ€Π°Π²Π»Π΅Π½ΠΈΠ΅ \ (\ hat {\ mathbf {i}} \ times \ hat {\ mathbf {j}} \) находится Π² \ (+ \ hat {\ mathbf {k}} \) ΠΊΠ°ΠΊ ΠΏΠΎΠΊΠ°Π·Π°Π½ΠΎ Π½Π° рисункС 17.5. Π’Π°ΠΊΠΈΠΌ ΠΎΠ±Ρ€Π°Π·ΠΎΠΌ, \ (\ hat {\ mathbf {i}} \ times \ hat {\ mathbf {j}} = \ hat {\ mathbf {k}} \).

    Рис. 17.5 Π’Π΅ΠΊΡ‚ΠΎΡ€Π½ΠΎΠ΅ ΠΏΡ€ΠΎΠΈΠ·Π²Π΅Π΄Π΅Π½ΠΈΠ΅ \ (\ hat {\ mathbf {i}} \ times \ hat {\ mathbf {j}} \)

    ΠžΡ‚ΠΌΠ΅Ρ‚ΠΈΠΌ, Ρ‡Ρ‚ΠΎ Ρ‚ΠΎ ΠΆΠ΅ ΠΏΡ€Π°Π²ΠΈΠ»ΠΎ примСняСтся для Π΅Π΄ΠΈΠ½ΠΈΡ‡Π½Ρ‹Ρ… Π²Π΅ΠΊΡ‚ΠΎΡ€ΠΎΠ² Π² направлСниях y ΠΈ z,

    \ [\ hat {\ mathbf {j}} \ times \ hat {\ mathbf {k}} = \ hat {\ mathbf {i}}, \ quad \ hat {\ mathbf {k}} \ times \ hat { \ mathbf {i}} = \ hat {\ mathbf {j}} \]

    По Π°Π½Ρ‚ΠΈΠΊΠΎΠΌΠΌΡƒΡ‚Π°Ρ‚ΠΈΠ²Π½ΠΎΠΌΡƒ свойству (1) Π²Π΅ΠΊΡ‚ΠΎΡ€Π½ΠΎΠ³ΠΎ произвСдСния

    \ [\ hat {\ mathbf {j}} \ times \ hat {\ mathbf {i}} = — \ hat {\ mathbf {k}}, \ quad \ hat {\ mathbf {i}} \ times \ hat {\ mathbf {k}} = — \ hat {\ mathbf {j}} \]

    Π’Π΅ΠΊΡ‚ΠΎΡ€Π½ΠΎΠ΅ ΠΏΡ€ΠΎΠΈΠ·Π²Π΅Π΄Π΅Π½ΠΈΠ΅ Π΅Π΄ΠΈΠ½ΠΈΡ‡Π½ΠΎΠ³ΠΎ Π²Π΅ΠΊΡ‚ΠΎΡ€Π° \ (\ hat {\ mathbf {i}} \) с самим собой Ρ€Π°Π²Π½ΠΎ Π½ΡƒΠ»ΡŽ, ΠΏΠΎΡ‚ΠΎΠΌΡƒ Ρ‡Ρ‚ΠΎ Π΄Π²Π° Π΅Π΄ΠΈΠ½ΠΈΡ‡Π½Ρ‹Ρ… Π²Π΅ΠΊΡ‚ΠΎΡ€Π° ΠΏΠ°Ρ€Π°Π»Π»Π΅Π»ΡŒΠ½Ρ‹ Π΄Ρ€ΡƒΠ³ Π΄Ρ€ΡƒΠ³Ρƒ, \ ((\ sin (0) = 0) \),

    \ [| \ hat {\ mathbf {i}} \ times \ hat {\ mathbf {i}} | = | \ hat {\ mathbf {i}} || \ hat {\ mathbf {i}} | \ sin (0) = 0 \]

    Π’Π΅ΠΊΡ‚ΠΎΡ€Π½ΠΎΠ΅ ΠΏΡ€ΠΎΠΈΠ·Π²Π΅Π΄Π΅Π½ΠΈΠ΅ Π΅Π΄ΠΈΠ½ΠΈΡ‡Π½ΠΎΠ³ΠΎ Π²Π΅ΠΊΡ‚ΠΎΡ€Π° \ (\ hat {\ mathbf {j}} \) с самим собой ΠΈ Π΅Π΄ΠΈΠ½ΠΈΡ‡Π½ΠΎΠ³ΠΎ Π²Π΅ΠΊΡ‚ΠΎΡ€Π° \ (\ hat {\ mathbf {k}} \) с самим собой Ρ‚Π°ΠΊΠΆΠ΅ Ρ€Π°Π²Π½Ρ‹ Π½ΡƒΠ»ΡŽ ΠΏΠΎ Ρ‚ΠΎΠΉ ΠΆΠ΅ ΠΏΡ€ΠΈΡ‡ΠΈΠ½Π΅,

    \ [| \ hat {\ mathbf {j}} \ times \ hat {\ mathbf {j}} | = 0, \ quad | \ hat {\ mathbf {k}} \ times \ hat {\ mathbf {k} } | = 0 \]

    ИмСя Π² Π²ΠΈΠ΄Ρƒ эти свойства, ΠΌΡ‹ ΠΌΠΎΠΆΠ΅ΠΌ Ρ‚Π΅ΠΏΠ΅Ρ€ΡŒ Ρ€Π°Π·Ρ€Π°Π±ΠΎΡ‚Π°Ρ‚ΡŒ алгСбраичСскоС Π²Ρ‹Ρ€Π°ΠΆΠ΅Π½ΠΈΠ΅ для Π²Π΅ΠΊΡ‚ΠΎΡ€Π½ΠΎΠ³ΠΎ произвСдСния Π² Ρ‚Π΅Ρ€ΠΌΠΈΠ½Π°Ρ… ΠΊΠΎΠΌΠΏΠΎΠ½Π΅Π½Ρ‚ΠΎΠ².Π”Π°Π²Π°ΠΉΡ‚Π΅ Π²Ρ‹Π±Π΅Ρ€Π΅ΠΌ Π΄Π΅ΠΊΠ°Ρ€Ρ‚ΠΎΠ²Ρƒ систСму ΠΊΠΎΠΎΡ€Π΄ΠΈΠ½Π°Ρ‚ с Π²Π΅ΠΊΡ‚ΠΎΡ€ΠΎΠΌ \ (\ overrightarrow {\ mathbf {B}} \), Π½Π°ΠΏΡ€Π°Π²Π»Π΅Π½Π½Ρ‹ΠΌ вдоль ΠΏΠΎΠ»ΠΎΠΆΠΈΡ‚Π΅Π»ΡŒΠ½ΠΎΠΉ оси x с ΠΏΠΎΠ»ΠΎΠΆΠΈΡ‚Π΅Π»ΡŒΠ½ΠΎΠΉ ΠΊΠΎΠΌΠΏΠΎΠ½Π΅Π½Ρ‚ΠΎΠΉ x \ (B_ {x} \). Π’ΠΎΠ³Π΄Π° Π²Π΅ΠΊΡ‚ΠΎΡ€Ρ‹ \ (\ overrightarrow {\ mathbf {A}} \ text {ΠΈ} \ overrightarrow {\ mathbf {B}} \) ΠΌΠΎΠΆΠ½ΠΎ Π·Π°ΠΏΠΈΡΠ°Ρ‚ΡŒ ΠΊΠ°ΠΊ

    \ [\ overrightarrow {\ mathbf {A}} = A_ {x} \ hat {\ mathbf {i}} + A_ {y} \ hat {\ mathbf {j}} + A_ {z} \ hat {\ mathbf {k}} \]

    \ [\ overrightarrow {\ mathbf {B}} = B_ {x} \ hat {\ mathbf {i}} \]

    соотвСтствСнно. ΠŸΡ€ΠΎΠΈΠ·Π²Π΅Π΄Π΅Π½ΠΈΠ΅ Π²Π΅ΠΊΡ‚ΠΎΡ€Π° Π² ΠΊΠΎΠΌΠΏΠΎΠ½Π΅Π½Ρ‚Π°Ρ… Π²Π΅ΠΊΡ‚ΠΎΡ€Π° Ρ€Π°Π²Π½ΠΎ

    .

    \ [\ overrightarrow {\ mathbf {A}} \ times \ overrightarrow {\ mathbf {B}} = \ left (A_ {x} \ hat {\ mathbf {i}} + A_ {y} \ hat {\ mathbf {j}} + A_ {z} \ hat {\ mathbf {k}} \ right) \ times B_ {x} \ hat {\ mathbf {i}} \]

    Π­Ρ‚ΠΎ становится,

    \ [\ begin {align}
    \ overrightarrow {\ mathbf {A}} \ times \ overrightarrow {\ mathbf {B}} & = \ left (A_ {x} \ hat {\ mathbf {i}} \ times B_ {x} \ hat {\ mathbf {i}} \ right) + \ left (A_ {y} \ hat {\ mathbf {j}} \ times B_ {x} \ hat {\ mathbf {i}} \ right) + \ left (A_ {z} \ hat {\ mathbf {k}} \ times B_ {x} \ hat {\ mathbf {i}} \ right) \\
    & = A_ {x} B_ {x} (\ шляпа {\ mathbf {i}} \ times \ hat {\ mathbf {i}}) + A_ {y} B_ {x} (\ hat {\ mathbf {j}} \ times \ hat {\ mathbf {i}} ) + A_ {z} B_ {x} (\ hat {\ mathbf {k}} \ times \ hat {\ mathbf {i}}) \\
    & = — A_ {y} B_ {x} \ hat {\ mathbf {k}} + A_ {z} B_ {x} \ hat {\ mathbf {j}}
    \ end {align} \]

    Π’Ρ‹Ρ€Π°ΠΆΠ΅Π½ΠΈΠ΅ ΠΊΠΎΠΌΠΏΠΎΠ½Π΅Π½Ρ‚Π° Π²Π΅ΠΊΡ‚ΠΎΡ€Π° для Π²Π΅ΠΊΡ‚ΠΎΡ€Π½ΠΎΠ³ΠΎ произвСдСния Π»Π΅Π³ΠΊΠΎ обобщаСтся для ΠΏΡ€ΠΎΠΈΠ·Π²ΠΎΠ»ΡŒΠ½Ρ‹Ρ… Π²Π΅ΠΊΡ‚ΠΎΡ€ΠΎΠ²

    \ [\ overrightarrow {\ mathbf {A}} = A_ {x} \ hat {\ mathbf {i}} + A_ {y} \ hat {\ mathbf {j}} + A_ {z} \ hat {\ mathbf {k}} \]

    \ [\ overrightarrow {\ mathbf {B}} = B_ {x} \ hat {\ mathbf {i}} + B_ {y} \ hat {\ mathbf {j}} + B_ {z} \ hat {\ mathbf {k}} \]

    Π΄Π°Π΅Ρ‚

    \ [\ overrightarrow {\ mathbf {A}} \ times \ overrightarrow {\ mathbf {B}} = \ left (A_ {y} B_ {z} -A_ {z} B_ {y} \ right) \ hat { \ mathbf {i}} + \ left (A_ {z} B_ {x} -A_ {x} B_ {z} \ right) \ hat {\ mathbf {j}} + \ left (A_ {x} B_ {y } -A_ {y} B_ {x} \ right) \ hat {\ mathbf {k}} \]

    Π’Π΅ΠΊΡ‚ΠΎΡ€Π½ΠΎΠ΅ Ρ€Π°Π·Π»ΠΎΠΆΠ΅Π½ΠΈΠ΅ ΠΈ Π²Π΅ΠΊΡ‚ΠΎΡ€Π½ΠΎΠ΅ ΠΏΡ€ΠΎΠΈΠ·Π²Π΅Π΄Π΅Π½ΠΈΠ΅: цилиндричСскиС ΠΊΠΎΠΎΡ€Π΄ΠΈΠ½Π°Ρ‚Ρ‹

    ВспомнитС Ρ†ΠΈΠ»ΠΈΠ½Π΄Ρ€ΠΈΡ‡Π΅ΡΠΊΡƒΡŽ систСму ΠΊΠΎΠΎΡ€Π΄ΠΈΠ½Π°Ρ‚, ΠΊΠΎΡ‚ΠΎΡ€ΡƒΡŽ ΠΌΡ‹ ΠΏΠΎΠΊΠ°Π·Ρ‹Π²Π°Π΅ΠΌ Π½Π° рисункС 17.6. ΠœΡ‹ Π²Ρ‹Π±Ρ€Π°Π»ΠΈ Π΄Π²Π° направлСния: Ρ€Π°Π΄ΠΈΠ°Π»ΡŒΠ½ΠΎΠ΅ ΠΈ ΠΊΠ°ΡΠ°Ρ‚Π΅Π»ΡŒΠ½ΠΎΠ΅ ΠΊ плоскости, Π° Ρ‚Π°ΠΊΠΆΠ΅ пСрпСндикулярноС ΠΊ плоскости.

    Рисунок 17.6 ЦилиндричСскиС ΠΊΠΎΠΎΡ€Π΄ΠΈΠ½Π°Ρ‚Ρ‹

    Π•Π΄ΠΈΠ½ΠΈΡ‡Π½Ρ‹Π΅ Π²Π΅ΠΊΡ‚ΠΎΡ€Ρ‹ располоТСны ΠΏΠΎΠ΄ прямым ΡƒΠ³Π»ΠΎΠΌ Π΄Ρ€ΡƒΠ³ ΠΊ Π΄Ρ€ΡƒΠ³Ρƒ, поэтому, ΠΈΡΠΏΠΎΠ»ΡŒΠ·ΡƒΡ ΠΏΡ€Π°Π²ΠΈΠ»ΠΎ ΠΏΡ€Π°Π²ΠΎΠΉ Ρ€ΡƒΠΊΠΈ, Π²Π΅ΠΊΡ‚ΠΎΡ€Π½ΠΎΠ΅ ΠΏΡ€ΠΎΠΈΠ·Π²Π΅Π΄Π΅Π½ΠΈΠ΅ Π΅Π΄ΠΈΠ½ΠΈΡ‡Π½Ρ‹Ρ… Π²Π΅ΠΊΡ‚ΠΎΡ€ΠΎΠ² задаСтся ΡΠΎΠΎΡ‚Π½ΠΎΡˆΠ΅Π½ΠΈΡΠΌΠΈ

    \ [\ hat {\ mathbf {r}} \ times \ hat {\ boldsymbol {\ theta}} = \ hat {\ mathbf {k}} \]

    \ [\ hat {\ boldsymbol {\ theta}} \ times \ hat {\ mathbf {k}} = \ hat {\ mathbf {r}} \]

    \ [\ hat {\ mathbf {k}} \ times \ hat {\ mathbf {r}} = \ hat {\ boldsymbol {\ theta}} \]

    ΠŸΠΎΡΠΊΠΎΠ»ΡŒΠΊΡƒ Π²Π΅ΠΊΡ‚ΠΎΡ€Π½ΠΎΠ΅ ΠΏΡ€ΠΎΠΈΠ·Π²Π΅Π΄Π΅Π½ΠΈΠ΅ удовлСтворяСт ΡƒΡΠ»ΠΎΠ²ΠΈΡŽ \ (\ overrightarrow {\ mathbf {A}} \ times \ overrightarrow {\ mathbf {B}} = — \ overrightarrow {\ mathbf {B}} \ times \ overrightarrow {\ mathbf {A}} \) Ρƒ нас Ρ‚Π°ΠΊΠΆΠ΅ Π΅ΡΡ‚ΡŒ

    \ [\ hat {\ boldsymbol {\ theta}} \ times \ hat {\ mathbf {r}} = — \ hat {\ mathbf {k}} \]

    \ [\ hat {\ mathbf {k}} \ times \ hat {\ boldsymbol {\ theta}} = — \ hat {\ mathbf {r}} \]

    \ [\ hat {\ mathbf {r}} \ times \ hat {\ mathbf {k}} = — \ hat {\ boldsymbol {\ theta}} \]

    НаконСц

    \ [\ hat {\ mathbf {r}} \ times \ hat {\ mathbf {r}} = \ hat {\ boldsymbol {\ theta}} \ times \ hat {\ boldsymbol {\ theta}} = \ hat { \ mathbf {k}} \ times \ hat {\ mathbf {k}} = \ overrightarrow {\ mathbf {0}} \]

    ΠŸΡ€ΠΈΠΌΠ΅Ρ€ 17.1 ΠŸΡ€ΠΎΠ΄ΡƒΠΊΡ‚Ρ‹ Π²Π΅ΠΊΡ‚ΠΎΡ€Π°

    Для Π΄Π²ΡƒΡ… Π²Π΅ΠΊΡ‚ΠΎΡ€ΠΎΠ² \ (\ overrightarrow {\ mathbf {A}} = 2 \ hat {\ mathbf {i}} + — 3 \ hat {\ mathbf {j}} + 7 \ hat {\ mathbf {k}} \) ΠΈ \ (\ overrightarrow {\ mathbf {B}} = 5 \ hat {\ mathbf {i}} + \ hat {\ mathbf {j}} + 2 \ hat {\ mathbf {k}} \), Π½Π°ΠΉΠ΄ΠΈΡ‚Π΅ \ (\ overrightarrow {\ mathbf {A}} \ times \ overrightarrow {\ mathbf {B}} \).

    РСшСниС:

    \ [\ begin {align}
    \ overrightarrow {\ mathbf {A}} \ times \ overrightarrow {\ mathbf {B}} & = \ left (A_ {y} B_ {z} -A_ {z} B_ {y } \ right) \ hat {\ mathbf {i}} + \ left (A_ {z} B_ {x} -A_ {x} B_ {z} \ right) \ hat {\ mathbf {j}} + \ left ( A_ {x} B_ {y} -A_ {y} B_ {x} \ right) \ hat {\ mathbf {k}} \\
    & = ((- 3) (2) — (7) (1)) \ hat {\ mathbf {i}} + ((7) (5) — (2) (2)) \ hat {\ mathbf {j}} + ((2) (1) — (- 3) (5) ) \ hat {\ mathbf {k}} \\
    & = — 13 \ hat {\ mathbf {i}} + 31 \ hat {\ mathbf {j}} + 17 \ hat {\ mathbf {k}}
    \ ΠΊΠΎΠ½Π΅Ρ† {Π²Ρ‹Ρ€ΠΎΠ²Π½Π΅Π½} \]

    ΠŸΡ€ΠΈΠΌΠ΅Ρ€ 17.2 Π—Π°ΠΊΠΎΠ½ Бинуса

    Для Ρ‚Ρ€Π΅ΡƒΠ³ΠΎΠ»ΡŒΠ½ΠΈΠΊΠ°, ΠΏΠΎΠΊΠ°Π·Π°Π½Π½ΠΎΠ³ΠΎ Π½Π° рисункС 17.7a, Π΄ΠΎΠΊΠ°ΠΆΠΈΡ‚Π΅ Π·Π°ΠΊΠΎΠ½ синусов, \ (| \ overrightarrow {\ mathbf {A}} | / \ sin \ alpha = | \ overrightarrow {\ mathbf {B}} | / \ sin \ beta = | \ overrightarrow {\ mathbf {C}} | / \ sin \ gamma \), ΠΈΡΠΏΠΎΠ»ΡŒΠ·ΡƒΡ Π²Π΅ΠΊΡ‚ΠΎΡ€Π½ΠΎΠ΅ ΠΏΡ€ΠΎΠΈΠ·Π²Π΅Π΄Π΅Π½ΠΈΠ΅.

    Рис. 17.7 (b) Π’Π΅ΠΊΡ‚ΠΎΡ€Π½Ρ‹ΠΉ Π°Π½Π°Π»ΠΈΠ·

    РСшСниС: Рассмотрим ΠΏΠ»ΠΎΡ‰Π°Π΄ΡŒ Ρ‚Ρ€Π΅ΡƒΠ³ΠΎΠ»ΡŒΠ½ΠΈΠΊΠ°, ΠΎΠ±Ρ€Π°Π·ΠΎΠ²Π°Π½Π½ΠΎΠ³ΠΎ трСмя Π²Π΅ΠΊΡ‚ΠΎΡ€Π°ΠΌΠΈ \ (\ overrightarrow {\ mathbf {A}}, \ overrightarrow {\ mathbf {B}} \) ΠΈ \ (\ overrightarrow { \ mathbf {C}} \), Π³Π΄Π΅ \ (\ overrightarrow {\ mathbf {A}} + \ overrightarrow {\ mathbf {B}} + \ overrightarrow {\ mathbf {C}} = 0 \) (рисунок 17.7Π±). ΠŸΠΎΡΠΊΠΎΠ»ΡŒΠΊΡƒ \ (\ overrightarrow {\ mathbf {A}} + \ overrightarrow {\ mathbf {B}} + \ overrightarrow {\ mathbf {C}} = 0 \), ΠΌΡ‹ ΠΈΠΌΠ΅Π΅ΠΌ \ (0 = \ overrightarrow {\ mathbf { A}} \ times (\ overrightarrow {\ mathbf {A}} + \ overrightarrow {\ mathbf {B}} + \ overrightarrow {\ mathbf {C}}) = \ overrightarrow {\ mathbf {A}} \ times \ overrightarrow {\ mathbf {B}} + \ overrightarrow {\ mathbf {A}} \ times \ overrightarrow {\ mathbf {C}} \). Π’Π°ΠΊΠΈΠΌ ΠΎΠ±Ρ€Π°Π·ΠΎΠΌ, \ (\ overrightarrow {\ mathbf {A}} \ times \ overrightarrow {\ mathbf {B}} = — \ overrightarrow {\ mathbf {A}} \ times \ overrightarrow {\ mathbf {C}} \) ΠΈΠ»ΠΈ \ ( | \ overrightarrow {\ mathbf {A}} \ times \ overrightarrow {\ mathbf {B}} | = | \ overrightarrow {\ mathbf {A}} \ times \ overrightarrow {\ mathbf {C}} | \).Из рисунка 17.7b Π²ΠΈΠ΄Π½ΠΎ, Ρ‡Ρ‚ΠΎ \ (| \ overrightarrow {\ mathbf {A}} \ times \ overrightarrow {\ mathbf {B}} | = | \ overrightarrow {\ mathbf {A}} || \ overrightarrow {\ mathbf { B}} | \ sin \ gamma \) ΠΈ \ (| \ overrightarrow {\ mathbf {A}} \ times \ overrightarrow {\ mathbf {C}} | = | \ overrightarrow {\ mathbf {A}} || \ overrightarrow {\ mathbf {C}} | \ sin \ beta \). Π‘Π»Π΅Π΄ΠΎΠ²Π°Ρ‚Π΅Π»ΡŒΠ½ΠΎ, \ (| \ overrightarrow {\ mathbf {A}} || \ overrightarrow {\ mathbf {B}} | \ sin \ gamma = | \ overrightarrow {\ mathbf {A}} || \ overrightarrow {\ mathbf {C} } | \ sin \ beta \), ΠΈ, ΡΠ»Π΅Π΄ΠΎΠ²Π°Ρ‚Π΅Π»ΡŒΠ½ΠΎ, \ (| \ overrightarrow {\ mathbf {B}} | / \ sin \ beta = | \ overrightarrow {\ mathbf {C}} | / \ sin \ gamma \).Аналогичный Π°Ρ€Π³ΡƒΠΌΠ΅Π½Ρ‚ ΠΏΠΎΠΊΠ°Π·Ρ‹Π²Π°Π΅Ρ‚, Ρ‡Ρ‚ΠΎ \ (| \ overrightarrow {\ mathbf {B}} | / \ sin \ beta = | \ overrightarrow {\ mathbf {A}} | / \ sin \ alpha \) Π΄ΠΎΠΊΠ°Π·Ρ‹Π²Π°Π΅Ρ‚ Π·Π°ΠΊΠΎΠ½ синусов.

    ΠŸΡ€ΠΈΠΌΠ΅Ρ€ 17.3 Π•Π΄ΠΈΠ½ΠΈΡ†Π° ΠΠΎΡ€ΠΌΠ°Π»ΡŒΠ½Π°Ρ

    НайдитС Π΅Π΄ΠΈΠ½ΠΈΡ‡Π½Ρ‹ΠΉ Π²Π΅ΠΊΡ‚ΠΎΡ€, пСрпСндикулярный \ (\ overrightarrow {\ mathbf {A}} = \ hat {\ mathbf {i}} + \ hat {\ mathbf {j}} — \ hat {\ mathbf {k}} \) ΠΈ \ (\ overrightarrow {\ mathbf {B}} = — 2 \ hat {\ mathbf {i}} — \ hat {\ mathbf {j}} + 3 \ hat {\ mathbf {k}} \).

    РСшСниС: Π²Π΅ΠΊΡ‚ΠΎΡ€Π½ΠΎΠ΅ ΠΏΡ€ΠΎΠΈΠ·Π²Π΅Π΄Π΅Π½ΠΈΠ΅ \ (\ overrightarrow {\ mathbf {A}} \ times \ overrightarrow {\ mathbf {B}} \) пСрпСндикулярно ΠΎΠ±ΠΎΠΈΠΌ \ (\ overrightarrow {\ mathbf {A}} \ text {ΠΈ} \ overrightarrow {\ mathbf {B}} \).Π‘Π»Π΅Π΄ΠΎΠ²Π°Ρ‚Π΅Π»ΡŒΠ½ΠΎ, Π΅Π΄ΠΈΠ½ΠΈΡ‡Π½Ρ‹Π΅ Π²Π΅ΠΊΡ‚ΠΎΡ€Ρ‹ \ (\ hat {\ mathbf {n}} = \ pm \ overrightarrow {\ mathbf {A}} \ times \ overrightarrow {\ mathbf {B}} / | \ overrightarrow {\ mathbf {A}} \ times \ overrightarrow {\ mathbf {B}} | \) пСрпСндикулярны ΠΎΠ±ΠΎΠΈΠΌ \ (\ overrightarrow {\ mathbf {A}} \ text {ΠΈ} \ overrightarrow {\ mathbf {B}} \). {1/2} \]

    ΠŸΡ€ΠΈΠΌΠ΅Ρ€ 17.4 ОбъСм ΠΏΠ°Ρ€Π°Π»Π»Π΅Π»Π΅ΠΏΠΈΠΏΠ΅Π΄Π°

    ΠŸΠΎΠΊΠ°ΠΆΠΈΡ‚Π΅, Ρ‡Ρ‚ΠΎ объСм ΠΏΠ°Ρ€Π°Π»Π»Π΅Π»Π΅ΠΏΠΈΠΏΠ΅Π΄Π° с Ρ€Π΅Π±Ρ€Π°ΠΌΠΈ, ΠΎΠ±Ρ€Π°Π·ΠΎΠ²Π°Π½Π½Ρ‹ΠΌΠΈ Π²Π΅ΠΊΡ‚ΠΎΡ€Π°ΠΌΠΈ \ (\ overrightarrow {\ mathbf {A}}, \ overrightarrow {\ mathbf {B}}, \ text {and} \) \ (\ overrightarrow {\ mathbf {C}} \) задаСтся ΠΊΠ°ΠΊ \ (\ overrightarrow {\ mathbf {A}} \ cdot (\ overrightarrow {\ mathbf {B}} \ times \ overrightarrow {\ mathbf {C}}) \).

    РСшСниС: ОбъСм ΠΏΠ°Ρ€Π°Π»Π»Π΅Π»Π΅ΠΏΠΈΠΏΠ΅Π΄Π° опрСдСляСтся ΠΊΠ°ΠΊ ΠΏΠ»ΠΎΡ‰Π°Π΄ΡŒ основания, умноТСнная Π½Π° высоту. Если основаниС ΠΎΠ±Ρ€Π°Π·ΠΎΠ²Π°Π½ΠΎ Π²Π΅ΠΊΡ‚ΠΎΡ€Π°ΠΌΠΈ \ (\ overrightarrow {\ mathbf {B}} \ text {ΠΈ} \ overrightarrow {\ mathbf {C}} \), Ρ‚ΠΎ ΠΏΠ»ΠΎΡ‰Π°Π΄ΡŒ основания опрСдСляСтся Π²Π΅Π»ΠΈΡ‡ΠΈΠ½ΠΎΠΉ \ ( \ overrightarrow {\ mathbf {B}} \ times \ overrightarrow {\ mathbf {C}} \).Π’Π΅ΠΊΡ‚ΠΎΡ€ \ (\ overrightarrow {\ mathbf {B}} \ times \ overrightarrow {\ mathbf {C}} = | \ overrightarrow {\ mathbf {B}} \ times \ overrightarrow {\ mathbf {C}} | \ hat { \ mathbf {n}} \), Π³Π΄Π΅ \ (\ hat {\ mathbf {n}} \) — Π΅Π΄ΠΈΠ½ΠΈΡ‡Π½Ρ‹ΠΉ Π²Π΅ΠΊΡ‚ΠΎΡ€, пСрпСндикулярный основанию (рис. 17.8).

    Рисунок 17.8 ΠŸΡ€ΠΈΠΌΠ΅Ρ€ 17.4

    ΠŸΡ€ΠΎΠ΅ΠΊΡ†ΠΈΡ Π²Π΅ΠΊΡ‚ΠΎΡ€Π° \ (\ overrightarrow {\ mathbf {A}} \) вдоль направлСния \ (\ hat {\ mathbf {n}} \) Π΄Π°Π΅Ρ‚ высоту ΠΏΠ°Ρ€Π°Π»Π»Π΅Π»Π΅ΠΏΠΈΠΏΠ΅Π΄Π°. Π­Ρ‚Π° проСкция даСтся ΠΏΡƒΡ‚Π΅ΠΌ взятия скалярного произвСдСния \ (\ overrightarrow {\ mathbf {A}} \) с Π΅Π΄ΠΈΠ½ΠΈΡ‡Π½Ρ‹ΠΌ Π²Π΅ΠΊΡ‚ΠΎΡ€ΠΎΠΌ ΠΈ Ρ€Π°Π²Π½Π° \ (\ overrightarrow {\ mathbf {A}} \ cdot \ hat {\ mathbf { n}} = \ тСкст {высота} \).Π‘Π»Π΅Π΄ΠΎΠ²Π°Ρ‚Π΅Π»ΡŒΠ½ΠΎ,

    \ [\ overrightarrow {\ mathbf {A}} \ cdot (\ overrightarrow {\ mathbf {B}} \ times \ overrightarrow {\ mathbf {C}}) = \ overrightarrow {\ mathbf {A}} \ cdot (| \ overrightarrow {\ mathbf {B}} \ times \ overrightarrow {\ mathbf {C}} |) \ hat {\ mathbf {n}} = (| \ overrightarrow {\ mathbf {B}} \ times \ overrightarrow {\ mathbf {C}} |) \ overrightarrow {\ mathbf {A}} \ cdot \ hat {\ mathbf {n}} = (\ text {area}) (\ text {height}) = (\ text {volume}) \ ]

    ΠŸΡ€ΠΈΠΌΠ΅Ρ€ 17.5 Π Π°Π·Π»ΠΎΠΆΠ΅Π½ΠΈΠ΅ Π²Π΅ΠΊΡ‚ΠΎΡ€Π°

    ΠŸΡƒΡΡ‚ΡŒ \ (\ overrightarrow {\ mathbf {A}} \) Π±ΡƒΠ΄Π΅Ρ‚ ΠΏΡ€ΠΎΠΈΠ·Π²ΠΎΠ»ΡŒΠ½Ρ‹ΠΌ Π²Π΅ΠΊΡ‚ΠΎΡ€ΠΎΠΌ ΠΈ ΠΏΡƒΡΡ‚ΡŒ \ (\ hat {\ mathbf {n}} \) Π±ΡƒΠ΄Π΅Ρ‚ Π΅Π΄ΠΈΠ½ΠΈΡ‡Π½Ρ‹ΠΌ Π²Π΅ΠΊΡ‚ΠΎΡ€ΠΎΠΌ Π² Π½Π΅ΠΊΠΎΡ‚ΠΎΡ€ΠΎΠΌ фиксированном Π½Π°ΠΏΡ€Π°Π²Π»Π΅Π½ΠΈΠΈ.ΠŸΠΎΠΊΠ°ΠΆΠΈΡ‚Π΅, Ρ‡Ρ‚ΠΎ \ (\ overrightarrow {\ mathbf {A}} = (\ overrightarrow {\ mathbf {A}} \ cdot \ hat {\ mathbf {n}}) \ hat {\ mathbf {n}} + (\ hat { \ mathbf {n}} \ times \ overrightarrow {\ mathbf {A}}) \ times \ hat {\ mathbf {n}} \)

    РСшСниС: ΠŸΡƒΡΡ‚ΡŒ \ (\ overrightarrow {\ mathbf {A}} = A _ {\ |} \ hat {\ mathbf {n}} + A _ {\ perp} \ hat {\ mathbf {e}} \), Π³Π΄Π΅ \ ( A _ {\ |} \) — это ΠΊΠΎΠΌΠΏΠΎΠ½Π΅Π½Ρ‚ \ (\ overrightarrow {\ mathbf {A}} \) Π² Π½Π°ΠΏΡ€Π°Π²Π»Π΅Π½ΠΈΠΈ \ (\ hat {\ mathbf {n}}, \ hat {\ mathbf {e}} \) — Π½Π°ΠΏΡ€Π°Π²Π»Π΅Π½ΠΈΠ΅ ΠΏΡ€ΠΎΠ΅ΠΊΡ†ΠΈΠΈ \ (\ overrightarrow {\ mathbf {A}} \) Π½Π° ΠΏΠ»ΠΎΡΠΊΠΎΡΡ‚ΡŒ, ΠΏΠ΅Ρ€ΠΏΠ΅Π½Π΄ΠΈΠΊΡƒΠ»ΡΡ€Π½ΡƒΡŽ \ (\ hat {\ mathbf {n}} \), Π° \ (A _ {\ perp} \) — ΠΊΠΎΠΌΠΏΠΎΠ½Π΅Π½Ρ‚ \ (\ overrightarrow {\ mathbf {A}} \) Π² Π½Π°ΠΏΡ€Π°Π²Π»Π΅Π½ΠΈΠΈ \ (\ hat {\ mathbf {e}} \).ΠŸΠΎΡΠΊΠΎΠ»ΡŒΠΊΡƒ \ (\ hat {\ mathbf {e}} \ cdot \ hat {\ mathbf {n}} = 0 \), Ρƒ нас Π΅ΡΡ‚ΡŒ \ (\ overrightarrow {\ mathbf {A}} \ cdot \ hat {\ mathbf { n}} = A _ {\ |} \). ΠžΠ±Ρ€Π°Ρ‚ΠΈΡ‚Π΅ Π²Π½ΠΈΠΌΠ°Π½ΠΈΠ΅, Ρ‡Ρ‚ΠΎ

    \ [\ hat {\ mathbf {n}} \ times \ overrightarrow {\ mathbf {A}} = \ hat {\ mathbf {n}} \ times \ left (A \ hat {\ mathbf {n}} + A_ {\ perp} \ hat {\ mathbf {e}} \ right) = \ hat {\ mathbf {n}} \ times A _ {\ perp} \ hat {\ mathbf {e}} = A _ {\ perp} (\ шляпа {\ mathbf {n}} \ times \ hat {\ mathbf {e}}) \]

    Π•Π΄ΠΈΠ½ΠΈΡ‡Π½Ρ‹ΠΉ Π²Π΅ΠΊΡ‚ΠΎΡ€ \ (\ hat {\ mathbf {n}} \ times \ hat {\ mathbf {e}} \) Π»Π΅ΠΆΠΈΡ‚ Π² плоскости, пСрпСндикулярной \ (\ hat {\ mathbf {n}} \), Π° Ρ‚Π°ΠΊΠΆΠ΅ пСрпСндикулярно \ (\ hat {\ mathbf {e}} \).Π‘Π»Π΅Π΄ΠΎΠ²Π°Ρ‚Π΅Π»ΡŒΠ½ΠΎ, \ ((\ hat {\ mathbf {n}} \ times \ hat {\ mathbf {e}}) \ times \ hat {\ mathbf {n}} \) Ρ‚Π°ΠΊΠΆΠ΅ являСтся Π΅Π΄ΠΈΠ½ΠΈΡ‡Π½Ρ‹ΠΌ Π²Π΅ΠΊΡ‚ΠΎΡ€ΠΎΠΌ, ΠΏΠ°Ρ€Π°Π»Π»Π΅Π»ΡŒΠ½Ρ‹ΠΌ \ (\ шляпа {\ mathbf {e}} \) (ΠΏΠΎ ΠΏΡ€Π°Π²ΠΈΠ»Ρƒ ΠΏΡ€Π°Π²ΠΎΠΉ Ρ€ΡƒΠΊΠΈ. Π˜Ρ‚Π°ΠΊ, \ ((\ hat {\ mathbf {n}} \ times \ overrightarrow {\ mathbf {A}}) \ times \ hat {\ mathbf { n}} = A _ {\ perp} \ hat {\ mathbf {e}} \). Π’Π°ΠΊΠΈΠΌ ΠΎΠ±Ρ€Π°Π·ΠΎΠΌ,

    \ [\ overrightarrow {\ mathbf {A}} = A _ {\ |} \ hat {\ mathbf {n}} + A _ {\ perp} \ hat {\ mathbf {e}} = (\ overrightarrow {\ mathbf { A}} \ cdot \ hat {\ mathbf {n}}) \ hat {\ mathbf {n}} + (\ hat {\ mathbf {n}} \ times \ overrightarrow {\ mathbf {A}}) \ times \ шляпа {\ mathbf {n}} \]

    .

alexxlab

Π”ΠΎΠ±Π°Π²ΠΈΡ‚ΡŒ ΠΊΠΎΠΌΠΌΠ΅Π½Ρ‚Π°Ρ€ΠΈΠΉ

Π’Π°Ρˆ адрСс email Π½Π΅ Π±ΡƒΠ΄Π΅Ρ‚ ΠΎΠΏΡƒΠ±Π»ΠΈΠΊΠΎΠ²Π°Π½. ΠžΠ±ΡΠ·Π°Ρ‚Π΅Π»ΡŒΠ½Ρ‹Π΅ поля ΠΏΠΎΠΌΠ΅Ρ‡Π΅Π½Ρ‹ *